Download as pdf or txt
Download as pdf or txt
You are on page 1of 350

Electrical Machines -1

Lecture Notes

Prepared By

Dr.Karimulla Peerla Shaik


Associate Professor
Department of EEE

Department of Electrical & Electronics Engineering

Malla Reddy College of Engineering & Technology


Maisammaguda, Dhullapally, Secunderabad-500100
2022-23
CONTENTS

Syllabus:
UNIT-I: D.C GENERATORS
UNIT-II: D.C. MOTORS
UNIT-III: TESTING OF D.C. MACHINES
UNIT-IV: SINGLE PHASE TRANSFORMERS
UNIT-V: TESTING OF TRANSFORMERS AND POLY-PHASE
TRANSFORMERS
MALLA REDDY COLLEGE OF ENGINEERING AND TECHNOLOGY
II YEAR B.Tech EEE –I SEM L/T/P/C
3/0/0/3
(R20A0202) ELECTRICAL MACHINES - I
COURSE OBJECTIVES:

1. To understand the basic working principle, constructional details and operational features of DC
Generators.

2. To study and understand the various characteristics DC Generators.

3. To learn the various methods of speed control of DC Motors.

4. To introduce the concept of regulation and its calculations.

5. To learn the concepts of single phase and three phase transformers circuits

UNIT – I
D.C GENERATORS: Principle of operation - constructional features - Action of commutator –
armature windings – lap and wave windings – simplex and multiplex windings – use of laminated
core – E. M.F Equation – Problems, Armature reaction – Cross magnetizing and de-magnetizing
AT/pole – compensating winding - Commutation – reactance voltage – methods of improving
commutation. Methods of Excitation – separately excited and self- excited generators – build-up of
E.M.F - critical field resistance and critical speed - causes for failure to self-excite and remedial
measures. Load characteristics of shunt, series and compound generators
UNIT – II
D.C. MOTORS: Principle of operation – Back E.M.F. - Torque equation – characteristics and
application of shunt, series and compound motors – Armature reaction and commutation. Speed
control of D.C. Motors - Armature voltage and field flux control methods. Motor starters (3 point
and 4-point starters).
UNIT – III
TESTING OF D.C. MACHINES: Losses – Constant & Variable losses – calculation of efficiency
condition for maximum efficiency. Methods of Testing – direct, indirect and regenerative testing –
brake test – Swinburne’s test – Hopkinson’s test – Field’s test – Retardation test – separation of
stray losses in a DC Motor test.
UNIT – IV
SINGLE PHASE TRANSFORMERS: Principle of operation - constructional features -Types -
minimization of hysteresis and eddy current losses- EMF equation - operation on no load and on
load - phasor diagrams. Equivalent circuit - losses and efficiency – regulation - All day efficiency -
effect of variations of frequency & supply voltage on iron losses.
UNIT – V
TESTING OF TRANSFORMERS AND POLY-PHASE TRANSFORMERS: OC and SC tests - Sumpner’s test -
predetermination of efficiency and regulation-separation of losses test- parallel operation with
equal and unequal voltage ratios - auto transformers- equivalent
circuit - comparison with two winding transformers. Poly-phase transformers – Poly-phase
connections - Y/Y, Y/Δ, Δ/Y, Δ/Δ and open Δ
TEXT BOOKS:

1. Electric machinery – A.E. Fitzgerald, C.Kingsley and S.Umans, Mc Graw Hill Companies,

2. A. E. Clayton and N. N. Hancock, “Performance and design of DC machines”, CBS


Publishers,2004.

3. M. G. Say, “Performance and design of AC machines”, CBS Publishers, 2002.

REFERENCE BOOKS:

1. Electric Machinery Fundamentals, Stephen J. Chapmen, Tata Mc Graw –Hill Publishers.

2. A. S. Langsdorf, “Alternating current machines”, McGraw Hill Education, 1984.

3. P. C. Sen, “Principles of Electric Machines and Power Electronics”, John Wiley & Sons, 2007.

4. Electrical Machines, P.S. Bimbra, Khanna Publishers.

5. Electric Machines by I.J. Nagrath & D.P. Kothari, Tata Mc Graw –Hill Publishers.

COURSE OUTCOMES:
At the end of this course the student would get

1. Explain the Constructional features of DC Generators, DC motors and transformers.

2. Understand different excitation and starting methods of DC machines.

3. Summarize Testing of different types of DC Generators and DC motors.

4. Carry out different testing methods and assess the performance of transformers.

5. Analyze single phase and three phase transformers.


Electrical Machines -1 (EM-1): Lecture Notes: (Dr.Karimulla P Sk)
Unit 1: D C Generators

UNIT – I
D.C GENERATORS
CONTENTS:
 Principle of operation- Constructional features
 Action of Commutator
 Armature windings – lap and wave windings – simplex and
multiplex windings
 Use of laminated core –
 E. M.F Equation – Problems,
 Armature Reaction – Cross magnetizing and demagnetizing
AT/pole – compensating winding
 Commutation – reactance voltage – methods of improving
commutation.
 Methods of Excitation – separately excited and self- excited
generators – build-up of E.M.F - critical field resistance and
critical speed - causes for failure to self-excite and remedial
measures.
 Load characteristics of shunt, series and compound generators
 Important concepts and Formulae:
 Illustrative examples

Page 1
Malla Reddy College of Engineering and Technology
Department of EEE (2022-23)
Electrical Machines -1 (EM-1): Lecture Notes: (Dr.Karimulla P Sk)
Unit 1: D C Generators

Introduction:

A DC generator is a rotating machine which converts mechanical energy into DC


electrical energy. It requires a prime mover like a Diesel engine, wind turbine or a
steam turbine to rotate the DC generator. An EMF is induced in a DC Generator
when there is a relative motion between a Magnetic field and a set of electrical
conductors. The EMF induced is called a dynamically induced EMF or motional
EMF . Normally the magnetic field is stationary and is obtained from stationary
field coils placed on the Stator poles and the conductors are placed on a rotating
shaft called Rotor. The basic constructional features of a DC generator and a DC
Motor are same, and the same DC machine can work either as a DC generator or
a DC motor.

The conversion of Mechanical energy into Electrical energy in DC generator is


based on the principle of electromagnetic Induction. According to Faradays laws
of Electromagnetic induction, whenever a conductor moves in a magnetic field a
dynamically induced EMF is produced across the conductor. When the terminals
of the conductor are connected to an electrical load the induced EMF enables a
current flow through the load. Thus a mechanical energy in the form of a
rotational motion given to a conductor is converted into Electrical energy. The
EMF induced in a single conductor is very small. Hence a large set of conductors
are used in practical generators and such a set of conductors placed on a rotating
round shaft is called an armature.

Principle of operation of DC Machines:

Let us consider a single turn of coil ABCD mounted on a cylindrical shaft and
rotated in an anticlockwise direction at constant angular velocity of ‘ω‘ rad/sec
within a uniform magnetic field of flux density B webers/mtrs2 as shown in the
figure below .

Page 2
Malla Reddy College of Engineering and Technology
Department of EEE (2022-23)
Electrical Machines -1 (EM-1): Lecture Notes: (Dr.Karimulla P Sk)
Unit 1: D C Generators

Let l be the length and b be the breadth of the rectangular coil in meters.
According to Faradays law the emf induced in a conductor is given by e =
− N.dØ/dt where e is the induced emf , N is the number of conductors , Ø is the
flux linkage and t is the time. The flux linkage Ø is given by : Ø = B.area of the
coil.cos ωt = B.l.b.Cos ωt

Since we are considering only one conductor the induced emf in the conductor is
given by:

e = -dØ/dt = -d(B.l.b.Cos ωt)/dt = B.l.b.ω .Sin ωt = Em Sin ωt where Em =


B.l.b.ω

As can be seen from the above equation for induced emf the voltage in a given
generator can be increased by either increasing the flux density ‘B’ or the
rotational speed ’ω’ .

The induced emf ‘e’at any position of the coil as a function of time ‘t’ as derived
above is then given by : e = Em Sin ωt where Em = B.l.b.ω. As can be seen
dØ/dt i.e rate of change of flux linkage is minimum (=0) when the coil is at
perpendicular position to the flux lines and hence the induced voltage e is also

Page 3
Malla Reddy College of Engineering and Technology
Department of EEE (2022-23)
Electrical Machines -1 (EM-1): Lecture Notes: (Dr.Karimulla P Sk)
Unit 1: D C Generators

minimum (=0) . We will call this as position Ѳ = 00 at the instant of say t = 0 sec.
And dØ/dt is maximum when the coil is at parallel position to the flux lines and
hence the induced voltage e is also maximum( = Em = B.l.b.ω) and this position
will then be Ѳ = 900 .When Ѳ = 1800 the induced emf is again zero and when Ѳ =
2700 the emf induced is again maximum but now it would be negative. When Ѳ =
3600 the coil is back to the original position and the induced emf is again equal to
zero. For the two pole generator shown in the figure one complete cycle of
change takes place in one rotation of the coil. A plot of the induced emf ‘e’ as
function of coil position Ѳ is an alternating voltage as shown in the figure below.

Fig: emf induced in a single turn generator in one full revolution

When the two terminals of the coil are connected to an external load (resistance
in this case) through two separate rings (called slip rings) mounted on the
armature current flows through the resistance and the current also would be
sinusoidal.

Action of commutator: We have seen that the output from a simple single turn
generator in one full revolution is a sinusoidal in nature (AC). Commutator is the
most important part of a DC Generator which converts the AC to DC. The current
flowing through the external load can be made unidirectional by replacing the
two slip rings with two split rings as shown in the figure below which is the basis
Page 4
Malla Reddy College of Engineering and Technology
Department of EEE (2022-23)
Electrical Machines -1 (EM-1): Lecture Notes: (Dr.Karimulla P Sk)
Unit 1: D C Generators

for the operation of a commutator in a practical DC Machine with more number


of Poles and multiple coils .

One slip ring is split into two equal segments P and Q which are insulated from
each other and the armature shaft. The two coils AB and CD are connected to the
two segments P and Q .Two fixed (stationary) brushes B1 and B2 sliding along
these two split rings will be collecting the current from the generator. During the
first half of the revolution segment P is positive and current flows along
ABPLMQCD through brush B1 which is positive and into brush B 2 into segment Q
which is negative. Next during the other half cycle, the location of the segments
AB & CD will reverse along with the respective segments P and Q . Now
conductor CD and segment Q are positive and current flows along DCQLMPBA
through the Brush B1 which is again positive and into the brush B2 which is again
negative as shown in the figure below.

Page 5
Malla Reddy College of Engineering and Technology
Department of EEE (2022-23)
Electrical Machines -1 (EM-1): Lecture Notes: (Dr.Karimulla P Sk)
Unit 1: D C Generators

In each half revolution the positions of the conductors AB & CD and the segments
P &Q reverse but the brushes B1&B2 are stationary and continue to collect current
from the Positive side and deliver current to the Negative side respectively. Hence
the voltage across the load will be a unipolar voltage as shown in the waveform
above. The changeover of brushes B1&B2 between segments P &Q takes place
when the voltage is minimum so as to avoid or minimize the arcing between the
split segments. In practical generators there will be more number of conductors
and also more number of Pole pairs and hence more number of split segments
are required and such a set of more number of split segments is called
commutator.

Constructional features of a DC Generator:

Page 6
Malla Reddy College of Engineering and Technology
Department of EEE (2022-23)
Electrical Machines -1 (EM-1): Lecture Notes: (Dr.Karimulla P Sk)
Unit 1: D C Generators

Fig: A simplified diagram of a dc machine:

Major parts of a DC generator:

 Main frame or Yoke


 Poles
 Armature
 Commutator
 Brushes ,bearings and shaft

The physical structure of the machine consists of two parts: the stator and the
rotor.
The stationary part consists of the main frame (yoke), and the pole pieces, which
project inward and provide a path for the magnetic flux. The ends of the pole
pieces that are near the rotor spread out over the rotor surface to distribute its
flux evenly over the rotor surface. These ends are called the pole shoes. The
exposed surface of a pole shoe is called a pole face, and the distance between the
pole face and the rotor is the air gap.
There are two principal windings on a dc machine:

Page 7
Malla Reddy College of Engineering and Technology
Department of EEE (2022-23)
Electrical Machines -1 (EM-1): Lecture Notes: (Dr.Karimulla P Sk)
Unit 1: D C Generators

 The armature windings: the windings in which a voltage is induced (rotor)


 The field windings: the windings that produce the main magnetic flux
(stator)
Because the armature winding is located on the rotor, a dc machine’s rotor is
mostly called an armature.
The terminal characteristic of a DC Machine is a plot of the output parameters of
the Machine against each other. For a DC Generator the output quantities are the
Terminal Voltage and the Line (Load) current.

Figure: Cross sectional view of a DC Machine

The air-gap is kept very small to keep the reluctance of the magnetic circuit low.
The armature is a laminated cylinder and is mounted on a shaft. The armature
laminations are about 0.4–0.6 mm thick and are insulated from one another. The
armature is laminated to reduce the eddy-current loss in the core. Slots are
stamped on the periphery of the armature laminations. The armature slots house
the armature windings. The stator core, the yoke and the poles may not be
laminated as they encounter DC flux.
Due to the presence of slots on the armature surface, there is flux pulsation at the
Page 8
Malla Reddy College of Engineering and Technology
Department of EEE (2022-23)
Electrical Machines -1 (EM-1): Lecture Notes: (Dr.Karimulla P Sk)
Unit 1: D C Generators

Stator pole-face. The stator pole shoes, therefore, should be laminated to reduce
the eddy-current loss. However, for mechanical reasons, in many cases the whole
of the pole core is laminated. In DC Machines of high ratings slots are cut on the
pole-faces to house a separate winding called the compensating winding. The
compensating winding is connected in series with the armature winding and
neutralises the effect of armature reaction. To neutralize the effect of armature
reaction in the space in between two poles, smaller poles, called interpoles, are
fixed on the yoke as shown in Fig. 2.2.

Figure: Cross sectional view of a DC Machine showing Interpoles

As mentioned earlier, the armature winding is placed inside the armature slots.
The slots are lined with tough insulating material. This slot insulation is folded
over the armature conductors. The conductors in the slots are secured in their
places by hard wooden wedges or fiber glass wedges. The armature windings are
first made on formers and then placed on slots.
Enamel insulated copper wires are used for the armature winding. Each armature
coil end is connected with each segment of the commutator.

Page 9
Malla Reddy College of Engineering and Technology
Department of EEE (2022-23)
Electrical Machines -1 (EM-1): Lecture Notes: (Dr.Karimulla P Sk)
Unit 1: D C Generators

A commutator is a cylindrical body mounted on the shaft along with the


armature. In fact, the armature core and the commutator form one single unit
mounted on the shaft. Brushes are placed on the commutator surface to supply
or collect current to the armature coils through the commutator segments. The
commutator segments are insulated from each other.
The function of the commutator is to convert alternating currents induced in the
armature conductors into direct currents in the external circuit in case of a DC
Generator operation. In the case of a dc motor the function of the commutator is
to produce a unidirectional torque. The commutator is of cylindrical structure and
is built up of a wedge-shaped segment of hard-drawn copper. Mica insulation is
provided between commutator segments. Brushes are made of carbon and are
housed in brush-holders.
A spring in the brush-holder maintains the desirable pressure on the carbon
brushes so that proper contact is maintained between the brushes and the
commutator surface.

Armature windings:
The armature windings are vital part of a DC Machine. This is where emf is
induced in the case of a Generator and and force is developed that results in the
turning of the rotor in the case of a Motor. The design of the armature winding is
more critical than the design of other parts of a DC machine. The armature
winding is housed in slots made on the armature surface. Formed coils are placed
on slots. The ends of the coils are joined with commutator segments.
Commutator: The commutator is made up of a number of commutator
segments. Coil-ends are connected to each commutator segment. The segments
of the commutator are made of hard-drawn copper and are separated by thin
sheets of mica or micanite ( insulator) .
The induced emf per conductor in a DC machine is small. The problem is how
these conductors are to be connected together so as to form a complete winding.
Figure below shows the cross-sectional view of the armature of a four-pole
machine.

For ease of understanding, a developed diagram of armature of Fig. (a) below is


drawn as shown in Fig. (b). Conductors should be so connected that the total emf
Page 10
Malla Reddy College of Engineering and Technology
Department of EEE (2022-23)
Electrical Machines -1 (EM-1): Lecture Notes: (Dr.Karimulla P Sk)
Unit 1: D C Generators

is maximum. Therefore, conductor 1 should be connected to conductor 6 shown


by dotted line as conductor 6 is placed below conductor 5 so that they occupy
identical positions under two adjacent poles. Similarly conductor 3 should be
connected with conductor 8 and so on.

Figure : (a) Cross-sectional view of the armature of a 4-pole DC Machine (b)


Incomplete developed diagram of the armature winding

Figure below shows the developed winding diagram of the 16 armature


conductors of Fig. (a) Shown earlier.

Figure: Armature winding of a DC Machine

Page 11
Malla Reddy College of Engineering and Technology
Department of EEE (2022-23)
Electrical Machines -1 (EM-1): Lecture Notes: (Dr.Karimulla P Sk)
Unit 1: D C Generators

The average pitch Ya, back pitch Yb, and the front pitch Yf are calculated as:
Ya = 16/4= 4
Ya = (Yb + Yf) /2
Yb – Yf = ±2
For progressive lap winding
Yb – Yf = 2
Yb = 5, Yf = 3

Figure gives the details of end connections of the conductors, connection of coils
with commutator segments, and the position of brushes on the
commutatorsurface with their polarities. This type of winding is called lap
winding. In the winding shown in Fig. 2.11, single-turn conductors are used. As
many as 16 conductors make eight coils. The coils are 1-6, 3-8, 5-10, 7-12, 9-14,
11-16, 13-2 and 15-4. The design of a lap winding of the type shown in Fig. 2.11 is
described as follows.
2.3.2 Lap Winding
In a lap winding, the finishing end of one coil is connected via the commutator
segment to the starting end of the adjacent coil situated under the same pole. In
this way all the coils are connected. The winding is known as lap winding because
the sides of successive coils overlap each other (see Fig. below). A coil may consist
of any number of turns. The number of slots required on the armature is equal to
the number of coil sides if two coil-sides are placed in each slot. With two coil-
sides in each slot, a two layer winding is obtained. While making a winding
diagram in a two-layer winding, all top coil-sides are numbered odd whereas the
bottom coil-sides are numbered even (shown by dotted lines) as shown in Fig..
For an eight-coil armature, therefore, eight slots are required on the armature
surface. The following terminologies are required to be understood for preparing
an armature winding diagram.

Pole Pitch: It is equal to the number of coil-sides per pole. For a single turn, eight
coil, four-pole armature pole pitch is calculated as:

Pole pitch ( Ya) = (No. of coils x 2)/ No. of poles = (8 x 2) /4 = 4

Page 12
Malla Reddy College of Engineering and Technology
Department of EEE (2022-23)
Electrical Machines -1 (EM-1): Lecture Notes: (Dr.Karimulla P Sk)
Unit 1: D C Generators

Figure: Position of coil-sides in slots of a two-layer armature winding

Coils and Coil-sides: The DC armature windings are double-layer type having at
least two coil-sides per slot. Each coil consists of an upper coil-side at the top of
one slot and a lower coil-side situated at the bottom of another slot. The distance
between the two coil-sides of a coil is approximately equal to the pole pitch. A coil
may be of single turn or of many turns. If two coil-sides are placed in one slot,
then the number of slots required on the armature for housing the coils is equal
to the number of coils of the winding. For low-speed high-voltage winding,
however, the number of coil-sides per slot is more than two. This is because the
winding will have a large number of coils and it may not be possible to have an
equal number of slots on the armature.
Back Pitch: The distance measured in terms of the number of armature
conductors (coil sides) between the two coil-sides of a coil measured around the
back of the armature, i.e., away from the commutator end of the armature is
called the back pitch, Yb.

Page 13
Malla Reddy College of Engineering and Technology
Department of EEE (2022-23)
Electrical Machines -1 (EM-1): Lecture Notes: (Dr.Karimulla P Sk)
Unit 1: D C Generators

(a) Lap winding (b) Wave winding


Figure: Shows back pitch Yb , front pitch yf resultant pitch Yr, and
commutator pitch Yc in (a) lap winding (b) wave winding

Front Pitch: The distance between two coil-sides connected to the same
commutator segment is called the front pitch, Yf .
Resultant Pitch: It is defined as the distance in terms of the number of coil-sides
between the start of one coil and the start of the next coil to which it is
connected.
Commutator Pitch: It is defined as the distance measured in terms of
commutator segments between the segments to which the two ends of a coil are
connected.
For calculating back pitch Yb and front pitch Yf for a lap winding, the following
relations are used:
(i) Yb – Yf = ± 2m Also, Yb = (Z/P)± 1
where m = 1 for simplex winding
= 2 for duplex winding
When Yb is greater than Yf , the winding is a progressive one, i.e., it progresses
from left to right. If Yb is less than Yf , the winding is called a retrogressive one,
i.e., it progresses from right to left.
(ii) The back pitch and front pitch must be odd.
(iii) The average pitch, Ya = (Yb + Yf)/2 should be equal to the pole pitch, i.e.,
equal to Z/P, where Zis the number of coil sides.

Page 14
Malla Reddy College of Engineering and Technology
Department of EEE (2022-23)
Electrical Machines -1 (EM-1): Lecture Notes: (Dr.Karimulla P Sk)
Unit 1: D C Generators

(iv) The commutator pitch is equal to m, i.e., equal to 1, 2, etc. for simplex, duplex
etc. type of winding.
(v) The number of parallel paths in the armature winding for a simplex lap winding
is equal to the number of poles, P.
(vi) The resultant pitch is always even, being the difference of two odd numbers.

Example 1 : Prepare a layout winding diagram for a simplex lap-type DC armature


winding. The winding is for 4 poles. The armature has 16 slots and 16
commutator segments.
Solution: Number of armature coils = Number of commutator segments = 16
Number of coil-sides (conductors) Z = 16 × 2 = 32
Back pitch Yb =(Z/P) ±1 = (32/4) ±1 = 9 or 7
Yb – Yf = 2
Yf = Yb – 2 = 9 – 2 (using Yb = 9) = 7
Yb = 9
Yf = 7
Since Yb > Yf , the winding is a progressive one.
As there are 32 coil-sides and 16 slots, the number of coil-sides per slot is 2. The
connection scheme of the coil-sides is shown in Figure . below

Figure: Scheme for connections of the coil-sides of a DC armature windings

Coil-side 1 is connected to coil-side 10 on the other side of the commutator (since


Yb is 9, coil-side 1 is connected to coil-side 1 + 9, i.e., 10). Coil-side 10 is connected

Page 15
Malla Reddy College of Engineering and Technology
Department of EEE (2022-23)
Electrical Machines -1 (EM-1): Lecture Notes: (Dr.Karimulla P Sk)
Unit 1: D C Generators

to coil-side 3 on the commutator end (Since Yf is 7, coil-side 10 is connected to


coilside10–7, i.e., 3). The winding progresses according to the above scheme. It
may be noted that each coil is used once and the winding is a closed one.
The layout diagram of the winding along with commutator connections and brush
positions is shown below.

Connections of the coil-sides are made as follows: for connections at the back end
of the armature, add the back pitch with the coil-side which is to be connected.
Thus coil-side 1 is to be connected with coil-side 1 + Yb, i.e., 1 + 9 = 10. On the
commutator end side, coil-side 10 is connected to coil-side 3. This is achieved by
subtracting Yf , i.e., 7 from coil-side number 10 (10 – 7 = 3). Coil-side 3 is now
connected to 3 + Yb = 3 + 9 = 12. In this way the winding is completed.
The positions of the four poles are also shown in Fig. 2.15. Eight coil-sides placed
in four slots are under each pole. Assuming a direction of rotation of the
armature, say anticlockwise in Fig. 2.15, the direction of the induced emf in the
armature conductors is determined by applying Fleming’s right-hand rule. The
direction of the current in the coil-sides under north poles will be downward and
under south poles upward as shown in Fig. 2.15.
The position of brushes can be determined by tracing the directions of current in
various coil-sides. From Fig. 2.15, it can be observed that directions of current in
coil-sides 1 and 8 are downward and they are connected to commutator segment
1. A brush placed on commutator segment 1 will have positive polarity. Similarly
in coil-sides 9 and 16, the current is upwards. The two coil-sides are connected to
commutator segment 5. The brush placed on commutator segment 5 will have
negative polarity. Similarly the positions of the other two brushes are fi xed. Two
positive brushes and two negative brushes are joined together to output
terminals A and B respectively.
The number of parallel paths of the armature winding across the output terminals
is four (equal-to the number of poles) which can be examined as follows: Redraw
the armature winding of Fig. 2.15 in a simplifi ed manner as shown in Fig. 2.16.
Between terminals A and B there are four parallel paths shown as M, N, O and P.
The total emf generated in the machine is equal to the emf generated in one
parallel path.

Page 16
Malla Reddy College of Engineering and Technology
Department of EEE (2022-23)
Electrical Machines -1 (EM-1): Lecture Notes: (Dr.Karimulla P Sk)
Unit 1: D C Generators

Figure: Lay out diagram for a Lap winding given in example -1


Page 17
Malla Reddy College of Engineering and Technology
Department of EEE (2022-23)
Electrical Machines -1 (EM-1): Lecture Notes: (Dr.Karimulla P Sk)
Unit 1: D C Generators

Figure: (a) Armature winding of a dc machine shown in a simplified manner


(b) Shows the number of parallel paths in the armature

Equalizer Connections in Lap Winding: As mentioned earlier, a simplex lap


winding has as many number of parallel paths as there are poles. The emf induced
in each parallel path may not be exactly equal due to a number of reasons, such
as the difference in the lengths of the air-gap under each pole, the difference in
the field strength due to some error in putting field windings, etc.

Unequal values of emf generated in the parallel paths will circulate a considerable
amount of current in the armature circuit without doing any useful work. This
circulating current will be large as the armature circuit resistance is generally very
low. This circulating current will generate heat and while circulating through the
brush contacts will cause commutation diffi culties (like sparking on the
commutator surface).

Page 18
Malla Reddy College of Engineering and Technology
Department of EEE (2022-23)
Electrical Machines -1 (EM-1): Lecture Notes: (Dr.Karimulla P Sk)
Unit 1: D C Generators

To overcome this problem arising from the circulating current, equalizer


connections are made in lap wound armatures. These equaliser connections or
equalisers are low-resistance copper conductors which connect those points in
the winding which under ideal conditions should be at equal potential. The
difference in potential between these points created due to reasons mentioned
earlier will be equalised as a result of flow of current through these low resistance
conductors which will bypass the current from flowing through the brushes.

2.3.3 Wave Winding


In a wave winding a coil-side under one pole is connected to a second coil-side
which occupies approximately the same position under the next pole through
back connection. The second coil-side is then connected forward to another coil-
side under the next pole (in the case of lap winding the second coil is connected
back through the commutator segment to a coil-side under the original pole). The
difference in lap and wave winding connections has been illustrated in Fig. 2.13(a)
and (b).
The characteristics of a wave winding are:
(i) Average pitch, Ya = (Yb ± Yf)/2 = (Z ±2)/P
If Ya is taken equal to Z/P, as is the case in a lap winding the winding after one
round will close itself without including all the coils which is not desirable.
Hence the product of the average pitch and the number of pairs of poles must be
two greater or less than the number of coil-sides.
Average pitch should be a whole number.
(ii) Both back pitch and front pitch should he odd numbers.
(iii) To make the average pitch a whole number, wave winding is not possible with
any number of coil-sides. For example if Z = 32 and P = 4,
Ya = (Z±2)/P = (32±2)/4 = 8 ½ or 7 ½
Thus wave winding is not possible with 32 coil-sides. In this case the number of
effective coil-sides needs to be 30.

Example-2 : Prepare a winding diagram for a 4 pole wave-connected armature of


a dc generator having 22 coil sides.

Ya = (Z ±2)/P = (22 ±2)/4 = 6 or 5

Page 19
Malla Reddy College of Engineering and Technology
Department of EEE (2022-23)
Electrical Machines -1 (EM-1): Lecture Notes: (Dr.Karimulla P Sk)
Unit 1: D C Generators

If Ya is taken to be odd, i.e., 5, then the front pitch and back pitch will be equal.
Thus, Ya = Yb = Yf = 5.
Connections of the coil sides will be as shown in figure. The connection diagram is
achieved by adding Yb and Yf with the coil numbers progressing in the forward
direction. Coil-side 1 is connected at the back with coil side 6 (1 + Yb = 6). Coil side
6 is connected at the front with coil-side 11 (6 + Yf = 11) and so on.

Figure: Connection diagram of the coil-sides for a DC wave winding

In Fig. 2.17 it is to be noted that coil-side 19 is connected with coil-side 2. This is


obtained by adding Yb to 19 which gives 24. Coil-side 24 does not exist as there
are in all 22 coil-sides. Therefore after 22 count two more numbers starting from
1. This gives coil-side 2. Similarly it can be seen that coil-side 20 is connected in
the front with coil-side 3. By adding Yf (= 5) to 20, the number 25 is obtained.
After 20 fi ve numbers are counted as 21, 22, 1, 2, and 3. Thus coil-side 20 should
be connected to coil-side 3. In this way, the whole winding is completed by
connecting all the coil-sides with one another. The actual layout diagram of the
winding along with the position of the poles and the direction of induced emf in
the coil-sides for a particular direction of rotation of the armature are shown in
Fig. 2.18. The positions of the four brushes are also shown in the figure.

The positions of brushes are fixed as follows: for ease of understanding, the
connection diagram of Fig. 2.17 is reproduced in Fig. 2.19. The directions of
current in the coil-sides are also shown by observing the directions from Fig. 2.18.
By carefully examining the directions of current in the coil-sides it is seen that
between points P and Q current gets divided in two parallel paths. From point P
the current fl ows to Q via two paths, viz. through 11-16-21- ... 6-11-18-13-

The point P in Fig. 2.19 is the separating point of the emf in the two sections of
the winding and therefore corresponds to the position of one of the brushes, viz.
the negative brush. For placing of the positive brush, it is seen from Fig. 2.19 that
Page 20
Malla Reddy College of Engineering and Technology
Department of EEE (2022-23)
Electrical Machines -1 (EM-1): Lecture Notes: (Dr.Karimulla P Sk)
Unit 1: D C Generators

at point Q current is coming out from both the coil-sides. Therefore, point Q
corresponds to the position of the positive brush.

Figure: Layout diagram for the wave winding of example 2.2

It may be noted from Fig. 2.18 that coil-sides 6 and 17 lie in the interpolar region.
The direction of current in these coil-sides will depend upon the direction of
current in the other coil side of the respective coils, viz. coils 1-6 and 17-22.
Dummy Coils: As mentioned earlier wave winding is possible with a particular
number of coil-sides. But if standard stampings with a definite number of slots are
to be used, the number of coil-sides needed to be placed in all the slots may be
more than the required number. In such a case, the extra coils are left
unconnected. These coils are called dummy coils. Dummy coils are used so as to
make the armature dynamically balanced. They, otherwise, do not contribute to
the induced emf or developed torque.

Page 21
Malla Reddy College of Engineering and Technology
Department of EEE (2022-23)
Electrical Machines -1 (EM-1): Lecture Notes: (Dr.Karimulla P Sk)
Unit 1: D C Generators

Example-3 : Calculate the winding pitches and draw developed and sequence
diagrams of the winding for a four-pole wave connected armature winding of a dc
generator having seven coils. In the diagram, show the position of poles and the
position and polarity of brushes.
Solution:
Number of coil-sides = 7 × 2 = 14
Ya = (Z ±2)/P = (14 ±2)/4 = 3 or 4
Ya should be an integer, Yb and Yf should be odd numbers.
Therefore, we choose
Ya = Yb = Yf = 3
The sequence and layout diagrams of the winding are shown in Fig. 2.20.

Figure: Layout and sequence diagram for a wave wound DC armature

Page 22
Malla Reddy College of Engineering and Technology
Department of EEE (2022-23)
Electrical Machines -1 (EM-1): Lecture Notes: (Dr.Karimulla P Sk)
Unit 1: D C Generators

Simplex and Multiplex windings:

Rotor (armature) windings are further classified according to the plex of their
windings. A simplex rotor winding is a single, complete, closed winding wound on
a rotor. A duplex rotor winding is a rotor with two complete and independent sets
of rotor windings. If a rotor has a duplex winding, then each of the windings will
be associated with every other commutator segment. One winding will be
connected to segments I, 3, 5, etc., and the other winding will be connected to
segments 2, 4, 6, etc. Similarly, a triplex winding will have three complete and
independent sets of windings, each winding connected to every third commutator
segment on the rotor. Collectively, all armatures with more than one set of
windings are said to have multiplex windings.

Use of laminated core :


Faraday's law is the fundamental property of magnetic fields involved in
transformer operation. The effect of Lenz's law in transformers is to predict the
polarity of the voltages induced in transformer windings.
Faraday's law also explains the eddy current losses mentioned previously. A time-
changing flux induces voltage within a ferromagnetic core in just the same
manner as it would in a wire wrapped around that core. These voltages cause
swirls of current to flow within the core, much like the eddies seen at the edges of
a river. It is the shape of these currents that gives rise to the name eddy currents.
These eddy currents are flowing in a resistive material (the iron of the core), so
energy is dissipated by them. The lost energy goes into heating the iron core. The
amount of energy lost to eddy currents is proportional to the size of the paths
they follow within the core. For this reason, it is customary to break up any
ferromagnetic core that may be subject to alternating fluxes into many small
strips, or laminntions, and to bui ld the core up out of these strips. An insulating
oxide or resin is used between the strips. so that the current paths for eddy
currents are limited to very small areas. Because the insulating layers are
extremely thin, this action reduces eddy current losses with very little effect on
the core's magnetic properties. Actual eddy current losses are proportional to the

Page 23
Malla Reddy College of Engineering and Technology
Department of EEE (2022-23)
Electrical Machines -1 (EM-1): Lecture Notes: (Dr.Karimulla P Sk)
Unit 1: D C Generators

square of the lamination thickness, so there is a strong incentive to make the


laminations as thin as economically possible.

EMF Equation:

This being very important for understanding of a Generator performance we will


derive a detailed expression for the exact induced emf in a generator in terms of
all the following DC Machine parameters.

Ø The flux from a pole (webers)

Z The total number of conductors on the armature

a The number of parallel paths

 In a practical machine all the conductors are not connected in series. They
are divided into groups of parallel conductors and then all the groups are
connected in series to get higher voltage. In each group there are ‘a’
conductors in parallel and hence there are ‘a’ parallel current paths and
each parallel path will have Z/a conductors in series.

N The Speed of rotation (RPM)

ω The speed (Radians/sec)

P The number of poles

Now consider one conductor on the armature. As this conductor makes one
complete revolution it cuts PØ webers of flux.

Since the induced emf in a conductor is its rate of cutting of flux lines ( Rate of
change of Flux linkage ) the emf ‘e’ induced in such a single conductor is equal to

e = PØ/ Time for one revolution in seconds = PØ/(60/N) = NPØ/60 volts

There are Z/a conductors in series in each parallel path.


Page 24
Malla Reddy College of Engineering and Technology
Department of EEE (2022-23)
Electrical Machines -1 (EM-1): Lecture Notes: (Dr.Karimulla P Sk)
Unit 1: D C Generators

∴ the total induced emf ‘E’ = (Z/a) NPØ/60 = ( NPØ Z )/ (a. 60)

EA = (Ø ZN/60).( P/a)

The armature conductors are generally connected in two methods. Viz. Lap
winding and Wave winding.

In Lap wound machines the number of parallel paths ‘a’ = P

∴ ‘E’ = (Ø ZN/60)

In Wave wound machines the number of parallel paths ‘a’ = 2

∴ ‘E’ = (Ø ZN/60).( P/2)

In general the emf induced in a DC machine can be represented as EA = Ka. Ø.N

Where Ka = ZP/60.a

Sometimes it is convenient to express the emf induced in terms of the angular


rotation ω (Rad/sec) and then the expression for emf becomes:

EA = (Ø ZN/60).( P/a) = (ZP/a). Ø. N/60 = (ZP/.a). Ø. (ω/2π) = (ZP/2πa).Ø.ω =


Ka. Ø.ω

(since N/60 RPS = 2π. N/60 Radians /sec = ω Radians /sec and ∴ N/60 = ω/2π)

Where Ka is the generalized constant for the DC machine’s armature and is given
by :

Ka = (ZP/2πa)

Where Ø is the flux/per pole in the machine (Webers), N is the speed of rotation
(RPM) ω is the angular speed (Radians/sec) and Ka is a constant depending on
the machine parameters.

Page 25
Malla Reddy College of Engineering and Technology
Department of EEE (2022-23)
Electrical Machines -1 (EM-1): Lecture Notes: (Dr.Karimulla P Sk)
Unit 1: D C Generators

And thus finally EA = Ka. Ø. ω and we can say in general, the induced voltage in
any DC machine will depend on the following three factors:
1. The flux Ø in the machine
2. The angular speed of rotation ω and
3. A constant representing the construction of the machine. (ZP/2πa)
(i.e. the number of conductors ‘Z’, the number of poles ‘P’ and the number of
parallel paths ‘a’ along with the other constant ‘2π’)

Armature Reaction:

If the magnetic field windings of a DC machine are connected to a power supply


and the rotor of the machine is turned by an external source of mechanical
power,then a voltage will be induced in the conductors of the rotor. This voltage
wilI be rectified into a DC output by the action of the machine's commutator.
Now connect a load to the terminaIs of the machine, and a current will flow in its
armature windings. This current flow will produce a magnetic field of its own,
which will distort the original magnetic field from the machine's poles. This
distortion of the flux in a machine as the load is increased is called armature
reaction.
It causes two serious problems in real DC machines.
The first problem caused by armature reaction is neutral-plane shift. The
magnetic neutral plane is defined as the plane within the machine where the
velocity of the rotor wires is exactly parallel to the magnetic nux lines, so that e ind
in the conductors in the plane is exactly zero.
To understand the problem of neutral-plane shift, examine Figure 8- 23. Figure 8-
23a shows a two-pole dc machine. Notice that the flux is distributed uniformly
under the pole faces. The rotor windings shown have voltages built up out of the
page for wires under the North Pole face and into the page for wires under the
South Pole face. The neutral plane in this machine is exactly vertical.
Now suppose a load is connected to this machine so that it acts as a generator.
Current will flow out of the positive terminal of the generator, so current will be
flowing out of the page for wires under the North Pole face and into the page for
wires under the South Pole face. This current flow produces a magnetic field from
Page 26
Malla Reddy College of Engineering and Technology
Department of EEE (2022-23)
Electrical Machines -1 (EM-1): Lecture Notes: (Dr.Karimulla P Sk)
Unit 1: D C Generators

the rotor windings, as shown in Figure 8- 23c. This rotor magnetic field affects the
original magnetic field from the poles that produced the generator's voltage in
the first place. In some places under the pole surfaces, it subtracts from the pole
flux, and in other places it adds to the pole flux. The overall result is that the
magnetic flux in the air gap of the machine is skewed as shown in Figure 8- 23d
and e. Notice that the place on the rotor where the induced voltage in a
conductor would be zero (the neutral plane) has shifted.
For the generator shown in Figure 8- 23, the magnetic neutral plane shifted in the
direction of rotation. If this machine had been a motor, the current in its rotor
would be reversed and the flux would bunch up in the opposite corners from the
bunches shown in the figure. As a result, the magnetic neutral plane would shift
the other way.

Page 27
Malla Reddy College of Engineering and Technology
Department of EEE (2022-23)
Electrical Machines -1 (EM-1): Lecture Notes: (Dr.Karimulla P Sk)
Unit 1: D C Generators

Figure 8-23: The development of armature reaction in a DC generator. (a)


Initially the pole flux is uniformly distributed, and the magnetic neutral plane is
vertical (b) the effect of the air gap on the pole flux distribution (c) the
armature magnetic field resulting when a load is connected to the machine (d)
both rotor and pole fluxes are shown indicating points where they add and
subtract (e) the resulting flux under the poles. The neutral plane has shifted in
the direction of motion.

Page 28
Malla Reddy College of Engineering and Technology
Department of EEE (2022-23)
Electrical Machines -1 (EM-1): Lecture Notes: (Dr.Karimulla P Sk)
Unit 1: D C Generators

In general, the neutral-plane shifts in the direction of motion for a generator and
opposite to the direction of motion for a motor. Furthermore, the amount of the
shift depends on the amount of rotor current and hence on the load of the
machine.
So what's the problem with neutral-plane shift? It 's just this: The commutator
must short out commutator segments just at the moment when the voltage
across them is equal to zero. If the brushes are set to short out conductors in the
vertical plane, then the voltage between segments is indeed zero until the
machine is loaded. When the machine is loaded, the neutral plane shifts, and the
brushes short out commutator segments with a finite voltage across them. The
result is a current now circulating between the shorted segments and large sparks
at the brushes when the current path is interrupted as the brush leaves a
segment. The end result is arcing and sparking at the brushes. This is a very
serious problem, since it leads to drastically reduced brush Iife, pitting of the
commutator segments, and greatly increased maintenance costs. Notice that this
problem cannot be fixed even by placing the brushes over the full-load neutral
plane, because then they wouId spark at no load.
In extreme cases, the neutral-plane shift can even lead to flashover in the
commutator segments near the brushes. The air near the brushes in a machine is
normally ionized as a result of the sparking on the brushes. Flashover occurs
when the voltage of adjacent commutator segments gets large enough to sustain
an arc in the ionized air above them. If flashover occurs, the resulting arc can even
melt the commutator's surface.
The second major problem caused by armature reaction is called flux weakening.
To understand flux weakening, refer to the magnetization curve shown in Figure
8-24. Most machines operate at flux densities near the saturation point.
Therefore, at locations on the pole surfaces where the rotor magnetomotive force
adds to the pole magnetomotive force, only a small increase in flux occurs. But at
locations on the pole surfaces where the rotor magnetomotive force subtracts
from the pole magnetomotive force, there is a larger decrease in flux. the net
result is that the total average flux under the entire pole face is decreased (see
Figure 8- 25).
Flux weakening causes problems in both generators and motors. In generators,
the effect of flux weakening is simply to reduce the voltage supplied by the
Page 29
Malla Reddy College of Engineering and Technology
Department of EEE (2022-23)
Electrical Machines -1 (EM-1): Lecture Notes: (Dr.Karimulla P Sk)
Unit 1: D C Generators

generator for any given load. In motors, the effect can be more serious. As the
early examples in this chapter showed, when the flux in a motor is decreased, its
speed increases. But increasing the speed of a motor can increase its load,
resulting in more flux weakening. It is possible for some shunt dc motors to reach
a runaway condition as a result of flux weakening, where the speed of the motor
just keeps increasing until the machine is disconnected from the power line or
until it destroys itself.

Figure 8-24: A typical magnetization curve shows the effects of pole saturation
where armature and pole magnetomotive forces add.

Page 30
Malla Reddy College of Engineering and Technology
Department of EEE (2022-23)
Electrical Machines -1 (EM-1): Lecture Notes: (Dr.Karimulla P Sk)
Unit 1: D C Generators

Reactance voltage:

The voltage rise in the short circuited coil due to inductive property of the coil,
which opposes the current reversal in it during the commutation period, is called the
reactance voltage. It is given by :
Reactance voltage = Coefficient of self-inductance(L) x Rate of change of current
(di/dt).
We know that the coil undergoes commutation when the two commutator
segments get short-circuited by the brush. During this period the current say I
changes from +I to -I . That means di= change in current = 2I.
The time taken for this change in current is given by dt = (Wb-Wc)/v where
Wb =Width of the brush (cms)
Wm =Width of the mica insulator between the commutator segments (cms)
V = peripheral (linear) velocity of the commutator (armature) (cm/sec)
Then reactance voltage = L.di/dt = L .2I.v/(Wb-Wc)

This reactance voltage also causes sparking at the brushes resulting in the same
phenomenon as that produced by neutral phase shifting due to armature reaction.

We can produce reversing e.m.f in two ways. By brush shifting. By using inter-poles or
commutating poles.

Methods of improving commutation:

Shifting of Brushes: By shifting the brushes to the new MNA, sparking due to
commutation can be avoided. The brushes are to be shifted by the same angle by
which the MNA has shifted due to loading. They are to be shifted in the forward
direction (in the direction of rotation) in a generator, and backward in a motor.
The disadvantage with this method is that the angle of shift will depend upon the
load on the machine and therefore is practically difficult to shift the brushes
continuously with change in load.

Commutating poles or Interpoles: The basic idea behind this approach is that if
the voltage in the conductors undergoing commutation can be made zero, then
there will be no sparking at the brushes. To accomplish this, small poles, called

Page 31
Malla Reddy College of Engineering and Technology
Department of EEE (2022-23)
Electrical Machines -1 (EM-1): Lecture Notes: (Dr.Karimulla P Sk)
Unit 1: D C Generators

commutating poles or interpoles, are placed midway between the main poles.
These commutating poles are located directly over the conductors being
commutated. By providing a suitable amount of flux with proper polarity from the
commutating poles, the voltage in the coils undergoing commutation can be
exactly canceled. If the cancellation is exact, then there will be no sparking at the
brushes. Exact cancellation of the voltage in the commutator segments is
accomplished for all values of loads by connecting the interpole windings in series
with the windings on the rotor, as shown in the figure below.
The commutating poles do not change the operation of the machine, because
they are so small that they affect only the few conductors about to undergo
commutation. The armature reaction under the main pole faces is unaffected,
since the effects of the commutating poles do not extend that far. This means
that the flux weakening problem in the machine is not solved by the commutating
poles.

Page 32
Malla Reddy College of Engineering and Technology
Department of EEE (2022-23)
Electrical Machines -1 (EM-1): Lecture Notes: (Dr.Karimulla P Sk)
Unit 1: D C Generators

Figure: Connection of commutating poles in a DC Generator

As the load increases and the rotor current increases, the magnitude of the
neutral-plane shift and the size of the L dildt effects increase too. Both these
effects increase the voltage in the conductors undergoing commutation.
However, the interpole flux increases too, producing a larger voltage in the
conductors that opposes the voltage due to the neutral-plane shift. The net result
is that their effects cancel over a broad range of loads. Note that interpoles work
for both motor and generator operation, since when the machine changes from
motor to generator, the current both in its rotor and in its interpoles reverses
direction. Therefore, the voltage effects from them still cancel.

The interpoles must induce a voltage in the conductors undergoing commutation


with such a polarity that is opposite to the voltage caused by neutral-plane shift
and L dildt effects. In the case of a generator, the neutral plane shifts in the
direction of rotation, meaning that the conductors undergoing commutation have
the same polarity of voltage as the pole they just left (see Figure 8- 29).

Figure : Compensating windings used to neutralize the effect of armature


reaction

Page 33
Malla Reddy College of Engineering and Technology
Department of EEE (2022-23)
Electrical Machines -1 (EM-1): Lecture Notes: (Dr.Karimulla P Sk)
Unit 1: D C Generators

To oppose this voltage, the interpoles must have the opposite flux, which is the
flux of the upcoming pole. In a motor, however, the neutral plane shifts opposite
to the direction of rotation, and the conductors undergoing commutation have
the same flux as the pole they are approaching. In order to oppose this voltage,
the interpoles must have the same polarity as the previous main pole. Therefore,
1. The interpoles must be of the same polarity as the next upcoming main pole in
a generator.
2. The interpoles must be of the same polarity as the previous main pole in a
motor.
The use of commutating poles or interpoles is very common, because they correct
the sparking problems of dc machines at a fairly low cost. They are almost always
found in any dc machine of 1 hp or larger. It is important to realize, though, that
they do nothing for the flux distribution under the pole faces, so the flux-
weakening problem is still present. Most medium-size, general-purpose motors
correct for sparking problems with interpoles and just live with the flux
weakening effects.
Compensating windings: For very heavy, severe duty cycle motors, the flux-
weakening problem can be very serious. To completely cancel armature reaction
and thus eliminate both neutral-plane shift and flux weakening, a different
technique was developed. This technique involves placing compensating windings
in slots carved in the faces of the poles parallel to the rotor conductors, to cancel
the distorting effect of armature reaction. These windings are connected in series
with the rotor windings, so that whenever the load changes in the rotor, the
current in the compensating windings also changes. Figure 8- 30 shows the basic

Page 34
Malla Reddy College of Engineering and Technology
Department of EEE (2022-23)
Electrical Machines -1 (EM-1): Lecture Notes: (Dr.Karimulla P Sk)
Unit 1: D C Generators

concept. In Figure 8-30(a), the pole flux is shown by itself. In Figure 8-30(b), the
rotor flux and the compensating winding flux are shown. Figure 8-30(c) represents
the sum of these three fluxes, which is just equal to the original pole flux by itself.
The major disadvantage of compensating windings is that they are expensive,
since they must be machined into the faces of the poles. Any motor that uses
them must also have interpoles, since compensating windings do not cancel
L di /dt effects. The interpoles do not have to be as strong, though, since they are
canceling only L di /dt voltages in the windings, and not the voltages due to
neutral-plane shifting. Because of the expense of having both compensating
windings and interpoles on such a machine, these windings are used only where
the extremely severe nature of a motor's duty demands them.

Important features of DC Generators:

 The terminal characteristic of a DC Machine is a plot of the output


quantities of the Machine against each other. For a DC Generator the
output quantities are the Terminal Voltage and the Line (Load) current.
 The various types of Generators differ in their terminal characteristics
(Voltage-Current) and therefore to the application to which they are suited.
 The DC Generators are compared by their Voltages, Power ratings, their
efficiencies and Voltage regulation. Voltage Regulation (VR) is defined by
the equation: VR = [(Vnl − Vfl) / Vfl ].100 % Where Vnl is the No load
terminal voltage and Vfl is the Full load terminal voltage. It is a rough
measure of the Generator’s Voltage- Current Characteristic. A positive
voltage regulation means a drooping characteristic and a negative
regulation means a rising characteristic.
 Since the speed of the prime movers affects the Generator voltage and
prime movers can have varying speed characteristics, The voltage
regulation and speed characteristics of the Generator are always compared
assuming that the Prime mover’s speed is always constant.

Page 35
Malla Reddy College of Engineering and Technology
Department of EEE (2022-23)
Electrical Machines -1 (EM-1): Lecture Notes: (Dr.Karimulla P Sk)
Unit 1: D C Generators

Methods of excitation: (The method by which the field current is generated)

The performance characteristics of a dc machine are greatly influenced by the


way in which the field winding is excited with direct current. There are two basic
ways of exciting a dc machine.

1. Separate excitation: The field is excited from a separate and independent DC


source as shown in fig(a) below. It is flexible as full and independent control of
both Field and Armature circuits is possible.

2. Self- excitation: The field is excited either from its own armature voltage
(Shunt Excitation: fig-b) or own armature current (Series excitation : fig-c)

The dc machine excitation is also classified in three other ways:

1. Shunt excitation : Here the field winding is excited in parallel with armature
circuit and hence the name shunt excitation. It is provided with a large number
(hundreds or even thousands) of turns of thin wire and therefore, has a high
resistance and carries a small current. Since the armature voltage of a dc machine
remains substantially constant, the shunt field could be regulated by placing an
external series resistance in its circuit.

2. Series excitation : Here the field winding has a few turns of thick wire and is
excited with armature current by placing it in series with armature, and therefore
it is known as series field winding. For a given field current, control of this field is
achieved by means of a diverter, a low resistance connected in parallel to series
winding. A more practical way of a series field control is changing the number of
turns of the winding by suitable tappings which are brought out for control
purpose.

3. Compound Excitation: In compound excitation both shunt and series fields are
excited. If the two fields aid each other such that the resultant air gap flux per
pole is increased (their ampere turns are additive), then the excitation is called

Page 36
Malla Reddy College of Engineering and Technology
Department of EEE (2022-23)
Electrical Machines -1 (EM-1): Lecture Notes: (Dr.Karimulla P Sk)
Unit 1: D C Generators

cumulative compound excitation as shown in Fig. (d). If the series field flux
opposes the shunt field flux such that the resultant air gap flux per pole is
decreased, then the excitation is called differential compound excitation as
shown in Fig. (e). The series field is so designed that the increase or decrease in
flux/pole is to a limited extent.

Further there are two types of compounding connections. Long Shunt and Short
shunt . In long shunt compound of Fig. (f ) the shunt field is connected across the
output terminals. In short shunt compound, the shunt field is connected directly
across the armature as shown in Fig. (g). There is no significant difference in
machine performance for the two types of connections. The choice between
them depends upon mechanical consideration or the reversing switches.

Figure below shows the physical arrangement of shunt and series field windings
on one pole of a machine.

Fig: Arrangement of shunt and series field windings on one pole of a machine.

Excellent and versatile ways of controlling the shunt and series excitations are
now possible by use of solid-state devices and associated control circuitry.

Page 37
Malla Reddy College of Engineering and Technology
Department of EEE (2022-23)
Electrical Machines -1 (EM-1): Lecture Notes: (Dr.Karimulla P Sk)
Unit 1: D C Generators

In showing the excitation diagrams of a dc machine, the field winding is shown to


be at 90° (electrical) with respect to the armature circuit which is the actual
spatial orientation of the magnetic fields produced by the field and armature
circuits in a DC machine.

Page 38
Malla Reddy College of Engineering and Technology
Department of EEE (2022-23)
Electrical Machines -1 (EM-1): Lecture Notes: (Dr.Karimulla P Sk)
Unit 1: D C Generators

Magnetization characteristics of DC Generators:

No load or Open circuit magnetization characteristic of any DC Machine is a plot


of the Field flux versus the magnetizing current. Since measurement of field flux is
difficult we use the relation for the emf induced in a DC machine EA = K. Ø.N from
which we can see that the induced voltage is proportional to the Flux in the
machine when the speed is maintained constant. Hence we conduct a test on the
given DC machine to obtain data on the induced voltage as a function of the field
current.

The diagram of the test setup required to obtain the above data is shown in the
figure below.

Page 39
Malla Reddy College of Engineering and Technology
Department of EEE (2022-23)
Electrical Machines -1 (EM-1): Lecture Notes: (Dr.Karimulla P Sk)
Unit 1: D C Generators

Fig: Test setup with a DC machine to obtain the No load magnetization


Characteristic

The prime mover gives the required mechanical energy to the DC Machine and it
can be a small Diesel engine. The rheostat connected between the DC Input and
the field winding is used to adjust and get the required field current. The field
current is initially set to Zero and the Armature volatage is measured. Then the
field current is gradually increased and the corresponding values of Armature
voltage are measured until the output voltage saturates. Next the field current is
brought back to zero gradually and the corresponding Armature voltages are
measured at a few points. The corresponding data on Armature voltage is plotted
against field current and is shown in the figure below.

IF

Fig: No load magnetization curve (or OCC) of a DC Machine ( Plot of Armature


Voltage Ea Vs.Field current IF)

Page 40
Malla Reddy College of Engineering and Technology
Department of EEE (2022-23)
Electrical Machines -1 (EM-1): Lecture Notes: (Dr.Karimulla P Sk)
Unit 1: D C Generators

Though the field current is zero we get a small value of Armature voltage as seen
at point 1 due to the residual magnetism present in the field coil. Subsequently
armature voltage increases with field current upto some point 3 and then the rate
of rise decreses. Finally at poin 4 field flux gets saturated and hence the emf also
gets saturated. The plot of armature voltage vs.field current is not same during
the field current reduction as that during the field current increase and this is due
to the property of magnetic hysteresis in the Ferro magnetic materials. In the
return path the induced volatage at zero field current is higher than that during
the field current increase. This is due to the combined effect of Hysterisis and the
residual magnetism.

Different Types of DC Generators and their Terminal ( or Load ) Characteristics:

The DC generators are classified according to the manner in which the field flux is
produced. Let us consider the following important types of DC Generators and
their characteristics along with their equivalent circuits.

The following notation is used uniformly in all the following


circuits/characteristics:

 VT = Generator’s Terminal Voltage


 IL = Load or line current
 IA = Armature current
 EA = Armature voltage
 RA = Armature Resistance
 IF = Field current
 VF = Field voltage
 RF = Field Resistance

Separately Excited Generator: In this type the field flux is derived from a separate
power source which is independent of the Generator. The equivalent circuit of
such a machine along with the governing equations is shown in the figure below.
Page 41
Malla Reddy College of Engineering and Technology
Department of EEE (2022-23)
Electrical Machines -1 (EM-1): Lecture Notes: (Dr.Karimulla P Sk)
Unit 1: D C Generators

Fig: Equivalent circuit of a separately excited DC Generator

The terminal characteristic of this type of Generators is a plot of VT vs. IL for a


constant speed ω and the governing equations are :

 The Load or line current IL = The armature current IA


 Generator’s Terminal Voltage = VT = (EA − IARA)
 IF = VF / RF

Since the internally generated voltage is independent of IA ,the terminal


characteristic of a Separately Excited Generator is a straight line as shown in the
figure below.

Page 42
Malla Reddy College of Engineering and Technology
Department of EEE (2022-23)
Electrical Machines -1 (EM-1): Lecture Notes: (Dr.Karimulla P Sk)
Unit 1: D C Generators

Fig: The terminal Characteristics of a Separately Excited DC Generator

When the load supplied by the generator increases, the load current IL increases
and hence the armature current IA also increases. When the armature current
increases, the IARA drop increases, so the terminal voltage of the generator droops
(falls). It is called a drooping characteristic.

Shunt Generator: In this the field flux is derived by connecting the Field directly
across the Armature terminals. The equivalent circuit of such a generator is
shown in the figure below along with the governing equations.

Page 43
Malla Reddy College of Engineering and Technology
Department of EEE (2022-23)
Electrical Machines -1 (EM-1): Lecture Notes: (Dr.Karimulla P Sk)
Unit 1: D C Generators

Fig: The equivalent circuit of a DC Shunt generator along with the relevant
governing equations

As could be seen, in this machine the armature current supplies both the load
current and the field current. Using the Kirchhoff’s voltage law the terminal
voltage is seen to be same as that of a separately excited voltage i.e. VT = (EA −
IARA ). In this the advantage is that no external supply is required for the field
circuit. But this leaves an important question. If the generator supplies its own
field current how does it get the initial field flux that is required to start the
machine and generate voltage when it is first turned on? This is explained below.

Build‐up of E.M.F, Critical Field Resistance and Critical Speed :

Voltage build up in a Shunt Generator :

The voltage build up in a shunt generator depends upon the presence of a


residual flux in the poles of the generator. When a Shunt generator first starts to
turn on an internal voltage is generated which is given by EA = k. Øres. ω. This
voltage( which may be just one or two volts ) appears at the generator terminals.
This causes a current to flow in the generator’s field coil IF = VT / RF. This produces

Page 44
Malla Reddy College of Engineering and Technology
Department of EEE (2022-23)
Electrical Machines -1 (EM-1): Lecture Notes: (Dr.Karimulla P Sk)
Unit 1: D C Generators

a m.m.f. in the poles which in turn increases the flux in them. The increase in the
flux causes an increase in EA = k.Ø↑. ω which in turn increases the terminal
voltage VT . When VT rises, IF increases further, increasing the flux more which
increases EA and so on. This voltage build up phenomenon is shown in the figure
below.

Fig: Voltage build up on starting in a DC Shunt generator

It is to be noted here that it is effect of magnetic saturation in the Pole faces


which eventually limits the build of the terminal voltage.

The voltage build up in the figure above shows up as though it is building up in


discrete steps. It is not so. These steps are shown just to make it clear the
phenomenon of positive feedback between the Generator’s internal voltage and
the field current. In the DC Shunt generator both EA and IF increase simultaneously
until the steady state conditions are reached.

Critical Resistance: For understanding the terms critical Resistance and critical
speed, the open circuit characteristic (OCC) or the magnetization characteristic of
a DC machine is shown again in the figure below along with air gap line and Rf
line. The extension of the liner portion of the magnetization curve, shown dotted

Page 45
Malla Reddy College of Engineering and Technology
Department of EEE (2022-23)
Electrical Machines -1 (EM-1): Lecture Notes: (Dr.Karimulla P Sk)
Unit 1: D C Generators

in the figure below is known as the air-gap line as it represents mainly the
magnetic behavior of the machine’s air-gap.

Fig: Open Circuit Characteristic of DC machine along with Air gap and Rf lines

As already explained in the topic Build up of EMF in a DC shunt generator: At the


instant of switching on the field after the armature has been brought to rated
speed, the armature voltage corresponds to a small residual value which causes a
small field current to flow. If the field is connected such that this current increases
the field mmf and therefore the induced emf, the machine voltage cumulatively
buildups and settles at a final steady value because of the saturation
characteristic of the machine’s magnetic circuit.
Since the generator is assumed to be on no-load during the build-up process, the
following circuit relationships apply with reference to the machines’ equivalent
circuit shown in the figure below.

Page 46
Malla Reddy College of Engineering and Technology
Department of EEE (2022-23)
Electrical Machines -1 (EM-1): Lecture Notes: (Dr.Karimulla P Sk)
Unit 1: D C Generators

Fig: The equivalent circuit of DC shunt generator

Ia = If
V = Ea – IfRa
The field current in a shunt generator being very small, the voltage drop IfRa can
be neglected so that : Vo= Ea(If) (magnetization
characteristic)
And for the field circuit:
Vo = If Rf
which is a straight line relationship, called the Rf -line as shown in the OCC plot
earlier. The no-load terminal voltage is the solution of the above two eqations for
Vo.Thus the intersection point P of the Rf -line with the magnetization
characteristic as shown in the OCC gives the no-load terminal voltage (V0) and the
corresponding field current. Further, it is easy to visualize from this figure that the
no-load voltage can be adjusted to a desired value by changing the field
resistance.
It can be seen in the figure below that as the field resistance is increased the no-
load voltage decreases.

Page 47
Malla Reddy College of Engineering and Technology
Department of EEE (2022-23)
Electrical Machines -1 (EM-1): Lecture Notes: (Dr.Karimulla P Sk)
Unit 1: D C Generators

Fig: Variation of No load voltage with field resistance


The no-load voltage is undefined for a field resistance (Rf3 = Rfc) whose line
coincides with the linear portion of the magnetization curve. With field resistance
even slightly more than this value, the machine does not excite to any appreciable
value and would give no-load voltage close to the residual value. The machine
with this much resistance in the field fails to excite and the corresponding
resistance is known as the critical resistance (Rfc).

Critical speed: Consider now the operation with fixed Rf and variable armature
speed as illustrated in the figure below. It can be observed that as the speed is
reduced, the OCC proportionally slides downwards so that the no-load voltage
reduces. At a particular speed, called the critical speed, the OCC becomes
tangential to the Rf line and as a result the generator would fail to excite.

Page 48
Malla Reddy College of Engineering and Technology
Department of EEE (2022-23)
Electrical Machines -1 (EM-1): Lecture Notes: (Dr.Karimulla P Sk)
Unit 1: D C Generators

Fig: Effect of speed on No load voltage

Causes of failure to excite in a Shunt Generator:

A shunt generator may not get excited in certain conditions. The causes of such
failure to excite, the method of detection and the corresponding remedial
measures are given in the table below.

S.No Cause Method of Remedy


Detection
1 Absence of residual Zero reading on Operate the
magnetism due to ageing Voltmeter after Generator as
rotating the separately excited
machine machine first and
then as separately
excited
2 Wrong field winding Voltmeter reading Interchange the field
connections. Due to this the decreases rather connections
flux gets produced in than increasing as
opposite direction to that of the field current is
the residual flux and they increased
cancel each other.
3 Field resistance is more than Voltmeter shows Field resistance to be
the Critical field resistance. zero reading reduced using
suitable field diverter
4 Generator is driven in This wipes out the Generator to be
opposite direction residual flux and driven in the proper
the machine fails direction
to excite

Terminal characteristics of a shunt generator :

Page 49
Malla Reddy College of Engineering and Technology
Department of EEE (2022-23)
Electrical Machines -1 (EM-1): Lecture Notes: (Dr.Karimulla P Sk)
Unit 1: D C Generators

The terminal characteristics of the shunt generator differ from that of the
separately excited generator because the amount of field current depends on its
terminal voltage. As the generator load is increased, the load current IL increases
and so IA = IF + IL↑ also increases . An increase in IA increases the IARA drop
causing VT = (EA -- IA ↑RA ) to decrease. This is precisely the same behavior we
have seen in the case of separately excited generator. However, in the shunt
generator when VT decreases the field current decreases ,hence the field flux
deceases thus decreasing the generated Voltage EA . Decreasing the EA causes a
further decrease in the terminal voltage VT = (EA↓ --IA RA ). The resulting
characteristic is shown in the figure below.

Fig: Terminal Characteristic of DC Shunt Generator

It can be noticed that the drop with load is steeper than that of a separately
excited motor due to the field weakening affect. This means that the regulation of
a Shunt Generator is worse than that of a Separately Excited Generator.

DC Series Generator: In this the field flux is derived by connecting the Field coil in
series with the Armature of the Generator as shown in the figure below.

Page 50
Malla Reddy College of Engineering and Technology
Department of EEE (2022-23)
Electrical Machines -1 (EM-1): Lecture Notes: (Dr.Karimulla P Sk)
Unit 1: D C Generators

Fig: Equivalent circuit of DC Series Generator along with the governing


equations

As shown, the armature current, load current and field current are same in a DC
series generator. i.e IA = IF = IL . Since the mmf produced by the fields is
given by = NI and the field current is more in the DC series generator , the
fileld winding is wound with lesser number of turns and also with a thicker gauge
so as to offer less field resistance since full load current flows through the field
winding .

The terminal characteristic of a DC Series Generator looks very much like the
magnetization curve of any other type of generator and is shown in the figure
below.

Page 51
Malla Reddy College of Engineering and Technology
Department of EEE (2022-23)
Electrical Machines -1 (EM-1): Lecture Notes: (Dr.Karimulla P Sk)
Unit 1: D C Generators

Fig: Terminal Characteristic of DC Series Generator

At no load however since there is no field current,armature voltage EA and also


the terminal voltage VT are very small ( generated by the small amount of
residual flux.) As the load increases ,field current rises hence EA also increases
rapidly. The IA (RA+RF ) drop also goes up but this rise is less predominant
compared to the rise in EA initaially and hence VT also rises initially. After some
time field flux gets saturated and hence the induced voltage EA will be constant
without any further rise. At this stage the resistive drop predominates and hence
the terminal voltage VT starts drooping.

DC Compound generator:

As we know in DC shunt Generator the terminal Voltage falls and in a DC series


generator the terminal voltage increases on loading. A compound DC Generator is
the one in which there will be both Series and shunt field coils. If they are wound
such that they aid each other then it is called a Cumulative Compound DC
Generator and if they are wound such that the two fields oppose each other, then
it is called a differential Compound DC Generator. The equivalent circuit diagram
of such Cumulative DC Generator along with relevant governing equations is
shown in the figure below.

Page 52
Malla Reddy College of Engineering and Technology
Department of EEE (2022-23)
Electrical Machines -1 (EM-1): Lecture Notes: (Dr.Karimulla P Sk)
Unit 1: D C Generators

Fig: Equivalent circuit of a Cumulative compound DC Generator

The circuit diagram is shown with standard dot convention on the field windings.

i.e. The current flowing into the dot side of the winding produces a positive mmf
.

And as can be seen that both IF in the shunt winding and IA in the series winding
flow into the dot side and hence both produce magnetic fields which are positive
and hence aid each other.

When the two fields are aiding each other we get a characteristic which will have
the combined effect of drooping (due to the shunt coil) and rising (due to the
field coil). Whichever coil current is more its effect will be more predominant. The
terminal characteristics of a cumulative compound DC Generator are shown in
the figure below for all the three cases.

Page 53
Malla Reddy College of Engineering and Technology
Department of EEE (2022-23)
Electrical Machines -1 (EM-1): Lecture Notes: (Dr.Karimulla P Sk)
Unit 1: D C Generators

Fig: Terminal Characteristics of a DC Compound Generator

1. If the Series field effect is more dominating than that of the Shunt field coil
then we get the Over compounded characteristic where the full load
terminal voltage is higher than the no load terminal voltage.
2. If the Series field effect is equal to that of the Shunt field coil then we get
the Flat compounded characteristic where the full load terminal voltage is
equal to the no load terminal voltage.
3. If the Shunt field effect is more dominating than that of the Series field coil
then we get the Under compounded characteristic where the full load
terminal voltage is lower than the no load terminal voltage.
The normal shunt characteristic is also shown in the figure for comparison.

Important concepts and Formulae:

 Voltage generated in a DC machine: EA = (Ø ZN/60). (P/a) and in terms


of angular speed ω :
EA = Ka Ø ω where Ka = ZP/2πa

Page 54
Malla Reddy College of Engineering and Technology
Department of EEE (2022-23)
Electrical Machines -1 (EM-1): Lecture Notes: (Dr.Karimulla P Sk)
Unit 1: D C Generators

Illustrative Examples:
Ex.1: Calculate the e.m.f. generated by a 6 pole DC Generator having 480
conductors and driven at a speed of 1200 RPM. The flux per pole is 0.012 webers.
(a) When the machine is lap wound (b) When the machine is wave wound
Solution: We know that the e.m.f. generated by a DC Generator is given by
EA = (Ø ZN/60)( P/a) where
Ø Flux per pole (webers) = 0.012Wb
Z Total number of conductors on the armature = 480
a The number of parallel paths = No of Poles P ( = 6 ) when Lap wound and
= 2 when
wave wound
N Speed of rotation of the machine (RPM) = 1200 RPM
P The number of poles = 6

(a) For Lap wound machine a = P = 6

Ea = [ (0.012 x 480 x 1200 ) / 60 ] [ 6/6] = 115.2 Volts

(b) For wave wound machine a = 2

Ea = [(0.012 x 480 x 1200 ) / 60 ] [ 6/2] = 345.6 Volts

Ex.2 : A 50 Kw ,250 V shunt generator operates at 1500 RPM .The armature has 6
poles and is lap wound with 200 turns. Find the induced e.m.f and the flux per
pole at full load given that the armature and the field resistances are 0.01 Ω and
125 Ω respectively.

Solution:

Page 55
Malla Reddy College of Engineering and Technology
Department of EEE (2022-23)
Electrical Machines -1 (EM-1): Lecture Notes: (Dr.Karimulla P Sk)
Unit 1: D C Generators

Output line current = Output power / Line voltage = 50 x 1000 / 250 =


200 A

Field current = Line Voltage / Field resistance = 250 / 125 = 2 A

Armature current in a shunt generator: = Il + If = 200 + 2 = 202 A

Induced e.m.f Ea : = Line Voltage + Armature drop (IaRa drop)

= 250 + 202 x 0.01 = 252.02 V

But we know that armature voltage in terms of the basic machine parameters
is also given by

EA = (Ø ZN/60)( P/a) where


Ø c: Flux per pole (webers) = To be determined
Z : Total number of conductors on the armature = Number of turns x 2 (
since each turn has two conductors ) = 200 x 2 = 400
a : The number of parallel paths = No of Poles P ( = 6 ) (since Lap wound )
N : Speed of rotation of the machine (RPM) = 1500 RPM
P : The number of poles = 6

∴ Ø = (EA x 60 x a/ ZNP) = 252.02 x 60 x 6 / 400 x 1500 x 6 = 0.025202 Wb

Ex.3: A shunt generator connected in parallel to supply mains is delivering a


power of 50 Kw at 250 V while running at 750 RPM. Suddenly its prime mover fails
and the machine continues to run as a motor taking the same 50 Kw power from
250 V mains supply. Calculate the speed of the machine when running as a motor
given that Ra = 0.01 Ω, Rf = 100 Ω and brush drop is 1 V per brush.

Solution:

Page 56
Malla Reddy College of Engineering and Technology
Department of EEE (2022-23)
Electrical Machines -1 (EM-1): Lecture Notes: (Dr.Karimulla P Sk)
Unit 1: D C Generators

First let us calculate the Voltage generated by the machine while running as a
generator under the given conditions:

Output line current = Output power / Line voltage = 50 x 1000 / 250 = 200 A

Field current = Line Voltage / Field resistance = 250 / 100 = 2.5 A

Armature current : Il + If = 200 + 2.5 = 202.5 A

Induced e.m.f Ea : = Line Voltage + Armature drop (IaRa drop)+ Brush


drop(two brushes )

= 250 + 202.5 x 0.01 + 2 x1 = 254.025 V

Next let us calculate the Voltage generated by the machine while running as a
motor under the given conditions :

Input line current = Input power / Line voltage = 50 x 1000 / 250 = 200 A

Field current = Line Voltage / Field resistance = 250 / 100 = 2.5 A

Armature current : Il - If = 200 - 2.5 = 197.5 A

Induced e.m.f or back e.m.f Eb : = Line Voltage - Armature drop(IaRa drop)


- Brush drop(two brushes )

= 250 – 197.5 x 0.01 - 2 x1 = 246.025 V

We know that the voltage induced in the machine is proportional to the speed
i. e

Generator armature voltage is proportional to Generator speed : Ea 𝖺 NG


and similarly

Motor back e.m.f is proportional to Motor speed : Eb 𝖺 NM

Page 57
Malla Reddy College of Engineering and Technology
Department of EEE (2022-23)
Electrical Machines -1 (EM-1): Lecture Notes: (Dr.Karimulla P Sk)
Unit 1: D C Generators

Hence Ea / NG = Eb / NM or NM = (Eb / Ea )NG = (246.025 / 254.025) x


750 = 726 RPM

Example 4 : The following figures give the open-circuit characteristics of a dc


shunt generator at 300 rpm:
If (A) 0 0.2 0.3 0.4 0.5 0.6 0.7
Voc (V) 7.5 93 135 165 186 202 215
The field resistance of the machine is adjusted to 354.5 W and the speed is 300
rpm.
(i) Determine graphically the no-load voltage.
(ii) Determine the critical field resistance.
(iii) Determine the critical speed for the given field resistance.
(iv)What additional resistance must be inserted in the field circuit to reduce the
no-load voltage to 175 V.

Solution:

Step-1 : Let us draw the Field resistance line corresponding to 354.5 Ω on the
OCC ( magnetization characteristic). This can be done by identifying a point
corresponding to a Voltage and current below the OCC corresponding to 354.5
Ω and extending the line joining this point with the origin.

1. V No load : This is the voltage corresponding to the point of intersection of the


OCC and the Rf line corresponding to 354.5 Ω and is seen to be = 195 V
2. Rf critical : To obtain this draw a line tangential to the OCC starting from the
Voltage at If = 0 A. Note down the voltage at which the tangential deviates
from the OCC and the corresponding If. Dividing this voltage by the
corresponding If we get the critical resistance viz. 90/0.2 = 450 Ω

Page 58
Malla Reddy College of Engineering and Technology
Department of EEE (2022-23)
Electrical Machines -1 (EM-1): Lecture Notes: (Dr.Karimulla P Sk)
Unit 1: D C Generators

3. Critical Speed: We know that as speed reduces the armature voltage reduces.
i.e. the OCC leans down wards with decrease in speed and becomes
tangential to the existing Rf line itself. So to find out the critical speed we have
to find out the new Ea from the OCC corresponding to the lesser speed which
deviates from the existing Rf line. This is done by dropping a vertical
perpendicular line from the point of deviation of the critical resistance line
from the original OCC and identifying its intercept on the existing Rf line. Then
by drawing a line parallel to the If axis from this point and locating its
intercept with the Voltage axis, the new Ea is found out.
Then Critical speed = Original RPM x new Ea / Original Ea = 300 x71/90 =
236.7 RPM
4. To find out the additional resistance to be introduced into the field to get a
new no load voltage of 175 V first we have to find out the value of If
corresponding to the new no load voltage. This can be directly read from the
OCC and then from these voltage and current values we can directly get the
new value of Rf and thus the additional value of R f to be introduced into the
field circuit.
Thus new Rf = 175/0.44 = 397.7 Ω and
The additional resistance to be introduced into the field = 397.7 – 354.5 =
43.2 Ω

Page 59
Malla Reddy College of Engineering and Technology
Department of EEE (2022-23)
Electrical Machines -1 (EM-1): Lecture Notes: (Dr.Karimulla P Sk)
Unit 2: D C Motors

UNIT – II
D.C. MOTORS

CONTENTS:

 Principle of operation
 Back E.M.F - Torque equation
 Characteristics and application of shunt, series and compound
motors
 Armature reaction and commutation.
 Speed control of D.C. Motors - Armature voltage and field flux
control methods.
 Motor starters (3 point and 4-point starters)
 Important concepts and Formulae
 Illustrative examples

Page 1
Malla Reddy College of Engineering and Technology
Department of EEE (2022-23)
Electrical Machines -1 (EM-1): Lecture Notes: (Dr.Karimulla P Sk)
Unit 2: D C Motors

Principle of operation: DC Motors are DC machines used as motors. A DC Motor


converts the input DC power into output rotational mechanical power from the
following principle. A current carrying conductor placed in a magnetic field
experiences a mechanical force given by F= i (l X B).

When a group of such conductors is placed on a rotor and are connected


properly the force experienced by the all the conductors together gets translated
into a torque on the rotor (armature) and it starts rotating. We will derive an
expression for such a Torque developed by a DC Motor from the first principles
and its equivalent circuit by equating the Electrical power given to the motor
(excluding the losses) to the mechanical power developed by the motor.

Torque developed by a DC Motor:

Consider the equivalent circuit of a DC motor as shown in the figure below.

Fig: Equivalent circuit of a DC motor

In this figure, the armature circuit is represented by an ideal voltage source EA and
the armature resistance RA. The field coils, which produce the magnetic flux in
the motor, are represented by inductor LF and the field resistance RF. The
separate external variable resistor used to control the amount of current in the
field circuit is also combined with the field resistance and is together shown as RF.

Page 2
Malla Reddy College of Engineering and Technology
Department of EEE (2022-23)
Electrical Machines -1 (EM-1): Lecture Notes: (Dr.Karimulla P Sk)
Unit 2: D C Motors

We know from the earlier study of generators that the voltage generated in a DC
Machine when It’s armature is rotating in a magnetic flux of Ø webers/pole is
given by EA = KA. Ø.ω where KA is given by:
KA = (ZP/2πa)

Now in the DC Motor also, when it is rotating, from the same fundamental
principle of Generator a Voltage is generated across the armature and it is now
called back EMF and is normally shown as Eb to distinguish it from the voltage
generated in the armature of a generator which was shown as EA .

The governing equation of the DC Motor armature circuit now becomes:

VT = Eb+ IaRA or Eb= VT - IaRA

(as against VT = EA - IaRA in the case of a generator where IA flows from armature
towards the external terminals i.e external load )

Since now an external voltage VT is applied to the motor terminals , direction of


armature current changes i.e. now it flows from external terminals towards the
armature.

The power delivered to the motor is given by : Pin = VT . Ia . From this, the loss of
power in the armature is equal to I a2RA and hence the net power given to the
motor armature is given by :
Pm = VT . Ia - Ia2RA = Ia (VT - IaRA ) = Ia . Eb
Pm = Ia . Eb
This net electrical power is converted into mechanical power. We know that in
mechanical rotational systems the power is equal to Torque times the speed. In
the SI system of units which is the present Industry standard it is given by :
P mech (watts ) = τ (Nw.mtrs ).ω (Radians/second )
For simplification if we ignore the mechanical losses in the motor,then :
Pm = Ia . Eb = P mech = τ .ω
i.e. τ .ω = Ia . Eb = Eb . Ia

Page 3
Malla Reddy College of Engineering and Technology
Department of EEE (2022-23)
Electrical Machines -1 (EM-1): Lecture Notes: (Dr.Karimulla P Sk)
Unit 2: D C Motors

Substituting the value of EA = KA . Ø . ω we got in generators here for Eb since


they are the same induced emfs we get τ .ω = Ia . KA. Ø.ω or
τ = KA. Ø . Ia
It is to be noted that this expression for the torque induced in a motor is similar to
the voltage induced in a DC Generator except that the speed ω in the DC
Generator is replaced by the Armature current Ia . The constant KA is same and is
given by KA = (ZP/2πa)

In general, the torque τ in the DC motor will depend on the following 3 factors:
1. The flux Ø in the machine
2. The armature current Ia in the machine
3. The same constant KA representing the construction of the machine

Types of DC Motors and their output (or terminal) Characteristics:

There are three important types DC Motors: DC separately excited, Shunt and
Series motors. We will explain their important features and characteristics briefly.

The terminal characteristic of a machine is a plot of the machine’s output


quantities versus each other. For a motor, the output quantities are shaft torque
and speed, so the terminal characteristic of a motor is a plot of its output torque
versus speed. (Torque/Speed characteristics)

They can be obtained from the Motor’s Induced voltage and torque equations we
have derived earlier plus the Kirchhoff’s voltage law around the armature circuit
and are again given below for quick reference.

 The internal voltage generated in a DC motor is given by: Eb = Ka. Φ.ω


 The internal Torque generated in a DC motor is given by: τ = Ka. Φ.Ia
 KVL around the armature circuit is given by : VT = Eb+ Ia.Ra

Where Φ = Flux per pole .... Webers


Ia = Armature current .... Amperes

Page 4
Malla Reddy College of Engineering and Technology
Department of EEE (2022-23)
Electrical Machines -1 (EM-1): Lecture Notes: (Dr.Karimulla P Sk)
Unit 2: D C Motors

VT = Applied terminal Voltage .... Volts


Ra = Armature resistance .... Ohms
ω = Motor speed .... Radians/sec
Eb = Armature Back EMF .... Volts
Ka = (ZP/2πa) : Motor Back EMF/Torque constant
From the above three equations we get the relation between Torque and speed
as:
ω = (VT / Ka. Φ ) -- (Ra/ Ka. Φ). Ia

ω = (VT / Ka. Φ ) -- [Ra/ (Ka. Φ)2+.τ


We will use this generalized equation in different types of motors and obtain their
Torque vs. Speed characteristics.

DC separately excited and Shunt Motors:

The Equivalent circuits of DC separately excited and Shunt Motors along with
their governing equations are shown in the figure below.

(a) Separately Excited (b) Shunt

Fig: Equivalent circuit of DC separately excited and Shunt Motors

In a separately excited DC motor the field and armature are connected to


separate voltage sources and can be controlled independently. In a shunt motor

Page 5
Malla Reddy College of Engineering and Technology
Department of EEE (2022-23)
Electrical Machines -1 (EM-1): Lecture Notes: (Dr.Karimulla P Sk)
Unit 2: D C Motors

the field and the armature are connected to the same source and cannot be
controlled independently. When the supply voltage to a motor is assumed
constant and is same to the field and armature circuits, there is no practical
difference in behavior between these two machines. Unless otherwise specified,
whenever the behavior of a shunt motor is described, it would be same as that of
a separately excited motor.
In both their cases, with a constant field current the field flux can be assumed to
be constant and then (Ka.Φ) Would be another constant K. Then the above
Generalized Torque speed relations would become:

ω = VT / K -- (Ra/ K). Ia

Substituting the value of Ia in terms of τ (Ia = τ / Ka.Φ = τ / K) we get

ω = VT / K -- [Ra/ (K)2].τ

This equation is just a straight line with negative slope. The resulting Speed/
Torque Characteristics of a DC Separately Excited /Shunt Motor for a rated
terminal voltage and full field current are shown in the figure below. It is a
drooping straight line.

Speed (ω )

Torque (T)

Page 6
Malla Reddy College of Engineering and Technology
Department of EEE (2022-23)
Electrical Machines -1 (EM-1): Lecture Notes: (Dr.Karimulla P Sk)
Unit 2: D C Motors

Fig: Speed/ Torque Characteristics of a DC Separately Excited/Shunt Motor

The no load speed is given by the Applied armature terminal voltage and the field
current. Speed falls with increasing load torque. The speed regulation depends on
the Armature circuit resistance. The usual drop from no load to full load in the
case of a medium sized motor will be around 5%. Separately excited motors are
mostly used in applications where good speed regulation and adjustable speed
are required.

If motor armature reaction is taken into account, then as its load increases, the
flux-weakening effects reduce its flux. From the motor speed equation above, the
effect of reduction in flux is to increase the motor’s speed at any given load over
the speed it would run at without armature reaction. Though at a first glance of
the Speed torque equation it may appear that the effect of reduction in flux is to
decrease the motor’s speed at any given load (since Φ2 is in the denominator)
actually since the first positive term contains V T which is much larger quantity
compared to the second negative term, IaRa drop the net effect would only be to
increase the motor’s speed at any given load. The torque-speed characteristic of a
shunt motor with armature reaction is shown below:

Fig: Torque-speed characteristic of the motor with armature reaction


considered

Motor’s Other Characteristics: Though the terminal characteristics (Speed vs.


Torque) are only important for analysis of a DC motor performance, study and

Page 7
Malla Reddy College of Engineering and Technology
Department of EEE (2022-23)
Electrical Machines -1 (EM-1): Lecture Notes: (Dr.Karimulla P Sk)
Unit 2: D C Motors

understanding of the other characteristics like speed vs. I a and Torque vs. Ia would
also give additional insight into the performance of the motor and hence they
are obtained from the basic equations and presented below:

 Speed vs. Ia : Eb = Ka. Φ.ω = VT - Ia.Ra


ω = (VT - Ia.Ra)/ Ka. Φ

 Torque vs. Ia: τ = Ka. Φ.Ia

Fig: Speed and torque vs. Armature current for a DC shunt motor

DC Series Motor:

The equivalent circuit of a DC Series motor is shown in the figure below.

Page 8
Malla Reddy College of Engineering and Technology
Department of EEE (2022-23)
Electrical Machines -1 (EM-1): Lecture Notes: (Dr.Karimulla P Sk)
Unit 2: D C Motors

Fig: Equivalent Circuit of a DC Series Motor

In a series motor the field current and armature current are same and hence the
field flux is directly dependent on the armature current. Hence during the initial
i.e unsaturated region of the magnetization characteristic the flux Φ can be
assumed to be proportional to the armature current.

Then Φ = Kf.Ia

And using this value in the first basic motor relation given earlier we get:

τ = Ka. Φ.Ia = Ka. K f.I a2

τ = Kaf.Ia2 ( where K af = Ka.K f )

Substituting the above two values of Φ and τ in the second basic motor equation

ω = (VT / Ka. Φ ) -- [Ra/ (Ka. Φ)2].τ

We get ω = VT / Ka. Kf.Ia -- [Ra/ (Ka. Kf .Ia)2].Kaf.Ia2

ω = VT / Kaf.Ia -- [Ra/ (Kaf .Ia)2].Kaf.Ia2

ω = VT / Kaf.Ia -- [Ra/ (Kaf)]

Page 9
Malla Reddy College of Engineering and Technology
Department of EEE (2022-23)
Electrical Machines -1 (EM-1): Lecture Notes: (Dr.Karimulla P Sk)
Unit 2: D C Motors

From the relation τ = Ka. Φ.Ia = Ka. Kf.Ia2 we get Ia = √τ/Kaf and substituting
this in the above equation ω = VT / Kaf.Ia -- [Ra/ (Kaf)]

We get ω = *VT /√( Kaf.τ)] -- [Ra/(Kaf)]

Where Ra is now the sum of armature and field winding resistances and Kaf = Ka.Kf
is the total motor constant. The Speed-Torque characteristics of a DC series motor
as obtained from the above relation are shown in the figure below.

Speed (ω )

Rated Speed Torque (τ )

Rated Torque

Fig: Speed-Torque characteristics of a DC series motor

Motor’s Other Characteristics: Though the terminal characteristics (Speed vs.


Torque) are only important for analysis of a DC motor performance, study and
understanding of the other characteristics like speed vs. I a and Torque vs. Ia would
also give additional insight into the performance of the motor and hence they are
obtained from the basic equations and presented below:

 Speed vs. Ia : Eb = Ka. Kf. Ia ω = VT - Ia.Ra


i.e. = Kaf. Ia .ω = VT - Ia.Ra and
ω = (VT - Ia.Ra) / Kaf. Ia = (VT / Kaf. Ia ) - (Ra
/ Kaf)
Page 10
Malla Reddy College of Engineering and Technology
Department of EEE (2022-23)
Electrical Machines -1 (EM-1): Lecture Notes: (Dr.Karimulla P Sk)
Unit 2: D C Motors

This is an inverse relationship and is shown plotted in the figure below.

 Torque vs. Ia: τ = Ka. Φ.Ia = K af.I a2

This is a direct relationship and is shown plotted in the figure below.

Saturation and armature reaction demagnetization both cause the flux per pole to
increase (with respect to Ia ) at a rate slower than the assumed linear relationship.
Actual characteristics are shown in dotted lines.

Fig: Speed Vs. Armature current in a Series Motor

Page 11
Malla Reddy College of Engineering and Technology
Department of EEE (2022-23)
Electrical Machines -1 (EM-1): Lecture Notes: (Dr.Karimulla P Sk)
Unit 2: D C Motors

Fig: Torque vs. Armature current in a DC Series Motor

Series motors are suitable for applications requiring high starting torque and
heavy overloads. Since Torque is proportional to square of the armature current,
for a given increase in load torque the increase in armature current is less in case
of series motor as compared to a separately excited motor where torque is
proportional to only armature current. Thus during heavy overloads power
overload on the source power and thermal overload on the motor are kept
limited to reasonable small values. According to the above Speed torque
equation, as speed varies inversely to the square root of the Load torque, the
motor runs at a large speed at light load. Generally the electrical machine’s
mechanical strength permits their operation up to about twice their rated speed.
Hence the series motors should not be used in such drives where there is a
possibility for the torque to drop down to such an extent that the speed exceeds
twice the rated speed.

DC Compound Motor:

Page 12
Malla Reddy College of Engineering and Technology
Department of EEE (2022-23)
Electrical Machines -1 (EM-1): Lecture Notes: (Dr.Karimulla P Sk)
Unit 2: D C Motors

A compound motor is a motor with both a shunt and a series field. Such a motor is
shown in the Figure below. The dots that appear on the two field coils have the
same meaning as the dots on a transformer: Current flowing into a dot produces a
positive magneto motive force. If current flows into the dots on both field coils,
the resulting magneto motive forces add to produce a larger total magneto
motive force.
This situation is known as cumulative compounding. If current flows into the dot
on one field coil and out of the dot on the other field coil , the resulting magneto
motive forces subtract. In the Figure below the round dots correspond to
cumulative compounding of the motor, and the squares correspond to differential
compounding.

Page 13
Malla Reddy College of Engineering and Technology
Department of EEE (2022-23)
Electrical Machines -1 (EM-1): Lecture Notes: (Dr.Karimulla P Sk)
Unit 2: D C Motors

Fig: Speed vs. Armature current in a DC Compound Motor Compared with other
Motors

Fig: Torque vs. Armature current in a DC Compound Motor Compared with


other Motors

Page 14
Malla Reddy College of Engineering and Technology
Department of EEE (2022-23)
Electrical Machines -1 (EM-1): Lecture Notes: (Dr.Karimulla P Sk)
Unit 2: D C Motors

Fig: Speed vs. Torque in a DC Compound Motor Compared with other Motors

The Torque-Speed Characteristic of a Cumulatively Compounded DC Motor :

 In the cumulatively compounded DC motor, there is a component of flux


which is constant and another component which is proportional to its
armature current (and thus to its load) which aid each other. Hence the flux
per pole increases with armature current and as consequence (n-Ia) curve
lies between that of a shunt motor( Ø – constant ) and series motor (Ø
𝖺 𝐼𝑎 ).
 Therefore, the cumulatively compounded motor has a higher starting
torque than a shunt motor (whose flux is constant) but a lower starting
torque than a series motor (whose entire flux is proportional to armature
current).
 In a sense, the cumulatively compounded DC motor combines the best
features of both the shunt and the series motors. Like a series motor, it has
extra torque for starting; like a shunt motor, it does not over speed at no
load.
 At light loads, the series field has a very small effect, so the motor behaves
approximately as a shunt DC motor. As the load gets very large, the series
flux becomes quite important and the torque-speed curve begins to look
like a series motor’s characteristic. A comparison of the torque-speed
characteristics of each of these types of machines is shown in Figures.

Page 15
Malla Reddy College of Engineering and Technology
Department of EEE (2022-23)
Electrical Machines -1 (EM-1): Lecture Notes: (Dr.Karimulla P Sk)
Unit 2: D C Motors

The Torque-Speed Characteristic of a Differentially Compounded DC Motor:

 In a differentially compounded dc motor, the shunt magneto motive force


and series magneto motive force subtract from each other. This means that
as the load on the motor increases, Ia increases and the flux in the motor
decreases. But as the flux decreases, the speed of the motor increases. This
speed increase causes another increase in load, which further increases Ia
further decreasing the flux, and Increasing the speed again. The result is
that a differentially compounded motor is unstable and tends to run away.
This instability is much worse than that of a shunt motor with armature
reaction. It is so bad that a differentially compounded motor is unsuitable
for any application.
 Because of the stability problems of the differentially compounded
DCmotor, it is almost never intentionally used.
 However, a differentially compounded motor can result if the direction of
power flow reverses in a cumulatively compounded generator. For that
reason, if cumulatively compounded DC generators are used to supply
power to a system, they will have a reverse-power trip circuit to disconnect
them from the line if the power flow reverses. No motor- generator set in
which power is expected to flow in both directions can use a differentially
compounded motor, and therefore it cannot use a cumulatively
compounded generator.

Typical terminal characteristics of differentially compounded dc motor are also


included in the Figures.

Speed control of DC Motors:

Speed control of DC Motors is easier as compared to the speed control of AC


motors and much wider range of speeds is possible. That is one reason why even
today they are preferred in modern industrial drives. From the two basic
equations of DC machines
Page 16
Malla Reddy College of Engineering and Technology
Department of EEE (2022-23)
Electrical Machines -1 (EM-1): Lecture Notes: (Dr.Karimulla P Sk)
Unit 2: D C Motors

 Eb = Ka. Φ.ω
 VT = Eb+ Ia.Ra

We have the expression for the speed ω = ( VT ‒ Ia.Ra )/ Ka. Φ. From this equation
we can (Since Ka is a constant and Ia is load dependent) easily see that the speed
can be controlled by two methods:

1. By varying the terminal voltage known as : Armature Voltage Control (


AVC ) and
2. By varying the field current and thus the flux per pole Φ known as : Flux
control

Let us study them one by one for all the three types of Motors

Speed control of DC Shunt Motor:

Armature Voltage Control (AVC):

This method involves changing the voltage applied to the armature of the motor
without changing the Voltage applied to the field. This is possible with a
separately excited DC Motor only and not with DC Shunt Motor. So first we shall
explain for a DC separately excited motor and extend the same logic to a shunt
Motor. If the armature terminal Voltage VT is increased, then the IA will rise since
[ IA = (VT ↑ -Eb)/RA]. As IA increases, the induced torque τ = Ka. Φ.Ia↑ increases,
making τind > τload , and the speed of the motor increases.
But, as the speed ω increases, Eb = Ka. Φ.ω↑ increases, causing the armature
current IA to decrease since [ IA = (VT ‒ Eb↑)/RA]. This decrease in IA decreases
the induced torque, causing τind to become equal to τload at a final higher steady
state rotational speed ω. Thus we can see that an increase in Armature voltage
results in a higher speed and the resulting Speed Torque characteristics with AVC
is shown in the figure below.

Page 17
Malla Reddy College of Engineering and Technology
Department of EEE (2022-23)
Electrical Machines -1 (EM-1): Lecture Notes: (Dr.Karimulla P Sk)
Unit 2: D C Motors

Fig: The effect of armature voltage speed control

Notice that the no-load speed of the motor is shifted by this method of speed
control , but the slope of the curve remains constant
The cause-and-effect behavior in this method of speed Control can be
summarized as below:

1. An increase in VT increases [ IA = (VT ↑ ‒ Eb)/RA


2. Increasing IA increases τind = Ka. Φ.Ia↑
3. Increasing τind makes τind > τload increasing ω.
4. Increasing ω increases Eb = Ka. Φ.ω↑
5. Increasing Eb decreases IA = (VT ‒ Eb↑)/RA
6. Decreasing IA decreases τind until τind = τload corresponding to a higher ω.

In the case of a DC Shunt motor since changing the voltage applied to the
armature of the motor without changing the Voltage applied to the field is not
possible, a Variable resistance is introduced in series with the Armature which
results in a reduction in the Armature current I A. Effectively reduction of Armature
current is equivalent to reduction in Armature voltage as seen in the above logic.
Hence we get the same type of Speed control as shown in the figure above except
that the characteristic with VA2 represents the nominal rated speed and that with
VA1 represents with additional resistance introduced in series with the Armature.
With this method, speed control is possible but speed can only be reduced from
the rated or nominal speed. Even for a separately excited DC Motor it can provide
Page 18
Malla Reddy College of Engineering and Technology
Department of EEE (2022-23)
Electrical Machines -1 (EM-1): Lecture Notes: (Dr.Karimulla P Sk)
Unit 2: D C Motors

speed control below Base speed only because armature voltage cannot exceed the
rated value.

Inserting a resistor in series with the armature circuit : If a resistor is inserted in


series with the armature circuit, the effect is to drastically increase the slope of
the motor's torque-speed characteristic, making it operate more slowly if loaded
as shown in the figure below. This fact can easily be seen from the basic Equation:
ω = (VT / Ka. Φ ) -- [Ra/ (Ka. Φ)2+.τ

The insertion of a resistor is a very wasteful method of speed control, since the
losses in the inserted resistor are very large. For this reason, it is rarely used.

Figure: Effect of Armature Resistance on the Speed Torque characteristic of a DC


shunt motor

Flux control:

Page 19
Malla Reddy College of Engineering and Technology
Department of EEE (2022-23)
Electrical Machines -1 (EM-1): Lecture Notes: (Dr.Karimulla P Sk)
Unit 2: D C Motors

Another method of Shunt motor speed control is to change the flux in the field. In
a shunt motor Field current and hence field flux cannot be changed without
changing the armature voltage. Hence flux control in Shunt motor is achieved by
changing the Field resistance. Field coil resistance being fixed we cannot reduce it
but increase the field circuit resistance by adding a variable resistance in series
with the field coil as shown in the figure below.

Accordingly, when the resistance increases, the field current decreases (IF↓ =
VT/RF↑), and as the field current decreases, the flux decreases. A decrease in flux
causes an instantaneous decrease in the back emf ( Eb↓ = Ka. Φ↓.ω ) which
causes an increase in the machine’s armature current since,
IA ↑ = (VT -- EB↓)/RA
The induced torque in a motor is given by τind = Ka. Φ↓.Ia↑ .

Here since the flux in this machine decreases while the current IA increases, which
way does the induced torque change?

Page 20
Malla Reddy College of Engineering and Technology
Department of EEE (2022-23)
Electrical Machines -1 (EM-1): Lecture Notes: (Dr.Karimulla P Sk)
Unit 2: D C Motors

From practical data it is seen that for an increase in field resistance the decrease
in flux is much lesser than the increase in armature current i.e. the increase in
current predominates over the decrease in flux.

Hence, τind increases i.e. τind > τload , and the motor speeds up.
However, as the motor speeds up, Eb rises, causing IA to fall. Thus, induced torque
τind too drops, and
Finally τind equals τload at a higher steady-state speed than the original speed .

The cause-and-effect behavior involved in this method of speed control is


summarized below :
1. Increasing RF causes IF↓ = VT/RF↑ to decrease
2. Decreasing IF↓ decreases Φ↓
3. Decreasing Φ↓ lowers Eb↓ = Ka. Φ↓.ω
4. Decreasing Eb↓ increases IA since IA ↑ = (VT -- EB↓)/RA
5. Increasing IA increases τind = Ka. Φ↓.Ia↑, with the change in IA being
dominant over the change in flux ).
6. Increasing τind makes τind > τload, and the speed ω increases.
7. Increasing ω increases Eb↑= Ka. Φ.ω↑ again.
8. Increasing Eb decreases Ia
9. Decreasing Ia decreases " τind until " τind = " τload at a higher speed ω.

Fig: Shunt Motor Speed control with Flux control (Change in field resistance)(
over the normal operating Range )
Page 21
Malla Reddy College of Engineering and Technology
Department of EEE (2022-23)
Electrical Machines -1 (EM-1): Lecture Notes: (Dr.Karimulla P Sk)
Unit 2: D C Motors

The Speed Torque characteristics with change in Field Resistance are shown in the
figure below. Notice that with flux control i.e. with insertion of additional
resistance in the field circuit, the flux in the machine decreases and hence:
 The no- load speed of the motor increases, while the slope of the torque-
speed curve becomes steeper and also
 Speeds above base speed only can be achieved. (as against with Armature
resistance insertion control , speeds below base speed only can be
achieved) since to achieve speed below base speeds field current has to be
increased beyond its rated value which is not permitted. In a normally
designed motor the maximum speed can be twice the rated speed and in
specially designed motors it can be up to six times the rated speed.

Other important Limitation of field resistance speed control:

The effect of increasing the field resistance on the output characteristic of a DC


shunt motor as seen and explained above is a consequence of the Equation

ω = (VT / Ka. Φ ) -- [Ra/ (Ka. Φ)2+.τ

which describes the technical characteristic of the motor. In this equation, the no-
load speed is proportional to the reciprocal of the flux in the motor, while the
slope of the curve is proportional to the reciprocal of the flux squared. Therefore,
a decrease in flux causes the slope of the torque- speed curve to become steeper.
The earlier figure shows the technical characteristic of the motor over the range
from no-load to full-load conditions. Over this range, an increase in field
resistance increases the motor's speed, as described above. Hence for motors
operating between no- load and full-load conditions, an increase in RF may
reliably be expected to increase the operating speed.
Now let us examine the figure shown below. This figure shows the technical
characteristic of the motor over the full range i.e. from no- load to stall
conditions. It is apparent from the figure that at very slow speeds an increase in
field resistance will actually decrease the speed of the motor. This effect occurs
because , at very low speeds, the increase in armature current caused by the
Page 22
Malla Reddy College of Engineering and Technology
Department of EEE (2022-23)
Electrical Machines -1 (EM-1): Lecture Notes: (Dr.Karimulla P Sk)
Unit 2: D C Motors

decrease in Eb. is no longer large enough to compensate for the decrease in flux in
the induced torque equation. With the flux decrease being actually larger than
the armature current increase, the induced torque decreases, and the motor
slows down.

Fig: Shunt Motor Speed control with Flux control (Change in field resistance)(
over the complete operating Range i.e. from no load to stall condition)

Some small DC motors used for control purposes actually operate at speeds close
to stall conditions. For these motors, an increase in field resistance might have no
effect, or it might even decrease the speed of the motor. Since the results are not
predictable, field resistance speed control should not be used in these types of dc
motors. Instead, the armature voltage method of speed control should be
employed.

Speed Control of Series DC Motors:

Unlike with the shunt dc motor, there is only one efficient way to change the
speed of a series dc motor. That method is to change the terminal voltage of the
motor. If the terminal voltage is increased, the first term in Equation

ω = *VT /√( Kaf.τ)] -- [Ra/(Kaf)]

is increased, resulting in a higher speed for any given torque.

Page 23
Malla Reddy College of Engineering and Technology
Department of EEE (2022-23)
Electrical Machines -1 (EM-1): Lecture Notes: (Dr.Karimulla P Sk)
Unit 2: D C Motors

The speed of DC series motors can also be controlled by the insertion of a series
or parallel (Diverter) resistor into the motor circuit as shown in the figures below
along with the resulting effect on Speed torque characteristics. But in this
technique large amount of power is dissipated as heat and thus wasted. Hence
this method is used only for intermittent periods during the start-up of some
motors.

Until the last 40 years or so, there was no convenient way to change VT, so the
only method of speed control available was the wasteful series resistance
method. That has all changed today with the introduction of solid-state control
circuits. We will study the techniques of obtaining variable terminal voltages
subsequently in another subject ‘Power Electronics ‘

Speed Control of Cumulatively Compounded DC Motor:

The techniques available for the control of speed in a cumulatively compounded


DC motor are the same as those available for a shunt motor:

1. Change the field resistance RF


2. Change the armature voltage VT
3. Change the armature resistance RA.

The analysis describing the methods and effects of changing RF or VT or RA are


similar to the analysis given earlier for the shunt motor.

Page 24
Malla Reddy College of Engineering and Technology
Department of EEE (2022-23)
Electrical Machines -1 (EM-1): Lecture Notes: (Dr.Karimulla P Sk)
Unit 2: D C Motors

The ward Leonard system:

The speed of a separately excited, shunt, or compounded dc motor can be varied


in one of three ways: by changing the field resistance, changing the armature
voltage, or changing the armature resistance. Of these methods, perhaps the
most useful is armature voltage control, since it permits wide speed variations
without affecting the motor's maximum torque.
A number of motor-control systems have been developed over the years to take
advantage of the high torques and variable speeds available from the armature
voltage control of DC motors. In the days before solid-state electronic
components became available, it was difficult to produce a varying DC volt age. In
fact, the normal way to vary the armature voltage of a dc motor was to provide it
with its own separate dc generator.

An armature voltage control system of this type known as ward Leonard Speed
control system is shown in the Figure below .

Figure: Ward Leonard DC Motor Speed control system

Page 25
Malla Reddy College of Engineering and Technology
Department of EEE (2022-23)
Electrical Machines -1 (EM-1): Lecture Notes: (Dr.Karimulla P Sk)
Unit 2: D C Motors

Figure: The operating range of a Ward-Leonard motor-control system. The


motor can operate as a motor in either the forward (quadrant - 1) or reverse
(quadrant -3) direction and it can also regenerate in quadrants 2 and 4.

In this an AC motor is serving as a prime mover for a DC generator, which allows


the motor's speed to be smoothly varied between a very small value and the base
speed. The speed of the motor can be adjusted above the base speed by reducing
the motor’s field current. With such a flexible arrangement, total motor speed
control is possible.

Furthermore, if the field current of the generator is reversed, then the polarity of
the generator's armature voltage will be reversed, too. This will reverse the
motor's direction of rotation. Therefore, it is possible to get a very wide range of
speed variations in either direction of rotation using a Ward-Leonard DC motor
control system.
Another advantage of the Ward-Leonard system is that it can "regenerate," or
return the machine’s energy of motion to the supply lines. If a heavy load is first
raised and then lowered by the DC motor of a Ward-Leonard system, when the
load is being lowered, the DC motor acts as a generator and supplying power back

Page 26
Malla Reddy College of Engineering and Technology
Department of EEE (2022-23)
Electrical Machines -1 (EM-1): Lecture Notes: (Dr.Karimulla P Sk)
Unit 2: D C Motors

to the power system. In this fashion, much of the energy required to lift the load
in the
first place can be recovered, reducing the machine's overall operating costs.
The possible modes of operation of the DC machine are shown in the torque-
speed diagram shown in the above Figure. When this motor is rotating in its
normal direction and supplying a torque in the direction of rotation, it is operating
in the first quadrant of this figure. If the generator's field current is reversed, that
will reverse the terminal voltage of the generator, in turn reversing the motor's
armature voltage. When the armature voltage reverses with the motor field
current remaining unchanged, both the torque and the speed of the motor are
reversed, and the machine is operating as a motor in the third quadrant of the
diagram. If the torque or the speed alone of the motor reverses while the other
quantity does not, then the machine serves as a generator, returning power to
the dc power system. Because a Ward-Leonard system permits rotation and
regeneration in either direction, it is called a four-quadrant control system.
The disadvantages of a Ward-Leonard system should be obvious. One is that the
user is forced to buy three full machines of essentially equal ratings, which is quite
expensive. Another is that three machines will be much less efficient than one.
Because of its expense and relatively low efficiency, the Ward-Leonard system has
been replaced in new applications by SCR-based controller circuits.

Principle of 3 point and 4 point starters:

Before studying the principle of operation of these starters let us understand the
basic principles underlying the starters.

 DC motors are by themselves self starting type. Once the appropriate field
and armature supply are given the motors start automatically. They do not
need any additional device for the purpose of starting.
 But DC motor starters are required for safe starting of the motors. Initially
just at the starting of the motor, the speed is zero and hence the back emf
Eb is also zero. In this condition if the Rated terminal voltage Vt is applied to
the motor we can see from the basic governing equation
Page 27
Malla Reddy College of Engineering and Technology
Department of EEE (2022-23)
Electrical Machines -1 (EM-1): Lecture Notes: (Dr.Karimulla P Sk)
Unit 2: D C Motors

Ia = ( Vt ‒ Eb )/Ra
That the motor draws excessive current which would be easily 10 to 15
times that of the nominal rated current of the motor. This excessive
current would flow till the motor develops the rated speed.
 During this transient period when the excessive current flows the torque
developed also would be excessive. With the result the motor would get
damaged both electrically and mechanically.
 To protect the motor from such damage, a resistance is introduced in
series with the motor as shown in the figure below which would be
withdrawn gradually in steps as the motor picks up speed.
 This is a basic arrangement of a DC motor starter and its operation is totally
manual. But practical starters have been developed with additional
protective and automatic starting features. They are called 3 point starters
and 4 point starters, the subject of our study.

3 Point starter:

The circuit diagram and the arrangement of a three point starter are shown in the
figure below.

Page 28
Malla Reddy College of Engineering and Technology
Department of EEE (2022-23)
Electrical Machines -1 (EM-1): Lecture Notes: (Dr.Karimulla P Sk)
Unit 2: D C Motors

Figure: 3 Point starter

Basic features and working principles:

 The basic component viz starter resistance comes in steps with contact
points brought out as studs 1,2,3.. Run.
 The three points are:
 L – The line terminal to be connected to the DC positive terminal
through a two pole switch
 A – The terminal to be connected to the terminal A1 of the armature.
 F – The terminal to be connected to the terminal F 1 of the Field
winding
 The other ends A2 of the armature and F2 of the field are connected to the
other contact of the two pole switch which gets connected to the negative
terminal of the DC power supply when switched on.
 Point L in turn is connected to the pivot point of the handle through a
protective device called OLR ( Over Load Relay )

Page 29
Malla Reddy College of Engineering and Technology
Department of EEE (2022-23)
Electrical Machines -1 (EM-1): Lecture Notes: (Dr.Karimulla P Sk)
Unit 2: D C Motors

 The handle which is spring loaded comes back to the OFF position under its
own force until locked in the RUN position due to the electromagnetic pull
of the other protective device known as NVC ( No Volt Coil)
 The field terminal F is connected to starting point 1 of the resistance in a
parallel path through the NVC.

Operation of the starter: The starter is gradually moved from the initial position
to the final RUN position manually against the spring force. When the handle
comes in contact with stud -1 , the field supply gets extended to the field coil
through the parallel path connected directly from stud -1 through NVC . In the
starter initially entire resistance comes in series with the armature and as the
handle is moved towards RUN, the portion of the series resistance that comes out
of the armature circuit gets added to the field circuit. Finally when the handle is
brought to the Run position, the entire resistance gets removed from the
Armature circuit and the motor runs at the rated speed. The handle is held in RUN
position due to the action of the NVC.

Action of the NVC: When the field current flows through the NVC it attracts the
handle with the soft iron piece and keeps it in contact the NVC electromagnet.
Hence NVC is also called as Hold On Coil. In addition to holding the handle in the
final RUN position, the NVC works as a safety/protection device by releasing the
handle back to the start position from the RUN position whenever there is a
power failure or when the field circuit breaks. Thus the entire starting resistance
comes into the armature circuit every time the motor is started from zero speed
and prevents high inrush currents during every fresh starting attempt after a
power failure.

Action of the OLR: As can be seen from the figure there is another protective
relay called OLR (Over Load Relay) which is also an electromagnet which works in
conjunction with an arm fixed on a fulcrum at one end and with a triangular iron
piece fixed on the other end. Whenever there is an overload current beyond a set

Page 30
Malla Reddy College of Engineering and Technology
Department of EEE (2022-23)
Electrical Machines -1 (EM-1): Lecture Notes: (Dr.Karimulla P Sk)
Unit 2: D C Motors

safety value, the electromagnet activates and pulls the arm upwards and the
triangular iron piece short circuits the two terminals which are connected to the
two ends of the NVC coil. Thus with any overload due to a fault in the motor or
associated circuit , the NVC gets deactivated and releases the handle back to the
initial safe start condition. After the fault is rectified the motor can be started
afresh with full resistance brought back into the armature circuit.

3 Point starter with a brass/copper arc:

In the earlier version of the 3 Point starter as we have seen, as the handle is
moved towards RUN position, the portion of the series resistance that comes out
of the armature circuit gets added to the field circuit. Thus finally when the motor
is running, the entire starter resistance gets added to the field circuit. But since
the starting resistance value is very small compared to field winding resistance,
this hardly reduces the field current and hence there is no any practical impact.
However this addition of resistance in the field circuit can be avoided by providing
a brass or copper arc with one end connected to the stud -1 and the other end
connected to the NVC as shown in the figure below.

Page 31
Malla Reddy College of Engineering and Technology
Department of EEE (2022-23)
Electrical Machines -1 (EM-1): Lecture Notes: (Dr.Karimulla P Sk)
Unit 2: D C Motors

Figure: 3 Point starter with Brass arc

With such an arrangement when the handle moves on the arc the field current
directly flows through the arc to the NVC thus avoiding the starting resistance.
With such an arc in place, the earlier parallel connection from stud -1 to the NVC
start terminal is no more required and hence is removed.

4 Point starter: The operation of the 4 point starter is explained along with the
schematic diagram shown below.

Figure: 4 Point starter

 The basic difference between a 3 Point starter and a four point starter is : In a
3 point starter NVC was connected in series with the field coil while in a 4
point starter the NVC is connected independently to the supply through a
fourth terminal termed as N in addition to L, F and A.
 With this arrangement any change in the field current due to the change field
control resistance will not affect the performance of the NVC. This ensures

Page 32
Malla Reddy College of Engineering and Technology
Department of EEE (2022-23)
Electrical Machines -1 (EM-1): Lecture Notes: (Dr.Karimulla P Sk)
Unit 2: D C Motors

that NVC always produces a force enough to hold on the handle irrespective of
the amount of field current. Adequate current required for the confirmed
operation of the NVC is obtained by adjusting the resistor R connected in
series with the NVC coil.
 However the 4 point starter has a separate disadvantage: Since now the NVC is
connected separately excluding the field current, it cannot detect the field
failure and hence the resulting over speed cannot be prevented.

Important concepts and Formulae:

 Torque generated in a DC machine : τ = Ka. Φ.Ia


 KVL around the armature circuit: VT = Eb+ Ia.Ra
 Generalized Torque vs. Speed equation in different types of motors:

ω = (VT / Ka. Φ ) -- [Ra/ (Ka. Φ)2+.τ


 Shunt Motor’s Other Characteristics:
Speed vs. Ia : ω = (VT - Ia.Ra)/ Ka. Φ
Torque vs. Ia: τ = Ka. Φ.Ia
 Series Motor:
ω = *VT /√( Kaf.τ)] -- [Ra/(Kaf)]
 Speed vs. Ia : ω = (VT - Ia.Ra) / Kaf. Ia = (VT / Kaf. Ia ) - (Ra
/ Kaf)
 Torque vs. Ia: τ = Ka. Φ.Ia = K af.I a2
 Speed control with armature voltage control is possible only below the
rated or nominal speed (also known as base speed).
 Speed control with flux control is possible only above the base speed

Illustrative examples:

Page 33
Malla Reddy College of Engineering and Technology
Department of EEE (2022-23)
Electrical Machines -1 (EM-1): Lecture Notes: (Dr.Karimulla P Sk)
Unit 2: D C Motors

Ex.1: A 500 V shunt motor with Rf = 250 Ω and Ra = 0.2 Ω runs at 2500 RPM taking
a current of 25 A from the mains supply . Calculate the resistance to be added to
the armature circuit to reduce the speed to 1500 RPM keeping the armature
current constant.

Solution:

First let us calculate the back e.m.f developed by the motor in the given first set
of conditions:

Field current If = Rated terminal voltage / Rf = 500 / 250 = 2 A

Armature current Ia = Il – If = ( 25 – 2 ) = 23 A
Back e.m.f Eb = VT - Ia Ra = 500 – 23 x 0.2 = 495.4 V
We know that the back e.m.f is proportional to the speed
∴ Eb1 / Eb2 = N1/ N2 i.e 495.4 / Eb2 = 2500/1500 ∴ Eb2 = 495.4 x 1500
/ 2500
= 297.24 V
But we also know that Eb2 = VT - Ia Ra2 ( Since the terminal voltage and the
armature current remain the same )
∴ 297.24 = 500 – 23 x Ra2 from which we get Ra2 = ( 500 – 297.24 ) /23 =
8.82 Ω
This is the total new resistance of the armature circuit (including the original
armature resistance of 0.2 Ω to get a speed of 1500 RPM)
Hence the new resistance to be added into the armature circuit = 8.82- 0.2 =
8.62 Ω
Ex.2: A DC shunt motor takes 22 A from 250 V supply. Ra = 0.5 Ω , Rf = 125 Ω.
Calculate the resistance required to be connected in series with the armature to

Page 34
Malla Reddy College of Engineering and Technology
Department of EEE (2022-23)
Electrical Machines -1 (EM-1): Lecture Notes: (Dr.Karimulla P Sk)
Unit 2: D C Motors

halve the speed (a ) when the load torque is constant ( b) When the load torque
is proportional to the square of the speed
Solution :
First let us calculate the speed of the motor when the load current I l is 22 A :
Field current If = Rated Terminal voltage / Field resistance = 250/125 = 2 A
Armature current Ia = Il - If = 22 -2 = 20 A
Back e.m.f Eb = VT - Ia Ra = 250 – 20 x 0.5 = 240 V
(a) we have to find out the New Ra when the speed is halved with torque
maintained constant :
We know that Torque T = Ka. Ø.Ia . In this case since change is only in the
armature resistance field current and hence flux Ø remains the same. Further
since the torque is maintained constant the armature currents are also equal and
hence Ia1 = Ia2 = 20 A
We also know that Eb = Ka. Ø.ω . As already explained, Ka. Ø remains same and
hence when the speed is halved the back e.m.f also gets halved.
Hence Eb2 = 120 V = VT – IaRa2 i.e 250 – 20 x Ra2 = 120 V i.e Ra2 = (250 -
120)/20 = 6.5Ω
Hence the Resistance to be added to halve the speed = Ra2 - Ra = 6.5-0.5 = 6.0 Ω
(b) Next we have to find out the New Ra when the speed is halved when torque is
proportional to square of speed.
2
When the torque is proportional to the square of the speed τ1 = K ω and τ =
1 2
K ω22
∴ τ1 / τ2 = K ω 12 / K ω 22 = ω 12 / ω22 = (1/0.5 )2 = 4
But Torque is also proportional to the product of flux (and hence field current
)and Armature current. Here field circuit is not disturbed and hence the field
current is same. Using this relation we can find out new armature current Ia 2
Page 35
Malla Reddy College of Engineering and Technology
Department of EEE (2022-23)
Electrical Machines -1 (EM-1): Lecture Notes: (Dr.Karimulla P Sk)
Unit 2: D C Motors

∴ τ1 / τ2 = K x If x Ia1 / K x If x Ia 2 = Ia1 / Ia 2 = 4 i.e Ia 2 = Ia1 / 4 = 20/4 =5 A


Next using the relation between the speeds and the back emfs we can find out
the armature resistance to be added.
ω1/ ω2 = 2 and also
ω1/ ω2 = Eb1/Eb2 = 240 / ( 250 – 5 x Ra2 ) i.e 250- 5Ra2 = 240/2 = 120 From
which we get
Ra2 = (250-120)/5 = 26 Ω ∴ Finally Resistance to b added is = 26-0.5 =
25.5 Ω
Ex.3: A 250 V DC series motor takes 40 A and runs at 1000 RPM. Find the speed at
which it runs if its torque is halved. Assume that the motor is operating in the
unsaturated region of its magnetization. Rf = 0.25 Ω Ra = 0.25 Ω
First we will use the relation between torque and armature current and get the
back e.mf when the torque is halved :
In a DC motor we know that the torque is proportional to Ø.I a. In the case of a
series DC motor flux is proportional to the armature current itself since I f = Ia .
Hence in a series motor τ 𝖺 I a2

Hence τ1/τ2 = Ia12 / Ia22 = 2 (Since torque is halved) Ia1 / Ia2 = √2

Ia1 = 40 A and Ia2 = 40/√2 = 28.28 A


Eb1 = 250 - 40 (0.25 + 0.25) = 230 V and Eb2 = 250 - 28.28 (0.25 + 0.25)
= 235.86 V
Next we will use the relation between back emf and speed and get the speed when
the torque is halved:
We know that Eb1 = Ka Ø1N1 and Eb2 = Ka Ø2N2 . But since the flux is
proportional to Ia the relations become Eb1 = K Ia1N1 and Eb2 =
K Ia2N2 where K is a new constant.

Page 36
Malla Reddy College of Engineering and Technology
Department of EEE (2022-23)
Electrical Machines -1 (EM-1): Lecture Notes: (Dr.Karimulla P Sk)
Unit 2: D C Motors

Hence Eb1/ Eb2 = K Ia1N1 / K Ia2N2 = Ia1N1 / Ia2N2 and N2 = ( Ia1 /


Ia2) ( Eb2/ Eb1) N1

Substituting the above values we get N2 = √2 (235.86/230)1000 = 1450 RPM


Ex.4: A 500 V DC shunt motor runs at 1900 RPM taking an armature current of
150 A. The armature resistance is 0.16 Ω. Find the speed of the motor when a
resistance is inserted in the field circuit which reduces the field current to 80 %
and the armature current is 75 A.
Solution:
We know that the back e.m.f of a DC motor is proportional to the Flux and speed.
And in the unsaturated region of the magnetization region the flux in turn is
proportional to the field current. So Back e.m.f is proportional to field current and
speed. We will find out the new speed by calculating the back e.m.fs [from the
relation (Eb = VT - Ia Ra )]and using the above proportionality relation in both the
conditions as below.
Eb1 = VT - Ia1 Ra = 500 – 150 x 0.16 = 476 V and is equal to Ka.Ø1. N1
Eb2 = VT - Ia2 Ra = 500 – 75 x 0.16 = 488 V and is equal to Ka . 0.8Ø1. N2
∴ 476 /488 = Ka.Ø1. N1 / Ka . 0.8Ø1. N2
And N2 = (488 /476)( N1 / 0.8 ) = (488 /476)( 1900 / 0.8 ) = 2435 RPM

Page 37
Malla Reddy College of Engineering and Technology
Department of EEE (2022-23)
Electrical Machines -1 (EM-1): Lecture Notes: (Dr.Karimulla P Sk)
Unit 3: Testing of D C Machines

UNIT – III
TESTING OF D.C. MACHINES

CONTENTS:

 Losses – Constant & Variable losses


 Calculation of efficiency
 Condition for maximum efficiency.
 Methods of Testing
 Direct, indirect and regenerative testing
 Brake test
 Swinburne’s test
 Hopkinson’s test
 Field’s test
 Retardation test
 Separation of stray losses in a DC motor test
 Important concepts and Formulae
 Illustrative examples

Page 1
Malla Reddy College of Engineering and Technology
Department of EEE (2022-23)
Electrical Machines -1 (EM-1): Lecture Notes: (Dr.Karimulla P Sk)
Unit 3: Testing of D C Machines

Losses:

DC Generators convert Mechanical power into Electrical power and DC Motors


convert Electric power to Mechanical power. In the process of conversion some
power is lost. The difference between the input power and the output power of a
machine is the Power loss that occurs inside the machine.

Constant & Variable losses:

The losses are broadly classified as constant losses and variable losses. Constant
losses are constant and are independent of the load where as the variable losses
are dependent on the load. They are further classified in detail as below.

Detailed Classification of Losses:

1. Electrical or Copper Losses (I2R Loss): Current flow through the resistance of
Armature and Field coils gives rise to I2R losses and since the coils are
normally made up of copper these losses are called Copper losses.
Armature copper loss: PA = I 2 R
A A
Field copper loss: 2
PF = I R F
F

2. Brush losses: The brush drop loss is the power lost across the contact potential
at the brushes of the machine. It is given by the equation:
PBD = VBD x IA
where PBD = brush drop loss
VBD = brush voltage drop
IA = armature current

The brush losses are calculated in this manner because the voltage drops
across a set of brushes are
approximately constant over a large range of armature currents. Unless
otherwise specified. The
brush voltage drop is usually assumed to be about 2 V.

Page 2
Malla Reddy College of Engineering and Technology
Department of EEE (2022-23)
Electrical Machines -1 (EM-1): Lecture Notes: (Dr.Karimulla P Sk)
Unit 3: Testing of D C Machines

3. Core Losses: Hysteresis and eddy current losses occurring in the Armature and
Field cores together are called core losses.
 Hysteresis loss: in an iron core is the loss of power due to the hysteresis
loop in the magnetization characteristic of the core in each cycle of the
alternating current applied to the core. In the case of DC machines though
there is no alternating current applied to the core, the change in the
magnetic flux within the machine due to its constructional features result in
a small hysteresis loss
 Eddy current losses: A time-changing flux induces voltage within a
ferromagnetic core in just the same manner as it induces voltage in the
conductors around the core of the armature. These voltages cause swirls of
current to flow within the core, much like the eddies seen at the edges of a
river. It is the shape of these currents that gives rise to the name eddy
currents. These eddy currents flowing in a resistive material (the iron of the
core) cause power loss thus heating the iron core and the resulting loss is
called eddy current loss. This loss is proportional to the thickness of the
core material and hence to minimize this loss the core is made up of thin
sheets called laminations instead of a single thick block. An insulating oxide
or resin is used between the strips so that the current paths for eddy
currents are limited to very small areas. Thus the eddy current losses have a
very little effect on the core's magnetic properties.
4. Mechanical Losses: They are associated with the mechanical effects and they
are mainly Friction and windage losses.
 Friction losses are losses caused by the friction in the bearings of the
machine and
 Windage losses are due to the friction between the moving parts of the
machine and the air flow in the machine housing.
5. Stray Losses: They are other miscellaneous losses that cannot be grouped into
any of the above categories.

Out of the above, Core Losses and Mechanical Losses are grouped under Constant
losses. Electrical or Copper Losses and Stray Losses are grouped under variable
losses.

Page 3
Malla Reddy College of Engineering and Technology
Department of EEE (2022-23)
Electrical Machines -1 (EM-1): Lecture Notes: (Dr.Karimulla P Sk)
Unit 3: Testing of D C Machines

The losses and their classification explained above is summarized in the form of a
tree and is shown below.

Figure: Classification of losses in DC Machine

The power flow in DC machines showing the stages where the different losses
occur is shown clearly in the figure below.

Power flow diagram:

One of the most convenient techniques for accounting for power losses and
showing them clearly in the order in which they occur in a machine is the power-
flow diagram. A power-flow diagram for a DC generator is shown in the figure (a)
below. In this figure, mechanical power is input into the machine, and then the
stray losses, mechanical losses, and core losses are subtracted. After they have
been subtracted, the remaining power is ideally converted from mechanical to
electrical form at the point labeled Pconv. The mechanical power that is converted
is given by:
PCONV = τind . ωm

Page 4
Malla Reddy College of Engineering and Technology
Department of EEE (2022-23)
Electrical Machines -1 (EM-1): Lecture Notes: (Dr.Karimulla P Sk)
Unit 3: Testing of D C Machines

and the resulting electric power produced is given by: PCONV = EA.IA

However, this is not the power that appears at the machine's terminals. Before
the terminals are reached, the electrical power losses like the copper losses and
the brush losses must be subtracted.

Figure: Power flow diagram of a DC Machine (a) Generator (b) Motor

In the case of dc motors, this power-flow diagram is simply reversed. The Power
flow diagram for a motor is shown in the above figure (b) above

Page 5
Malla Reddy College of Engineering and Technology
Department of EEE (2022-23)
Electrical Machines -1 (EM-1): Lecture Notes: (Dr.Karimulla P Sk)
Unit 3: Testing of D C Machines

Efficiency:

The efficiency of a DC Machine is defined as η = (Pout/Pin). 100 % = [(Pin ‒ Ploss) /


Pin] x 100 %

Using this basic relation and from a clear understanding of the above Power flow
the η calculations when the machine is working as a Generator and as a Motor are
given below.

Efficiency calculations of Generator:

 If IL is the load current supplied by the Generator at a terminal voltage of


VT then the output power is given by Pout = VT.IL
 The armature current IA = IL + IF

 Armature copper loss PA = IA2 RA


 Field copper loss PF = IF2 2RF
 Total losses = I R + I 2 R +W where W is the sum
A A F F C C
of the core losses and
stray losses. (also known as constant
losses)
 Therefore Input = Pout+ Total losses = Pout+ I A2 RA + I F2 RF
+WC

Hence η = (Pout/Pin). 100 % = (VT.IL )/( VT.IL + I A2 RA + I 2F RF


+WC).100%

Efficiency calculations of Motor:

 If IL is the line current taken by the Motor at a terminal voltage of VT then


the input power is given by Pin = VT.IL
Page 6
Malla Reddy College of Engineering and Technology
Department of EEE (2022-23)
Electrical Machines -1 (EM-1): Lecture Notes: (Dr.Karimulla P Sk)
Unit 3: Testing of D C Machines

 The losses are same as in the Generator

 Therefore output Pout = Pin-- Total losses = Pin-( I A2 RA + I F2 RF


+WC )

Hence η = (Pout/Pin). 100 % = [{Pin-( I A2 RA + I F2 RF +WC )}/ (VT.IL


) ] . 100%
Of these losses ( I 2 R +W ) are called constant losses P since they are almost
F F C C
independent of load. The armature copper losses i.e. (I A2.RA ) is called the variable
loss and is dependent on the load. The variable loss varies approximately as the
square of load current. We say approximately since loss varies as the square of
the armature current and not as the square of the load current. Hence if we know
the loss at full load, the loss at half load, one fourth load etc can be calculated.

Condition for Maximum efficiency:

The condition for maximum efficiency is developed by differentiating the


expression for efficiency as a function of load current and equating it to zero since
the variable losses are dependent on the load current.

Generator:
The efficiency is obtained as: η = (Pout/Pin). 100 % = (VT.IL )/( VT.IL + IA2 RA + IF2 RF
+WC).100%
Neglecting the field current which is small compared to armature current we get

η = (VT.IL )/( VT.IL + IL2 RA +WC).100% = 1/[1 + IL2 RA /(VT.IL ) +WC/(VT.IL )].100
= 1/ [1 + IL RA / VT +(WC/(VT IL )].100
The efficiency is maximum when the denominator is maximum. Hence the
condition for maximum efficiency becomes: d/dIL [1 + IL RA / VT +(WC/(VT IL )] = 0
i.e. RA / VT - (WC/(VT IL2 ) = 0

Page 7
Malla Reddy College of Engineering and Technology
Department of EEE (2022-23)
Electrical Machines -1 (EM-1): Lecture Notes: (Dr.Karimulla P Sk)
Unit 3: Testing of D C Machines

And finally the condition for maximum efficiency becomes: IL2RA = WC


Which means Variable losses = Constant
Losses
And the current at maximum efficiency becomes:
IL = √ WC/ RA = √ Constant Losses/Armature resistance

Motor:
The efficiency is obtained as: η = (Pout/Pin). 100 % = [{Pin-( I 2 ARA + I 2 FRF +WC )}/
(VT.IL ) ] . 100%
= [{VT.IL -( I A2 RA + IF2 RF +WC )}/ (VT.IL ) ] . 100%
Neglecting the field current which is small compared to armature current we get
η = *,VT.IL - ( IL2 RA + WC )}/ (VT.IL ) ] . 100%
= 1 – [(IL2 RA + WC )/ (VT.IL ) ] . 100
η becomes maximum when the term in the square brackets becomes minimum
and thus the condition for maximum efficiency becomes d/d IL [(IL2 RA + WC )/
(VT.IL ) ] = 0 which again finally becomes :

IL2RA = WC
Or Variable losses = Constant Losses

And the current at maximum efficiency also becomes:

IL = √ WC/ RA = √ Constant Losses/Armature resistance

Both same as that for the generator.

Testing of DC machines:

Involves the measurement of the various losses and then finding out the
efficiency of the machine by various methods. The methods are broadly classified
as:

Page 8
Malla Reddy College of Engineering and Technology
Department of EEE (2022-23)
Electrical Machines -1 (EM-1): Lecture Notes: (Dr.Karimulla P Sk)
Unit 3: Testing of D C Machines

1. Direct 2. Indirect and 3. Regenerative methods of testing

1. Direct method of testing : In this method the DC machine is actually loaded to


the required extent, the Input and output are measured and then the η is
calculated as η = Output/Input
1. This method is generally employed only for small motors. The motor is
loaded by a friction pulley arrangement.(braking)
2. The main drawback of this method is accuracy of output power
measurement is limited.
3. Difficult to provide braking load for a large capacity motor.
2. Indirect method of testing: In this method the machine is not subjected to full
load. First on no load the constant losses are measured and then efficiency is
estimated at various loads. Swinburne’s test and Hopkinson’s test come under
this category. Only shunt motors can be tested using these methods. Series
motors cannot be tested with this method since they cannot be run on no
load.
3. Regenerative method of testing: In this method a motor generator is pair is
used which are powered by each other. Thus only losses are drawn from the
mains power supply. Hopkinson’s test comes in this category.

Brake Test: This is a direct method of testing. In this method the motor is put on a
direct friction load arrangement with a belt and a pulley as shown in the figure
below.

Page 9
Malla Reddy College of Engineering and Technology
Department of EEE (2022-23)
Electrical Machines -1 (EM-1): Lecture Notes: (Dr.Karimulla P Sk)
Unit 3: Testing of D C Machines

Figure: Brake Test setup

By adjusting the tension in the pulley the motor can be subjected from no load to
its full load capability. Since the load is applied by the physical braking action, the
test is called the Brake test.

The tension in the belt is adjusted by using the handle. The tension (kgf) is
obtained from the spring balance readings. The net force applied on the pulley by
this braking arrangement is given by:

Net force = (W1 – W2) Kgf = 9.81(W1 – W2) Nw

Where R = Radius of the pulley in meters

N = Speed in RPM

W1 and W2 = Spring balance readings on the tight side and on


the slack side of the

Pulley respectively.

Page 10
Malla Reddy College of Engineering and Technology
Department of EEE (2022-23)
Electrical Machines -1 (EM-1): Lecture Notes: (Dr.Karimulla P Sk)
Unit 3: Testing of D C Machines

With this force exerted on the pulley, the load torque applied on the motor shaft
is given by:

τ load = Net force x Radius of the pulley = 9.81(W1 – W2)R Nw.m

With this applied load torque τ load , the output power(mechanical ) of the motor
is given by:

Pout = τ load x ω = τ load x2πN/60 W

The input power (electrical) to the motor is given by : Pin = VI

Thus we have η = Pout/ Pin = [τ load x2πN/60+/VI

Apart from the efficiency, we can also find out all the characteristics like Torque
vs Speed , Speed vs Armature current and Torque vs armature current of the
motor by noting down the currents and voltage along with the speed N at various
load settings. The speed is measured by using physical contact type Tachometer.

Advantages:

1. Efficiency can be found out in the actual working conditions.


2. The method is simple and easy to perform.
3. The test can be performed on any type of DC machine.

Disadvantages:

1. Due to the friction lot of energy is wasted in the form of heat. Hence the
test is quite expensive and is suitable for only small machines.
2. Since heat energy is not accounted for, the efficiency observed would be
inaccurate to that extent.

Swinburne’s test:

Page 11
Malla Reddy College of Engineering and Technology
Department of EEE (2022-23)
Electrical Machines -1 (EM-1): Lecture Notes: (Dr.Karimulla P Sk)
Unit 3: Testing of D C Machines

This is a test to determine the efficiency of any DC Machine (Motor or Generator)


without conducting the actual test at the required load. The test is conducted just
at no load and the constant losses are found out when the machine is running as
a motor. Then the efficiency is found out by calculating the variable losses at the
required load. This method is formulated by Sir James Swinburne and hence it is
called Swinburne’s test. This comes under the category indirect method of testing.
The test setup required to conduct this test is shown in the figure below.

Figure: Swinburne’s Test Setup

The machine is run as a motor on no load at normal terminal voltage VT , at


normal speed and the armature current IA & field current IF(Ish in figure ) are
measured.

 Then the no load armature current INL = IA + IF


 Variable losses on no load = IA2 .RA (Machine’s armature
resistance can be measured directly and these losses can be calculated)

Page 12
Malla Reddy College of Engineering and Technology
Department of EEE (2022-23)
Electrical Machines -1 (EM-1): Lecture Notes: (Dr.Karimulla P Sk)
Unit 3: Testing of D C Machines

 Input to the motor = VT. INL = Total losses (Since the machine is on no
load there is no output. i.e. the entire input power on no load goes as
losses.)
 Therefore constant losses PC = (Total losses - Variable losses) = (VT. INL)-( I A2
.RA)

Using these constant losses PC , the efficiency of the machine can be estimated
at any other load when working either as a Motor or as a Generator.

Working as a Generator delivering a load current of IL amperes at a terminal


voltage of VT volts:

Power output = VT. IL

Armature current IA = IL+ IF (IF is same as obtained in the No load test )

Variable loss = IA2 R A (RAis obtained from the no load test or from
Machine data)

Efficiency = (output/Input) = [output/(output + Total losses)] = (VT. IL)/( VT. IL+


IA2RA+PC)

(PC is obtained from the No load test and IA2RA is calculated using IA
corresponding to the required IL at which the efficiency is to be calculated)

Working as a Motor drawing a load current of IL amperes from a supply terminal


voltage of VT volts:

 Power in put = VT. IL


Armature current IA = IL- IF (IF is same as obtained in the No load test )

Variable loss = IA2 RA (RA is obtained from the no load test or from
Machine data)

Page 13
Malla Reddy College of Engineering and Technology
Department of EEE (2022-23)
Electrical Machines -1 (EM-1): Lecture Notes: (Dr.Karimulla P Sk)
Unit 3: Testing of D C Machines

Efficiency = (output/Input) = [(Input-Total losses)/ Input] = [VT. IL-


(IA2RA+P C)]/(V T. IL)

(PC is calculated and obtained from the No load test and IA2R Ais calculated
using IA corresponding to the required IL at which the efficiency is to be calculated)

Advantages of Swinburne’s test:

 This is a very simple to determine the efficiency of the machine at any load
just by conducting the no load test.
 The power required is very less compared to the direct full load test.

Disadvantages of Swinburne’s test:

 This test can be done on Shunt machines only.


 The speed and flux are assumed constant. But the speed will fall with
loading. Fall in speed results in lesser friction and windage losses. Change in
flux will change the core losses.
 The temperature of the machine changes while running on load. Hence the
assumption that RA is same as that of the No load test is not correct.
 These reasons contribute to the difference in the efficiency obtained from
the Swinburne’s test and actual load test.

Hopkinson’s test: In this set up two identical DC machines are coupled


mechanically and tested together. One of the machines works as a motor and
drives the other machine which works as generator and it’s electrical output in
turn is connected back to the motor. Hence this comes under the regenerative
category and is also called a Back to Back test. The motor is connected to the
mains supply and it draws power from the mains only to compensate for the
losses in the two machines since the major power required by each machine is

Page 14
Malla Reddy College of Engineering and Technology
Department of EEE (2022-23)
Electrical Machines -1 (EM-1): Lecture Notes: (Dr.Karimulla P Sk)
Unit 3: Testing of D C Machines

derived from the other machine. Since the power consumption is only to the
extent of the losses they can be tested up to full load.

Figure below shows the Hopkinson’s test up with all the measured voltage and
current parameters marked clearly.

Figure: Hopkinson’s Test Setup

(Ammeters are to be redesignated as ‘A2 to A4’ , ‘A3 to A2 ‘ and ‘A4 to A3’ so


that the current designations will match )

Initially the switch S is kept open and the Motor is run and brought to the rated
speed by adjusting the field current using the field rheostat R1 . The Generator
Voltage is adjusted by adjusting it’s field rheostat R2 until the voltmeter reads
zero volts. (This indicates that the Generator voltage is same as that of the Motor
both in amplitude and polarity) This prevents flow of any high circulating currents
when the switch S is closed and the two machines are connected back to back.

Page 15
Malla Reddy College of Engineering and Technology
Department of EEE (2022-23)
Electrical Machines -1 (EM-1): Lecture Notes: (Dr.Karimulla P Sk)
Unit 3: Testing of D C Machines

Now the switch is closed thus bringing the machines back to back and the load on
both the machines can be increased gradually by increasing the Generator
excitation or decreasing the Motor excitation. The readings from all the meters
are taken at every load setting for further evaluation. Let us spell out clearly all
the parameters for ease of further derivations.

V = Terminal Voltage (Supply Voltage)

I1 = Current drawn from mains supply

I2 = Current supplied by the generator to the motor

I3 = Excitation Current of Generator

I4 = Excitation Current of Motor

Ra = Armature resistance of each machine

1. Equal efficiency : Let us now first find out the Efficiency ‘η’ assuming it to be
same for both the machines:

Input to the motor = V (I1 + I2)

Output of the motor = η x Input to the motor = η x V (I1 + I2)

This output of the motor is given as input to the generator. Hence

Input to the Generator = η x V (I1 + I2)

Output of the generator = η x Input to the generator = η x η x V (I1 + I2) = η2 x V


(I1 + I2)

But the output of the generator can also be given as = V I2 and equating these
two we get

η2 x V (I1 + I2) = V I2 from which we get

Page 16
Malla Reddy College of Engineering and Technology
Department of EEE (2022-23)
Electrical Machines -1 (EM-1): Lecture Notes: (Dr.Karimulla P Sk)
Unit 3: Testing of D C Machines

η = √ [ I2 / (I1 + I2) ]

2. Un equal efficiency: Let us now find out the Efficiency ‘η’ assuming it to be
unequal for the two machines:

In this analysis, the stray losses (Constant Losses) are assumed to be same for
both the machines where as the field and armature copper losses are different
(Since when the efficiencies are different it means that the currents are not the
same and hence the copper losses also will not be same)

So first let us determine the Copper losses of the two machines independently and
then the constant losses of both machines together can taken as the difference
between the input power from the main supply and the total copper losses.

Armature copper losses in Generator = (I2 + I3 )2 x Ra

Armature copper losses in Motor = (I1+I2 – I4 )2 x Ra

Field copper losses in Generator = V I3

Field copper losses in Motor = V I4

We know that the total losses in both the machines put together are equal to the
input power from the mains supply i.e V I1

Hence we can take that the total stray losses for both the machines put together
are the difference between the input power and all the copper losses put
together. Hence

Stray losses of both machines together = V I1 – [(I2 + I3 )2 x Ra + (I1+I2 – I4 )2 x Ra +


V I3 + V I4] = Say Ws and stray losses of each machine = Ws/2

Page 17
Malla Reddy College of Engineering and Technology
Department of EEE (2022-23)
Electrical Machines -1 (EM-1): Lecture Notes: (Dr.Karimulla P Sk)
Unit 3: Testing of D C Machines

Now that we know the variable losses (Armature and Field Copper losses) and the
constant (stray) losses for both the machines we can easily find out the
efficiencies of both Generator and Motor using the above data as shown below.

Efficiency of Generator:

Total losses = Generator’s variable losses + Stray losses of one machine

= [(I2 + I3 )2 x Ra + V I3 + Ws/2 ]

Output of Generator = V I2

Efficiency of Generator ηG = Output/ Input = Output/ ( Output + Losses)

= V I2 / V I2 + [(I2 + I3 )2 x Ra + V I3 + Ws/2 ]

Efficiency of Motor:

Total losses = Motor’s variable losses + Stray losses of one machine

= [(I1 +I2 – I4 )2 x Ra + V I4 + Ws/2 ]

Input to Motor = V ( I1 + I2)

Efficiency of Motor ηM = Output/ Input =( Input – losses ) / Input

= [ V ( I1 + I2) - [(I1 +I2 – I4 )2 x Ra + V I4 +


Ws/2 ]] / V ( I1 + I2)

Field’s test:

Introduction and Methodology:

This test is for finding out the losses and efficiency of DC Series Motors by direct
testing, since series motors cannot be tested on no load. The test is named after
the inventor of test method ‘MB Field ‘. Series motors which are normally used
for Traction are available as pairs and hence this test is devised on two motors

Page 18
Malla Reddy College of Engineering and Technology
Department of EEE (2022-23)
Electrical Machines -1 (EM-1): Lecture Notes: (Dr.Karimulla P Sk)
Unit 3: Testing of D C Machines

which are coupled mechanically. The test setup is shown in the figure below. One
machine works as motor supplying power to the other one working as Generator.
Their mechanical and iron (core) losses (put together called as stray losses or
constant losses) are made equal by: (i) running them with equal speed and
(ii) by connecting their both field windings in the motor armature circuit to the
Motor input supply such that both the machines are equally excited . The load
resistance is adjusted till the Motor draws the rated current as read by ammeter
A1. In this condition all the other parameters are noted down from the respective
meter readings as per the nomenclature given below.

Figure: Field’s Test Setup

Nomenclature:

Page 19
Malla Reddy College of Engineering and Technology
Department of EEE (2022-23)
Electrical Machines -1 (EM-1): Lecture Notes: (Dr.Karimulla P Sk)
Unit 3: Testing of D C Machines

V = Supply voltage

V1 = Motor Supply voltage

V2= Generator output voltage connected across the variable load resistance R

I1 = Motor armature current and also the field current of both Motor and
Generator

I2 = Generator armature current

Also Let RA = Motor armature resistance and RF = Motor field resistance ( which
can be measured independently or can be taken from the machine data )

From this data obtained in this full load condition, the stray losses, copper losses
and then efficiency of both the Motor and the Generator can be found out as
below:

Stray Losses:

Input to the total Motor Generator test setup: V I1

Output of Generator = V2 I2

Total losses of both motor and generator = WT = V I1• V2 I2

But total losses of both motor and generator WT are also equal to (Armature
Copper losses + Field Copper losses + Stray losses ).Thus

WT = V I1– V2 I2 = (I 12 + I 22) RA + 2I12 RF + WS

And total Stray Losses = Total Losses – Total Armature and Field copper losses

= VI1 – V2 I2– [(I12 + I22) RA + 2I12 RF ] and

Stray losses per machine = Total Stray Losses/2 = WS = [V I1 – V2 I2 – [(I12 +


I22) RA + 2I12 RF ]] / 2

Page 20
Malla Reddy College of Engineering and Technology
Department of EEE (2022-23)
Electrical Machines -1 (EM-1): Lecture Notes: (Dr.Karimulla P Sk)
Unit 3: Testing of D C Machines

Now using this value of stray losses of each machine, the efficiency of the
machine as a Motor and as a Generator can be found out as below.

η as a Motor:

Input to Motor = V1 I1

Total losses = Armature and Field Copper losses + Stray losses = I 2(R
1 +A
RF) + WS

Output of Motor = Input – Losses = V1 I1 – {I 12(R A+ R )F + W S}

Efficiency of the motor = ηM = Output/Input = [V1 I1 – {I1 2(RA + RF) + WS}] /


V1 I1

η as a Generator:

This not very important because the machine is working in separately excited
condition. However just for completion sake let us find it out.

Output of Generator = V2 I2

Total losses = Armature and Field Copper losses + Stray losses =


I22RA+ I 21R +F W S

Input to the Generator = Output + Losses = V2 I2 + I 22R A+ I 12R F+ W S

Efficiency of the Generator = ηG = Output/Input = V2 I2 / (V2 I2 + I22RA +


I1 2RF + WS)

Retardation test or Running down test: This is an indirect test similar to the
Swinburne’s test where in the constant (Stray losses) losses are first determined
and then the efficiency at any load when the machine is working both as a
Generator and Motor are estimated on the same lines. However the constant
losses in this test are determined by a different principle i.e. by finding out the

Page 21
Malla Reddy College of Engineering and Technology
Department of EEE (2022-23)
Electrical Machines -1 (EM-1): Lecture Notes: (Dr.Karimulla P Sk)
Unit 3: Testing of D C Machines

kinetic energy spent by a rotating mass during the process of retardation from the
rated speed to zero speed and then calculating the rate of change of kinetic
energy which is equal to the Power Loss.

The setup to conduct this test is shown in the figure below.

Test Procedure: The motor is started and taken to a speed higher than the rated
speed of the machine. Then the supply to the motor is cutoff by moving the Two
Pole Two way switch from the supply side to the open terminals. The armature
then slows down with its’ own inertia and its stored energy is used up to supply
the constant rotational losses (stray losses) like iron, friction, windage etc. This
power loss is found out from the following principle.

If ‘I’ is the moment of Inertia of the Armature, and ‘ω’ is the angular velocity, then
the kinetic energy of the armature is given by:

KE = ½ Iω2 and

Page 22
Malla Reddy College of Engineering and Technology
Department of EEE (2022-23)
Electrical Machines -1 (EM-1): Lecture Notes: (Dr.Karimulla P Sk)
Unit 3: Testing of D C Machines

Figure: Retardation test setup

The power loss which is the rate of change of Energy is given by:

PL = d/dt (½ Iω2 ) = I. ω . dω/dt

Substituting the value of ‘ω’ in terms of the speed in RPM ‘N’ ( 2πN/60) we get

PL = (2π/60)2 I. N . dN/dt

So, to find out the stray losses we must know ‘I’, the moment of inertia and
dN/dt, the rate of change in speed. The method of finding out these quantities is
given below.

The method of finding out dN/dt :

When the motor is cutoff from the input supply, the speed starts falling down.
The motor back e.m.f. as read by the Voltmeter V1 connected across the motor is
noted down as a function of time. Since we know that the back e.m.f is

Page 23
Malla Reddy College of Engineering and Technology
Department of EEE (2022-23)
Electrical Machines -1 (EM-1): Lecture Notes: (Dr.Karimulla P Sk)
Unit 3: Testing of D C Machines

proportional to the speed we can convert the V1 reading into speed in RPM and
plot it as a function of time as shown in the figure below. In this figure, at the
point C corresponding the rated speed a tangent AB is drawn whose slope gives
the rate of change in speed dN/dt.

Thus: dN/dt = OA(RPM)/OB(Time)

Figure: Speed fall during the retardation test

Determination of Moment of Inertia (I):

Method -1: Using Flywheel

The same test as earlier is repeated after adding a flywheel of known Moment of
Inertia ‘I1’ to the armature shaft of the motor and the resulting rate of change of
speed is obtained. Let us call the earlier rate of change as dN/dt 1 and the second
rate of change with added inertia as dN/dt2.

Then, since the losses can be assumed to be same with or without the new
flywheel we have the following relations:

In the first case without flywheel: PL = (2π/60)2 I. N . dN/dt1

In the second case with added flywheel: PL = (2π/60)2 (I+ I1). N .


dN/dt

After equating the two equations and simplifying we get : I = I1 (t1) /( t2– t1)

Page 24
Malla Reddy College of Engineering and Technology
Department of EEE (2022-23)
Electrical Machines -1 (EM-1): Lecture Notes: (Dr.Karimulla P Sk)
Unit 3: Testing of D C Machines

Method -2: Without using a Flywheel (Using Resistance Braking)

 First, the switch is taken from supply side to the open condition and then
the time taken ( say t1) for the motor to slow down from a speed slightly
higher than the rated speed to a speed slightly lower than the rated speed
(say δN) is noted down with just the Armature alone like in the earlier
method step -1 (without any added external inertia)
 Then again the switch S is moved from the supply position to the Resistance
position quickly and again the time taken(say t2) for the same change in
speed (same δN)is noted down. By this effectively we are connecting an
electrical load across the armature in which the stored electrical power is
dissipated thus providing an additional retarding torque.
 This additional power loss due to the resistance is given by the product of
the Average Voltage across the Armature (say V ) and the average current
(say IA ) that flows into the Braking Resistance R i.e. I A2 ( R A+R )x V = say W’
 Then the powers dissipated during the above two steps are given by

1. W = (2π/60)2 I. N . dN/dt1 (Just due to the armature


Inertia)
2. W + W’ = (2π/60)2 I. N . dN/dt2 (Due to the armature Inertia and
the braking
resistance )

Separation of Constant losses:

The theory required for the purpose of Separation of Constant losses is


explained below.

At any given excitation:

 Friction losses and hysteresis losses are both proportional to speed .

Page 25
Malla Reddy College of Engineering and Technology
Department of EEE (2022-23)
Electrical Machines -1 (EM-1): Lecture Notes: (Dr.Karimulla P Sk)
Unit 3: Testing of D C Machines

 Windage losses and eddy current losses are both proportional to square of
speed.
 Hence Friction losses = AN Watts, Windage losses = BN2 Watts, Hysteresis
losses = CN Watts, and Eddy current losses = DN2 Watts where N = speed
and A,B,C and D are constant coefficients
 Further the coefficient C of hysteresis losses is proportional to B max1.6 and
2
the coefficient D of Eddy current losses is proportional to B max

The other standard relation: For a motor on no load, power input to the armature
is the sum of the armature copper losses and the above losses.

Hence from a no load test we can get the constant losses as usual and then
equate them to the constant losses with the above categorization as shown
below:

Power input to the armature = V.Ia watts.

Armature copper losses = I a2.R awatts

Constant Losses = W = V.Ia – I a2.R a= (A + C)N + (B + D)N2

W/N = (A+C) + (B+D)N.

First the test is conducted with rated field current as per the following procedure:

1. The motor is started on no load with field current set to rated value by
adjusting the field auto transformer.
2. The armature voltage is increased till the speed is about 200 rpm more than
the rated value.
3. Now, the speed is gradually decreased by decreasing the armature voltage,
the values of armature voltage, armature current and speed are noted
down.

Page 26
Malla Reddy College of Engineering and Technology
Department of EEE (2022-23)
Electrical Machines -1 (EM-1): Lecture Notes: (Dr.Karimulla P Sk)
Unit 3: Testing of D C Machines

4. From this data W is calculated as explained above and the graph between
W/N & N is plotted as shown in the figure below.

Figure : Plot of PNL / N vs. N

5. The graph between PNL /N & N is a straight line from which (A+C) and (B+D)
can be found. (A+C) is the y axis intercept and (B+D) is the slope of the
straight line.
6. In order to separate out A, B, C and D, the test is repeated again with
reduced field current and the graph between PNL /N & N is drawn again as
shown in the figure which is a straight line given by PNL /N = (A+C’) N +
(B+D’) N from which we can again find out (A+C’) and (B+D’).

At the reduced excitation, friction and windage losses are still AN and BN2,
but hysteresis losses become C’N and eddy current losses become D’ N2.
We can now obtain (A+C’) and (B+D’) as before from the second straight
line .

Page 27
Malla Reddy College of Engineering and Technology
Department of EEE (2022-23)
Electrical Machines -1 (EM-1): Lecture Notes: (Dr.Karimulla P Sk)
Unit 3: Testing of D C Machines

As already indicated the coefficient of hysteresis loss C is proportional to Bmax1.6,


B 2. Since the core is
and the coefficient of eddy current loss D is proportional to max
common we can replace maximum flux density Bmax with maximum flux density
Ømax in the core and then the ratio of fluxes corresponding to the normal and
reduced excitation would become :

(C/C’) = ( Ømax / Ø’max)1.6 also equal to (Eb/E’b)1.6 since Back e.m.f is


proportional to Ømax

(D/D’) = ( Ømax / Ø’max)2 also equal to (Eb/E’b)2 since Back e.m.f is


proportional to Ømax

During the two tests we can get the values of back e.m.f s from the measured
values of armature supply voltage V , armature current Ia and armature resistance
Ra using the relation: Eb = V– Ia.Ra and thus get the values of (C/C’) and (D/D’) .
We can also get the values of (C’-C) and (D’-D) by subtracting (A+C) from (A+C’)
and (B+D) from (B+D’)

From these values we can get all the four coefficients A,B,C and D and thus
separate the constant losses PNL into Friction, Windage, Hysteresis and Eddy
current losses

Since the change in speed δN is same in both the cases, dividing the expression 2
by expression 1 we get

(W + W’ )/W = (1/ dt2) / (1/ dt1) = dt1/ dt2 = t1/ t2 and after
simplification we get

W = W’ *t1/( t1 - t2)] which is equal to the stray losses ( Mechanical plus


Core losses )

Page 28
Malla Reddy College of Engineering and Technology
Department of EEE (2022-23)
Electrical Machines -1 (EM-1): Lecture Notes: (Dr.Karimulla P Sk)
Unit 3: Testing of D C Machines

Important concepts and Formulae:

 The efficiency of a DC Machine is defined as η = (Pout/Pin). 100 %

Efficiency of Generator:
η = (Pout/Pin). 100 % = (VT.IL )/( VT.IL + I A2 RA + IF2 RF +WC).100%

Efficiency of Motor:
η = (Pout/Pin). 100 % = [{VT.IL -( I A2 RA + I F2 RF +WC )}/ (VT.IL ) ] . 100%

 The condition for maximum efficiency :


 Constant losses PC = Variable losses ( IL2RA ) or IL = √( PC/ RA)

Illustrative examples:

Ex.1: A DC shunt motor having a full load efficiency (η )of 85 % takes a line current
of 27A from 250 Volts mains on full load. If R a = 0.5Ω and Rf = 125 Ω, find the
constant losses, load current for maximum efficiency and the maximum
efficiency.
Solution:
Input power at full load = Full load current x Rated voltage = 250 x 27 = 6750 W
Output power = Input power x η = 6750 x 0.85 ( η = 85% ) = 5737.5 W
Hence Total losses = Input power − Output power = 6750 − 5737.5 = 1012.5 W
We know that Total losses = Variable losses ( Ia2Ra ) + constant losses.
If = Rated Terminal Voltage/ Field resistance = 250 / 125 = 2 A
For the Shunt motor armature current Ia = Il − If = 27 − 2 = 25 A
Variable losses = Ia2Ra = 252 x 0.5 = 312.5

Page 29
Malla Reddy College of Engineering and Technology
Department of EEE (2022-23)
Electrical Machines -1 (EM-1): Lecture Notes: (Dr.Karimulla P Sk)
Unit 3: Testing of D C Machines

Constant losses = Total losses − Variable losses = 1012.5 − 312.5 = 700 W


We know that the condition for maximum efficiency is: Variable losses =
Constant losses

i.e. Ia2Ra at maximum efficiency = 700 ∴ Ia @max. η = √700 /0.5 = √ 1400 =


37.42 A
∴The load current at maximum η : Il@max. η = Ia@max. η + If = 37.42 + 2 =
39.42
Input power at maximum η = Il @max. η x Rated terminal voltage = 39.42 x 250 =
9855 W

O/P power at maximum η = I/P power at maximum η−Total losses = 9855–(700


+700) = 8455 W
(Since variable losses = constant losses = 700)
Maximum efficiency = Out power at maximum efficiency/ Input power at
maximum efficiency
= 8455 / 9855 = 0.858 or 85.8 %
Ex.2: A 100 Kw 500 V DC shunt machine when run as a motor on no load at rated
speed and voltage takes a line current of 10 A and a shunt field current of 2.5 A .
Resistance of the armature is 0.15 Ω. Estimate the efficiency of the DC machine
when running as a generator (a) at full load (b) at half full load.
First the constant losses of the Machine are obtained from the data we have when
the machine is run as a motor on no load at rated speed and voltage:
Input power on no load = Rated voltage x Input current on no load = 500 x 10 =
5000 W
Field current If = 2.5 A

No load Armature current Ia = Il no load – If = ( 10 – 2.5 ) = 7.5 A

Page 30
Malla Reddy College of Engineering and Technology
Department of EEE (2022-23)
Electrical Machines -1 (EM-1): Lecture Notes: (Dr.Karimulla P Sk)
Unit 3: Testing of D C Machines

Variable loss at no load = (Ia on no load )2 x Ra = 7.52 x 0.15 = 8.4375 W


Constant Losses = ( Input power– Variable losses)(on no load) = 5000 - 8.4375
= 4991.56 W
Next we will calculate the efficiency in different conditions:
(a) Running as a generator at full load:
Full load output (line) current = 100 x 1000 / 500 = 200 A
Full load armature current = Full load line current + Field current = 200 + 2.5 =
202.5 A
Variable (Armature copper) losses on full load = Ia2 Ra = 202.52 x 0.15 = 6150.94
W
Total losses @ full load = Constant Losses + Variable losses on full load = 4991.56
+ 6150.94
= 11142.5 W
Efficiency at full load (Working as Generator) = Output / Input = Output / Output
+ Total losses @ full load = 100000 / 100000 + 11142.5 = 0.8997 or 89.97
%
(a) Running as a generator at half full load:
Half Full load output (line) current = 50 x 1000 / 500 = 100 A
Half Full load armature current = Half Full load line current + Field current = 100 +
2.5 = 102.5 A
Variable (Armature copper) losses on half full load = Ia2 Ra = 102.52 x 0.15 =
1575.94 W
Total losses @ half load = Constant Losses + Variable losses on half load = 4991.56
+ 1575.94
= 6567.5 W

Page 31
Malla Reddy College of Engineering and Technology
Department of EEE (2022-23)
Electrical Machines -1 (EM-1): Lecture Notes: (Dr.Karimulla P Sk)
Unit 3: Testing of D C Machines

Efficiency at half full load (Working as Generator) = Output / Input = Output /


Output + Total losses @ full load = 50000 / 50000 + 6567.5 = 0.8839 or
88.39 %
Ex.3: A 500 V DC shunt machine takes 5A when running light (on no load) at rated
speed and rated voltage as a motor. Calculate the out output power and
efficiency when the machine is run as a Motor and taking an Input current of 80
A. Calculate the line current at which the efficiency is maximum and the value of
maximum efficiency. Ra = 0.2 Ω and Rf = 250 Ω
First the constant losses of the Machine are obtained from the data we have
when the machine is run as a motor on no load at rated speed and voltage:
Input power on no load = Rated voltage x Input current on no load = 500 x 5 =
2500 W
Field current If = Rated voltage / Rf = 500 / 250 = 2 A

No load Armature current Ia = Il no load – If = ( 5 – 2 ) = 3 A


Variable loss at no load = (Ia on no load )2 x Ra = 32 x 0.2 = 1.8 W
Constant Losses = ( Input power– Variable losses)(on no load) = 2500 – 1.8 =
2498.2 W
Next we will calculate the output power and efficiency when the Machine is
running as a motor and taking an input current of 80 A :
Armature current = Line current (Input Current) - Field current = 80 - 2 = 78 A
Variable (Armature Copper) losses (with armature current of 78A) = Ia2 Ra =782 x
0.2 = 1216.8 W
Total losses = Constant Losses + Variable losses at 80 A line current = 2498.2 +
1216.8 = 3715 W
Input Power = 500 x 80 = 40000 W

Page 32
Malla Reddy College of Engineering and Technology
Department of EEE (2022-23)
Electrical Machines -1 (EM-1): Lecture Notes: (Dr.Karimulla P Sk)
Unit 3: Testing of D C Machines

Out Put Power at 80 A line current (Working as Motor) = Input Power – Total
losses at 80 A line current = 40000 -3715 = 36285 W
Efficiency at 80 A line current (Working as Motor) = Output Power /Input Power
= 36285/40000 = 0.9071 or 90.71 %
Finally we will calculate the line current at which the efficiency is maximum and
the value of maximum efficiency:
We know that the condition for maximum efficiency is: Variable losses =
Constant losses
i.e. Ia2Ra ( variable Losses ) at maximum efficiency = 2498.2 W
∴ Ia @max. η = √2498.2 /0.2 = √ 12491 = 111.76 A
∴The line current at maximum η : Il@max. η = Ia@max. η + If = 111.76 + 2 =
113.76 A
Input power at maximum η = Il @max. η x Rated terminal voltage = 113.76 x 500 =
56880 W
O/P power at maximum η = I/P power at maximum η−Total losses =
56880 – (2498.2 + 2498.2) = 51883.6 W
(Since variable losses = constant losses = 2498.2)
Maximum efficiency = Out power at maximum efficiency/ Input power at
maximum efficiency
= 51883.6 /56880 = 0.9122 or 91.22 %

Example: 4

The following readings are obtained when doing a load test on d.c. shunt motor
using a brake drum. Spring balance readings =10kg, and 35 kg, Diameter of drum

Page 33
Malla Reddy College of Engineering and Technology
Department of EEE (2022-23)
Electrical Machines -1 (EM-1): Lecture Notes: (Dr.Karimulla P Sk)
Unit 3: Testing of D C Machines

=40 cm, speed of the motor = 950 r.p.m, Applied voltage = 200 V Line
current =30 A . Calculate the output power and efficiency.

Solution : We have W1 =10 kg and W2 = 35 kg


Diameter of the drum D = 40 cm, R =d/2 =20 cm = 0.2m
I = 30 A, V =200 volts, N =950 r.p.m
Applied Torque τ (Newton meters ) = Net Force in (Newtons) x Diameter of the
drum (mtrs)

=(W2 – W1) × 9.81 × R


= (35 -10) × 9.81 × 0.2 = 49.05 N-m
2𝜋 𝑁 𝛕 2𝜋 ×950×49.05
Pout = τ (Nw.mtrs.) x ω (Rad/sec) =
60
= 60
= 4879.67 W = 4.87 kW
Pin = V. I =(200) (30) =6000 W =6 kW
𝑷𝒐𝒖𝒕 4.87
% Efficiency of motor = × 100 = × 100
𝑷𝒊𝒏 6
= 81.32%
∴ Power output of DC shunt motor = 4.87 kW
% Efficiency of DC shunt motor = 81.32

Example 5: A retardation test is carried out on a 1000 r.p.m. d.c. machine. The
time taken for the speed to fall from 1030 r.p.m. to 970 r.p.m. is

i) 36 seconds with no excitation

ii) 15 seconds with full excitation


iii) 9 seconds with full excitation and the armature supporting an extra load of
10 A at 219 V.
Calculate : 1) The moment of inertia of the armature in kg.m2

Page 34
Malla Reddy College of Engineering and Technology
Department of EEE (2022-23)
Electrical Machines -1 (EM-1): Lecture Notes: (Dr.Karimulla P Sk)
Unit 3: Testing of D C Machines

2) Iron loss
3) Mechanical loss at the mean speed of 1000 r.p.m.

Solution: It is to be remembered that,


1. When armature is slowing down and there is no excitation, then kinetic
energy is used to overcome mechanical losses only. Iron losses are absent as
excitation is absent.
2. When excitation is given, kinetic energy is used to overcome both mechanical
as well as iron losses. Total called stray losses.
3. If moment of inertia is in kg-m2, then loss of energy is in watts.
The mechanical losses are,
𝑑𝑁
W = {2π}2 I . N
m 𝑑𝑡 1
60
The total stray losses are,
𝑑𝑁
W = {2π}2 I . N
s 𝑑𝑡 3
60
Now dN = 1030 -970 = 60 r.p.m.
t1 = 36 sec, t2 = 15 sec, t3 = 9 sec
The additional loss W’ due to additional load,
W’ = VI = 219×10 = 2190 W
𝑡3 9
Ws = W’ × 𝑡2−𝑡3 =2190 × 15−9 =3285 W
𝑑𝑁
3285 = {2π}2 I . N
60 𝑑𝑡 2
60
i.e. 3285 = {2π}2 × I × 1000 ×
60 15
I = Moment of inertia = (3285x602x15)/(2π)2x1000x60
= 74.88 kg-m2
𝑊𝑚 𝑡2 15
= = 36
Ws 𝑡1
15 15
Wm = Ws × 36 = 3285 × 36
= 1368.75 W

Page 35
Malla Reddy College of Engineering and Technology
Department of EEE (2022-23)
Electrical Machines -1 (EM-1): Lecture Notes: (Dr.Karimulla P Sk)
Unit 3: Testing of D C Machines

Iron losses = Ws – Wm = 3285 – 168.75 = 1916.85 W

Example 6: A Retardation test is made on a separately excited d.c. machine as a


motor. The induced voltage falls from 220 V to 190 V in 30 seconds on opening
the armature circuits and 5 seconds on suddenly changing the armature
connection from supply to a load resistance taking 12 A (average). Find the
efficiency of the machine when running as a motor and taking a current of 22 A
on a supply of 250 V. The resistance of its armature is 0.5 Ω and that of its field
winding is 250 ohms.

220+190
Solution : Average voltage across load = = 205 V
2
Iav = 12 A, Armature resistance Ra = 0.5 Ω, Field resistance Rf =
250 Ω
∴ Power absorbed by the load resistance ,
W’ = (205) (12) = 2460 W
t1 = 30 sec, t2 = 5 sec
Let W = Stray losses
W t2 5 5
We have, = = = = 0.2
W′ t1−t2 30−5 25
∴ Stray losses W = 2460×0.2 = 492 W
V 250
Input current = 22 A, If = Rf =250 = 1 A
Armature current Ia = Input current – If = 22-1 =21 A
Armature copper loss = Ia2 x Ra = (21)2 (0.5) = 220.5 W
Shunt field copper loss = Vs x If = (250) (1) = 250 W
Total losses in the machine = Stray losses + Armature copper loss + Shunt field
copper loss

= 492 + 220.5 + 250 = 962.5 W


Motor input = Vs x IL = (250) (22) =5500 W

Page 36
Malla Reddy College of Engineering and Technology
Department of EEE (2022-23)
Electrical Machines -1 (EM-1): Lecture Notes: (Dr.Karimulla P Sk)
Unit 3: Testing of D C Machines

Motor output = Motor input – Losses = 5500 – 962.5


= 4537.5 W
% Efficiency of the machine running as a motor ,
Motor output
ηm = Motor input
×100
4537 .5
% ηm (Running as motor) = ×100 = 82.5%
5500

Example 7: The Hopkinson’s test on two shunt machines gave the following
results for full load conditions. Line voltage 250 volts, line current excluding field
current is 50 A, motor armature current 380 A, field currents for generator and
motor are 5 A, 4.2 A respectively. Draw the circuit diagram with the above data
and Calculate the efficiency of each machine. Armature resistance of each
machine is 0.02Ω

Solution : The connections and current distribution based on the given data are
shown in the figure below.

Armature copper loss for generator = I a 2 Ra = (330)2 × 0.02 =


2178 W
Armature copper loss for motor = Ia2 Ra = (380)2 × 0.02 =
2888 W
Power drawn loss from supply = VI =250 × 50 = 12500 W
Stray losses for both machines = VI – Total armature

Page 37
Malla Reddy College of Engineering and Technology
Department of EEE (2022-23)
Electrical Machines -1 (EM-1): Lecture Notes: (Dr.Karimulla P Sk)
Unit 3: Testing of D C Machines

copper loss
= 12500 – 2178 – 2888 =
7434 W
7434
Stray loss for each = = 3717 W
2
For motor : Field copper loss = V If = 250 × 4.2 = 1050 W
Total loss = 2888 +3717 + 1050 =
7655 W
Motor input = V× Ia + V × If = 250 × 380 +250 × 4.2
= 96050 W
Motor output = Motor input - Total loss
= 88395 W
88395
% ηm = 96050 × 1000 = 92.03 %
For generator : Field copper loss = V If = 250 × 5 = 1250 W
Total loss = 2178 + 3717 + 1250 =
7145 W
Generator input = Generator output + Total
loss
Generator output = V Ia = 250 × 330 = 82500
W
Generator input = 82500 + 7145 = 89645 W
82500
% ηm = 89645 × 100 = 92.02 %

Example 8: The Hopkinson’s test on two similar machines gave the following
full load results: Line current = 48 A, Line voltage = 110 V, Motor armature
current = 230 A
the field currents of motor and generator are 3 A and 3.5 A respectively.
Armature resistance of each machine is 0.035 ohms. Draw the circuit diagram
with the above data and Calculate the efficiency of each machine assuming a
brush contact drop of 1 V per brush.

Page 38
Malla Reddy College of Engineering and Technology
Department of EEE (2022-23)
Electrical Machines -1 (EM-1): Lecture Notes: (Dr.Karimulla P Sk)
Unit 3: Testing of D C Machines

Solution: The connections and current distribution based on the given data are
shown in the figure below.
Brush contact loss for motor = 230 × 2 = 460 W
Brush contact loss for generator = 188.5 × 2 = 377 W
Motor armature copper loss = Ia2 Ra = (230)2 (0.035) =
1851.5 W
Generator armature copper loss = Ia2 Ra = (188.5)2 (0.035) =
1243.62 w
Motor field copper loss = V If = (110)(3) = 330 W
Generator field copper loss = V If (110)(3.5) = 385 W
Motor and generator field copper loss = 330 + 385 = 715 W
Motor and generator armature copper loss = 1851.5 + 1243.62 = 3095.12
W
Total supply power taken = (110)(48) = 5280 W

Stray losses for both the machines =


5280 – 715 – 3095.12 – 460 – 377 = 632.88 W
632.88
Stray losses for each machine = = 316.44 W
2
Motor efficiency :
Motor armature copper loss = 1851.5 W, Motor field copper
loss = 330 W
Stray loss = 316.44 W; Brush loss = 460
Total motor losses = 1851.5 + 330 + 316.44 + 460 =
2957.94 W
Motor input = (110×3) + (110×230) = 25630
W

Page 39
Malla Reddy College of Engineering and Technology
Department of EEE (2022-23)
Electrical Machines -1 (EM-1): Lecture Notes: (Dr.Karimulla P Sk)
Unit 3: Testing of D C Machines

Motor output = Motor input – Total losses = 25630 – 2957.94


= 22672.06 W
Machine output 22672 .06
Motor efficiency : ηm = ×100 = ×
Manchine input 25630
100 = 88.45 %
Generator armature copper loss = 1243.62 W, Generator field
copper loss = 385 W
Stray losses = 316.44 W, Brush contact loss for
generator = 377 W
Total generator losses = 1243.62 + 385 + 316.44 +377 =
2322.06 W
Generator = Generator output + Total
generator losses
= (110)(185) + 2322.06 = 22672.06
W
Generator output (110)(185)
Generator efficiency, ηg = Generator input × 100 = 22672 .06 ×
100 = 89.75 %

Example 9: Two shunt machines loaded for the Hopkinson’s test take 15 A at
200 V from the supply. The motor current is 100 A and the shunt currents are 3
A and 2.5 A for the generator and the motor respectively. The resistance of
each armature is 0.05 Ω. Draw the circuit diagram with the above data and
calculate the efficiency of each machine for its particular conditions of loading.

Solution: The connections and current distribution are shown in the figure
below.

Page 40
Malla Reddy College of Engineering and Technology
Department of EEE (2022-23)
Electrical Machines -1 (EM-1): Lecture Notes: (Dr.Karimulla P Sk)
Unit 3: Testing of D C Machines

For generator :
IL = 15 A , Ia = 88 A, If = 3 A, Ra = 0.05 Ω, Vs =
200 V
∴ Armature copper loss = I2 a R a = 387.2 W
∴ Field copper loss = If V = 600 W
For motor : Ia = 97.5 A, If = 2.5 A, Ra = 0.05 Ω, Vs = 200 V
∴ Armature copper loss = I2 a R a = 475.3125 W
∴ Field copper loss = If V = 2.5x200 = 500 W
Power drawn from supply = VI = 200 × 15 = 3000 W
Total copper losses for set = 387.2 + 600 + 475.3125 + 500 = 1962.51
W
∴ Stray losses for both machine = 3000 – 1962.51 = 1037.48 W
1037 .48
∴ Stray losses for each machine = = 518.74 W
2
Efficiency of motor : Pin = VI = 200 × 100 = 20000 W
Total losses = 518.74 + 500 + 475.3125 = 1494.0525
W
Pin −Losses 20000 −1494.0525
∴ %η= × 100 = ×
Pin 20000
100 = 92.53 %
Efficiency of generator : Motor output = Generator input
Pin (gen) = 20000 – 1494.0525 = 18505.9475 W
Total losses = 518.74 + 387.2 + 600 = 1505.94 W

Page 41
Malla Reddy College of Engineering and Technology
Department of EEE (2022-23)
Electrical Machines -1 (EM-1): Lecture Notes: (Dr.Karimulla P Sk)
Unit 3: Testing of D C Machines

Pin −Losses 18505 .9475−1505 .94


∴ %η= × 100 = × 100
Pin 18505 .9475
= 91.86 %
Example 10: A Field’s test on two mechanically coupled similar motors with
their fields connected in series and with one machine running as motor and the
other as a generator gave the following data :
Motor : Armature current 40 A, armature voltage 200 V the drop across its
field winding 15 V.
Generator : Armature current 32 A, armature voltage 160 V, the drop across its
field winding 15 V.
The resistance of each armature is 0.4 Ω. Draw the circuit diagram with the
above data and calculate the efficiency of each machine at this load.

Solution: The circuit diagram with the above data is shown in the figure below.

I1 = Motor input current = 40 A


V1 = Voltage across motor armature = 200 V
I2 = Load current = 32 A
V2 = voltage across generator = 160 V
Ra, Rf = Armature and series field resistance of each machine
Total input voltage to Motor (excluding that of Generator field) = V = 200 + 15 =
215 V

Page 42
Malla Reddy College of Engineering and Technology
Department of EEE (2022-23)
Electrical Machines -1 (EM-1): Lecture Notes: (Dr.Karimulla P Sk)
Unit 3: Testing of D C Machines

Total input power to motor = V. I1 = 215 × 40 = 8600 W


Generator output = V2. I2 = 160 × 32 = 5120 W
Total losses in the two machines = input – output = 8600 – 5120 = 3480 W
Series field resistance for both machines,
15
Rf = (V/I1) = 40
= 0.375 Ω
Total Cu loss = (Ra + 2 Rf) I2 1+ I2 2Ra
= (0.4 + 2 × 0.375) (40)2 + (32)2 (0.4)
= 1840 + 409.6 = 2249.6 W
Stray losses of the two machines = Total losses – Total Cu loss
= 3480 – 2249.6 = 1230.4 W
1230 .4
Stray losses for each machine = Total stray Losses / 2=
2
= 615.2 W
For Motor :
Motor input = V . I1 = 200 × 40 = 8000 W
Armature and field Cu loss = (Ra + Rf) I 12 = (0.4 + 0.375) (40)2
= 1240 W
Total losses = Armature and field Cu loss + Stray losses
= 1240 + 615.2 = 1855.2 W
Motor output = Motor input – Total losses
= 8000- 1855.2 = 6144.8 W
Motor output 6144 .8
Motor efficiency = Motor input
× 100 = × 100
8000
=76.81%
For Generator :
Armature Cu loss = I22 Ra = (32)2 (0.4) = 409.6 W
Series field Cu loss = V. I1 = 15 × 40 = 600 W
Total losses = Armature Cu loss + Series field Cu loss + Stray loss
= 409.6 + 600 + 615.2 = 1624.8 W
Output of generator = 5120 W

Page 43
Malla Reddy College of Engineering and Technology
Department of EEE (2022-23)
Electrical Machines -1 (EM-1): Lecture Notes: (Dr.Karimulla P Sk)
Unit 3: Testing of D C Machines

Input to generator = Output + Losses = 5120 + 1624.8


= 6744.8 W
output 5120
Efficiency of generator = × 100 = × 100 = 75.91 %
input 6744 .8
Example 11: The total iron loss in a d.c. machine is 7 kW at the rated speed
and excitation. With the same excitation and speed reduced by 30 % the total
iron losses were found to be 4 kW. Calculate the hysteresis and eddy current
losses at i) Full speed ii) Half of full speed
Solution: Hysteresis loss Ph = AN, Eddy current loss Pe = BN2
And, Pi = Ph + Pe = AN + BN2
Let the rated speed N = 1 then,
Pi = A + B = 7 kW (given)
At new speed N’ = N – 30% of N = 1 – 0.3 = 0.7,
Pi = 0.7A + (0.7)2 B = 4 kW
i.e. 0.7A + 0.49B = 4 kW
Solving equations (1) and (2),
A = 2.7143, B = 4.2857
i) At full speed,
Ph = A = 2.7143 kW and Pe = B = 4.2857 kW
ii) At half of full speed,
Ph = A × (0.5)1 = 1.3515 kW
and Pe = B × (0.5)2 = 1.07142 kW

Example 12: When running on no load, a 400 V shunt motor takes 5 A.


Armature resistance is 0.5 Ω and field resistance 200 Ω. Find the output of the
motor and efficiency when running on full load and taking a current of 50 A. Also,
find the percentage change in speed from no load to full load.

Page 44
Malla Reddy College of Engineering and Technology
Department of EEE (2022-23)
Electrical Machines -1 (EM-1): Lecture Notes: (Dr.Karimulla P Sk)
Unit 3: Testing of D C Machines

Solution : V = 400 V, INL = 5 A, Ra = 0.5 Ω, Rf = 200 Ω, IFL = 50 A


V 400
If = Rsh = 200 = 2 A
IaNL = INL – If = 5 – 2 = 3 A
Ebnl =V - IaNL Ra = 400 – 3 ×0.5 = 398.5 V
No load power input = Total losses
= V × INL = 400 × 5 = 2000 W
∴ Constant losses = 2000 – I2anl Ra = 2000 -(3)2 × 0.5
= 1995.5 W
On full load, IaFL = IFL - Ish = 50 – 2 = 48 A
∴ Armature copper losses = IaFL2 Ra
EbFL = V - IaFL Ra = 400 – 48 × 0.5 = 376 V
Total losses = Constant losses + Armature copper losses
=1995.5 + 1152 = 3147.5 W
Pin (FL) = V IFL = 400 × 50 = 20000 W
∴ Pout (FL) =Pin (FL) – Total losses = 20000 – 3147.5
= 16852.5 W
∴ % η = Pout (FL) × 100 = 16852 .5 × 100 = 84.26 %
FL
Pin (FL) 20000
Eb
N𝖺 𝖺 Eb (Since Ø is constant )
Ø
398.5
∴ Nnl /Nfl = Ebnl / EbFL = = 1.05984
376
i.e. Nnl = 1.05984 NFL
∴ % Change is speed = [(Nnl - NFL)/ NFL ]× 100
1.05984 NFL −NFL
= NFL
× 100
= 5.984 %

Page 45
Malla Reddy College of Engineering and Technology
Department of EEE (2022-23)
Electrical Machines -1 (EM-1): Lecture Notes: (Dr.Karimulla P Sk)
Unit 4: Single Phase Transformers

UNIT – IV
SINGLE PHASE TRANSFORMERS

CONTENTS:

 Principle of operation
 Constructional features -Types
 Minimization of hysteresis and eddy current losses
 EMF equation Operation on no load and on load
 Phasor diagrams
 Equivalent circuit Losses and efficiency
 Regulation
 All day efficiency
 Effect of variations of frequency & supply voltage on iron losses.
 Illustrative Problems

Page 1
Malla Reddy College of Engineering and Technology
Department of EEE (2022 -23)
Electrical Machines -1 (EM-1): Lecture Notes: (Dr.Karimulla P Sk)
Unit 4: Single Phase Transformers

Introduction:

A transformer is a device used to transfer Electric power from one circuit at a


certain voltage level to another circuit at a different voltage level. The electrical
power is transferred by Magnetic Induction between two coils in the magnetic
circuit of the transformer. It is electrical equipment which has the highest
efficiency since there are no moving parts. Transformers can carry only AC
Electrical power. Transformers are available in single phase and three phases.
We will study single phase transformers in this unit.

Types and applications of transformers:


1. Step up/Unit transformers – Usually located at the output of a generator in
Generating stations. Its function is to step up the voltage level to transmit
power with minimum losses.
2. Step down/Substation transformers – Located at main distribution or
secondary level transmission substations. Its function is to lower the
voltage levels for first level distribution
3. Distribution Transformers – located at small distribution substation. It
lowers the voltage levels
for second level distribution purposes.
4. Special Purpose Transformers - E.g. Potential Transformer (PT) , Current
Transformer (CT) etc which are used for metering applications.

Principle of operation of Transformers:

In a generator a voltage is induced in a coil which moves past a stationary


magnetic field emanating from field coils and the field flux is constant. But since
the flux that links with the coil is changing, a voltage is induced in the coil which is
proportional to the rate of change of flux linkage. In a transformer though both
the coils and the magnetic circuit are stationary, a voltage is induced in the
secondary since the current which flows in the primary is alternating and it
produces a continuously changing flux. According to Faraday’s laws of
electromagnetic Induction when the magnetic flux linking with a conductor (or
Coil) changes, an EMF is induced in that.
Page 2
Malla Reddy College of Engineering and Technology
Department of EEE (2022 -23)
Electrical Machines -1 (EM-1): Lecture Notes: (Dr.Karimulla P Sk)
Unit 4: Single Phase Transformers

In a single phase transformer there are two coils one known as primary and the
other known as secondary both wound on he same magnetic circuit (core)of high
permeability (or low reluctance) as shown in the figure below.

Fig: Single Phase Transformer

An alternating Voltage is applied to the primary coil. It sets up an alternating


current in the primary coil and hence an alternating flux in the magnetic circuit
(Core).This magnetic flux links with the secondary coil and induces in it an
alternating voltage due to the changing flux linkage. The EMF induced in the
secondary coil will be supplied to the load.

Ideal Transformer:

An ideal transformer has no losses i.e. no I2R losses (its windings have no
resistance) and no core losses. In other words it consists of two pure inductive
coils and a lossless core. But practically it is not possible to realize such an Ideal
transformer.
In an Ideal transformer the no load current will be lagging the applied voltage by
900 since the current flows through a purely inductive primary coil. The flux Ø
induced in the magnetic circuit also will be lagging the applied voltage V1 since
the current in the primary is entirely a magnetizing current (the coil is assumed to
be totally inductive) and will be in phase with the magnetic flux. The voltages
induced in the primary and secondary coils E1 & E2 will be lagging the flux by 900

Page 3
Malla Reddy College of Engineering and Technology
Department of EEE (2022 -23)
Electrical Machines -1 (EM-1): Lecture Notes: (Dr.Karimulla P Sk)
Unit 4: Single Phase Transformers

and in phase opposition to the applied Voltage V1. All these relations are clearly
shown in the No load phasor diagram given below.

Fig: No load Phasor Diagram of an Ideal Single phase Transformer

Types and constructional features:


The essential parts of a Transformer are :

 Coils and the Laminated core


 Tank or container to house the coils & the core
 Suitable medium called the transformer oil to insulate the core and the
windings from the tank
 Bushings (porcelain) for insulating and bringing out the coil terminals.

There are two types of core construction called core type and shell type and are
shown in the figure below.
 In Core Form the transformer windings surround the two sides of a
rectangular core.
 In Shell Form a three legged laminated core is used with the windings
wrapped around the centre leg.

Page 4
Malla Reddy College of Engineering and Technology
Department of EEE (2022 -23)
Electrical Machines -1 (EM-1): Lecture Notes: (Dr.Karimulla P Sk)
Unit 4: Single Phase Transformers

Fig: Core type transformer Fig: Shell type


transformer

The core is constructed of laminations to minimize the eddy current losses. The
laminations are made of thin sheet steel with high silicon content to produce high
permeability and a low hysteresis loss. The laminations are insulated from each
other by a coat of varnish. The laminations are cut in the form of strips of shape L,
E & I called core stampings as shown in the figure below and then stacked
together to get the Core and shell type cores.

Page 5
Malla Reddy College of Engineering and Technology
Department of EEE (2022 -23)
Electrical Machines -1 (EM-1): Lecture Notes: (Dr.Karimulla P Sk)
Unit 4: Single Phase Transformers

Fig: Types of core stampings

EMF Equation of a Transformer:

 Number of turns in the primary and secondary windings : N1 and N2


 Maximum value of flux in the core : Øm
 Frequency : f
 Time for change of Flux in one cycle : 1/f
 Time for change of flux from zero to Øm : 1/4f
 Hence rate of change of flux : Øm/1/4f
 Average e.m.f. induced in each turn : 4f Øm
 Form factor for a sine wave : 1.11
 RMS value of voltage in a turn : 1.11 x 4f
Øm = 4.44 f Øm
 RMS value of voltage induced in the primary : 4.44 f Øm
N1
 RMS value of voltage induced in the secondary : 4.44 f Øm
N2

It is to be noted that when a sinusoidal voltage V1 is applied to the primary


winding leading to the flow of current and thus generation of flux in the core,
voltages are induced in both the primary and secondary windings as given by the
equations above. Voltage E1 & E2 are opposite in polarity to V1 and the voltage E2
gives rise to the load current when the secondary is connected to a load.
We get the relation between the two voltages by dividing the expressions for E1
with that of E2 as
Page 6
Malla Reddy College of Engineering and Technology
Department of EEE (2022 -23)
Electrical Machines -1 (EM-1): Lecture Notes: (Dr.Karimulla P Sk)
Unit 4: Single Phase Transformers

E1/E2 = N1/N2
And this ratio is called the turns ratio ‘a’ or ‘k’ of the transformer.
Since the primary VoltAmp (VA) is nearly equal to that of the secondary due to
the high efficiency of transformers we get the relation : E1 I1 = E2I2 i.e I1/I2
= E2/E1 = N2/N1 = 1/a or 1/k

Dot Convention:
Given that the primary circuit's voltage is positive at a specific end of the coil,
what would the polarity of the secondary circuit's voltage be? In real
transformers, it would be possible to tell the secondary 's polarity only if the
transformer were opened and its windings examined. To avoid this necessity,
transformers utilize the dot convention. The dots appearing at one end of each
winding in the figure tell the polarity of the voltage and current on the secondary
side of the transformer. The relationship is as follows:
I. If the primary voltage is positive at the dotted end of the winding with respect
to the undotted end, then the secondary voltage will be positive at the dotted
end also. Voltage polarities are the same with respect to the dots on each side of
the core.
2. If the primary current of the transformer flows into the dotted end of the
primary winding, the secondary current will flow out of the dotted end of the
secondary winding.

A practical Transformer on no load:

When the primary winding of a transformer is energized with an AC source of


power and the secondary is left open without connecting to any load, the
transformer is said to be on No load.

Page 7
Malla Reddy College of Engineering and Technology
Department of EEE (2022 -23)
Electrical Machines -1 (EM-1): Lecture Notes: (Dr.Karimulla P Sk)
Unit 4: Single Phase Transformers

Fig: Phasor diagram of a practical single phase transformer on No Load

In a practical Transformer the primary winding cannot be a pure reactance and it


would have some resistance also. Then the primary no-load current I0 would lag
behind the applied voltage by an angle α which is slightly less than 900 and it can
be resolved into the following two components as shown in the Phasor diagram .

 Iμ = I0 Sin α along the direction of flux called the magnetizing current and
is responsible for the generation of flux Ø in the core of the transformer
 Iw = I0 Cos α along the direction of the primary voltage V1 called the
working (active)component and is responsible to cover the no load losses (
Hysteresis, eddy current and small resistive)or core losses

In this diagram the applied voltage V1 and the flux Ø will have a phase difference
of 900 and are taken as reference.

The no load losses are given by: PNL = V1.IW

E1 and E2 are the induced voltages in the primary and secondary and both lag
behind the flux by 900 since the induced voltage is equal to the rate of change of
flux linkages.

Page 8
Malla Reddy College of Engineering and Technology
Department of EEE (2022 -23)
Electrical Machines -1 (EM-1): Lecture Notes: (Dr.Karimulla P Sk)
Unit 4: Single Phase Transformers

Equivalent circuit of a real transformer:

The equivalent circuit of a transformer is a circuit which will take into account all
the major imperfections in a practical transformer and are modeled appropriately
as equivalent resistors and inductors as explained below.

To develop the equivalent circuit of a real transformer, the following losses have
to be taken into account in order to accurately model the transformer into its
equivalent circuit:

 Copper (I2R) Losses – Resistive heating losses in the primary and secondary
windings of the transformer.

They are modeled by placing a resistor R P in the primary circuit and a resistor R S
in the secondary circuit.

 Eddy current Losses – Resistive heating losses in the core of the


transformer.
 Hysteresis Losses – These are associated with the rearrangement of the
magnetic domains in the core during each half-cycle.

Both theses losses produce a core loss current IH+E or ICL (Iw) which is a current
proportional to the voltage applied to the core. Since this is in phase with the
applied voltage this loss is modeled as a resistance RC across the primary voltage
source.

 Leakage flux – The fluxes ØLP and ØLS which escape the core and pass
through only one of the transformer windings are called leakage fluxes.
They then produce self-inductances in the primary and secondary coils.

They are modeled as equivalent Inductive reactances XP and XS in the primary and
secondary circuits in series with the resistors RP and RS .

Page 9
Malla Reddy College of Engineering and Technology
Department of EEE (2022 -23)
Electrical Machines -1 (EM-1): Lecture Notes: (Dr.Karimulla P Sk)
Unit 4: Single Phase Transformers

Apart from the above losses the transformer draws a magnetizing current I μ (or
Im) and since this current lags behind the applied voltage by 900 it is modeled as a
reactance XM

Complete Equivalent circuit of a Practical Transformer incorporating all the above


aspects is shown in the figure below.

Fig: Complete Equivalent circuit of a Practical Transformer

For ease of circuit analysis and mathematical calculation this complete equivalent
circuit is simplified by referring the impedances in the secondary to the primary
and vice versa as shown below.

Page 10
Malla Reddy College of Engineering and Technology
Department of EEE (2022 -23)
Electrical Machines -1 (EM-1): Lecture Notes: (Dr.Karimulla P Sk)
Unit 4: Single Phase Transformers

Fig: Equivalent circuit of a Transformer (a) referring to the primary (b) referring
to the secondary

 When the Secondary is referred to the Primary the Secondary side


parameters referred to the Primary will be come: a.VS , IS/a , a2. RS
and a2. XS

 When the Primary is referred to the Secondary the Primary side


parameters referred to the Secondary will be come: Vp/a , a.Ip , Rp /
a2 and . Xp/ a2

Approximate Equivalent circuit of a Transformer:

The derived equivalent circuit is detailed but it is complex for practical


engineering applications. The main problem in analysis & calculations is

Page 11
Malla Reddy College of Engineering and Technology
Department of EEE (2022 -23)
Electrical Machines -1 (EM-1): Lecture Notes: (Dr.Karimulla P Sk)
Unit 4: Single Phase Transformers

representation of the excitation and the eddy current & hysteresis losses which
add an extra branch.

In practical situations, the excitation current Iμ & the core loss current ICL will be
relatively small as compared to the load current, which makes the resultant
voltage drop across Rp and Xp to be very small. Hence Rp and Xp may be lumped
together with the secondary referred impedances to form an equivalent
impedance. In some cases, the excitation current Iμ & the core loss current ICL are
neglected entirely due to their small magnitude.

The equivalent circuits with these simplifications are shown in the figure below.

Fig: Simplified equivalent circuits (a) Referred to the primary side (b) Referred to
the secondary side

Page 12
Malla Reddy College of Engineering and Technology
Department of EEE (2022 -23)
Electrical Machines -1 (EM-1): Lecture Notes: (Dr.Karimulla P Sk)
Unit 4: Single Phase Transformers

(c) With no excitation branch, referred to the primary side (d) With no excitation
branch, referred to the secondary side

Voltage Regulation of a Transformer:

The output voltage of a transformer varies with the load even if the input voltage
remains constant. This is because of the voltage drop in it’s series impedance as
seen in the equivalent circuit. It is normally defined for Full load as a percentage
as given below:

% Voltage Regulation (%VR) = [ (VS NL – VS FL )/ VS NL ] . 100 %

The transformer phasor diagram:

To determine the voltage regulation of a transformer, we must understand the


voltage drops within the transformer and they can be seen easily with the help of
the Phasor diagram ON LOAD .

To develop the Phasor diagram on load let us consider once again the simplified
equivalent circuit referred to the secondary side:

Page 13
Malla Reddy College of Engineering and Technology
Department of EEE (2022 -23)
Electrical Machines -1 (EM-1): Lecture Notes: (Dr.Karimulla P Sk)
Unit 4: Single Phase Transformers

Fig: Simplified equivalent circuit Referred to the secondary side

Ignoring the excitation branch (since the current flow through the branch is
considered to be small), Series impedances (Req + jXeq) will be more predominant.
Hence Voltage Regulation depends on the magnitude of the series impedance and
the phase angle of the load current flowing through the secondary of the
transformer with respect to the secondary voltage.

Phasor diagram will represent the effects of these factors on the voltage
regulation. A phasor diagram consists of current and voltage vectors. In this
Phasor diagram the secondary voltage VS is assumed as the reference.

Applying Kirchoff Voltage Law to this simplified equivalent circuit we get the
relation :

VP/a = VS + IS . Reqs + IS . Xeqs

From this equation, the phasor diagram can be visualized and is shown below for
a transformer operating at a lagging power factor. For lagging loads, VP / a > VS
so the voltage regulation with lagging loads is > 0.
From the Phasor diagram it can be seen that the addition is Vector addition and
not an algebraic addition.

Fig: Phasor diagram of a Transformer on load with lagging power factor

Page 14
Malla Reddy College of Engineering and Technology
Department of EEE (2022 -23)
Electrical Machines -1 (EM-1): Lecture Notes: (Dr.Karimulla P Sk)
Unit 4: Single Phase Transformers

When the power factor is unity also VS is lower than VP and so VR > 0. But VR is
smaller than before (during lagging PF) as shown in figure (a) below .

With a leading power factor , VS is higher than the referred VP (VP /a) so VR < 0 as
shown in figure(b)
below .

Fig: Phasor diagram of a Transformer on load with (a) unity power factor and (b)
leading power factor

Losses:

As we know a transformer is a device used to transfer Electric power from one


circuit at a certain voltage level to another circuit at a different voltage level. In
this process of conversion some power is lost. The difference between the input
power and the output power is termed as the Power loss or Losses .
Constant & Variable losses:

Page 15
Malla Reddy College of Engineering and Technology
Department of EEE (2022 -23)
Electrical Machines -1 (EM-1): Lecture Notes: (Dr.Karimulla P Sk)
Unit 4: Single Phase Transformers

The losses are broadly classified as constant losses and variable losses. Constant
losses are constant and are independent of the load where as the variable losses
are dependent on the load. They are further classified in detail as below.

Detailed Classification of Losses:

1. Electrical or Copper Losses (I2R Loss): Current flow through the resistance of
the transformer coils give rise to I2R losses and since the coils are normally
made up of copper these losses are called Copper losses.
Primary copper loss: Pp = I p2 R p
Secondary copper loss: Ps = Is2 Rs

2. Core Losses: Hysteresis and eddy current losses occurring in the transformer
core are together called core losses.
3.
 Hysteresis loss: in an iron core is the loss of power due to the hysteresis
loop in the magnetization characteristic of the core in each cycle of the
alternating current applied to the core. It is proportional to the area of the
Hysteresis loop and is given by :

Hysteresis loss = Kh B m 1.67 f v watts


Where Kh = Hysteresis constant and it depends on the core material
Bm = Maximum flux density in the core
f = Frequency of operation
v = Volume of the core.

 Eddy current losses: A time-changing flux induces voltage within a


ferromagnetic core in just the same manner as it induces voltage in the
coils around the core of the transformer. These voltages cause swirls of
current to flow within the core, much like the eddies seen at the edges of a
river. It is the shape of these currents that gives rise to the name eddy
currents. These eddy currents flowing in a resistive material (the iron of the
core) cause power loss thus heating the iron core and the resulting loss is

Page 16
Malla Reddy College of Engineering and Technology
Department of EEE (2022 -23)
Electrical Machines -1 (EM-1): Lecture Notes: (Dr.Karimulla P Sk)
Unit 4: Single Phase Transformers

called eddy current loss. This loss is proportional to the thickness of the
core material and hence to minimize this loss the core is made up of thin
sheets called laminations instead of a single thick block. An insulating oxide
or resin is used between the strips so that the current paths for eddy
currents are limited to very small areas. Thus the eddy current losses have a
very little effect on the core's magnetic properties.
The eddy current loss is given by:
Eddy current loss = Ke B2mf2 t2 watts/ unit volume
Where Ke = Eddy current constant
t = Thickness of the core

Out of the above, Core Losses are also called as Constant losses since the flux in
the core is almost constant as supply voltage V 1 at rated frequency f is always
constant at all the loads and thus the flux density Bm is also constant on which
both hysteresis and eddy current losses are dependent . Electrical or Copper
Losses are also called as variable losses.

It is to be noted that there are no mechanical losses in transformers because there


are no moving parts.

Minimization of hysteresis and eddy current losses:

 Hysteresis losses are minimized by using high grade core material like
silicon steel having very high permeability and low hysteresis loop.
 Eddy current loss is proportional to the thickness of the core material and
hence this loss is minimized by making up the core with thin sheets called
laminations instead of a single thick block. An insulating oxide or resin is
used between the strips so that the current paths for eddy currents are
limited to very small areas.

Effect of variations of frequency and supply voltage on iron losses:


Page 17
Malla Reddy College of Engineering and Technology
Department of EEE (2022 -23)
Electrical Machines -1 (EM-1): Lecture Notes: (Dr.Karimulla P Sk)
Unit 4: Single Phase Transformers

We know that the iron losses in a transformer include two types of losses i.e.
Hysteresis losses & Eddy current losses and are given by the relations:

Hysteresis loss = Kh B m 1.67 f v watts and -----


(1) Eddy current loss
= Ke B2m f2 t2 watts/ unit volume ----- (2)
It is also known that for a transformer the voltage V is given by :
V = 4.44 f Øm N = 4.44 f Bm A N
From which we can see that for constant A and N
𝑉
Bm α -----
𝐹
(3)

From the above three equations we can see that:

 As voltage changes (with frequency remaining constant), the maximum flux


density changes and thus both eddy current and hysteresis losses also
change. As voltage increases, the maximum flux density in the core
increases and total iron losses increase.
 As frequency increases, as per equation (3) the flux density in the core
decreases but the iron loss increases as per equations (1) and (2).Hence the
net effect depends on the relative effect of flux density Bm and frequency f.
A closer look at equations (1) and (2) then reveals that :
 In Hysteresis losses Bm is of higher degree compared to frequency
and is more dominant. Hence the net Hysteresis losses decrease
with increase in frequency .
 In Eddy current losses Bm and f are of the same degree. Hence the
net Eddy current losses remain the same with increase or decrease
in frequency .

Volt-Ampere Rating of a Transformer:

Page 18
Malla Reddy College of Engineering and Technology
Department of EEE (2022 -23)
Electrical Machines -1 (EM-1): Lecture Notes: (Dr.Karimulla P Sk)
Unit 4: Single Phase Transformers

When electrical power is transferred from primary winding to secondary there are
few power losses in between. These power losses appear in the form of heat
which increases the temperature of the device. This temperature must be
maintained below a certain limiting value as it causes deterioration of the
insulation in the Transformer.

Hence the rating of the transformer is decided by the temperature rise which in
turn is decided by the losses.

The copper losses (I2R) in the transformer depend on the current ‘I’ through the
winding while the iron or core loss depends on the voltage ‘V’ as frequency of
operation is constant.

As losses depend on V and I only and not on the load power factor, the rating of
the transformer is specified as a product of these two parameters V × I called
Volt-Ampere Rating or VA rating ( or kVA rating for bigger machines )

On both sides of the transformer i.e. primary and secondary VA rating remains
same. This rating is more generally expressed in kVA (kilo volt amperes rating) for
large capacity machines.

Efficiency of a Transformer:

Efficiency of a Transformer is defined as (Same as defined for motors and


generators):
Efficiency η = (Pout / Pin ).100% = (Pout / Pout + Losses).100%
Losses incurred in a transformer:

 Copper losses ( I2R )


 Core losses ( Hysteresis losses and Eddy current losses)

Therefore, for a transformer, efficiency may be calculated using the following


relation :

Page 19
Malla Reddy College of Engineering and Technology
Department of EEE (2022 -23)
Electrical Machines -1 (EM-1): Lecture Notes: (Dr.Karimulla P Sk)
Unit 4: Single Phase Transformers

From the phasor diagram we know that Pout in a transformer is given by Pout =
VS.IS. Cos Ɵ. Hence

η = (VS.IS. Cos Ɵ / VS.IS. Cos Ɵ + PCU+PCORE).100%

All Day Efficiency of a Transformer:


The ordinary or commercial efficiency of a transformer is defined as the ratio of
the output power to the input power.

Transformers efficiency cannot be judged by ordinary or commercial efficiency as


the load on certain transformers like distribution transformers (which serve
residential and commercial loads) fluctuates throughout the day. Though the
distribution transformers are energized for all 24 hours, they deliver very light
loads for major portion of the day, rated or full load for minor portion of the day,
and 50 to 75% load most of the time.

But the primary of such transformers is energized at it’s rated voltage for all the
24 hours to provide continuous supply to the consumer. As we know, there are
various losses in the transformer such as iron and copper loss. The iron loss takes
place in the core of the transformer and it occurs for the whole day in the
distribution transformer. The second type of loss known as copper loss takes
place in the windings of the transformer also known as the variable loss. It occurs
only when the transformers are in the loaded condition and it’s value depends on
the load. For no load the copper losses are very small and on full load it is at the
rated value. Hence the normal power efficiency as defined earlier cannot give the
efficiency of such transformers in it’s true sense.

Hence, the performance of such transformers cannot be judged by the


commercial or ordinary efficiency, but the efficiency is calculated or judged by All

Page 20
Malla Reddy College of Engineering and Technology
Department of EEE (2022 -23)
Electrical Machines -1 (EM-1): Lecture Notes: (Dr.Karimulla P Sk)
Unit 4: Single Phase Transformers

Day Efficiency also known as operational efficiency or energy efficiency which is


computed by calculating the energy consumed during 24 hours.

Definition: All day efficiency means the power consumed by the transformer
throughout the day. It is defined as the ratio of output power to the input power
in kWh or wh of the transformer over 24 hours.

Mathematically, it is represented as:

All day efficiency of the transformer depends on their loading pattern.

Condition for Maximum Efficiency :

The load current at which the efficiency attains maximum value is denoted as I sm
and maximum efficiency is denoted as ηmax. The efficiency is a function of load i.e.
load current Is , assuming the load power factor cos ∅s and the secondary
terminal voltage Vs as constant. The condition for Maximum Efficiency can be
obtained by differentiating the expression for the efficiency with respect to IS and
then equating the differential to zero. So for maximum efficiency,
dη Vs Is cos ∅s
=0 While η =
dI s V s I s cos ∅ + P +I2s R
s i
dη d Vs Is cos se
∅s
∴ = { }=0
dI s dI s Vs Is cos ∅s + Pi +I2s Rse

Effectively we can equate the numerator of to zero.
dI s
∴ (Vs Is cos ∅s + Pi + I2sRse ) (Vs cos Øs) – (Vs Is cos ∅s ) (Vs cos Øs +
2Is Rse) = 0
Cancelling (Vs cos Øs) from both the terms we get,
Vs Is cos Ø + Pi + I2Rse - Vs Iscos Øs - 2 I2 Rse = 0 i.e. Pi - I2 Rse = 0
s s s s

Page 21
Malla Reddy College of Engineering and Technology
Department of EEE (2022 -23)
Electrical Machines -1 (EM-1): Lecture Notes: (Dr.Karimulla P Sk)
Unit 4: Single Phase Transformers

∴ Pi = I2s Rse = PCu


So the condition for maximum efficiency is :
Copper losses = Iron losses i.e. Pi = PCu

Load current Is at maximum Efficiency:

For ηmax, I2s@max .η Rse = Pi


P
i
∴ The load current @max efficiency is given by: Is@max.η = √ R
se

….

We can get Load current Is at maximum Efficiency in terms of the full load current
Is@FL (or rated current) also:
Pi Pi
Is@ max .η 1 Pi =√ =√
∴ = √
(Is )@F.L (Is )@F.L. Rse {(Is )@F.L}2 Rse (PCu )@F.L.
Pi
∴ Is@ max . η = (Is )@F. L √
(PCu )@F.L.

kVA supplied at Maximum Efficiency:

 For constant Vs the kVA supplied is a function of load current.


Pi
∴ kVA @ η max = Is@ max . η Vs = Vs (Is)@F.L × √ = (kVA ratimg) ×
(PCu )F.L.
Pi
√ (P )F.L.
Cu

Effect of Power Factor on Efficiency:

The efficiency of a transformer is given by,

Page 22
Malla Reddy College of Engineering and Technology
Department of EEE (2022 -23)
Electrical Machines -1 (EM-1): Lecture Notes: (Dr.Karimulla P Sk)
Unit 4: Single Phase Transformers

𝑂𝑢𝑡 𝑝𝑢𝑡 𝐼𝑛𝑝𝑢𝑡 −𝐿𝑜𝑠𝑠𝑒𝑠 𝐿𝑜𝑠𝑠𝑒𝑠


η = = = 1-
𝐼𝑛𝑝𝑢𝑡 𝐼𝑛𝑝𝑢𝑡 𝐼𝑛𝑝𝑢𝑡
….. (1)
Now, Input = output + Losses = Vs Is cos Ø + Losses
𝐿𝑜𝑠𝑠𝑒𝑠
Using this in equation (1), η=1- 𝑉
𝑠 𝐼𝑠 𝑐𝑜𝑠 Ø +𝐿𝑜𝑠𝑠𝑒𝑠
𝐿𝑜𝑠𝑠𝑒𝑠
𝑉𝑠 𝐼 𝑠
∴ η = 1-
𝐿𝑂𝑠𝑠𝑒𝑠
cos Ø+ 𝑉 𝑠 𝐼𝑠
𝐿𝑜𝑠𝑠𝑒𝑠
Let = x and using this in the above equation we get
𝑉𝑠 𝐼𝑠

𝑥
η = 1-
𝑥+𝐶𝑜𝑠 Ø

Thus as the power factor of the load i.e. cos Ø becomes higher, the second term
in the above equation becomes lesser and efficiency will become more.

The family of efficiency curves (as a function of load current) are obtained as
power factor increases, are shown in the figure below. From these plots we can
get the following two important observations.

 Maximum efficiency comes at a value lower than the 100% full load
current.
 As per factor increases, the efficiency increases.

Page 23
Malla Reddy College of Engineering and Technology
Department of EEE (2022 -23)
Electrical Machines -1 (EM-1): Lecture Notes: (Dr.Karimulla P Sk)
Unit 4: Single Phase Transformers

Figure : Family of efficiency curves (as a function of load current) as power


factor increases

Can D.C. Supply be used for Transformers? Explain.

The D.C. Supply cannot be used for the transformers for the following reasons.

 The transformer works on the principle of mutual induction for which, current
in one coil must change uniformly. If D.C. supply is given, the current will not
change due to constant supply and transformer will not work.
 Practically a transformer is designed with a low winding resistance to reduce
the copper losses and thus increase the efficiency. For D.C., the inductive
reactance XL is zero as D.C. frequency is zero. So total impedance of winding
is very low for D.C. Thus winding will draw higher current if D.C. supply is
given to it. This may cause the burning of windings due to extra heat
generated and may cause permanent damage to the transformer.

Page 24
Malla Reddy College of Engineering and Technology
Department of EEE (2022 -23)
Electrical Machines -1 (EM-1): Lecture Notes: (Dr.Karimulla P Sk)
Unit 4: Single Phase Transformers

Illustrative Examples:

Example 1 : A single phase 50 Hz transformer has 100 turns on the primary and
400 turns on the secondary winding. The net cross-sectional area of core is 250
cm2. If the primary winding is connected to a 230 V, 50 Hz supply, determine,
i) The e.m.f. induced in the secondary winding
ii) The maximum value of flux density in the core.
Solution: We have the data:

N1 = 100, N2 = 400, A = 250 cm2, E1 = 230 V.


N
Hence turns ratio : K = 2 = 400 = 4
N1 100
E2
Now K= i.e. E2 = KE1
E1
i) ∴ The e.m.f. induced in the secondary winding = E2 = 4 × 230 = 920 V
ii) We know that the e.m.f. induced in the secondary winding is given by E1 =
4.44 Øm f N1 where Øm is the maximum value of flux in the core.
i.e. 230 = 4.44 Øm ×50 × 100
∴ Øm = 0.01045 Wb
Øm 0.01045
∴The maximum value of flux density in the core Bm = = = 0.4182
A 250 × 10−4
Wb/m2

Example 2: A single phase transformer is connected to a 230 V, 50 Hz supply.


The net cross sectional area of the core is 50 cm2. The number of turns of the
primary is 460 and the secondary is 80. Determine i) Transformation ratio
ii) Peak value of the flux density in the core iii) e.m.f. in the secondary winding.
Solution : Refer example 1 above for the procedure and verify :
N
i) K = 2= 0.1739 ii) 0.4504 Wb/m2 iii) 40 V
N1

Page 25
Malla Reddy College of Engineering and Technology
Department of EEE (2022 -23)
Electrical Machines -1 (EM-1): Lecture Notes: (Dr.Karimulla P Sk)
Unit 4: Single Phase Transformers

Example 3: 2.4kV/115 V transformer has sinusoidal flux expressed by 0.113


sin188.5 t. Determine the primary and and secondary turns.
Solution : We have the data:
E1 = 2.4 kV, E2 = 115 V, and Ø = 0.113 sin 188.5 t Wb
Comparing Ø = 0.113 sin 188.5 t with the standard sinusoid expression Ø = Øm sin
ω t, we get
Øm = 0.113 Wb and ω = 188.5 rad/sec.
ω 188.5
∴f= = = 30 Hz
2π 2π
Induced e.m.f is given by E1 = 4.44 Øm f N1 . Substituting the above values of E1,
Øm, and f in this equation we get
2.4 ×103
N1 = 4.44 × 0.113 ×30 = 159.45 = 160
E2 N2 E2 160 ×115
We know that the turns ratio = = N1 and from which N2 = x N1=
E1 E1 2.4 ×103
= 7.666 ≈ 8

Example 4: A 6300/210 V, 50 Hz, single phase transformer has per turn e.m.f of
about 9 volts and maximum flux density of 1.2 T. Find the number of high voltage
and low voltage turns and the net cross sectional area of the core.
Solution: We have the data:
E1 = 6300 V, E2 = 210 V, f = 50 Hz, Bm = 1.2 T and E/turn = 9V
Hence E1 = (E/turn) × N1 i.e. 6300 = 9 × N1
Thus N1 = 6300/ 9 = 700
And similarly
E2 = (E/turn) × N2 i.e. 210 = 9 × N2
Thus N2 = 210 / 9 = 23.33 ≈24
But we also know that E1 is given by: E1 = 4.44 Ømf N1
i.e. 6300 = 4.44 Øm × 50 × 700
∴ Øm = 6300 /4.44 × 50 × 700 = 0.04054 Wb

Page 26
Malla Reddy College of Engineering and Technology
Department of EEE (2022 -23)
Electrical Machines -1 (EM-1): Lecture Notes: (Dr.Karimulla P Sk)
Unit 4: Single Phase Transformers

∅m
And we have Bm = a
∅m 0.04054
∴ a= = 1.2
= 0.03378 m2 = 337.8 cm2
Bm

Example 5:The e.m.f per turn for a single phase, 2310/220 volts, 50Hz
transformer is 13 volts. Calculate.
i) The number of primary and secondary turns.
ii) The net cross sectional area of the core, for a maximum flux density of 1.4 T.
Solution : Refer example 4 for the procedure and verify :
N1 = 178, N2 = 17,
a = 417.5 cm2

Example 6: A 1-phase transformer has 500 primary and 1200 secondary turns.
The net cross sectional area of the core is 75 cm2. If the primary winding be
connected to 400 V, 50 Hz supply, calculate the peak value of flux density in the
core and voltage induced in the secondary winding.
Solution : Refer example 1 above for the procedure and verify the answers
Bm = 0.4804 Wb/ m2, E2 = 960 V.

Example 7: A single-phase transformer has 400 primary and 1000 secondary


turns. The net cross-sectional area of the core is 60 cm2. If the primary winding is
connected to a 50Hz supply at 500 V. Calculate.
Solution : Refer example 1 for the procedure and verify the answer :
i) 0.938 Wb/m2, ii) 1250 V

Example 8: The no load current of a transformer is 4 A at 0.25 p.f when supplied


at 250 V, 50 Hz. The number of turns on the primary winding is 200. Calculate
i) The r.m.s value of the flux in the core ii) The core loss iii) The magnetizing
current.
Solution :
Page 27
Malla Reddy College of Engineering and Technology
Department of EEE (2022 -23)
Electrical Machines -1 (EM-1): Lecture Notes: (Dr.Karimulla P Sk)
Unit 4: Single Phase Transformers

We have the data : V1 = 250 V, f = 50 Hz, No load current I0= 4A, P.F = 0.25 and
N1 = 200
i) We know that V1 is given by : V1 = 4.44 Øm f N1 from which we get Øm =
250
4.44 50 200
m
∴ Øm = 5.63 mWb, Ø(RMS) = = 3.98 mWb
2
ii) Core loss = V1 I0 P.F = 250x 4x 0.25 = 250 W
iii) P.F = cos θ = 0.25 from which we get Sin θ = 0.9682
Then we can get magnetizing current as Im = I0 sin θ = 4 × 0.9682 =
3.873 A

Example 9: A 100 kVA, 2400/24 V, 50 Hz single phase transformer has an exciting


current of 0.64 Amps and a core loss of 700 watts, when its high voltage side is
energized at rated voltage and frequency. Calculate the two components of the
exciting current.
Solution : We have the data
No load current I0 = 0.64 A, Core losses ( or iron losses or no load losses ) Pi = 700
W, In put rating V1 = 2400 V
We know that the no load or Iron losses are given by : Pi = V1 I0 (p.f) which is =
700 Watts
From the given data then we get : 700 = 2400 × 0.64 × (p.f) from which we get
p.f = 700 /(2400 × 0.64) = 0.4557
we also know that power factor = cos θ and hence cos θ = 0.4557, and thus Sin θ
= 0.8901
The two components of exciting currents i.e core loss current Ic and
magnetizing current Im are then given by
∴ Ic = I0 cos θ = 0.64 × 0.4557
= 0.2916 A
∴ Im = I0 sin θ = 0.64 × 0.8901

Page 28
Malla Reddy College of Engineering and Technology
Department of EEE (2022 -23)
Electrical Machines -1 (EM-1): Lecture Notes: (Dr.Karimulla P Sk)
Unit 4: Single Phase Transformers

= 0.5696 A

Example 10: A2000/200 V single phase transformer gives 0.5 A and 40 W as


ammeter and wattmeter readings when supply is given to the low voltage winding
and high voltage winding is kept open. Find : i) The magnetizing component,
ii) The iron component, and iii) The power factor of no-load current.
Solution : From the given data we observe that the readings are on LV side
(secondary) and h.v. winding is kept open.
∴ I0 = 0.5 A, W0 = 40 W,
V0 = 200 V
W0 = V0I0 cos θ
40
i.e. Cos θ = = 0.4
0.5 200
Sin θ = 0.9165
i) Im = I0 Sin θ = 0.5 × 0.9165
= 0.4582 A
ii) Ic = I0 Cos θ = 0.5 × 0.4 = 0.2 A
iii) No load power factor = Cos θ = 0.4 lagging

Example 11: A single phase transformer with 10:1 turns ratio and rated at 50
kVA, 2400/240 volts, 50 Hz, is used to step down the voltage of a distribution
system. The L.V. side voltage is kept constant at 240 V. Find the value of load
impedance on L.V. side so that the transformer will be loaded fully. Find also the
maximum flux in the core if L.V. side has 23 turns.
Solution : Vs = 240 V, Vp = 2400 V, Ns = 23, 50 kVA
50 103
(Is)F.L., = VA rating/Vs = = 208.333 A
240
240
∴ Zs = Load impedance on F.L = VS/ IsFL = = 1.152 Ω
208.333
Vs ( Voltage on LV side ) = 4.44 × Øm× 50 × 23

Page 29
Malla Reddy College of Engineering and Technology
Department of EEE (2022 -23)
Electrical Machines -1 (EM-1): Lecture Notes: (Dr.Karimulla P Sk)
Unit 4: Single Phase Transformers

i.e. 240 = 4.44 × Øm × 50 × 23


From which we get Øm(Maximum flux in core ) = 240/( 4.44 × 50 × 23) = 0.047 Wb

Example 12: A 15 kVA, 2400/240 V, 60 Hz transformer has a magnetic core of 50


cm2 cross section and a mean length of 66.7 cm. The application of 2400 V causes
magnetic field intensity of 450 AT/m (RMS) and a maximum flux density of 1.5 T.
Determine i)The turn’s ratio, ii) The numbers of turns in each winding iii) The
magnetizing current.
Solution : We have the data : Vp = 2400 V, Vs = 240 V, a = 50 cm2, mean length I =
66.7 cm,
maximum flux density Bm = 1.5 T, magnetic filed intensity H = 450 AT/m
(R.M.S) with
Vp = 2400 V is applied to primary, f = 60 Hz
i) Turns ratio K =Vp/Vs = Np/Ns = 2400/240 = 10
ii) maximum flux Øm = Bm× a = 1.5 × 50 × 10 4 = 7.5 × 10 3 Wb,(Tesla=
Wb/m2)
We know that Vp = 4.44 Øm fNp
2400
From which we get Np = = 1201.2 = say 1200
4.44  7.5 103  60
K = Np/Ns = 10 i.e. Ns = Np/10 = 1200/10 = 120
iii) AT ( on 2400 V side ) = H × I = 450 × 66.7 × 10 2 = 300.15 AT
(For calculating the magnetizing current we have to take that side where voltage
is applied)
300.15
Hence magnetizing current I0 ( on 2400 V side ) = AT/Np =
1200
= 0.2501 A (RMS)

From here to be explained

Page 30
Malla Reddy College of Engineering and Technology
Department of EEE (2022 -23)
Electrical Machines -1 (EM-1): Lecture Notes: (Dr.Karimulla P Sk)
Unit 4: Single Phase Transformers

Example 13: A 6600/400 V single phase transformer has primary resistance of 2.5
Ω and secondary resistance of 0.01 Ω. Calculate total equipment resistance
referred to primary and secondary.
Solution : The given values are,
R1 = 2.5 Ω, R2 = 0.01 Ω
K = 6600/400 = 16.5
For finding equivalent resistance referred to primary, transfer R 2 to primary as
R2eq,
R2eq = K2R2= 0.01x16.52 = 2.7225 Ω
R1e = R1 + R2eq = 2.5 + 2.7225 = 5.2225 Ω
It can be observed that primary is high voltage side and hence high resistance
side hence while transferring R2 from low voltage to high voltage side , its value
increases.
Similarly to find total equivalent resistance referred to secondary, first calculate
R1 referred to secondary as
R1eq = R1/ K2 = 2.5/16.52
= 0.00918 Ω
∴ R2e = R2 + R1eq = 0.01 + 0.00918 = 0.01918 Ω

Example 14: A 20 kVA, 2000/200 V single phase transformer has the following
parameters. H.V. Winding: R1 = 3 Ω, X1 = 5.3 Ω, L.V. winding: R2 = 0.05 Ω, X2 = 0.1
Ω. Find the voltage regulation at, i) Power factor of 0.8 lagging. ii) UPF iii) 0.707
power factor leading.

Solution : We have the data


V1 = 2000 V, V2 = 200 V, R1 = 3 Ω, R2 = 0.05 Ω, X = 5.3 X1 = 5.3 Ω, X2 = 0.1 Ω.
K = V1/V2= 2000/200 = 10
From this data let us find out equivalent resistances referred to primary.
R1eq = R1 + R2eq = R1 + K2R2= 3 + 0.05 x 102 = 8 Ω
X1eq = X1 + X2eq = X1 + K2X2 = 5.3 + 0.1 x 102 = 15.3 Ω
Page 31
Malla Reddy College of Engineering and Technology
Department of EEE (2022 -23)
Electrical Machines -1 (EM-1): Lecture Notes: (Dr.Karimulla P Sk)
Unit 4: Single Phase Transformers

20 103
(I1)FL = VA rating / Primary voltage = = 10 A
2000
i) Cos Ø = 0.8 lagging, Sin Ø = 0.6
∴ % Regln. = (Voltage drop with full load current /Input Voltage)
= [ I1 {R1eq Cos Ø + X1eq Sin Ø} / V1 ] x 100
= [ 10 {8x 0.8 + 15.3x 0.6} / 2000 ] x 100 = 7.79 %
ii) cos Ø = 1, sin Ø = 0
∴ % Regln = (Voltage drop with full load current /Input Voltage)
= [ I1 {R1eq Cos Ø + X1eq Sin Ø} / V1 ] x 100
= [ 10 {8 x 1 + 15.3 x 0} / 2000 ] x 100 = 4%
iii) Cos Ø = 0.707 leading, Sin Ø = 0.707
∴ % Regln = (Voltage drop with full load current /Input Voltage)
= [ I1 {R1eq Cos Ø • X1eq Sin Ø} / V1 ] x 100
= [ I1 {8x 0.8 - 15.3x 0.6} / 2000] x 100 = - 2.5805 %
Example 15: A 230/460 volts transformer has a primary resistance of 0.2 Ω and
reactance of 0.5 Ω and the corresponding values for secondary are 0.75 Ω and 1.8
Ω respectively. Find the secondary terminal voltage when supplying 10 amps at
0.8 power factor lagging.
Solution : We have the data:
V1 = 230 , V2 = 460 V, R1 = 0.2 Ω, X1 = 0.5 Ω, R2 = 0.75Ω, X2 = 1.8 Ω
K = = 230/460 = 0.5
We have to find out the secondary terminal voltage while supplying a load
current of 10Amps. Since it is supplying current on the secondary side we have
to find out the equivalent circuit parameters referred to secondary side.
∴ R2e = R2 + R1e = R2 + R1/ K2= 0.75 + 0.2/(0.5)2 = 1.55 Ω
Similarly X2e = X2 + X1e = X2 + X1/K2= 1.8 + 0.5/(0.5)2 = 3.8 Ω
We have to find the secondary terminal voltage at Cos Ø = 0.8 lagging and load
current of I2 = 10 A. Sin Ø = 0.6 ( from the value of Cos Ø = 0.8)

Page 32
Malla Reddy College of Engineering and Technology
Department of EEE (2022 -23)
Electrical Machines -1 (EM-1): Lecture Notes: (Dr.Karimulla P Sk)
Unit 4: Single Phase Transformers

∴ Voltage drop = I2[R2ecos Ø + X2esinØ] = 10 [1.55 × 0.8 + 3.8 ×0.6] = 35.2 V


∴ Secondary terminal voltage V2 = 460 - Voltage drop = 460 - 35.2 = 424.8 V

Example 16: A 100 kVA, 6.6 kV/415 V single phase transformer has an effective
impedance of (3+j8) Ω referred to HV side. Estimate the full load voltage
regulation at
i) 0.8 pf lagging ii) 0.8 pf leading
Solution : Refer example 14 for the procedure with R1e = 3 Ω,
X1e = 8 Ω and verify the answers as,
i) + 1.652 % ii) -0.551 %

Example 17: A 230/460 V transformer has a primary resistance of 0.2 Ω and the
corresponding values for the secondary are 0.75 Ω and 1.8 Ω respectively. Find
the secondary terminal voltages when supplying 10 A at 0.8 p.f lagging and its
voltage regulation.
Solution : We have the data K = 230/460 = 0.5 , R1 = 0.2 Ω, X1 = 0.5 Ω
R2 = 0.75 Ω, X2 = 1.8 Ω, I2 = 10 A, cos Ø = 0.8 lag
Since we have the current on the load side we will find out the equivalent
resistance/reactance on secondary side.
R2e = R2 +R1/ K2 = 0.75 + (2)2× 0.2 = 1.55 Ω
X2e = X2 + X1/ K2= 1.8 + (2)2× 0.5 = 3.8 Ω
∴ Voltages drop = I2 [R2e cos Ø + X2e sinØ]
= 10 [1.55 ×0.8 + 3.8 × 0.6] = 35.2 V
∴ V2 (terminal ) = V2 - drop = 460 - 35.2 = 424.8 V
35.2
∴ % Reg = Voltagedrop × 100 = × 100 = + 7.652 %
V2 460

Example 18 : 1 kV/2kV transformer has 750 W hysteresis losses and 250 W eddy
current losses. When the applied voltage is doubled and frequency is halved, find
Page 33
Malla Reddy College of Engineering and Technology
Department of EEE (2022 -23)
Electrical Machines -1 (EM-1): Lecture Notes: (Dr.Karimulla P Sk)
Unit 4: Single Phase Transformers

the new losses.


Solution : The hysteresis loss is given by,
Ph B1.6
m f

The eddy current loss is given by,


Pe Bm2 f 2
V
But Bm
f
2
1.6 V 
∴ Ph  V  × f and Pe    × f 2
 f   f 
1.6
 V 1.6  f 
P f
From this the ratio of hysteresis losses is given by : 1 
h1
=      12
Ph2  V2   f1  f2
2 2 2
 P  V   f   f 
and the ratio of eddy current losses is given by:  e1  =  1       
2 1

 Pe 2   V2   f1   f 2 
Now voltage is doubled and frequency is halved and hence V2 = 2V1 and f2 = 0.5
f1
750
∴ The new hysteresis loss is given by: = (0.5)1.6× (0.5)1.6× 2
Ph 2
250
and new eddy current loss is given by: = (0.5)2× (0.5)2× (2)2
Pe 2
From which we get P h 2 = 3446.095 W and P e2 = 1000 W
∴ Total new iron loss = Ph2 + Pe 2 = 4446.095 W

Example 19: The flux in a magnetic core is alternating sinusoidally at a frequency


of 600 Hz. The maximum flux density is 2 Tesla. The eddy current loss is 15 W.
Find the eddy current loss in the core if the frequency is raised to 800 Hz and the
maximum flux density is reduced to 1.5 Tesla.
Solution : The eddy current loss is given by: Pe = KeBm2f2
At f1 = 600 Hz, Bm1 = 2 T, Pe1 =15 W
∴ 15 = Ke× (2)2× (600)2

Page 34
Malla Reddy College of Engineering and Technology
Department of EEE (2022 -23)
Electrical Machines -1 (EM-1): Lecture Notes: (Dr.Karimulla P Sk)
Unit 4: Single Phase Transformers

From which we get Ke = 15/(4 x 360000) = 1.04166 ×105


Then at f2 = 800 Hz and Bm = 1.5 T we get
Pe2 = 1.04166 ×105 × (1.5)2 × (800)2 = 15 W
The eddy current loss remains same.

Example 20 : A 2300/230 V, 50 Hz single phase transformer has the transformer


parameters : R1 = 0.3 Ω; R2e = 0.295 Ω; X1 = 0.375 Ω; X2e = 0.685 Ω ; Rc1 = 4 kΩ ; Xm1
= 1000 Ω. The secondary load impedance ZL = (0.4 + j0.3) Ω. Find the voltage
regulation, losses of the transformer using approximate equivalent circuit.
Solution : K = 2300/230 = 10
Rc1 = R0 = 4 kΩ, Xm1 = X0 = 1000 Ω

Let us find out equivalent circuit referred to primary because we are given the
values of secondary referred to primary i.e. R2e and X2e

R1e = R1 + R2e = 0.595 Ω,


X1e = X1 + X2e = 1.06 Ω

Because we have taken equivalent circuit referred to primary. Let us find out the
value of load impedance referred to primary
ZLe (ZL’ ) = ZLxK2= ZLx102= (0.4+j0.3) = 40 + j30 Ω
Page 35
Malla Reddy College of Engineering and Technology
Department of EEE (2022 -23)
Electrical Machines -1 (EM-1): Lecture Notes: (Dr.Karimulla P Sk)
Unit 4: Single Phase Transformers

Total equivalent impedance referred to primary Z1e = (40+0.595)+j(30+1.06)=


(40.595 + j31.06) Ω
Since we have taken primary as reference V1 = 2300 /00 V will be the reference
voltage. If
V1
I1’ =
R1e  jX1e  Z L
= 2300 /00 / (40.595 + j31.06)
= 2300 /00 / 51.114 /37.420 = 45/-37.420 A
= 35.739 - j 27.344 A
V1 2300 V1
Ic = = = 0.575 A, Im = = 2.3 A
R0 4 10 3
X0
∴ I0 = Ic - j Im = 0.575 - j 2.3 A
∴ I1 = I0 + I1’ = 36.314 - j 29.644 A
= 46.8772/- 39.220 A
Core loss = Pi = I C2 Rc = 1322.5 W
Copper loss = (I 1’)2 R 1e = 452× 0.595 = 1204.875 W
Secondary voltage = I2 ZL = I 1’ x K x ZL
= 45 x 10 (0.4)2  (0.3)2 45 x 10 = 225 V
230  225
∴ % Reg = ×100 = 2.22 %
225

Example 21: A single phase transformer has 500 turns on the primary and 40
turns on the secondary winding. The mean length of the magnetic path in the
iron core is 150 cm and the joints are equivalent to an air gap of 0.1 mm. When a
potential difference of 3000 V is applied to the primary, maximum flux density is
1.2 Wb/m2. Calculate:
i) The cross- sectional area of the core ii) No-load secondary voltages.
iii) The no load current drawn by the primary iv) Power factor on no-load
Given that AT/cm for a flux density of 1.2 Wb/m2 in iron to be 5, the
corresponding iron loss to be 2 watt/kg at 50 Hz , the density of iron as 7.8
Page 36
Malla Reddy College of Engineering and Technology
Department of EEE (2022 -23)
Electrical Machines -1 (EM-1): Lecture Notes: (Dr.Karimulla P Sk)
Unit 4: Single Phase Transformers

gram/cm2 and μ0 = 4π x 10-7

Solution : E1 = 3000 V, Bm = 1.2 Wb/m2, N1 = 500, f= 50 Hz, length of the magnetic


path in iron li = 150cm,
length of the magnetic path in air gap = lg = 0.1 mm = 0.1 x 10-3 mtrs
∴ E1 = 4.44 f ØmN1
3000
i.e. Øm = = 0.027 Wb
4.44 50 500
m 0.027
i) Bm = i.e. a = = 0.0225 m2
a 1.2
= 225 cm2
E2 N2 40
ii) = i.e. E2 = No load secondary voltage = 3000 × = 240
E1 N1 500
V
iii) AT/cm = 5 for iron path
AT = 5×Ii = 5 × 150 = 750
B 1.2
AT for air gap = H × Ig =  lg =  0.1103 = 95.5
0 4 107
∴ Total AT for given Bm = 750 + 95.5 = 845.5 = N1Im
845.5
∴ Im = = 1.691 A …… Since Bm (Maximum value )is used
500
we get Im (max)
1.691
∴ Im(rms ) = I m (max) = = 1.196 A ….Magnetizing current
2 2
Volume of iron = Ii× a = 150 × 225 = 33750 cm3
Density of iron = 7.8 gm/cm3
∴ Mass of iron = Volume × Density = 33750 × 7.8 = 263250 gm
Iron loss = 2 W/kg i.e. Total iron loss = 263250 × 10-3× 2 = 526.5 W
Totalironloss
∴ Ic = Core loss component = = 526.5 = 0.176 A
E1 3000

Page 37
Malla Reddy College of Engineering and Technology
Department of EEE (2022 -23)
Electrical Machines -1 (EM-1): Lecture Notes: (Dr.Karimulla P Sk)
Unit 4: Single Phase Transformers

(Core loss component is working component which is in phase with the applied
voltage.
Hence Ic = Power/Voltage)
∴ I0 = I C2  I m2 = (0.176)2  (1.196)2 = 1.208 A
IC
iv) cos Ø0 = = 0.176 = 0.1457 lagging.
I0 1.208

Page 38
Malla Reddy College of Engineering and Technology
Department of EEE (2022 -23)
Electrical Machines -1 (EM-1): Lecture Notes: (Dr.Karimulla P Sk)
Unit 4: Single Phase Transformers

Additional Problems:

Example 1 : A 4 kVA, 200/400 V, 50 Hz, single phase transformer has equivalent


resistance referred to primary as 0.15 Ω. Calculate,
i) The total copper losses on full load.
ii) The efficiency while supplying full load at 0.9 p.f. lagging.
iii) The efficiency while supplying half load at 0.8 p.f. leading.
Assume total iron losses equal to 60 W.
Solution : The given values are,
V1 = 200 V, V2 = 400 V, S = 4 kVA , R1e = 0.15 Ω, pi = 60 W
K = V1/V2 = 200/400 = 0.5

From the problem we can see that we have to find out the efficiency while
supplying in different conditions. Hence we have to get the parameters reflected
to secondary side. So we have to calculate

R2e = R1e/ K2 = 0.15/(0.5)2 = 0.6 Ω


𝑘𝑉𝐴 4×103
(I2) F.L. = = = 10 A
𝑉2 400
i) Total copper losses on full load,
(Pcu) F.L. = [(I2) F.L.]2 R2e = (10)2 × 0.6 = 60 W
ii) ƞ @ cos Ø = 0.9 lagging and full load
𝑉 𝐴 𝑟𝑎𝑡𝑖𝑛𝑔 cos ∅
∴ % ηFL = × 100
𝑉 𝐴 𝑟𝑎𝑡𝑖𝑛𝑔 cos ∅ + 𝑃𝑖+(𝑃𝑐𝑢 )𝐹.𝐿
4 ×103×0.9
∴ % ηFL = 4 ×103 ×0.9+60+60 × 100 = 96.77%
iii) cos Ø = 0.8 leading half load i.e. n = 0.5
(pcu) H.L. = n2 × (Pcu)F.L. = (0.5)2 × 60 = 15 W
𝑛×(𝑉 𝐴 𝑟𝑎𝑡𝑖𝑛𝑔 ) cos ∅
∴ % ηHL = × 100
𝑛×(𝑉 𝐴 𝑟𝑎𝑡𝑖𝑛𝑔 ) cos ∅+𝑃𝑖+(𝑃𝑐𝑢 )𝐻.𝐿.
0.5 ×4×103×0.8
= 0.5×4×103×0.8+60+15 × 100 = 95.52 %

Page 39
Malla Reddy College of Engineering and Technology
Department of EEE (2022 -23)
Electrical Machines -1 (EM-1): Lecture Notes: (Dr.Karimulla P Sk)
Unit 4: Single Phase Transformers

Example 2: The maximum efficiency of a 500 kVA, 3300/500 Volts, 50Hz, single-
phase transformer is 97 % and occurs at 3/4th full load with unity p.f. If the
impedance is 10 %, calculate the regulation at full load and 0.8 p.f. lagging.

Solution : ηmax = 97 % occurs at n = ¾ = 0.75 and unity p.f


𝑛𝑉𝐴 cos ∅
% ηmax = 𝑛𝑉𝐴 cos ∅+2𝑃 × 100 ( since at ηmax Pi = Pcu )
𝑖
0.75 ×500 ×103×1
i.e. 0.97 =
0.75×500×103×1+2𝑃 𝑖

From which we get Pi = Iron losses = 5798.969 W


Also Pcu @ n= 0.75 = 5798.969 as ηmax occurs at 3/4th load
We know that Pcu at any fractional load n = n2 (Pcu)F.L.
i.e 5789.969 = (0.75)2 × (Pcu)F.L.
From which (Pcu) @F.L. = Full load copper loss = 5789.969 /(0.75)2 =
10309.278 W
𝑉𝐴 𝑟𝑎𝑡𝑖𝑛𝑔 500 ×103
(I2)F.L. = = = 151.515 A
𝑉1 3300

We are given the % impedance as 10% which can be taken on either primary side
or secondary side for calculating the regulation. Accordingly the % resistance and
%reactance also are to be calculated on either primary side or secondary side. Let
us calculate on primary side.

Now (Pcu)F.L. = [(I1)F.L.]2 Rle


10309 .278
i.e Rle = (151.515)2
= 0.4491 Ω
%Rle (in terms of primary side equivalent resistance ,voltage and current) =
𝐼1 𝑅𝑙𝑒
× 100
𝑉1
151.515 ×0.4491
= 3300
× 100
= 2.062 %

Page 40
Malla Reddy College of Engineering and Technology
Department of EEE (2022 -23)
Electrical Machines -1 (EM-1): Lecture Notes: (Dr.Karimulla P Sk)
Unit 4: Single Phase Transformers

% Zle = 10 % (Given data )


% Xle = √(𝑍𝑙𝑒 )2 − (𝑅𝑙𝑒 )2 = √(10)2 − (2.062)2
= 9.785 %
But % Xle (again in terms of primary side equivalent resistance ,voltage and
𝐼 𝑋
current) = 1 𝑙𝑒 × 100
𝑉1
151.515𝑋𝑙𝑒
i.e. 9.785 = × 100 from which X le = 9.785 x 3300/(151.515 x
3300
100)

= 2.311 Ω
𝐼1 [𝑅𝑙𝑒 𝑐𝑜𝑠 ∅+𝑋𝑙𝑒 sin ∅]
% Regulation = × 100
𝑉1
151.515[0.4491 ×0.8+2.1311 ×0.6]
= 3300
× 100 (Given Cos Ø = 0.8 lag we
get Sin Ø =0.6)
= 7.52 %

Example 3:The maximum efficiency of a 50 kVA transformer is 97.4 % and occurs


at 90 % of the full load at unity power factor. Calculate the efficiency of
transformer at,
i) Full load 0.8 power factor lagging
ii) Half load 0.9 power factor.
Solution : We have the data :

Rating: 50 kVA, ƞmax = 97.4 %, n= 0.9 for ƞmax at cos ∅ = 1.


𝑛𝑉𝐴 𝑟𝑎𝑡𝑖𝑛𝑔 ×cos ∅
From this data using the relation %ƞmax = 𝑛𝑉𝐴 𝑟𝑎𝑡𝑖𝑛𝑔 ×cos ∅+2𝑃 × 100
𝑖
(Since Pi = Pcu for ƞmax)
0.9×50×103×1
we have 97.4 = 0.9×50×103 ×1+2𝑃 × 100
𝑖
From which we get Pi = 600.616 W

We know that efficiency at Full load and any p.f cos Ø is given by :

Page 41
Malla Reddy College of Engineering and Technology
Department of EEE (2022 -23)
Electrical Machines -1 (EM-1): Lecture Notes: (Dr.Karimulla P Sk)
Unit 4: Single Phase Transformers

𝑉𝐴 𝑟𝑎𝑡𝑖𝑛𝑔 ×cos ∅
% ȠFL = x100
𝑉𝐴 𝑟𝑎𝑡𝑖𝑛𝑔 ×cos ∅+𝑃𝑖+(𝑃𝐶𝑢 )𝐹.𝐿.

We have the data of all the parametrers in this equation except Pcu @FL which can
be found from the relation
𝑃𝑖
kVA for ƞ max = kVA rating ×√
(𝑃𝐶𝑈 )𝐹.𝐿.
600.616
∴ 0.9 ×[50×103] = [50×103] × √
(𝑃𝐶𝑢 )𝐹.𝐿.
From which Pcu @FL = Full load copper loss = 741.5012 W

i) Efficiency at Full load and cos Ø = 0.8 lag :


𝑉𝐴 𝑟𝑎𝑡𝑖𝑛𝑔 ×cos ∅
% ȠFL = 𝑉𝐴 𝑟𝑎𝑡𝑖𝑛𝑔 ×cos ∅+𝑃 𝑖+(𝑃 𝐶𝑢 )𝐹.𝐿. × 100
50×103×0.8
= × 100
50×103×0.8+600.616+741.5012
=96.7536 %
ii) Half load, n = 0.5, cos Ø = 0.9
ƞ𝑉𝐴 𝑟𝑎𝑡𝑖𝑛𝑔 ×cos ∅
% ȠHL = × 100
ƞ𝑉𝐴 𝑟𝑎𝑡𝑖𝑛𝑔 ×cos ∅+𝑃𝑖+𝑛2 (𝑃𝐶𝑈 )𝐹.𝐿.
0.5×50 ×103×0.8
= 0.5 ×50 ×103 ×0.8+600.616+(0.5)2×741.5012 × 100
= 96.2186 %

Example 4:The efficiency of a 1000 kVA, 110/200 V, 50 Hz single phase


transformer is 98.5 % at half full load at 0.8 power factor leading and 98 % at full
load, UPF. Determine
i) Iron loss ii) Copper loss iii) Maximum efficiency at UPF.
Solution : We have the data :

Rating : 1000 kVA, V1 =110 V, V2 = 200 , Ƞ@1/2FL and cos Ø = 0.8 = 98.5 %,

Using this data in the basic efficiency relation

Page 42
Malla Reddy College of Engineering and Technology
Department of EEE (2022 -23)
Electrical Machines -1 (EM-1): Lecture Notes: (Dr.Karimulla P Sk)
Unit 4: Single Phase Transformers

𝑛×𝑉𝐴 𝑟𝑎𝑡𝑖𝑛𝑔 ×cos ∅


% ƞHL = × 100
𝑛 ×𝑉𝐴 𝑟𝑎𝑡𝑖𝑛𝑔 ×cos ∅+𝑃 𝑖 +𝑛2 (𝑃 𝐶𝑈 )𝐹,𝐿,

we get :
𝑛×𝑉𝐴 𝑟𝑎𝑡𝑖𝑛𝑔 ×cos ∅
% ƞHL = × 100 (in this case n =
𝑛 ×𝑉𝐴 𝑟𝑎𝑡𝑖𝑛𝑔 ×cos ∅+𝑃𝑖 +𝑛2(𝑃 𝐶𝑈 )𝐹,𝐿,
0.5)
0.5 ×1000 ×103 ×0.8
98.5 = × 100
0.5 ×1000 ×103 ×0.8+𝑃 𝑖 +(0.5)2×(𝑃 𝐶𝑢 )𝐹.𝐿.
From which we get Pi + 0.25 (Pcu) F.L. = 6091.3705
……(1)
Similarly in the second case:
Ƞ@FL and cos Ø = 1 = 98 %,
𝑛×𝑉𝐴 𝑟𝑎𝑡𝑖𝑛𝑔 ×cos ∅
% ƞ@FL = 𝑛 ×𝑉𝐴 𝑟𝑎𝑡𝑖𝑛𝑔 ×cos ∅+𝑃𝑖 +𝑛2 (𝑃 𝐶𝑈 )𝐹,𝐿, × 100 (in this case n
= 1)

1000 ×103 ×1
98 = 1000 ×103 ×1+𝑃 +(𝑃 × 100
𝑖 𝐶𝑈 )𝐹.𝐿.

From which we get: Pi + (Pcu) F.L. = 20408.1632


……..(2)
Solving equations (1) and (2),
i) Pi = 1319.1062 W …….Iron loss
ii) (Pcu) F.L. = 19089.0569 W ……Full load copper loss
iii) For ƞmax with cos Ø = 1, we have to get the fractional kVA for maximum
efficiency which is given by
𝑃𝑖
kVA for ƞ max = kVA rating × √ (𝑃
𝑐𝑢 )𝐹.𝐿.

1319.1062
= 1000 × √ 19089.0569 = 262.874 kVA
At ƞmax, Pi = Pcu = 1319.1062 W
[𝑉𝐴 𝑓𝑜𝑟 ƞ𝑚𝑎𝑥 ] ×cos ∅
∴ % ƞ max = [𝑉𝐴 𝑓𝑜𝑟 ƞ × 100
𝑚𝑎𝑥 ] ×cos ∅ + 2𝑃𝑖

Page 43
Malla Reddy College of Engineering and Technology
Department of EEE (2022 -23)
Electrical Machines -1 (EM-1): Lecture Notes: (Dr.Karimulla P Sk)
Unit 4: Single Phase Transformers

262.874 ×103 ×1
= × 100
262.874 ×103 ×1+2 ×1319.1062
= 99 %

Example 5: A 100 Kva, 50 Hz, 440/11 Kv, single phase transformer has an
efficiency of 98.5 % when supplying full load current at 0.8 p.f. lagging and an
efficiency of 99 % when supplying half load current at unity power factor. Find the
core losses and copper losses corresponding to full load current. At what value of
load current will the maximum efficiency be attained?

Solution : Refer example 4 for the procedure and verify the answers :
Pi = 267.309 W, Pcu(FL) = 950.964 W.
𝑉𝐴 100 ×103
I2 (FL) = 𝑉 = 11 × 103
= 9.0909 A
2
𝑃𝑖
I2m = I2 (FL) ×√
𝑃𝑐𝑢 (𝐹𝐿)
= 4.822 A

Example 6:A 25 kVA, 2000/200 V, 50 Hz transformer has maximum efficiency at


75 % of full load. Its per unit resistance and impedance are 0.012 and 0.05
respectively. Determine its efficiency and voltage regulation at half of the full load
and 0.8 p.f lagging.

Solution : kVA for ηmax = 0.75 × kVA = 0.75 × 25 = 18.75 kVA


𝐼1 𝑅1𝑒
Per unit R = 0.012 =
𝑉1
Where I1 = full load current
𝑉𝐴 25 ×103
I1(FL) = = 2000
= 12.5 A
𝑉1

Page 44
Malla Reddy College of Engineering and Technology
Department of EEE (2022 -23)
Electrical Machines -1 (EM-1): Lecture Notes: (Dr.Karimulla P Sk)
Unit 4: Single Phase Transformers

12.5 𝑅1𝑒
∴ 0.012 = i.e. R1e = 1.92 Ω
2000
𝐼1 𝑋𝑙𝑒
Per unit X = 0.05 =
𝑉1
0.05 ×2000
i.e. X1e = = 8Ω
12.5
1
At half full load, I1(HL) = 2 × 12.5 = 6.25 A
cos Ø = 0.8, sin Ø = 0.6
𝐼1 (𝐻𝐿)[𝑅1𝑒 𝑐𝑜𝑠 ∅+𝑋1𝑒 sin ∅]
% Reg = × 100 = 1.98 %
𝑉1
𝑃𝑖
kVA for ηmax = kVA × √
𝑃𝑐𝑢 (𝐹𝐿)
𝑃𝑖 𝑃𝑖
∴ 18.75 = 25 × √ i.e. = 0.5625
𝑃𝑐𝑢 (𝐹𝐿) 𝑃𝑐𝑢 (𝐹𝐿)
2 2
PCu (FL) = [I1(FL) Rle = 12.5 × 1.92 = 300 W
Pi = 0.5625 × 300 = 168.75 W
0.5 ×𝑉𝐴 cos ∅
∴ % ηHL = 0.5 ×𝑉𝐴 cos ∅+𝑃 +[(0.5)2𝑃 (𝐹𝐿)]
× 100 (n = 0.5)
𝑖 𝑐𝑢
0.5 × 25 × 103 × 0.8
= 0.5 ×25 ×103 × 0.8+168.75+(0.52 ×300 × 100
= 97.62 %
Example 7: A 250/500 V, 50 Hz single phase transformer has the following
equivalent circuit parameters: Rt1 = 0.715 Ω ; Xt1 = 0.375 Ω ; Rc1 = 780Ω ; Xm1 =
265Ω
Determine the voltage regulation ,losses and efficiency of the transformer when it
is delivering 6 kVA at 0.8 pf leading and at rated voltage.

Solution :
Rt1 = R1e = 0.175 Ω, Xt1 = X1e = 0.375 Ω
Rc1 = Ro = 780 Ω, Xml = Xo = 265 Ω
Because the kVA of 6 given is while delivering we have to calculate using the
secondary equivalent parameters. So the given primary parameters are to be
referred to secondary.

Page 45
Malla Reddy College of Engineering and Technology
Department of EEE (2022 -23)
Electrical Machines -1 (EM-1): Lecture Notes: (Dr.Karimulla P Sk)
Unit 4: Single Phase Transformers

250
K = 500 = 0.5, R2e = R1e / K2 = 0.175/(0.5)2 =0.7 Ω
X2e = X1e/ K2 = 0.375 /(0.5)2 = 1.5 Ω
𝑉𝐴 6 ×103
I2 = = = 12 A (Since 6 kVA given is on
𝑉2 500
load side )
𝐼2 (𝑅2𝑒 𝐶𝑜𝑠 ∅−𝑋2𝑒 sin ∅)
∴ % Reg = × 100 ( negative sign is because
𝑉2
Cos Ø = 0.8 is leading)
12(0.7 ×0.8−1.5 ×0.6)
= 500 𝑉1 250
× 100 = - 0.816%
𝑉1
R = i.e. I = = = 0.3205 A
0 c 780
𝑉𝐼1𝑐 𝑅0
X= i.e. I = 𝑉1
=
250 = 0.9434 A
o m 265
𝐼𝑚 𝑋0
𝐼𝑚
∴ tan Ø = = 2.9435, Ø = 71.2350
𝐼𝐶
Io = √𝐼2 + 𝐼2 = 0.9963 A
𝑐 𝑚
∴ Pi = Iron losses = V1I0 cos Ø0 = 80.1242 W
Pcu = Copper losses = 𝐼22 R2e = 122 × 0.7 = 100.8 W
𝑉𝐴 cos ∅
∴ %η= × 100
𝑉𝐴 cos ∅+𝑃𝑖+𝑃𝑐𝑢
6 ×103 ×0.8
= 6 ×103 ×0.8+80.1242 +100.8 × 100
= 96.36 %

Example 8: A 25 kVA, 1-phase transformer, 2200/220 V, has a primary Resistance


of 2.0 Ω and a secondary resistance of 0.05 Ω. Find the Equivalent secondary
resistance and the full load efficiency at 0.8 p.f , if the iron loss of the transformer
is 75 % of the full load copper loss.

Solution : We have the data : R1 = 2 Ω R2 = 0.05 Ω, K = 2200/220 = 10


∴ Equivalent secondary resistance = R2e = R2 + R1e = 0.05 + 2/(10)2 = 0.07 Ω
25 ×103
I2 (FL) = 220
= 113.6363 A,

Page 46
Malla Reddy College of Engineering and Technology
Department of EEE (2022 -23)
Electrical Machines -1 (EM-1): Lecture Notes: (Dr.Karimulla P Sk)
Unit 4: Single Phase Transformers

Pcu(FL) = [I2(FL)]2 R2e


∴ Pcu (FL) = 903.925 W, Pi = 0.75 × 903.925 = 677.944 W
25 ×103 ×0.8
∴ %η= 25 ×103 ×0.8+903.925 + 677.944
× 100
= 92.67 %

Example 9: The full load copper and iron losses of a 15 kVA single phase
transformer are 320 W and 200 W respectively. Calculate the efficiency of the
transformer at unity power factor at full load and half load.
Solution : ARting = 15 kVA, Pi = 200 W, Pcu (FL) = 320 W
𝑉𝐴 cos ∅
%ηFL = × 100
𝑉𝐴 cos ∅ +𝑃 𝑖 +𝑃 𝐶𝑈 (𝐹𝐿)

We have to find out both FL and HFL efficiency with Cos Ø = 1


15 ×103 ×1
Hence %ηFL = 15 ×103 ×1+200+320 × 100 = 96.649 %

%ηHL = 𝑛𝑉𝐴 cos ∅


𝑛𝑉𝐴 cos ∅ +𝑃 +𝑛2 𝑃 × 100 (n = 0.5, and cos Ø = 1)
𝑖 𝑐𝑢 (𝐹𝐿)

0.5 ×15 ×103 ×1


= 0.5 ×15 ×103 ×1+200+[(0.5)2 ×320] × 100 = 96.401 %

Page 47
Malla Reddy College of Engineering and Technology
Department of EEE (2022 -23)
Electrical Machines -1 (EM-1): Lecture Notes: (Prof.K.Subhas)
Unit 5: Testing of Transformers &Three Phase Transformers

UNIT – V
TESTING OF TRANSFORMERS AND THREE-PHASE
TRANSFORMERS

CONTENTS:

 OC and SC tests
 Sumpner’s test
 Predetermination of efficiency and regulation
 Separation of losses test
 Parallel operation with equal and unequal voltage ratios
 Auto transformers- equivalent circuit
 Comparison with two winding Transformers
 Poly phase Transformers
 Poly phase Connections
 Y/Y, Y/Δ, Δ/ Δ, Δ/Y open Δ connections
 Illustrative examples

Page 1
Malla Reddy College of Engineering and Technology
Department of EEE (2021-22)
Electrical Machines -1 (EM-1): Lecture Notes: (Prof.K.Subhas)
Unit 5: Testing of Transformers &Three Phase Transformers

Open Circuit (OC) and Short Circuit (SC) tests:

They are conducted on the transformer to find out the transformer losses and
from them to determine the circuit constants that are used to represent the
equivalent circuit. From these parameters the transformer efficiency and
regulation can also be calculated. These tests are conducted without actually
loading the transformer to its full load and hence the power consumed during the
test is very small as compared to its full load (rated) power.

Open Circuit or No Load test:

The test setup to conduct the OC test is shown in the figure below.

Fig: Test Setup to conduct the Open Circuit or No Load test

Low voltage side is designated as Primary and High voltage side is designated as
Secondary. Voltmeter V1 , Ammeter A and wattmeter W are connected in the
primary as shown . Voltmeter V2 is connected in the open circuited Secondary.
Since the secondary is open circuited a small value of no load current called I0
flows in the primary and this is measured by the ammeter A. The power loss in
the transformer is due to core losses and a very small I2R loss in the primary.
There is no I2R loss in the secondary since the secondary is open and there is no
secondary current. Since I2R loss in the primary also is very small the no load
current is very small (usually 2 to 5 % of the full load current). The core loss is
dependent on the flux which in turn depends on the applied voltage. Since full

Page 2
Malla Reddy College of Engineering and Technology
Department of EEE (2021-22)
Electrical Machines -1 (EM-1): Lecture Notes: (Prof.K.Subhas)
Unit 5: Testing of Transformers &Three Phase Transformers

rated voltage is applied to the primary in this test, full rated flux will be set up and
the full Core losses will be present. And these core losses will be constant at all
loads. Since the I2R loss in the primary is very small compared to the core losses,
they can be ignored and the full power consumed in the primary as read by
wattmeter W can be regarded as the core losses. With this understanding the
readings of the various meters in the OC test are as follows.

 Ammeter reading : No load current I0


 Volt meter reading : Applied rated Primary voltage V1
 Wattmeter reading : Input power totally consumed as Core losses PCL

From these measurements the parameters RC and XM shown in the equivalent


circuit can be computed as shown below.

No load power factor : Cos Ɵ = PCL / V1.I0 (Since Input power = PCL = V1.I0.
Cos Ɵ )

We already know from the equivalent circuit description that ICL is the core loss
current corresponding to RC and IM is the magnetizing current corresponding to
XM and in terms of the present measurements are given by :
ICL = I0. Cos Ɵ and IM = I0. Sin Ɵ

Therefore RC and XM are given by :


RC = V1 / ICL and XM = V1 / IM

Short Circuit test:

The test setup to conduct the SC test is shown in the figure below.

Page 3
Malla Reddy College of Engineering and Technology
Department of EEE (2021-22)
Electrical Machines -1 (EM-1): Lecture Notes: (Prof.K.Subhas)
Unit 5: Testing of Transformers &Three Phase Transformers

HV side LV side

Fig: Test Setup to conduct the Short Circuit test

In this test high voltage side is designated as Primary (where an input voltage is
applied) and the low voltage side is designated as Secondary which is usually
short-circuited by a thick conductor(or sometimes through an Ammeter to read
additionally the secondary load current).Voltmeter V, Ammeter A and wattmeter
W are connected in the primary as shown.
A very low voltage through a Variac (Variable auto transformer) is applied to the
primary gradually from zero Volts to about 5 to 10 % of the rated primary value
till the primary current is just equal to the rated primary current. Since the
secondary is short circuited we will get rated primary current with a low value of
voltage itself. Since the applied voltage is very low the flux produced is also very
low. Hence the core losses also will be low and can be neglected. Now since the
rated currents are flowing in both the Primary and the secondary the input power
will be mostly consumed as copper losses. Since the secondary is short circuited
the secondary voltage is zero and the entire input voltage V1 drops in the total
equivalent impedance Z EQP of the transformer reflected to the primary.

i.e. V1 = IPSC . Z EQP

The readings in the short circuit test are as follows:

Page 4
Malla Reddy College of Engineering and Technology
Department of EEE (2021-22)
Electrical Machines -1 (EM-1): Lecture Notes: (Prof.K.Subhas)
Unit 5: Testing of Transformers &Three Phase Transformers

 Ammeter reading : Primary current IPSC( with secondary short


circuited )
 Volt meter reading : Applied Primary voltage V1
 Wattmeter reading : Input power totally consumed as Copper losses
PCUL

With this notation the power factor Cos ƟSC in this test is given by:

PCUL = V1. IPSC . Cos ƟSC

From the above readings and the governing equations we can calculate the
Equivalent Resistance, Equivalent Impedance and Equivalent Reactance
parameters of the transformer referred to the primary as below.
Equivalent Resistance: R EQP = PCUL / I2 PSC

Equivalent Impedance: Z EQP = V1 / IPSC

Equivalent Reactance : X EQP = √ Z 2EQP -- R2EQP

Sumpner's Test :
Sumpner's test or back to back test on transformer is another method for
determining transformer efficiency, voltage regulation and heating under loaded
conditions. Short circuit and open circuit tests on transformer can give us
parameters of equivalent circuit of transformer, but they cannot help us in finding
the heating information. Unlike O.C. and S.C. tests, actual loading is simulated in
Sumpner's test. Thus the Sumpner's test gives more accurate results of regulation
and efficiency than O.C. and S.C. tests.

Sumpner's test or back to back test can be employed only when two
identical transformers are available. Both transformers are connected to supply
such that one transformer is loaded on another. Primaries of the two identical
Page 5
Malla Reddy College of Engineering and Technology
Department of EEE (2021-22)
Electrical Machines -1 (EM-1): Lecture Notes: (Dr.Karimulla P Sk)
Unit 5: Testing of Transformers &Three Phase Transformers

transformers are connected in parallel across a supply. Secondaries are connected


in series opposition such that their e.m.f's are opposite to each other. Another
low voltage supply is connected in series with secondaries to get the readings, as
shown in the circuit diagram shown below.

Figure : Sumpner’s Test setup

In the above diagram, T1 and T2 are identical transformers. Their secondaries are
connected in voltage opposition, i.e. both the emf's E EF and EGH cancel each other,
as transformers are identical. In this case, as per superposition theorem, no
current flows through secondary. And thus the no load test is simulated. The
current drawn from V1 is 2I0, where I0 is equal to no load current of each
transformer. Thus , input power measured by wattmeter W 1 is equal to iron losses
of both the transformers.

Page 6
Malla Reddy College of Engineering and Technology
Department of EEE (2022-23)
Electrical Machines -1 (EM-1): Lecture Notes: (Dr.Karimulla P Sk)
Unit 5: Testing of Transformers &Three Phase Transformers

i.e. Iron loss per transformer Pi = W1/2.

Now, a small voltage V2 is injected into secondary with the help of a low voltage
transformer. The voltage V2 is adjusted such that, the rated current I2 flows
through the secondary. In this case, both primaries and secondaries carry rated
current. Thus short circuit test is simulated and wattmeter W2 shows total full
load copper losses of both transformers. i.e. Copper loss per transformer PCu =
W2/2.

From above test results, the full load efficiency of each transformer can be given
as:

And Output = kVA rating x Cos θ (power factor)

Predetermination of efficiency and regulation:

Calculation of efficiency:

We have obtained Iron losses Pi as the input power W0 from the OC test and then
Full load ( or at any load ) copper losses Pcu as the input power Psc from the SC
test.

Using this data we can predetermine efficiency and regulation at any load and
power factor as shown below.

% η at full (rated load) = *,Rated KVA x cos Ɵ- / , Rated KVA x cos Ɵ + Pi + Psc } ]
x 100

Page 7
Malla Reddy College of Engineering and Technology
Department of EEE (2022-23)
Electrical Machines -1 (EM-1): Lecture Notes: (Dr.Karimulla P Sk)
Unit 5: Testing of Transformers &Three Phase Transformers

Similarly at any load which is a fraction of full load we can predetermine the
efficiency

% η at any load = *,n x Rated KVA x cos Ɵ- / , n x Rated KVA x cos Ɵ + Pi + n2Psc
} ] x 100

Calculation of Regulation:

We know that % Regulation @FL is defined as: % Regulation@FL =[ (Input –


output at FL )/Input] x100

= [{Drop across the equivalent resistance REQ and equivalent


XEQ @FL}/Input] x 100

From the SC test we can obtain the equivalent circuit parameters. We shall use
the following designation and get the expression for regulation by using the
circuit parameters referred to both primary and secondary.

VP = rated primary voltage IP = rated primary current


(FL)

VS = rated secondary voltage IS = rated secondary current


(FL)

REQP = Equivalent resistance referred to primary REQS = Equivalent resistance


referred to secondary XEQP = Equivalent reactance referred to primary XEQS =
Equivalent reactance referred to secondary

Page 8
Malla Reddy College of Engineering and Technology
Department of EEE (2022-23)
Electrical Machines -1 (EM-1): Lecture Notes: (Dr.Karimulla P Sk)
Unit 5: Testing of Transformers &Three Phase Transformers

Then

% Regulation (referred to primary)@FL = [{IP (REQP x Cos Ɵ + XEQP x Sin Ɵ) /VP }]


x 100

% Regulation (referred to secondary)@FL = [{IS (REQS x Cos Ɵ + XEQS x Sin Ɵ)


/(VP/a)}] x 100

Where ‘a’ is the transformer turns ratio

Regulation at any other load can be calculated using the same relation as above
except that the currents Ip and Is are to be multiplied by the corresponding load
fractions ‘n’ and the relations will become

% Regulation (referred to primary)@any fractional FL = [{nIP (REQP x Cos Ɵ +


XEQP x Sin Ɵ) /VP }] x 100

% Regulation (referred to secondary)@any fractional FL = [{nI S (REQSx Cos Ɵ + XEQS


x Sin Ɵ) /(VP/a)}] x 100

Transformer equivalent circuits (Both Complete and simplified) referred to both


Primary and secondary are appended below again for a quick reference :

Page 9
Malla Reddy College of Engineering and Technology
Department of EEE (2022-23)
Electrical Machines -1 (EM-1): Lecture Notes: (Dr.Karimulla P Sk)
Unit 5: Testing of Transformers &Three Phase Transformers

Fig: Simplified equivalent circuits (a) Referred to the primary side (b) Referred to
the secondary side
(c) With no excitation branch, referred to the primary side (d) With no excitation
branch, referred to the secondary side
Parallel operation of transformers:

Need for Parallel operation of transformers:

When the load outgrows the capacity of an existing transformer, it may be


economical to install another one in parallel with it rather than replacing it with a
single larger unit. Also, sometimes in a new installation, two units in parallel,
though more expensive, may be preferred over a single unit for reasons of
reliability—half the load can be supplied with one unit out. Further, the cost of
maintaining a spare is less with two units in parallel. However, when spare units
are maintained at a central location to serve transformer installations in a certain
region, single-unit installations would be preferred. It is, therefore, seen that
Page 10
Malla Reddy College of Engineering and Technology
Department of EEE (2022-23)
Electrical Machines -1 (EM-1): Lecture Notes: (Dr.Karimulla P Sk)
Unit 5: Testing of Transformers &Three Phase Transformers

parallel operation of the transformer is quite important and desirable under


certain circumstances.

Conditions for successful parallel operation of transformers:

The satisfactory and successful operation of transformers connected in parallel on


both sides requires that they fulfill the following conditions:

(i) The transformers must be connected properly as far as their polarities are
concerned so that the net voltage around the local loop is zero. A wrong polarity
connection results in a dead short circuit.
(ii) Three-phase transformers must have zero relative phase displacement on the
secondary sides and must be connected in a proper phase sequence. Only the
transformers of the same phase group can be paralleled. For example, Y/Y and
Y/D transformers cannot be paralleled as their secondary voltages will have a
phase difference of 30°. Transformers with +30° and –30° phase shift can,
however, be paralleled by reversing the phase-sequence of one of them.
(iii) The transformers must have the same voltage-ratio to avoid no-load
circulating current when transformers are in parallel on both primary and
secondary sides. Since the leakage impedance is low, even a small voltage
difference can give rise to considerable no-load circulating current and extra I2R
loss.
(iv) There should exist only a limited disparity in the per-unit impedances (on their
own bases) of the transformers. The currents carried by two transformers (also
their kVA loadings) are proportional to their ratings if their ohmic impedances (or
their pu impedances on a common base) are inversely proportional to their
ratings or their per unit impedances on their own ratings are equal. The ratio of
equivalent leakage reactance to equivalent resistance should be the same for all
the transformers. A difference in this ratio results in a divergence of the phase
angle of the two currents, so that one transformer will be operating with a higher,
and the other with a lower power factor than that of the total output; as a result,
the given active load is not proportionally shared by them.

Parallel Transformers on No-load:

Page 11
Malla Reddy College of Engineering and Technology
Department of EEE (2022-23)
Electrical Machines -1 (EM-1): Lecture Notes: (Dr.Karimulla P Sk)
Unit 5: Testing of Transformers &Three Phase Transformers

The parallel operation of transformers can be easily conceived on a per phase


basis. Figure below shows
two transformers paralleled on both sides with proper polarities but on no-load.

Fig: Two transformers connected in parallel

The primary voltages V1 and V2 are obviously equal. If the voltage-ratio of the two
transformers are not identical, the secondary induced emf’s, E1 and E2 though in
phase will not be equal in magnitude and the difference (E1 – E2)will appear across
the switch S. When secondaries are paralleled by closing the switch, a circulating
current appears even though the secondaries are not supplying any load. The
circulating current will depend upon the total leakage impedance of the two
transformers and the difference in their voltage ratios. Only a small difference in
the voltage-ratios can be tolerated.

Parallel Operation of Two Ideal Transformers:

Now we will consider ideal case of two transformers having the same voltage
ratio and their voltage triangles are equal in size and shape.i.e. apart from their
impedances being same, their X/R ratios are also same. The circuit shown in the
figure below consists of two transformers in parallel.
Page 12
Malla Reddy College of Engineering and Technology
Department of EEE (2022-23)
Electrical Machines -1 (EM-1): Lecture Notes: (Dr.Karimulla P Sk)
Unit 5: Testing of Transformers &Three Phase Transformers

Fig: Two ideal transformers connected in parallel

The corresponding phasor diagram is shown in the figure below.

As seen from this figure the impedance voltage triangles of both the
transformers are same. I1 and I2 are the currents flowing through transformers 1
and 2 which are in parallel. These currents are in phase with the load current and
are inversely proportional to the respective impedances.

Fig: Phasor diagram of two ideal transformers connected in parallel

Current sharing : Let us now find out the distribution of the given load current for
this condition between the two transformers. Let I be the total current and I1 and
I2 be the currents shared by the two transformers.

Then applying KCL to the transformer secondaries ,


I = I1 + I2
Secondary voltage,
Page 13
Malla Reddy College of Engineering and Technology
Department of EEE (2022-23)
Electrical Machines -1 (EM-1): Lecture Notes: (Dr.Karimulla P Sk)
Unit 5: Testing of Transformers &Three Phase Transformers

V2 = E - I1 ZA = E – I2Z2
Also I1 Z1 = I2 Z2
I1 / I2 = Z2 / Z1
Applying current divider formulae we get
I1 = I Z2 / (Z1 + Z2 )
and I2 = I Z1 /(Z1 + Z2)

Parallel Operation of Transformers with Equal Voltage Ratios:

Let us now consider the case of two transformers connected in parallel having
equal voltage ratios. The two transformers are having no load secondary voltage
same. i.e. E1 = E2 = E. These voltages are in phase with each other. This is possible
if the magnetizing currents of the two transformers are not much different. With
this case the primaries and secondaries of the two transformers can be connected
in parallel and no current will circulate under no load condition. This is
represented in the figure below.

Fig: Two transformers with equal voltage ratios connected in parallel

If we neglect magnetizing components, the two transformers are represented as


shown in the figure below .

Page 14
Malla Reddy College of Engineering and Technology
Department of EEE (2022-23)
Electrical Machines -1 (EM-1): Lecture Notes: (Dr.Karimulla P Sk)
Unit 5: Testing of Transformers &Three Phase Transformers

Fig: Simplified equivalent circuit neglecting the magnetizing componenets

The phasor diagram under this case is shown in the figure below .

The two impedances Z1 and Z2 are in parallel. The values of Z1 and Z2 are with
respect to secondary.The impedances Z1 and Z2 are equal in magnitude but not in
phase. i.e. the ratios X1/R1 and X2/R2 are not equal.

Fig: Phasor diagram of the two transformers with equal voltage ratios connected in
parallel

Current distribution:

Z1 and Z2 are in parallel therefore the equivalent impedance is given by,


1/Zeq = 1/Z1 + 1/Z2 Zeq = Z1 Z2 /(Z1 + Z2 )
As seen from the phasor diagram
I1 Z1 = I2 Z2 = I Zeq
I1 = I Zeq/ Z1 = I Z2 /( Z1 + Z2 )
Page 15
Malla Reddy College of Engineering and Technology
Department of EEE (2022-23)
Electrical Machines -1 (EM-1): Lecture Notes: (Dr.Karimulla P Sk)
Unit 5: Testing of Transformers &Three Phase Transformers

I2 = I Zeq/ = I Z1/( Z1 + Z2
Multiplying both terms of above equation by voltage V 2,
V2 I1 = V2 I Z2 /( Z1 + Z2 )
V2 I2 = V2 I Z1/( Z1 + Z2)

But V2 I x 10-3 is Q i.e. the combined load in KVA

From this , the KVA carried by each transformer is calculated as,

The above expressions are useful in determining the values of Q1 and Q2 in


magnitude and in phase.

The two transformers work at different power factor. One operates at high p.f.
while the other at low p.f. If the impedances Z1 and Z2 are equal both in
magnitude and quality i.e. ( X1/ R1 = X2/R2 ), then both transformers operate at the
same p.f. which is the p.f. of the load.

Parallel Operation of Transformers with Unequal Voltage Ratios:

Now we will consider the case of two transformers working in parallel and having
unequal voltage ratio. This is shown in the figure below. The voltage ratios of the
two transformers are not equal. The parallel operation under this case is still
possible. But as seen previously there would be a circulating current under no
load condition.

Page 16
Malla Reddy College of Engineering and Technology
Department of EEE (2022-23)
Electrical Machines -1 (EM-1): Lecture Notes: (Dr.Karimulla P Sk)
Unit 5: Testing of Transformers &Three Phase Transformers

Fig: Two transformers with unequal voltage ratios connected in parallel

The phasor diagram of this transformer is shown below:

Fig: Phasor diagram of two transformers with unequal voltage ratios connected in parallel

As can be seen the two transformers apart from having unequal output voltages
E1 and E2 , their impedances Z1 and Z2 and their X/R ratios are also unequal.
Φ1 and Φ2 are the power factor angles of these two transformers and Φ is the
combined p.f. angle.

Current distribution (Load sharing):

Let us consider that the voltage ratio of transformer-1 is slightly more than that of
transformer-2. So the induced e.m.f E1 is greater than E2. Thus the resultant

Page 17
Malla Reddy College of Engineering and Technology
Department of EEE (2022-23)
Electrical Machines -1 (EM-1): Lecture Notes: (Dr.Karimulla P Sk)
Unit 5: Testing of Transformers &Three Phase Transformers

terminal voltage will be (E1 - E2 ) which will cause a circulating current Ic under no
load condition.
Ic = (E1 - E2)/(Z1 + Z2 )
From the circuit diagram we have,
E1 = V2 + I1 Z1
E2 = V2 + I2 Z2
Also, IL = I1 + I2
V2 = IL ZL = ( II + I2 ) ZL
E1 =(I1 + I2) ZL + II Z1 ......................... (1)
E2 =(I1 + I2 ) ZL + I2 Z2 ........................ (2)
Subtracting equation (b) from (a) we have,
E1 - E2 = I1 Z1 - I2 Z2 from which we get
I1 = [(E1 - E2) + I2 Z2] /Z1.................... (3)
and I2 = [ I1 Z1- (E1 - E2)]/Z2 ......................(4)

Substituting the value of I1 from (3) in equation (2) we get :

From which we get: I2 = [E2 Z1 - (E1 - E2)ZL) / (Z1 Z2 + ZL (Z1 + Z2)+ ............ (5)

Similarly by Substituting the value of I2 from equation (4) in equation (1) we get :

E1 = I1Z1 + [ I1 + { I1Z1 – (E1- E2)}/Z2]ZL

From which we get: I1 = [E1 Z2 + (E1 - E2)ZL)/ (Z1 Z2 + ZL (Z1 + Z2)+........... (6)

If impedances Z1 and Z2 are small in comparision with load impedance ZL then the
product Z1 Z2 in the above equations (6) and (5) for currents I1 and I2 may be
neglected so that finally we get the current distribution between the two
transformers as,

Page 18
Malla Reddy College of Engineering and Technology
Department of EEE (2022-23)
Electrical Machines -1 (EM-1): Lecture Notes: (Dr.Karimulla P Sk)
Unit 5: Testing of Transformers &Three Phase Transformers

Where the second term (E1 - E2) / (Z1 + Z2) is the circulating current Ic which adds
to the load shared by transformer-1 whose output voltage E1 is higher and
subtracts from the load shared by transformer-2 whose output E1 is lower. Hence
transformer-1 gets overloaded and the transformers will not share the load
according to their ratings.

Introduction:

Almost all the major power generation and distribution systems in the world
today are three-phase ac systems. Since three-phase systems play such an
important role in modern life, it is necessary to understand how transformers are
used in them.
Transformers for three-phase circuits can be constructed in two ways. One
approach is simply to take three single-phase transformers and connect them in a
three-phase bank. An alternative approach is to make a three-phase transformer
consisting of three sets of windings wrapped on a common core.
These two possible types of transformer construction are shown in the figures
below.
The construction of a single three-phase transformer is the preferred practice
today, since it is lighter, smaller, cheaper, and slightly more efficient. The older
construction approach was to use three separate transformers. That approach
had the advantage that each unit in the bank could be replaced individually in the
event of trouble, but that does not outweigh the advantages of a combined three

Page 19
Malla Reddy College of Engineering and Technology
Department of EEE (2022-23)
Electrical Machines -1 (EM-1): Lecture Notes: (Dr.Karimulla P Sk)
Unit 5: Testing of Transformers &Three Phase Transformers

phase unit for most applications. However, there are still a great many
installations consisting of three single-phase units in service.

Fig: A three-phase transformer bank composed of independent transformers.

Fig: A three-phase transformer wound on a single three-legged core.

Three-Phase Transformer Connections:


A three-phase transformer consists of three transformers, either separate or
combined on one core. The primaries and secondaries of any three-phase
transformer can be independently connected in either a Wye (Y) or a Delta (∆ ).
This gives a total of four possible connections for a three-phase transformer bank:
I. Wye(star)-Wye(star) (Y-Y)
2. Delta-Delta (∆-∆)
3. (star)Wye-Delta (Y -∆)

Page 20
Malla Reddy College of Engineering and Technology
Department of EEE (2022-23)
Electrical Machines -1 (EM-1): Lecture Notes: (Dr.Karimulla P Sk)
Unit 5: Testing of Transformers &Three Phase Transformers

4. Delta-Wye(star) (∆-Y)

The key to analyzing any three-phase transformer bank is to look at a single


transformer in the bank. Any single transformer in the bank behaves exactly like
the single-phase transformers already studied. The impedance, voltage regulation,
efficiency, and similar calculations for three-phase transformers are done on a
per-phase basis, using exactly the same techniques already developed for single-
phase transformers.
The advantages and disadvantages of each type of three-phase transformer
connection are explained below along with the relevant connection diagrams.

WYE-WYE CONNECTION:
The Y-Y connection details of three-phase transformers are shown in the figure
below.

Figure (a): Star-Star(Y-Y) connection representation

Page 21
Malla Reddy College of Engineering and Technology
Department of EEE (2022-23)
Electrical Machines -1 (EM-1): Lecture Notes: (Dr.Karimulla P Sk)
Unit 5: Testing of Transformers &Three Phase Transformers

Figure (b): Star-Star(Y-Y) connection Phasor diagram

Figure (c): Star-Star(Y-Y) connection wiring diagram


In a Y-Y connection, the primary voltage on each phase of the transformer is given
by VØP = VLP / √3. The primary-phase voltage is related to the secondary-phase
voltage by the turns ratio of the transformer. The phase voltage on the secondary
is then related to the line voltage on the secondary by VLS = √3VØS . Therefore,
overall the voltage ratio ‘a’ of the transformer is then given by:

Advantage/Application: This is useful and economical for low power high voltage
transformers because the phase voltage is 1/√3 times the line voltage. Hence the
number of turns per phase and the strength of insulation required would be less.

Disadvantages:
1. If loads on the transformer circuit are unbalanced, then the voltages on the
phases of the transformer can become severely unbalanced.
2. Third-harmonic voltages can be large.

Page 22
Malla Reddy College of Engineering and Technology
Department of EEE (2022-23)
Electrical Machines -1 (EM-1): Lecture Notes: (Dr.Karimulla P Sk)
Unit 5: Testing of Transformers &Three Phase Transformers

When a three-phase set of voltages is applied to a Y - Y transformer, the voltages


in any phase will be 1200 apart from the voltages in any other phase. However,
the third-harmonic components of each of the three phases will be in phase with
each other, since there are three cycles in the third harmonic for each cycle of the
fundamental frequency. There will always be some third-harmonic components in
a transformer because of the nonlinearity of the core, and these components add
up.
The result is a very large third-harmonic component of voltage on top of the 50 or
6O-Hz fundamental voltage. This third-harmonic voltage can be larger than the
fundamental voltage itself.
Both the unbalance problem and the third-harmonic problem can be solved using
one of the two following techniques:
1. Solidly ground the neutrals of the transformers, especially the primary winding’s
neutral. This connection permits the additive third-harmonic components to
cause a current flow in the neutral instead of building up large voltages. The
neutral also provides a return path for any current imbalances in the load.
2. Add a third (tertiary) winding connected in ∆ to the transformer bank. If a third
∆ connected winding is added to the transformer, then the third-harmonic
components of voltage in the ∆ will add up, causing a circulating current flow
within the winding. This suppresses the third-harmonic components of voltage in
the same manner as grounding the transformer neutrals.
The ∆ connected tertiary windings need not even be brought out of the
transformer case, but they often are used to supply lights and auxiliary power
within the substation where it is located. The tertiary windings must be large
enough to handle the circulating currents, so they are usually made about one-
third the power rating of the two main windings.

Page 23
Malla Reddy College of Engineering and Technology
Department of EEE (2022-23)
Electrical Machines -1 (EM-1): Lecture Notes: (Dr.Karimulla P Sk)
Unit 5: Testing of Transformers &Three Phase Transformers

One or the other of these correction techniques must be used any time a Y-Y
transformer is installed. In practice, very few Y-Y transformers are used, since the
same jobs can be done by one of the other types of three-phase transformers.

DELTA-DELTA CONNECTION:
The ∆- ∆ connection details are shown in the figure below.

Figure (a): Delta-Delta (∆-∆) connection representation

Figure (b): Delta-Delta (∆-∆) connection Phasor diagram

Page 24
Malla Reddy College of Engineering and Technology
Department of EEE (2022-23)
Electrical Machines -1 (EM-1): Lecture Notes: (Dr.Karimulla P Sk)
Unit 5: Testing of Transformers &Three Phase Transformers

Figure (c): Delta-Delta (∆-∆) connection wiring diagram

In a ∆- ∆ connection, VLP = VØP and VLS = VØS, so the relationship between primary
and secondary line voltages is given by:

Advantages/Application:
 Transformers with this configuration are economical for high power low
voltage application since the number of turns required for a given line
voltage are more (since line voltage is same as phase voltage)
 This transformer has no phase shift associated with it and no problems with
unbalanced loads or harmonics.
 For the secondary voltage to be perfect sinusoidal the magnetizing currents
must contain third harmonic components. The Delta configuration provides
a closed path for the circulation of third harmonic components of current.
Hence the flux remains sinusoidal thus resulting in better sinusoidal
voltages.

Page 25
Malla Reddy College of Engineering and Technology
Department of EEE (2022-23)
Electrical Machines -1 (EM-1): Lecture Notes: (Dr.Karimulla P Sk)
Unit 5: Testing of Transformers &Three Phase Transformers

 The phase current is lesser than the Line current (by 1/√3). Hence the
conductor cross sectional area can be smaller thus resulting in saving of
conductor material.

Disadvantages:

 Due to the nonavailability of the Neutral point this configuration is not


suitable for three phase four wire systems
WYE-DELTA CONNECTION:
The Y - ∆ connection details of three-phase transformers is shown in the figures
below. In this connection, the primary line voltage is related to the primary phase
voltage by VLP = √3VØP while the secondary line voltage is equal to the secondary
phase voltage VLS = VØS . The voltage ratio of each phase is:

VØP / VØS = a
so the overall relationship between the line voltage on the primary side of the
bank and the line voltage on the secondary side of the bank is:

VLP / VLS = √3VØP / VØS = √3 a

Page 26
Malla Reddy College of Engineering and Technology
Department of EEE (2022-23)
Electrical Machines -1 (EM-1): Lecture Notes: (Dr.Karimulla P Sk)
Unit 5: Testing of Transformers &Three Phase Transformers

Figure (a): Wye-Delta (Y-∆) connection representation

Figure (b): Wye -Delta (Y-∆) connection Phasor diagram

Page 27
Malla Reddy College of Engineering and Technology
Department of EEE (2022-23)
Electrical Machines -1 (EM-1): Lecture Notes: (Dr.Karimulla P Sk)
Unit 5: Testing of Transformers &Three Phase Transformers

Figure (c): Wye -Delta (Y-∆) connection wiring diagram

Advantages/Application:
 This connection is advantageous/economical for high power high voltage
step down power transformers. Primary in star configuration can be used
for higher voltage since line voltage is √3 times the phase voltage and thus
the number of turns required per phase will be lesser for a higher line
voltage. The delta side with lower line voltage (line voltage being equal to
phase voltage) can be used as secondary.
 The neutral available in primary can be earthed to avoid distortion
 Hence transformers with this type of connection are used in the main
receiving end of a transmission line where a step down transformer is
required.
 The Y - ∆ connection has no problem with third-harmonic components in its
voltages, since they are consumed in a circulating current on the ∆ side.
 This connection is also more stable with respect to unbalanced loads, since
the ∆ partially redistributes any imbalance that occurs. i.e. Load side
(secondary) large unbalanced loads can be handled satisfactorily.
Disadvantages:
 This arrangement has one problem. Because of this type of connection,
the secondary voltage is shifted 300 relative to the primary voltage of the
transformer which can be further positive shift or negative shift. The fact
that a phase shift has occurred can cause problems in paralleling the
secondaries of two transformer banks together. The phase angles of
transformer secondaries must be equal if they are to be paralleled, which
means that attention must be paid to the direction of the 30 0 phase shift
occurring in each transformer bank to be paralleled together.

Star/Delta (Y/D) Connection (Alternate explanation for Phase grouping):


Star connection is formed on primary side by connecting together 1 suffixed
terminals with 2 suffixed terminals connected to appropriate lines. The delta is

Page 28
Malla Reddy College of Engineering and Technology
Department of EEE (2022-23)
Electrical Machines -1 (EM-1): Lecture Notes: (Dr.Karimulla P Sk)
Unit 5: Testing of Transformers &Three Phase Transformers

formed by connecting c1a2, a1b2 and b1c2 with the lines connected to these
junctions being labeled as a, b and c respectively as shown in Fig. (a). The phasor
diagram is drawn in Fig. (b). It is seen from the phasor diagram on the delta side
that the sum of voltages around delta is zero. This is a must as otherwise closed
delta would mean a short circuit. It is also observed from the phasor diagram that
phase a to neutral voltage (equivalent star basis) on the delta side lags by – 30° to
the phase-to-neutral voltage on the star side. This is also the phase relationship
between the respective line to- line voltages. This connection, therefore, is known
as – 30°-connection. Or YD1 representing 1O clock position. With this notation
secondary Delta lags the primary star by 300.

Fig: -300 connection Fig: +300


connection

The + 30°-connection follows from the phasor diagram of Fig. (a) above with the
corresponding connection diagram shown in Fig. (b).

Delta/Star (D/Y) Connection (Alternate explanation for Phase grouping):


This connection is simply the interchange of primary and secondary roles in the
star/delta connection. One just interchanges capital and small letter suffixing in

Page 29
Malla Reddy College of Engineering and Technology
Department of EEE (2022-23)
Electrical Machines -1 (EM-1): Lecture Notes: (Dr.Karimulla P Sk)
Unit 5: Testing of Transformers &Three Phase Transformers

the above figures .But what was the – 30°-connection will now be the + 30°-
connection and vice versa.
DELTA-WYE CONNECTION:
∆-Y connection details of three-phase transformers are shown in the figures
below. In a ∆-Y connection, the primary line voltage is equal to the primary-phase
voltage VLP = VØP , while the secondary voltages are related by VLS = √3VØS .
Therefore, the line-to-line voltage ratio of this transformer connection is given by
:

Figure (a): Delta - Wye (∆-Y) connection representation

Page 30
Malla Reddy College of Engineering and Technology
Department of EEE (2022-23)
Electrical Machines -1 (EM-1): Lecture Notes: (Dr.Karimulla P Sk)
Unit 5: Testing of Transformers &Three Phase Transformers

Figure (b): Delta - Wye (∆-Y) connection Phasor diagram

Figure (c): Delta - Wye (∆-Y) connection wiring diagram

Advantages/Application:
 This connection is advantageous/economical for high power high voltage
step up power transformers. Primary in Delta configuration can be used for
lower voltage (line voltage being equal to phase voltage) And secondary in
Wye (star) configuration can be used for higher voltage since line voltage is
√3 times the phase voltage and thus the number of turns required per
phase will be lesser for a higher line voltage.
 Hence transformers with this type of connection are used at the starting
(Generating station) end of a transmission line where a step up transformer
is required.

Page 31
Malla Reddy College of Engineering and Technology
Department of EEE (2022-23)
Electrical Machines -1 (EM-1): Lecture Notes: (Dr.Karimulla P Sk)
Unit 5: Testing of Transformers &Three Phase Transformers

Disadvantages:
This connection has the same disadvantages and the same phase shift as the Y - ∆
transformer. The connection shown in the figure above makes the secondary
voltage differ the primary voltage by 30° as in Y -∆.

The Open ∆ (or V-V) Connection:


In some situations a full transformer bank may not be used to accomplish three
phase transformation. For example, suppose that a ∆ - ∆ transformer bank
consisting of three separate transformers has a damaged phase which has to be
removed for repair.

The resulting configuration is known as open ∆ (or V-V) Connection and is shown
in the figure below.

Figure: Third transformer (VBR) removed from the Three transformer Bank and
the corresponding Phasor diagram

Page 32
Malla Reddy College of Engineering and Technology
Department of EEE (2022-23)
Electrical Machines -1 (EM-1): Lecture Notes: (Dr.Karimulla P Sk)
Unit 5: Testing of Transformers &Three Phase Transformers

If the open Delta primary is now excited from a balanced three phase supply, then
the voltage across the gap where the third transformer used to be would exactly
be the same voltage that would be present if the third transformer were still
there .
Thus, the open-delta connection lets a transformer bank work as a three phase
transformer with only two transformers, allowing some reduced power flow to
continue even with a damaged phase removed.
Power Delivered in Open Delta configuration:
How much apparent power can the bank supply with one of its three
transformers removed? At first, it seems that it could supply two-thirds of its
rated apparent power, since two-thirds of the transformers are still present.
Things are not that simple. To understand what happens when a transformer is
removed, let us see the figures (a) and (b) below. Figure (a) shows ∆ - ∆
connection and figure (b) shows V-V connection.

Page 33
Malla Reddy College of Engineering and Technology
Department of EEE (2022-23)
Electrical Machines -1 (EM-1): Lecture Notes: (Dr.Karimulla P Sk)
Unit 5: Testing of Transformers &Three Phase Transformers

Fig(a) : ∆ - ∆ connection Fig(a) : V- V


connection

We know that the power output from a three phase system is √3 VLIL Cos φ
where Cos φ is the power factor. Hence in figure (a) ∆ - ∆ capacity = √3 VLIL Cos
φ = √3 VL √3IPh Cos φ (since IL = √3IPh)

= 3VL IPh Cos φ


But in figure (b) V-V capacity = √3 VLIL Cos φ = √𝟑 VL IPh Cos φ (since IL = IPh)

Therefore V-V capacity / ∆ - ∆ capacity = √𝟑 VL IPh Cos φ / 𝟑 VL IPh Cos φ = 1/√𝟑


= 0.577=57.7%

Summary conclusion:

The total load carried by an open ∆ (V- V) connection is 57.7 % of the total
capacity of the ∆- ∆ and not 2/3rd (66.6 % ) of the total capacity of the ∆- ∆ as
might be expected. Thus for example, in a ∆- ∆ bank of three transformers each of
100kVA if one transformer is removed, then the total capacity of the resulting V-V
bank becomes 57.7 % of 300kVA i.e. 173.2kVA and not 200kVA.
In other words in V-V configuration the resultant capacity becomes 86.6% of the
rated capacity of the two transformers put together. i.e. 0.866 x 200kVA = 173.2
kVA .

The factor 0.866 is called utility factor


= (operating capacity of the transformers in V-V) / (Available capacity of the
transformers in V-V)

V – V Configuration is intentionally employed in the following applications:

Page 34
Malla Reddy College of Engineering and Technology
Department of EEE (2022-23)
Electrical Machines -1 (EM-1): Lecture Notes: (Dr.Karimulla P Sk)
Unit 5: Testing of Transformers &Three Phase Transformers

 When the three phase load is too small to warrant the installation of full
three phase transformer bank.
 When one of the three transformers in a ∆ - ∆ bank is disabled service is at
a reduced capacity is adequate , till the faulty transformer is repaired and
restored.
 When it is expected that in the future , the total load will increase
necessitating the closing of the open ∆.
Illustrative examples:
Example1: The following test results were obtained for a 20 kVA,2400/240 V
distribution transformer : O.C test(l.v side)240 V,1.066 A,126.6 WS.C test (h.v
side) 57.5V, 8.34 A, 284 W Calculate a) Equivalent circuit parameters when
referred to h.v side and draw it. b) Efficiency of the transformer at half full-load
with 0.8 power factor lagging. C) Regulation at full load 0.8 power factor lagging.

( JNTU May-15)

Solution : From O.C test , V0 = 240 V, I0 = 1.066 A, W0 = 126.6 W,


Measurements are on LV, side i.e. secondary hence results will give parameters
referred to secondary.
W0 126.6
∴ cosØ0 =   0.4948, 0  60.34o
V0 I 0 2401.066
IC  I 0 cos0  1.066 0.4948  0.5275A
i)
I m  I 0 sin 0  1.066 0.8689  0.9263A
V 240
R ' 0   454.97
0
I 0 0.5275

V0
X '  259.095
0
Im
V1 2400
K   10
V2 240
R0  R0 'K 2  45.497k

Page 35
Malla Reddy College of Engineering and Technology
Department of EEE (2022-23)
Electrical Machines -1 (EM-1): Lecture Notes: (Dr.Karimulla P Sk)
Unit 5: Testing of Transformers &Three Phase Transformers

X 0  X 0 ' xK 2  25.91k .................... Referred to h.v side

From S.C. test, Vsc = 57.5 V, Isc = 8.34 A, Wsc = 284 W


The meters are on h.v. side hence we get parameters referred to h.v. side.

Wsc 284
R1e    4.083k
2
I le 8.342
V
∴ Z1e  I sc  6.8944
sc

X 1e  Z 1e2  R1e2  5.555

The equivalent circuits referred to h.v. side is shown in the figure below.

ii) From O.C test,


Pi = iron loss = 126.6 W
From S.C test Pcu = 284 W for Isc = 8.34 A
VA 20103
I 1 (FL)    8.34A
V1 2400

As I1(FL) = Isc, Wsc = Pcu(FL) = 284 W


nVAcos
∴ % HL  nVAcos  P  N 2 P (FL) 100 …..n = 0.5 on Half load
i cu

Page 36
Malla Reddy College of Engineering and Technology
Department of EEE (2022-23)
Electrical Machines -1 (EM-1): Lecture Notes: (Dr.Karimulla P Sk)
Unit 5: Testing of Transformers &Three Phase Transformers

0.5  20103  0.8


 100
0.5  20103  0.8  126.6  [0.52  284]

= 97.589 %

iii) I1(FL) = 8.34 A, V1 = 2400 V, cos Ø = 0.8

I1 (FL)[R1e cos  X1e sin]


∴ %R  100
V1

8.34[4.083 0.8  5.55 0.6]


 100
2400

= 2.293 %

Example 2 : Two similar 200 kVA, single phase transformers gave the following
results in Sumpner’s test : Mains wattmeter W1 = 4 kW, Series wattmeter W2 = 6
kW at full load current. Find out individual transformer efficiencies at i) Full load
at unity p.f.and ii) Half load at 0.8 p.f. lead. (JNTUK April-12)

Solution : The given values are: Rating = 200 kVA, W1 = 4 kW, W2 = 6 kW


W1 = iron loss of both the transformers = 4 kW
W1 4
∴ Pi = Iron loss for each transformer =  = 2 kW
2 2
W2 = Full load copper loss for both the transformers = 6 kW
W2 6
∴ (P )F.L. = Full load copper loss for each transformer =  = 3 kW
cu
2 2

i) At full load,
VAratingcos2
% 100 with cos Ø2 = 1
VAratingcos2  Pi  ( pcu )F.L

Page 37
Malla Reddy College of Engineering and Technology
Department of EEE (2022-23)
Electrical Machines -1 (EM-1): Lecture Notes: (Dr.Karimulla P Sk)
Unit 5: Testing of Transformers &Three Phase Transformers

200103 1
 100
200103 1  2 103  3103

= 97.56 %
1
ii) At half load, cos Ø2 = 0.8 and n = = 0.5
2

∴ %   n  (VArating)  cos2
n  (VArating)  cos  P  n2  (P )F.L 100
2 i cu

(Pcu)H.L. = n2 ×(Pcu) F.L. where n = Fraction of full load

0.5  200103  0.8


∴ %   100
0.5  200103  0.8  2 103  (0.5)2  3 103

= 96.67 %

Example 3: A 200 V, 60 Hz single phase transformer has hysteresis and eddy


current losses of 250 watts and 90 watts respectively. If the transformer is now
energised from 230 V, 50 Hz supply. Calculate its core losses. Assume
Steinmentz’s constant equal to 1.6 . (JNTUK April-12)

Solution : Let Ph = Hysteresis loss and Pe = Eddy current loss


Then they are given by Ph = Kh f Bmx where x= Steinmentz’s constant =1.6
and Pe = Ke f2 B m2
E = 4.44 f N BmA where Øm = BmA
E
∴ B α i.e. P = K f (E/f)1.6
m h h
f
and Pe = Kef2 (E/F)2
For E = 200 V, f = 60 Hz, Ph = 250 W and Pe = 90 W

Page 38
Malla Reddy College of Engineering and Technology
Department of EEE (2022-23)
Electrical Machines -1 (EM-1): Lecture Notes: (Dr.Karimulla P Sk)
Unit 5: Testing of Transformers &Three Phase Transformers
1.6
 200 
250 = Kh ×60×   i.e. Kh = 0.607
 60 
2
2  200
90 = Ke × 60    i.e. Ke = 0.00225
 60 
Now E = 230 V, f = 50 Hz
1.6
 230 
∴ Ph = 0.607 ×50×   = 348.795 W
 50 
2
2  230 
Pe = 0.00225 ×50 ×   = 119.025 W
 50 
∴ Core loss = Ph + Pe = 467.82 W at 230 V, 50 Hz

Example 4: In a test for determination of the losses of a 440V,50 Hz transformer,


the total iron losses were found to be 2500 W at normal voltage and frequency.
When the applied voltage and frequency were 220 V and 25 Hz, the iron losses
were found to be 850W. Calculate the eddy current loss at normal voltage and
frequency. (JNTU Feb-10 ,May-15)

Solution : We have the data: V1 = 440 V, f1= 50 Hz, V2 = 220 V, f2 = 25 Hz

V1 = 8.8 and V2
∴ = 8.8
f1 f2

Thus V/f is constant hence flux density Bm remains constant.

∴ Ph = A f and Pe = B f2

∴ Pi = Ph + Pe = A f + B f 2

∴In test - 1: 2500 = A x 50 + B x 2500 ………(1)


And in test - 2 850 = A x 25 + B x 625 ...................... (2)

Solving we get A = 18 and B = 0.64

Page 39
Malla Reddy College of Engineering and Technology
Department of EEE (2022-23)
Electrical Machines -1 (EM-1): Lecture Notes: (Dr.Karimulla P Sk)
Unit 5: Testing of Transformers &Three Phase Transformers

Thus eddy current loss a normal voltage and frequency is,

Pe = B f2 = 0.64 × (50)2 = 1600 W

Example 5:Two single phase transformer with equal turns have impendance of
(0.5 +j3) ohm and (0.6 +j10) ohm with respect to the secondary.If they operate in
parallel, determine how they will share a total load of 100 kW at p.f. 0.8 lagging ?

( JNTU April-04, Nov-04)

Solution : We have the data: Z1 = 0.5 + j3 Ω, Z2 = 0.6 +j10 Ω,

Total load = 100 kW


p.f. of load = 0.8 lag.
Cos Ø = 0.8, Ø = cos -1 0.8 = 36.860 lag
100
kVA of load = = 1245
0.8
Hence Q = 125/-36.860 kVA
 Z2 
Load shared by transformer 1 = Q  
 1
Z  Z 2 
0 
= [125 / -36.86 ]  0.6  j10 
(0.5  j3)  (0.6  j10) 
 
0
(125/_- 36.860)(10.017/_ 86.56 )
=
1.1  j13
1252.125/_ 49.70 0
= = 95.97 /-35.46 kVA
13.046/_ 85.160

p.f. = cos 35.460 = 0.8145 lag


 Z 2 


Load shared by transformer 2 = Q  
 1
Z  Z 2 

Page 40
Malla Reddy College of Engineering and Technology
Department of EEE (2022-23)
Electrical Machines -1 (EM-1): Lecture Notes: (Dr.Karimulla P Sk)
Unit 5: Testing of Transformers &Three Phase Transformers


= [125 /- 36.860]  0.5  j3 
(0.5  j3)  (0.6  j10) 
 

(125/_ 36.860 )(3.041/_ 80.530 )
=
13.046  85.160

380.125/_ 43.670 0
= 0 = 29.13 /- 41.49 kVA
13.046/_ 85.16

Example 6: Two single phase transformer A and B of equal voltage ratio are
running in parrallel and supplying a load of 1000 A at 0.8 p.f. lag. The equivalent
impedance of the two transformers are (2+j3)and (2.5+j5) ohms respectively.
Calculate the current supplied by each transformer and the ratio of the kW output
of the two transformer.

Solution : For transformer A, ZA = 2+ j 3 Ω


For transformer B, ZB = 2.5+ j 5 Ω
Current supplied by transformer A,
I.ZB
IA =
Z A  ZB
……..(1)
Current supplied by transformer B,
I.Z A
IB =
Z A  ZB
....…..(2)
Taking ratio of equations (1) and (2) we have,
I Z 2.5  j5 5.5901/_ 63.430
A
 B
 
 

IB ZA 2  j3 3060555/_ 56.300
= 1.5504 /7.130
IA
∴ = 1.54+j 0.1924 I = (1.54 + j 0.1924) I
A B
IB
Total current I = 1000 A and p.f. of total current = 0.8 lag

Page 41
Malla Reddy College of Engineering and Technology
Department of EEE (2022-23)
Electrical Machines -1 (EM-1): Lecture Notes: (Dr.Karimulla P Sk)
Unit 5: Testing of Transformers &Three Phase Transformers

∴ Cos Ø = 0.8, Ø = cos -1 0.8 = 36.860


∴ I = 1000 /- 36.860 A = (800-j 600) A
We have, I = IA + IB
∴ 800 - J 600 = [1.54 + j 0.1924] IB + IB
= [2.54 + j 0.1924] IB
800  j600 1000/_ 36.860
∴ IB = 
2.54  j0.1924 2.5472/_ 4.330
= 392.58 / - 41.190 A
∴ IB = 392.58 /- 41.190 A = 295.42 -j258.53 A
Now, IA = (1.54+ j 0.1924) IB
= [1.5519 <7.120][392.58<-41.190]
∴ IA = 609.24 /- 34.070 A = 504.66 - j 341.29 A
The ratio of kW outputs is nothing but the ratio of inphase components of the
two currents.
outputofTransformerA 504.66 = 1.7

outputofTransformerB 295.42

Example 7: Two transformers A and B are connected in parallel to a load of (2+


j1.5) Ω Their impedance in secondary terms are ZA = (0.15+j0.5) Ω and ZB =
(0.1+j0.6) Ω.Their no load terminal voltages are E A = 207 /00 V, EB = 205 /00 volts.
Find the power output and power factor of each transformer.

Solution : We have the data : ZL = (2 + j1.5 )Ω , ZA = (0.15 + j0.5)Ω , ZB = (0.1 +


j0.6) Ω , EA = 207 /00 volts and EB = 205 /00 volts.
Using the formulae for IA and IB

E A Z B  Z L (E A  EB )
IA =
Z A Z B  Z L (Z A  Z B )
And IB = EB Z A  Z L (E A  EB )
Z A Z B  Z L (Z A  Z B )

Page 42
Malla Reddy College of Engineering and Technology
Department of EEE (2022-23)
Electrical Machines -1 (EM-1): Lecture Notes: (Dr.Karimulla P Sk)
Unit 5: Testing of Transformers &Three Phase Transformers

[207/_ 00 ][0.1  j0.6]  (2  j1.5)[207/_ 00  205/_ 00 ]


IA =
(0.15  j0.5)(0.1  j0.6)  (2  j1.5)[(0.15  j0.5)  (0.1  j0.6)]

Simplifying we get IA = (42.196 /-38.840) A =(32.866-j26.463) A

and similarly we get I B = EB Z A  Z L (E A  EB )


Z A Z B  Z L (Z A  Z B )
[205/_ 0 ][0.15  j0.5]  (2  j1.5)[207/_ 00  205/_ 00 ]
0
=
(0.15  j0.5)(0.1  j0.6)  (2  j1.5)[(0.15  j0.5)  (0.1  j0.6)]
Solving, IB = (33.5534 /-42.890) A
= (24.5832 - j 22.8362) A
Now total current is given by,
IL = IA + IB
= (32.866 - j 26.463) + (24.5832 - j 22.8362)
= (57.4492 - j 49.2992) A
= 75.70 /-40.630 A
The load voltage, VL = IL ZL = (75.70 /- 40.630)(2+j 1.5)
= (75.70 /- 40.630) (2.5 /36.860)
= 189.25 /- 3.770 volts
The angle between VL and IA can be calculated as,
ØA = (-38.840) - (-3.770) = -35.070
∴ p.f. = cos ØA = cos (35.07)
= 0.8184 (lagging)
The angle between VL and IB can be Calculated as,
ØB = (-42.890) - (-3.770) = -39.120
p.f. = cos ØB = cos (39.12)
= 0.7758 (lagging

Page 43
Malla Reddy College of Engineering and Technology
Department of EEE (2022-23)
Electrical Machines -1 (EM-1): Lecture Notes: (Dr.Karimulla P Sk)
Unit 5: Testing of Transformers &Three Phase Transformers

Power output of transformer A = VL IA cos ØA


= 189.25×42.196×0.8184
= 6535.40 W = 6.5354 Kw

Power output of transformer B = VL IB cos ØB


= (189.25) (33.5534) (0.7758)
= 4926.31 W = 49263 kW

Illustrative Examples on three phase transformers:

Example 1: An ideal 3-Ø step down transformer connected in delta/star delivers


power to a balanced 3 - ∅ load of 120 Kva at 0.8 pf. The input line voltage is 11 Kv
and the turn’s ratio of transformer (Phase to Phase ) is 10. Determine the line
voltage, line currents, and phase voltages, phase currents on both primary and
secondary sides.

Page 44
Malla Reddy College of Engineering and Technology
Department of EEE (2022-23)
Electrical Machines -1 (EM-1): Lecture Notes: (Dr.Karimulla P Sk)
Unit 5: Testing of Transformers &Three Phase Transformers

Solution:
𝑁1 𝑉𝑝 ℎ 1
∴ K= = 10 = and V = 11 Kv
ph1
𝑁2 𝑉𝑝 ℎ 2

𝑉𝑝 ℎ 1 11 ×10 3
∴ Vph2 = = 10
= 1100 V
10

∴ VL2 = √3 Vph2 = 1.9052 kV


Load VA = √3 VL2 IL2 and is given as 120kVA
120 ×103
i.e. IL2 =
√3 ×1.9052 ×10 3
∴ IL2 = 36.36 A
∴ Iph2 = IL2 = 36.36 A
𝐼𝑝 ℎ 2 36.36
K=
𝐼𝑝 ℎ 1
= 10 i.e. Iph1 = 10
= 3.636 A

∴ IL1 = √3 Iph1 = √3 × 3.636 = 6.298 A


Example 2: A bank of three single phase transformers has its h.v. terminals
connected to 3 wire, 3-phase, 11kV system. It’s l.v. terminals are connected to a 3
wire, 3-phase load rated at 1500 kVA , 2200 V. specify the voltage, current and
kVA ratings of each transformer for both h.v

and l.v windings for the following connections.


i) Y -Δ ii) Δ - Y iii) Y - Y

Solution:
The load is 1500 kVA hence the rating of each transformer is (1500/3) = 500 kVA
and is same for all configurations. Since input is 11kV and output is 2200V, in all
configurations the line voltages are also same on both primary side and secondary
side. . i.e. VL1=11kV and VL2 = 2,200 V. The voltage and current ratings are
specified on Phase basis only and hence lets us find out VPh and I Ph on both HV
side and LV side for the given four configurations.

Page 45
Malla Reddy College of Engineering and Technology
Department of EEE (2022-23)
Electrical Machines -1 (EM-1): Lecture Notes: (Dr.Karimulla P Sk)
Unit 5: Testing of Transformers &Three Phase Transformers

i) 11 kV Y and 2200 V Δ
11103
Vph1 = = 6350.8529 V
3
Vph1 Iph1 = 500 × 103 i.e. Iph1 = 78.729 A
Vph2 = 2200 V
Vph2 Iph2 = 500 × 103 i.e. Iph2 = 227.2727 A
ii) 11 kV Δ and 2200 V Y
2200
Vph1 = 11 kV and Vph2 = 3 = 1270.1705 V
500103
∴ Iph1 = 11103 = 45.45 A
500103
∴ Iph2 = 1270.1705 = 393.6479 A
iii) 11 kV Y and 2200 V Y
11103 2200
Vph1 = 3 = 6350.8529 V, Vph2 = 3 = 1270.1705 V
500103
∴ Iph1 = 6350.8529 = 78.729 A
1500103
∴ Iph2 = 1270.1705 = 393.6479 A

Example 3 : A 3-Ø, 1200 kVA , 6.6/1.1kV transformer has Delta/Star connection.


The per phase resistance is 2 Ω and 0.03 Ω on primary and secondary respectively.
Calculate the efficiency on full load at 0.9 p.f. lagging, if iron losses are 20 kW.

Solution : VL1 = 6.6 kV, VL2 = 1.1 kV, Vph1 = VL1, Vph2 = VL2/√3

1200103 1200103
I1(FL) = 3VL1 = 3  6.6 103 = 104.9727 A (line value )

∴ I (Ph) = I1 (FL) = 60.606 A


1 3

Page 46
Malla Reddy College of Engineering and Technology
Department of EEE (2022-23)
Electrical Machines -1 (EM-1): Lecture Notes: (Dr.Karimulla P Sk)
Unit 5: Testing of Transformers &Three Phase Transformers

K = (VPh1/VPh2)= (6.6 x 103)/ (1.1 x 103/√3) =10.393

∴ R1e = R1 + R’2 = R1+K2R2 = 2 + (10.393)2(0.03) x = 5.24 Ω

∴ Pcu(FL) = 3× [I1(ph)]2 ×R1e = 3 × (60.606)2 × 5.24 = 57740.9313 W


VA cos
∴ %ղFL = VA cos  P  P (FL) 100
i cu

1200103  0.9 
  100
= 1200103  0.9  20103  57740.9313 = 93.285%

Example 4: A 5000 kVA, 3 phase transformer 6.6 /33 kV, Δ/ Y has a no load loss of
15 kW and full load of 50 kW. The impedance drop at full load is 7%. Calculate the
primary voltage when a load of 3200 kW at 0.8 p.f. is delivered at 33 kV.

Solution: Secondary is star connected with VL2 = 33 kV.

VA  5000103
∴ IL2 =  = 87.4773 A = Iph2
3VL2 3  33103

7 3
Impedance drop per phase = 7 % of V = × 3310 = 1333.6791 V
Ph2
100 3

1333.6791 1333.6791
∴ Z2e =  = 15.2459 Ω/ph
I ph2 87.4773

Pcu(FL) = Total loss - No load loss = 50 -15 = 35 kW

35103
But, Pcu(FL) = 3 × I 22ph R2e i.e. R2e = = 1.524 Ω
3  (87.4773)2

∴ X2e = Z 2e2  R 2e2 = 15.169 Ω/ph

When load is P2 = 3200 kW, cos Ø = 0.8, VL2 = 33 kV

Page 47
Malla Reddy College of Engineering and Technology
Department of EEE (2022-23)
Electrical Machines -1 (EM-1): Lecture Notes: (Dr.Karimulla P Sk)
Unit 5: Testing of Transformers &Three Phase Transformers

3200103

IL2 = P2 = 69.98 A = I2ph
3VL2 cos 3  33103  0.8


I 2 ph [R2e cos  X 2e sin]
∴ %R= × 100
V2 ph

69.98[1.524 0.8  15.169 0.6]


= × 100 = 3.79 %
(33103 / 3

Thus primary voltage must be increased by 3.79 % to maintain 33 kV at the


secondary.

∴ V1 = 6.6+3.79 % of 6.6 = 6.8501 kV

Example 5 : A 500 kVA, 3-phase, 50 Hz transformer has a voltage ratio (line


voltage) of 33/11 kV and is delta/star connected. The resistances per phase are :
high voltage 35 Ω, low voltage 0.876 Ω and the iron loss is 3050 W. Calculate the
value of efficiency at full load and one-half of full load with 0.8 lagging power
factor.

Solution : Since primary is Delta : VL1 = Vph1 = 33 kV

11
Since Secondary is Star: VL2 = 11 kV, Vph2 = = 6.35 kV
3

Vph1  33
∴ K=  = 5.1975
Vph2 6.35

R1 = 35 Ω, R2 = 0.876 Ω

∴ R1e = R1 + R’2 = R1 + K 2 R2 = 58.6643 Ω (R’2 = R2 referred to


primary)
VA 500103 I (FL)
I1(FL) =  = 8.7477 A Therefore : I1ph(FL)= 1

3VL1 3  3310 3
3

Page 48
Malla Reddy College of Engineering and Technology
Department of EEE (2022-23)
Electrical Machines -1 (EM-1): Lecture Notes: (Dr.Karimulla P Sk)
Unit 5: Testing of Transformers &Three Phase Transformers

I1 (FL)
∴ Pcu (FL) = 3× I12ph (FL) × R1e and I1ph =
3
2
 8.7477 
=3×   × 58.6643 = 4489.1245 W
 3 

∴ %ղFL = VA cos
× 100
VA cos  P  P (FL)
i cu

500103  0.8
= × 100
500103  0.8  3050 4489.12

= 98.15 % 

%ղHL = 0.5 VAcos × 100
0.5 VA cos  P  [(0.5)2  P (FL)]

i cu

0.5 VAcos × 100 = 97.956 %


=
0.5 VA cos  3050 [(0.5)  4489.12]
2

Page 49
Malla Reddy College of Engineering and Technology
Department of EEE (2022-23)
TESTS ON
TRANSFORMER

1. Open Circuit Test or No Load Test

2. Short Circuit Test

3. Polarity Test
Open Circuit Test or No-load Test

The low voltage side is supplied with


rated voltage while the high voltage side
is left open. Electrical measurements are
done on the low voltage side.
Pcoreloss  Poc S oc  Eoc I oc
2 2
Eoc Eoc
Rm  Xm 
Poc Qoc

Qoc  S oc  Poc
2 2

where:

Poc = reading of the wattmeter during the test (watt)


Eoc = reading of the voltmeter during the test (volt)
Ioc = reading of the ammeter during the test (ampere)
Example:

The no-load current of a transformer is


4A at 0.25 pf when supplied at 250 V,
60 Hz. Determine the
a. core loss
b. resistance representing the core
loss
c. magnetizing reactance
A.
Pcoreloss  Poc
Poc  Eoc Ioc pf  25040.25  250watts

B.
2
Eoc 2502
Rm    250
Poc 250

C.
Soc  Eoc Ioc  2504  1000VA
Qoc  Soc  Poc  10002  2502  968.24VAR
2 2

2
2
Eoc 250
Xm    64.55
Qoc 968.24
Short Circuit Test

The low voltage side is short circuited while


the high voltage side is supplied with voltage
adjusted so that the high side will draw rated high
side current. Electrical measurements are done on
the high voltage side.
Pcopper loss (rated load)  Psc
P sc E sc
Re( high )  2 Z e( high ) 
I sc I sc

X e( high )  Z e( high )  Re( high )


2 2

where:

Psc = wattmeter reading during the test (watt)


Isc = ammeter reading during the test (ampere)
Esc = voltmeter reading during the test (volt)
Re = equivalent resistance referred to the high side
Xe = equivalent reactance referred to the high side
Ze = equivalent impedance referred to the high side
Example:

A short circuit test was performed upon a 10-


kVA, 2300/230 volt transformer with the
following results: Esc = 137 volts; Psc = 192W;
Isc = 4.34 A. Calculate in primary terms the
equivalent resistance and reactance of the
transformer.
E sc 137
Ze    31.567
I sc 4.34
Psc 192
Re  2   10.193
I sc 4.34 2

X e  Z e  Re 
2 2
31.567  10.193
2 2

X e 29.88
Polarity Test

where:
Vp = test input voltage (volt)
V = voltmeter reading (volt)
If V>Vp, the polarity is additive
If V< Vp, the polarity is subtractive
Example:

A 2300/230 volt distribution transformer is tested for


polarity in accordance with the standard method. If 120
volts is impressed across the high voltage windings,
determine the voltmeter reading if the transformer has
a. additive polarity
b. subtractive polarity

E1 N1  N2 
  E2  E1  
E2 N 2  N1 
 230 
E2  120   12volts
 2300 
Reading = 120 + 12 = 132 volts for additive
Reading = 120 – 12 = 108 volts for subtractive
TRANSFORMER LOSSES
Copper Loss
- I2R loss in the primary and secondary windings.

Pcu  I R1  I 2 R2 Pcu  I Re1  I 2 Re 2


2 2 2 2
1 1

Core Loss or Iron Loss


- eddy current loss + hysteresis loss

Peddy  ke ( f m )  ke ' E g
2 2

1.6
Eg
Physteresis  kh f  kh ' 0.6
1.6
m
f
where:

Pe = eddy current loss (watt)


Ph = hysteresis loss (watt)
ke, kh = proportionality constant
βm = maximum flux density (tesla)
Eg = supply voltage (volt)
f = frequency of supply voltage (hertz)
Example 1:

In a 400V, 50 c/s transformer, the


total iron loss is 2500 W. When the
supplied voltage is 220V at 25 c/s,
the corresponding loss is 850W.
Calculate the eddy current loss at
normal frequency and voltage.
 Eg
1.6

Pe  ke E g  Ph  kh  0.6 
2
 f 
 
 Eg 1.6

 Pe  Ph  ke E g  kh  0.6 
2
Pcore
 f 
 
When supply is 400V, 50Hz:

 400 
1.6
2500  ke ( 400)  kh  0.6 
2

 50 
2500  160000ke  1392.88kh
kh  1.795  114.87ke  Eq.1

When supply is 220V, 25Hz:

 2201.6

850  ke (220)  kh  0.6 
2

 25 
850  48400ke  811.17kh  Eq.2
Substitute Eq. 1 in Eq. 2:

850  48400ke  811.17(1.795  114.87ke )


ke  0.01353

Pe  ke E g  0.01353(400)
2 2

Pe  2165watts
The efficiency of the transformer is the ratio of its output power
(power drawn by the load) to the input power (power developed
by the transformer).

Poutput
 Poutput  V2 I 2 pf Pinput  Poutput  Plosses
Pinput

where:
Poutput = output power or power delivered to the load
Plosses = power losses
V2 = load voltage (volt)
I2 = secondary current or load current (ampere)
pf = power factor
TRANSFORMER BANKS
FOR
THREE-PHASE CIRCUITS
1. Delta-to-delta (Δ-Δ) connection
- This connection is seldom used in three-
phase transformers.
2. Delta-to-wye (Δ-Y) connection
- It is often used for distribution service
where a four-wire secondary distribution
circuit is desired.
3. Wye-to-delta (Y-Δ) connection
- This is extensively used for power
transmission and distribution.
4. Wye-to-wye (Y-Y) connection
- It is used when tying together two high-voltage
transmission system of unequal voltage.
Seatwork:
1. The following data were obtained when a
short circuit test was performed upon a 100
kVA, 2400/240V distribution transformer:
Esc = 72 volts; Isc = 41.6 A; Psc = 1180W.
All instruments are on the high side during
the short circuit test. Calculate the
equivalent resistance and reactance of the
transformer.
2. The no-load loss of a transformer at rated
voltage is 100watts at 30 Hz and 300 watts
at 60 Hz. What is the hysteresis loss at 60 Hz
and rated voltage? (Note: Since not specified, the
maximum flux density (βm) is assumed constant.)
Assignment:

When a 220V, 60Hz is impressed on a


certain transformer at no load, the
total core loss is 200W. When the
frequency of the impressed voltage is
changed to 25 Hz and the magnitude
of the voltage is made such as to
maintain the same maximum flux
density as before, the core loss falls to
75W. Calculate the hysteresis losses at
60 Hz.
Board Exam Problems
on
AC MACHINES
(Transformers)
1. The high-voltage coil of a transformer is wound with 700 turns of wire, and the low-
voltage coil is wound with 292 turns. When used as a step-up transformer (the low-
voltage coil is used as the primary), the load current is 10.5 A. Find the load
component of the primary current.
A. 43.5 A B. 4.38 A C. 25.18 A D. 2.518 A
Solution:
𝑁𝑝 𝐼𝑠
=
𝑁𝑠 𝐼𝑝
𝐼𝑠 𝑁𝑠 (700)(10.5)
𝐼𝑝 = =
𝑁𝑝 292

 𝑰𝒑 = 𝟐𝟓. 𝟏𝟕 𝑨𝒎𝒑𝒆𝒓𝒆𝒔

REE – May 2008


2. A transformer has a primary winding of 2, 000 turns and of 2, 400 Volts and current
of 8.66 − 𝑗5 Ampere with an impedance 𝑍2 connected across the secondary winding.
If the secondary winding has 500 turns, what is the value of the secondary current?
A. 20 − 𝑗34.64 𝐴 B. 𝟑𝟒. 𝟔𝟒 − 𝒋𝟐𝟎 𝑨 C. 34.64 + 𝑗20 𝐴 D. 20 + 𝑗34.64 𝐴
Solution:
𝑁𝑝 𝐼𝑠
=
𝑁𝑠 𝐼𝑝
𝐼𝑠 𝑁𝑠 (8.66−𝑗5)(2,000)
𝐼𝑝 = =
𝑁𝑝 500
 𝑰𝒑 = 𝟑𝟒. 𝟔𝟒 − 𝒋𝟐𝟎 𝑨𝒎𝒑𝒆𝒓𝒆𝒔

3. A 120 V to 27.5 V, 400 Hz step-down transformer is to be operated at 60 Hz. What is


the highest safe input voltage?
A. 200 V B. 400 V C. 120 V D. 18 V
Solution:
𝐸1 𝑓1
=
𝐸2 𝑓2
𝐸1 𝑓2 (120)(60)
𝐸2 = =
𝑓1 400
 𝑬𝟐 = 𝟏𝟖 𝑽𝒐𝒍𝒕𝒔

REE – September 2011


4. When a welding transformer is used in a resistance welding, it will
A. step up voltage B. step down voltage
C. step up current D. step down current
∗ 𝑁𝑜𝑡𝑒:
𝐼𝑛 𝑎 𝑤𝑒𝑙𝑑𝑖𝑛𝑔 𝑡𝑟𝑎𝑛𝑠𝑓𝑜𝑟𝑚𝑒𝑟, 𝑎 ℎ𝑖𝑔ℎ 𝑣𝑎𝑙𝑢𝑒 𝑜𝑓 𝑐𝑢𝑟𝑟𝑒𝑛𝑡 𝑖𝑠 𝑛𝑒𝑒𝑑𝑒𝑑 𝑡𝑜 𝑚𝑒𝑙𝑡 𝑒𝑎𝑠𝑖𝑙𝑦 𝑡ℎ𝑒 𝑖𝑟𝑜𝑛.
5. A 4, 600/230 V, 60 Hz step-down transformer has core dimension of 76.2 mm by
111.8 mm. A maximum flux density of 0.93 𝑊𝑏/𝑚2 is to be used. Assuming 9
percent loss of area due to stacking factor of laminations, calculate the primary and
secondary turns required.
A. 2, 395 and 120 B. 120 and 2, 395 C. 2, 180 and 109 D. 109 and 2, 180
Solution:
𝐴𝑒𝑓𝑓 = (1 − 0.09)(76.2 𝑚𝑚 × 111.8 𝑚𝑚)
1𝑚 2
𝐴𝑒𝑓𝑓 = 7, 752.436𝑚𝑚2 ( )
1,000 𝑚𝑚
 𝐴𝑒𝑓𝑓 = 7.752 × 10 −3
𝑚 2

∅𝑚 = 𝛽𝑚 𝐴𝑒𝑓𝑓
𝑊𝑏
∅𝑚 = (0.93 ) (7.752 × 10−3 𝑚2 )
𝑚2
 ∅𝑚 = 7.209 𝑚𝑊𝑏
𝐸𝑟𝑚𝑠 = 4.44𝑓∅𝑁
𝐸𝑟𝑚𝑠 4,600
𝑁𝑝 = =
4.44𝑓∅ (4.44)(60)(7.209×10−3 )
 𝑵𝒑 = 𝟐, 𝟑𝟗𝟓 𝒕𝒖𝒓𝒏𝒔
𝐸𝑟𝑚𝑠 230
𝑁𝑠 = =
4.44𝑓∅ (4.44)(60)(7.209×10−3 )
 𝑵𝒔 = 𝟏𝟐𝟎 𝒕𝒖𝒓𝒏𝒔

REE – October 1997


6. A small single-phase transformer has 10.2 watts no-load loss. The core has a volume
of 750 cubic cm. The maximum flux density is 10, 000 gauss and the hysteresis
constant of the core is 5 × 10−4 , using the Steinmetz law to find the hysteresis,
determine the eddy current loss.
A. 4.55 Watts B. 5.55 Watts C. 3.55 Watts D. 2.55 Watts
Solution:
∗ 𝑆𝑡𝑒𝑖𝑛𝑚𝑒𝑛𝑡𝑧 𝑙𝑎𝑤
𝑃ℎ 𝛼 𝛽𝑚 1.6
∗ 𝑘ℎ = (5 × 10−4 )(750)
 𝑘ℎ = 0.375
𝑑𝑦𝑛𝑒−𝑐𝑚 1𝑁 1𝑚
𝑃ℎ = (0.375)(60)(10, 000)1.6 ( )( )
𝑠𝑒𝑐 105 𝑑𝑦𝑛𝑒 100 𝑐𝑚
 𝑃ℎ = 5.652 𝑊𝑎𝑡𝑡𝑠
𝑃𝑒 = 𝑃𝑖𝑛 − 𝑃ℎ
𝑃𝑒 = 10.2 − 5.652
 𝑷𝒆 = 𝟒. 𝟓𝟒𝟖 𝑾𝒂𝒕𝒕𝒔
REE – September 2006
7. The primary of transformer has 200 turns and is excited by a 240 V, 60 Hz source.
What is the maximum value of the core flux?
A. 4.04 mWb B. 4.40 mWb C. 4.13 mWb D. 4.32 mWb
Solution:
𝐸 240
∅𝑚 = = (4.44)(60)(200)
4.44𝑓𝑁
 ∅𝒎 = 𝟒. 𝟓𝟎 𝒎𝑾𝒃

REE – September 2008


8. A transformer is rated 1 kVA, 220/110 V, 60 Hz. Because of an emergency this
transformer has to be used on a 50 Hz system. If the flux density in the transformer
core is to be kept the same as at 60 Hz and 220 V, what is the kilovolt-ampere rating
at 50 Hz.
A. 0.890 kVA B. 0.833 kVA C. 0.909 kVA D. 0.871 kVA
Solution:
𝑆𝛼𝑓
𝑆1 𝑓1
=
𝑆2 𝑓2
𝑆1 𝑓2 (1)(50)
𝑆2 = =
𝑓1 60
 𝑺𝟐 = 𝟎. 𝟖𝟑𝟑 𝒌𝑽𝑨

9. A single-phase transformer has a no-load power input of 250 Watts, when supplied
at 250 V, 50 Hz has a p.f of 0.25. What is the magnetizing component of the no-load
current?
A. 4.00 A B. 3.87 A C. 1.00 A D. none of these
Solution:
𝑃 250
𝑆= =
𝑝.𝑓 0.25
 𝑆 = 1, 000 𝑉𝐴
𝑝𝑓 = 𝑐𝑜𝑠 −1 0.25
 𝑝𝑓 = 75.52°
𝑆 = 𝑉𝐼
1,000∠75.52
𝐼= = 4∠75.52° 𝐴𝑚𝑝𝑒𝑟𝑒𝑠
250
 𝐼 = 1 + 𝑗 3.873 𝐴𝑚𝑝𝑒𝑟𝑒𝑠
 𝑰𝒎 = 𝟑. 𝟖𝟕𝟑 𝑨𝒎𝒑𝒆𝒓𝒆𝒔
REE – September 2011
10. A 4, 400 V, 60 Hz transformer has a core loss of 840 Watts, of which one-third is
eddy current loss. What is the core loss when the x’former is connected to a 4, 600 V,
50 Hz source?
A. 977 Watts B. 907 Watts C. 927 Watts D. 944 Watts
Solution:
1
𝑃𝑐𝑜𝑟𝑒 = 𝑃𝑒𝑑𝑑𝑦
3
1
𝑃𝑒1 = (840)
3
 𝑃𝑒1 = 280 𝑊𝑎𝑡𝑡𝑠
𝑃𝑐𝑜𝑟𝑒 = 𝑃ℎ + 𝑃𝑒
𝑃ℎ1 = 840 − 280
 𝑃ℎ1 = 560 𝑊𝑎𝑡𝑡𝑠
𝑃𝑒2 𝑘2 (𝐸2 )2
=
𝑃𝑒1 𝑘1 (𝐸1 )2
4,6002
𝑃𝑒2 = 280 ( )
4,4402
 𝑃𝑒2 = 306.033 𝑊𝑎𝑡𝑡𝑠
𝐸 1.6
𝑘2 ( 20.6 )
𝑃ℎ2 𝑓2
= 𝐸1 1.6
𝑃ℎ1 𝑘1 ( 0.6 )
𝑓1
4,6001.6
( )
500.6
𝑃ℎ2 = 560 [ 4,4001.6
]
( )
600.6
 𝑃ℎ2 = 670.788 𝑊𝑎𝑡𝑡𝑠
𝑃𝑐𝑜𝑟𝑒2 = 𝑃ℎ2 + 𝑃𝑒2 = 670.788 + 306.033
 𝑷𝒄𝒐𝒓𝒆𝟐 = 𝟗𝟕𝟔. 𝟖𝟐𝟏 𝑾𝒂𝒕𝒕𝒔

REE – September 2004


11. In an ideal transformer, what is the efficiency?
A. 100% B. 90% C. 80% D. 70%
12. A 100 kVA distribution transformer has a full-load copper loss of 1, 180 Watts. For
what kilowatt load, at a power factor of 0.71, will the copper losses in the
transformer be 1, 500 Watts?
A. 90.25 B. 71 C. 112.75 D. 80.05
Solution:
𝑃𝑐𝑢−𝑎𝑛𝑦 2 𝑆𝑎𝑛𝑦 2
2 =( )
𝑃𝑐𝑢−𝐹𝐿 𝑆𝑟𝑎𝑡𝑒𝑑
𝑃
𝑆=
𝑝.𝑓
1,500 𝐾𝑊𝑙𝑜𝑎𝑑 /0.71 2
=( )
1,180 100
𝐾𝑊𝑙𝑜𝑎𝑑 2 1,500
( ) = 1002 ( )
0.71 1,180
1,500
𝐾𝑊𝑙𝑜𝑎𝑑 = √(0.71)2 (100)2 ( )
1,180

 𝑲𝑾𝒍𝒐𝒂𝒅 = 𝟖𝟎. 𝟎𝟓𝟎 𝒌𝑾

13. Given a 10-kVA transformer with full-load losses amounting to 70 Watts in the iron
and 140 Watts in the copper. Calculate the efficiency at half-load unity power factor.
A. 98.62% B. 97.97% C. 97.28% D. 97.94%
Solution:
𝑃𝐶𝑢 𝐻𝐿 𝑆𝐻𝐿 2
=( )
𝑃𝐶𝑢 𝐹𝐿 𝑆𝐹𝐿
140(52 )
𝑃𝐶𝑢 𝐻𝐿 =
102
 𝑃𝐶𝑢 𝐻𝐿 = 35 𝑊𝑎𝑡𝑡𝑠
𝑃𝑜 𝐻𝐿 5,000
ɳ 𝐻𝐿 = × 100% = × 100%
𝑃𝑜 𝐻𝐿+𝑃𝑐𝑜𝑟𝑒 +𝑃𝐶𝑢 𝐻𝐿 5,000+35

 ɳ 𝑯𝑳 = 𝟗𝟕. 𝟗𝟒%

REE – April 2004


14. Instrument transformers are used in indicating and metering and with protective
devices, they are used for .
A. measuring B. detecting C. relaying D. sensing

REE – September 2003


15. What type of transformer bank is used to convert 2-phase to 3-phase power?
A. open-delta B. scott-T C. wye-delta D. delta-wye
16. A 100-kVA 2, 400/240-Volt 60 cycle transformer has the following constants: 𝑟𝑝 =
0.42 Ω, 𝑋𝑝 = 0.72 Ω; 𝑟𝑠 = 0.0038 Ω, 𝑋𝑠 = 0.0068 Ω. What is the equivalent
impedance in primary terms?
A. 0.016 Ω B. 1.612 Ω C. 0.161 Ω D. 16.12 Ω
Solution:
𝑍𝑒 𝑝 = (𝑟𝑝 + 𝑎2 𝑟𝑠 ) + 𝑗(𝑋𝑝 + 𝑎2 𝑋𝑠 )
𝑉𝑝 2,400
𝑎= =
𝑉𝑠 240
 𝑎 = 10
𝑍𝑒 𝑝 = (0.42) + (102 )(0.0038) + 𝑗(0.72) + (10)2 (0.0068)
𝑍𝑒 𝑝 = 0.8 + 𝑗1.4 = 1.61∠60.26°
 |𝒁𝒆 𝒑 | = 𝟏. 𝟔𝟏 𝒐𝒉𝒎𝒔

17. Calculate the all-day efficiency of a 100-kVA transformer operating under the
following conditions: 6 hours on a load of 50 kW at 0.73 power factor; 3 hours on a
load of 90 kW at 0.82 power factor; 15 hours with no load on secondary. The iron
loss is 1, 000 Watts and the full-load copper loss is 1, 060 Watts.
A. 96.31% B. 94.87% C. 95.33% D. 95.29%
Solution:
𝑃𝑜𝑢𝑡 = (50, 000)(6) + (90, 000)(3) + (0)(15)
 𝑃𝑜𝑢𝑡 = 570 𝑘𝑊ℎ𝑟
𝑃𝑐𝑜𝑟𝑒 = (1, 000)(24)
 𝑃𝑐𝑜𝑟𝑒 = 24 𝑘𝑊ℎ𝑟
𝑃𝐶𝑢 1 𝑘𝑉𝐴 2
= (𝑘𝑉𝐴 1 )
𝑃𝐶𝑢 𝐹𝐿 𝐹𝐿

50/0.73 2
𝑃𝐶𝑢 1 = 1, 060 ( )
100
 𝑃𝐶𝑢 1 = 497.279 𝑊𝑎𝑡𝑡𝑠
90/0.82 2
𝑃𝐶𝑢 2 = 1, 060 ( )
100
 𝑃𝐶𝑢 2 = 1, 276.919 𝑊𝑎𝑡𝑡𝑠
 𝑃𝐶𝑢 3 = 0 𝑊𝑎𝑡𝑡𝑠
𝑃𝐶𝑢 𝑡𝑜𝑡𝑎𝑙 = 𝑃𝐶𝑢 1 (6) + 𝑃𝐶𝑢 2 (3) + 𝑃𝐶𝑢 3 (15)
𝑃𝐶𝑢 𝑡𝑜𝑡𝑎𝑙 = (487.279)(6) + (1, 276.919)(3) + (0)(15)
 𝑃𝐶𝑢 𝑡𝑜𝑡𝑎𝑙 = 6.814 𝑘𝑊ℎ𝑟
𝑃𝑜𝑢𝑡 570
ɳ𝑎𝑙𝑙 𝑑𝑎𝑦 = × 100% = × 100%
𝑃𝑜𝑢𝑡 +𝑃𝑐𝑜𝑟𝑒+𝑃𝐶𝑢 570+24+6.814
 ɳ𝒂𝒍𝒍 𝒅𝒂𝒚 = 𝟗𝟒. 𝟖𝟕%
REE – September 2005
18. A 50-kVA, single-phase transformer has 96% efficiency when it operates at full-load
unity power factor for 8 hours per day. What is the all-day efficiency of the
transformer if the copper loss is 60% of full-load losses?
A. 92% B. 90% C. 89.5% D. 93%
Solution:
50
0.96 =
50+𝑃𝑙𝑜𝑠𝑠𝑒𝑠 𝐹𝐿
50
𝑃𝑙𝑜𝑠𝑠𝑒𝑠 𝐹𝐿 = − 50
0.96
 𝑃𝑙𝑜𝑠𝑠𝑒𝑠 𝐹𝐿 = 2.083 𝑘𝑊
𝑃𝐶𝑢 𝐹𝐿 = (0.6)(2, 083)
 𝑃𝐶𝑢 𝐹𝐿 = 1, 249.8 𝑊
𝑃𝑐𝑜𝑟𝑒 = 𝑃𝑙𝑜𝑠𝑠 + 𝑃𝐶𝑢 𝐹𝐿 = 2, 083 − 1, 249.8
 𝑃𝑐𝑜𝑟𝑒 = 833.2 𝑊𝑎𝑡𝑡𝑠
𝑃𝑜𝑢𝑡 𝑡𝑜𝑡𝑎𝑙 = (50 𝑘𝑉𝐴)(1.0)(8)
 𝑃𝑜𝑢𝑡 𝑡𝑜𝑡𝑎𝑙 = 400𝑘𝑊ℎ𝑟
𝑃𝑐𝑜𝑟𝑒 𝑡𝑜𝑡𝑎𝑙 = (833.2)(24)
 𝑃𝑜𝑢𝑡 𝑡𝑜𝑡𝑎𝑙 = 19.997 𝑘𝑊ℎ𝑟
𝑃𝐶𝑢 𝑡𝑜𝑡𝑎𝑙 = (1, 249.8)(8)
 𝑃𝐶𝑢 𝑡𝑜𝑡𝑎𝑙 = 9.998 𝑘𝑊ℎ𝑟
400
ɳ𝑎𝑙𝑙 𝑑𝑎𝑦 = × 100%
400+19.997+9.998
 ɳ𝒂𝒍𝒍 𝒅𝒂𝒚 = 𝟗𝟑. 𝟎𝟐%

Asst. EE – October 1991


19. A 10 kVA, 2, 400/240 V, single-phase transformer has the following resistances and
leakage reactances;
𝑟𝑝 = 3 Ω 𝑟𝑠 = 0.03 Ω
𝑋𝑝 = 15 Ω 𝑋𝑠 = 0.15 Ω
Find the primary voltage required to produce 240 V at the secondary terminals at
full-load, when the load power factor is 0.8 lagging.
A. 2, 400 V B. 2, 496.5 V C. 2, 348 V D. 2, 445.5 V
Solution:
2,400
𝑎=
240
 𝑎 = 10
𝑎𝑉𝑠 = 10(240)
 𝑎𝑉𝑠 = 2, 400
10,000 ∠𝑐𝑜𝑠 −1 (0.8)
𝐼𝑡 =
2,400
 𝐼𝑡 = 4.17 ∠ − 36.87° 𝐴𝑚𝑝𝑒𝑟𝑒𝑠
𝑍𝑒 𝑝 = [3 + (10)2 (0.03)] + 𝑗[15 + (10)2 (0.15)]
 𝑍𝑒 𝑝 = 6 + 𝑗30 𝑜ℎ𝑚𝑠
𝑉𝑝 = 𝐼𝑡 𝑍𝑒 𝑝 + 𝑎𝑉𝑠
𝑉𝑝 = (4.17 ∠ − 36.87)(6 + 𝑗30 ) + 2, 400
𝑉𝑝 = 2, 496.526 ∠1.95° 𝑉𝑜𝑙𝑡𝑠
 |𝑽𝒑 | = 𝟐, 𝟒𝟗𝟔. 𝟓𝟐𝟔 𝑽𝒐𝒍𝒕𝒔

20. A 500 kVA, single-phase, 13, 200/2, 400 Volts transformer has 4% reactance and
1% resistance. The leakage reactance and resistance of the high voltage (primary)
winding are 6.34 Ω and 1.83 Ω, respectively. The core loss under rated condition is
1, 800 Watts. Calculate the leakage reactance and resistance of the low voltage
(secondary) winding.
A. 7.56 Ω and 1.66 Ω B. 13.69 Ω and 3.42 Ω
C. 0.25 Ω and 0.055 Ω D. 13.9 Ω and 3.48 Ω
Solution:
(𝑉𝑏 )2 (13,200)2
𝑍𝑏 = =
𝑆𝑏 500,000
 𝑍𝑏 = 348.48 𝑜ℎ𝑚𝑠
𝑅𝑒 𝑝 = 𝑅𝑝𝑢 𝑍𝑏𝑎𝑠𝑒 = (0.01)(348.48)
 𝑅𝑒 𝑝 = 3.4848 𝑜ℎ𝑚𝑠
𝑋𝑒 𝑝 = 𝑋𝑝𝑢 𝑍𝑏𝑎𝑠𝑒 = (0.04)(348.48)
 𝑋𝑒 𝑝 = 13.9392 𝑜ℎ𝑚𝑠
𝑟𝑒𝑓𝑒𝑟𝑟𝑒𝑑 𝑡𝑜 𝑝𝑟𝑖𝑚𝑎𝑟𝑦
𝑅𝑒 𝑝 = 𝑟𝑝 + 𝑎2 𝑟𝑠
13,200
𝑎=
2,400
 𝑎 = 5.5
𝑅𝑒 𝑝 −𝑟𝑝 3.4848−1.83
𝑟𝑠 = =
𝑎2 5.52
 𝒓𝒔 = 𝟎. 𝟎𝟓𝟓
𝑋𝑒 𝑝 = 𝑋𝑝 + 𝑎2 𝑋𝑠
𝑋𝑒 𝑝−𝑋𝑝 13.9392−6.34
𝑋𝑠 = =
𝑎2 5.52
 𝑿𝒔 = 𝟎. 𝟐𝟓𝟏 𝒐𝒉𝒎𝒔

21. In Problem No.20, calculate the %V.R and efficiency of the transformer at full-load,
0.85 p.f. lagging and 2, 400 Volts.
A. 4% and 97.8% B. 6% and 95.4% C. 5% and 96.8% D. 3% and 98.4%
Solution:
𝑟𝑝 1.83
=
𝑎2 5.52
𝑟𝑝
 = 0.0605 𝑜ℎ𝑚𝑠
𝑎2
𝑋𝑝 6.34
=
𝑎2 5.52
𝑋𝑝
 = 0.21 𝑜ℎ𝑚𝑠
𝑎2
−1
𝜃 = 𝑐𝑜𝑠 (0.85)
 𝜃 = 31.79°
500,000 ∠−31.79°
𝐼𝑇 =
2,400
 𝐼𝑇 = 208.33 ∠ − 31.79°
𝑟𝑝 𝑋𝑝
𝑍𝑒 𝑠 = ( + 𝑟𝑠 ) + 𝑗 ( + 𝑋𝑠 ) = (0.0605 + 0.055) + 𝑗(0.21 + 0.25)
𝑎2 𝑎2
 𝑍𝑒 𝑠 = 0.1155 + 𝑗0.46 Ω
𝑉𝑝 = 𝐼𝑇 𝑍𝑒 𝑠 + 𝑉𝑠
𝑉𝑝 = (208.33 ∠ − 31.79°)(0.1155 + 𝑗0.46) + 2, 400
 𝑉𝑝 = 2, 471.89 ∠1.6° 𝑉𝑜𝑙𝑡𝑠
2,471.89−2,400
%𝑉𝑅 = × 100
2,400
 %𝑽𝑹 = 𝟑%
𝑃𝑜𝑢𝑡 = (500 𝑘𝑉𝐴)(0.85)
 𝑃𝑜𝑢𝑡 = 425 𝑘𝑊
 𝑃𝑐𝑜𝑟𝑒 = 1, 800 𝑊
𝑃𝑐𝑢 = 𝐼𝑇 2 𝑅𝑇 = (208.33)2 (0.1155)
 𝑃𝑐𝑢 = 5, 012.86 𝑊
425,000
ɳ= × 100
425,000+1,800+5,012.86
 ɳ = 𝟗𝟖 %
22. An 11, 000/230 V, 150 kVA, single-phase, 50 Hz transformer has a core loss of 1.4
kW and a full-load copper loss of 1.6 kW. What is the value of maximum efficiency at
unity p.f?
A. 98.17% B. 98.04% C. 97.22% D. 97.64%
Solution:
𝑃𝑐𝑢 𝑚𝑎𝑥 = 𝑃𝑐𝑜𝑟𝑒
 𝑃𝑐𝑢 𝑚𝑎𝑥 = 1, 400 𝑊
𝑃𝑐𝑢 𝑚𝑎𝑥 𝑘𝑉𝐴𝑚𝑎𝑥 2
=( )
𝑃𝑐𝑢 𝐹𝐿 𝑘𝑉𝐴𝐹𝐿

1,400
𝑘𝑉𝐴𝑚𝑎𝑥 = √(1502 ) ( )
1,600

 𝑘𝑉𝐴𝑚𝑎𝑥 = 140.31 𝑘𝑉𝐴


𝑘𝑊𝑚𝑎𝑥 = 𝑘𝑉𝐴𝑚𝑎𝑥 (𝑝. 𝑓) = (140.31)(1.0)
 𝑘𝑊𝑚𝑎𝑥 = 140.13 𝑘𝑊
140.13
ɳ= × 100
140.13+2(1.4)
 ɳ = 𝟗𝟖. 𝟎𝟒 %

23. A 300-kVA, single-phase transformer is designed to have a resistance of 1.5% and


maximum efficiency occurs at a load of 173.2 kVA. Find its efficiency when
supplying full-load at 0.8 p.f. lagging at normal voltage and frequency.
A. 97.56% B. 96.38% C. 98.76% D. 95.89%
Solution:
𝑃𝑐𝑢 𝐹𝐿
𝑟𝑝.𝑢 =
𝑆𝐹𝐿
𝑃𝑐𝑢 𝐹𝐿 = 𝑟𝑝.𝑢 𝑆𝐹𝐿 = (0.015)(300, 000)
 𝑃𝑐𝑢 𝐹𝐿 = 4, 500 𝑊𝑎𝑡𝑡𝑠
𝑃𝑐𝑢 𝑚𝑎𝑥 173.2 2
=( )
4,500 300
 𝑃𝑐𝑢 𝑚𝑎𝑥 = 1, 499.91 𝑊𝑎𝑡𝑡𝑠
𝑃𝑐𝑢 𝑚𝑎𝑥 = 𝑃𝑐𝑜𝑟𝑒
𝑃𝑜𝑢𝑡 300(0.8)
ɳ𝐹𝐿 = × 100 = (300)(0.8)+1.5+4.5 × 100
𝑃𝑜𝑢𝑡 +𝑃𝑐𝑜𝑟𝑒 +𝑃𝑐𝑢 𝐹𝐿

 ɳ𝑭𝑳 = 𝟗𝟕. 𝟓𝟔 %
REE – September 2002
24. A 20 kV/7.87 kV autotransformer has 200 A current in the common winding. What
is the secondary line current?
A. 143.52 B. 200 C. 56.48 D. 329
Solution:
𝐼𝑐
=𝑎−1
𝐼𝑝
20,000
𝑎=
7,870
 𝑎 = 2.54
200
= 2.54 − 1
𝐼𝑝

 𝐼𝑝 = 129.76 𝐴𝑚𝑝𝑒𝑟𝑒𝑠
𝐼𝑠 = 𝐼𝑝 + 𝐼𝑐 = 129.76 + 200
 𝑰𝒔 = 𝟑𝟐𝟗. 𝟕𝟔 𝑨𝒎𝒑𝒆𝒓𝒆𝒔

25. An autotransformer is adjusted for an output voltage of 85.3 Volts when operated
from a 117 Volts line. The variable power load draws 3.63 kW at unity power factor
at this setting. Determine the transformed power and the connected power from the
source to the load.
A. 980 Watts and 2, 650 Watts B. 1, 343 Watts and 2, 287 Watts
C. 1, 815 Watts and 1, 815 Watts D. 1, 210 Watts and 2, 420 Watts
Solution:
117
𝑎=
85.3
 𝑎 = 1.37
1 1
𝑃𝑡𝑟𝑎𝑛𝑠 = 𝑃𝑖𝑛 (1 − ) = (3, 630) (1 − )
𝑎 1.37
 𝑷𝒕𝒓𝒂𝒏𝒔 = 𝟗𝟖𝟎. 𝟑𝟔 𝑾𝒂𝒕𝒕𝒔
𝑃𝑖𝑛 3,630
𝑃𝑐𝑜𝑛 = =
𝑎 1.37
 𝑷𝒄𝒐𝒏 = 𝟐, 𝟔𝟒𝟗. 𝟔𝟒 𝑾𝒂𝒕𝒕𝒔

REE – April 2006


26. What would happen if you connect a transformer to a dc circuit with a voltage of
20% of nameplate ratings after a steady state condition is reached?
A. No voltage is registered at the secondary
B. Rated no-load current flows to the secondary
C. primary current is equal to voltage over equivalent primary impedance
D. Voltage is established at secondary
27. A short-circuit test was performed upon a 10 kVA, 2, 300/230-Volt transformer
with the following results: 𝐸𝑠𝑐 = 137 𝑉𝑜𝑙𝑡𝑠; 𝑃𝑠𝑐 = 192 𝑊𝑎𝑡𝑡𝑠; 𝐼𝑠𝑐 = 4.34 𝐴𝑚𝑝𝑒𝑟𝑒𝑠.
Calculate in secondary terms the transformer equivalent.
A. 29.88 Ω B. 2.988 Ω C. 0.2988 Ω D. 298.8 Ω
Solution:
𝑃𝑠𝑐 192
𝑅= =
𝐼𝑠𝑐 2 4.34 2
 𝑅 = 10.19 Ω
𝐸𝑠𝑐 137
𝑍= =
𝐼𝑠𝑐 4.34
 𝑍 = 31.57 Ω
𝑋 = √𝑍 2 − 𝑅 2 = √31.572 − 10.192
 𝑿 = 𝟐𝟗. 𝟖𝟖 Ω

REE – April 2007


28. A transformer is rated 500 kVA, 4, 800/480 V, 60 Hz when it is operated as a
conventional two winding transformer. This transformer is to be used as a
5280/4800 V stepdown autotransformer in a power distribution system. In the
autotransformer, what is the transformer rating when used in this manner?
A. 5 MVA B. 6 MVA C. 5.5 MVA D. 6.5 MVA
Solution:
𝑆 500,000
𝐼𝑝 = 𝑉 =
𝑝 4,800

 𝐼𝑝 = 104.17 𝐴𝑚𝑝𝑒𝑟𝑒𝑠
𝑆 500,000
𝐼𝑠 = 𝑉 =
𝑠 4,80
 𝐼𝑠 = 1, 041.7 𝐴𝑚𝑝𝑒𝑟𝑒𝑠
𝐼𝑝 ′ = 𝐼𝑠
 𝐼𝑝 ′ = 1, 041.7 𝐴𝑚𝑝𝑒𝑟𝑒𝑠
𝑆 = 𝑉𝑝 ′𝐼𝑝 ′ = (5, 280)(1, 041.7)
 𝑺 = 𝟓. 𝟓 𝑴𝑽𝑨

REE – September 2008


29. Two identical transformers bank on open delta serve a balanced three-phase load of
26 kVA at 240 V, 60 Hz. What is the minimum size of each in kVA needed to serve
this load?
A. 25 B. 10 C. 30 D. 15
Solution:
𝑆∅ 𝑙𝑜𝑎𝑑 26,000
𝑆∅ 𝑟𝑎𝑡𝑒𝑑 = (√3)
=
√3

 𝑺∅ 𝒓𝒂𝒕𝒆𝒅 = 𝟏𝟓 𝒌𝑽𝑨
30. Two single-phase, 100 kVA transformers are connected in V (open delta) bank
supplying a balanced three-phase load. If the balanced three-phase load is 135 kW
at 0.82 p.f lagging and 0.823 efficiency, determine the overload kVA on each
transformer.
A. 10.5 B. 5.5 C. 15.5 D. 20.5
Solution:
𝑃 135,000
𝑆𝐿 = =
𝑝.𝑓 0.82
 𝑆𝐿 = 164.634 𝑘𝑉𝐴
𝑃𝑜𝑢𝑡
ɳ=
𝑃𝑖𝑛
135
𝑃𝑖𝑛 =
0.823
 𝑃𝑖𝑛 = 164.034 𝑘𝑊
𝑃𝑖𝑛 164.034 𝑘𝑊
𝑆𝑖𝑛 = =
𝑝.𝑓 0.82
 𝑆𝑖𝑛 = 200.041 𝑘𝑉𝐴
𝑆𝑖𝑛 200.041
𝑆∅ = =
√3 √3
 𝑆∅ = 115.494 𝑘𝑉𝐴
 𝑆∅ 𝑟𝑎𝑡𝑒𝑑 = 100 𝑘𝑉𝐴
𝑆𝑂.𝐿 = 𝑆𝐿∅ − 𝑆∅𝑟𝑎𝑡𝑒𝑑 = 115.494 − 100
 𝑺𝑶.𝑳 = 𝟏𝟓. 𝟒𝟗𝟒 𝒌𝑽𝑨

31. In problem No. 30, determine the p.f of each transformer secondary.
A. 0.820 lagging and 0.820 lagging B. 0.996 lagging and 0.424 leading
C. 0.996 lagging and 0.424 lagging D. 0.410 lagging and 0.410 lagging
Solution:
𝜃 = 𝑐𝑜𝑠 −1 (0.82)
 𝜃 = 34.92°
𝑝. 𝑓1 = 𝑐𝑜 𝑠(30 + 𝜃) = 𝑐𝑜 𝑠(30 + 34.92)
 𝒑. 𝒇𝟏 = 𝟎. 𝟒𝟐𝟒 𝒍𝒂𝒈𝒈𝒊𝒏𝒈
𝑝. 𝑓2 = 𝑐𝑜 𝑠(30 − 𝜃) = 𝑐𝑜 𝑠(30 − 34.92)
 𝒑. 𝒇𝟐 = 𝟎. 𝟗𝟗𝟔 𝒍𝒂𝒈𝒈𝒊𝒏𝒈

REE – April 2005


32. What is the normal secondary circuit current of a current transformer?
A. 15 A B. 20 A C. 5 A D. 10 A
33. In Problem No.30, what is the minimum size in kVAR of a capacitor bank to be
connected across the load so that each transformer is loaded 96% of its rated
capacity?
A. 87 kVAR B. 114 kVAR C. 27 kVAR D. 66 kVAR
Solution:
𝑆∅ = (0.96)(𝑆∅𝑟𝑎𝑡𝑒𝑑 ) = (0.96)(100)
 𝑆∅ = 96 𝑘𝑉𝐴
 𝑃𝑇 = 164.034 𝑘𝑊
 𝑆 = 200.041 𝑘𝑉𝐴
𝑄 = √𝑆 2 − 𝑃𝑇 2 = √(200.041)2 − (164.034)2
 𝑄 = 114.5 𝑘𝑉𝐴𝑅
𝑆𝑛𝑒𝑤 = (96 𝑘𝑉𝐴)(√3)
 𝑆𝑛𝑒𝑤 = 166.277 𝑘𝑉𝐴
𝑆 = 𝑃2 + 𝑄2
2

𝑄 = √𝑆𝑛𝑒𝑤 2 − 𝑃𝑇 2 = √(166.277)2 − (164.034)2


 𝑸 = 𝟐𝟕. 𝟐𝟐 𝒌𝑽𝑨𝑹

34. A polarity test is performed upon a 1, 150/115 V transformer. If the input voltage is
116, calculate the voltmeter reading if the polarity is subtractive.
A. 127.6 V B. 106 V C. 126 V D. 104.4 V
Solution:
𝑠𝑢𝑏𝑡𝑟𝑎𝑐𝑡𝑖𝑣𝑒
1,150
𝑎= 115
 𝑎 = 10
116
𝑉𝑟𝑒𝑎𝑑𝑖𝑛𝑔 = 116 − 10
 𝑽𝒓𝒆𝒂𝒅𝒊𝒏𝒈 = 𝟏𝟎𝟒. 𝟒 𝑽𝒐𝒍𝒕𝒔

35. A 20:1 potential transformer is used with a 150-V voltmeter. If the instrument
deflection is 118 Volts, calculate the line voltage.
A. 3, 000 V B. 2, 850 V C. 2, 360 V D. 2, 242 V
Solution:
𝑉𝐿
𝑎=
𝑉𝑟𝑒𝑎𝑑𝑖𝑛𝑔
20 𝑉𝐿
=
1 118
 𝑽𝑳 = 𝟐, 𝟑𝟔𝟎 𝑽𝒐𝒍𝒕𝒔
REE –September 2010
36. A three-phase wye-delta connected, 50 MVA, 345/34.5 kV transformer is protected
by differential protection. The current transformer on the high side for differential
protection is 150:5. What is the current on the secondary side of CT’s?
A. 3.83 A B. 2.53 A C. 4.50 A D. 4.83 A
Solution:
𝑆 50,000,000
𝐼∅ 𝑝 = = 345,000
3𝑉∅𝑝 3( )
√3

 𝐼∅ 𝑝 = 83.67 𝐴𝑚𝑝𝑒𝑟𝑒𝑠
𝑠𝑖𝑛𝑐𝑒 𝑤𝑦𝑒 − 𝑐𝑜𝑛𝑛𝑒𝑐𝑡𝑖𝑜𝑛
𝐼∅ 𝑝 = 𝐼𝐿 𝑝
 𝐼𝐶𝑇 𝑝 = 𝐼𝐶𝑇 𝑝 = 83.67 𝐴𝑚𝑝𝑒𝑟𝑒𝑠
𝐿 ∅
𝐼𝑝 𝑁𝑝
𝑎= =
𝐼𝑠 𝑁𝑠
150 83.67
=
5 𝐼𝐶𝑇 𝑠

(83.67)(5)
𝐼𝐶𝑇 𝑠 =
∅ 150
 𝐼𝐶𝑇 𝑠 = 2.789 𝐴𝑚𝑝𝑒𝑟𝑒𝑠

𝐼𝐶𝑇 𝑠𝑒𝑐 = √3 (2.789)


 𝑰𝑪𝑻 𝒔𝒆𝒄 = 𝟒. 𝟖𝟑𝟏 𝑨𝒎𝒑𝒆𝒓𝒆𝒔

REE – October 2000


37. The CT ratio and PT ratio used to protect a line are 240 and 2,000, respectively. If
the impedance of each line is 10 Ω, what is the relay impedance to protect the line
from fault?
A. 83.33 ohms B. 1.2 ohms C. 48, 000 ohms D. 12 ohms
Solution:
𝑃𝑇𝑟 2,000
𝑍𝑟 = =
𝐶𝑇𝑟 240
 𝑍𝑟 = 8.33 𝑜ℎ𝑚𝑠
𝑍𝑙𝑖𝑛𝑒
𝑍𝑟 =
𝑍𝑟𝑒𝑙𝑎𝑦
10
𝑍𝑟𝑒𝑙𝑎𝑦 =
8.33
 𝒁𝒓𝒆𝒍𝒂𝒚 = 𝟏. 𝟐 𝒐𝒉𝒎𝒔
38. Two transformers 1 and 2 are connected in parallel supplying a common load of 120
kVA. Transformer 1 is rated 50 kVA, 7, 620/240-V single-phase and has an
equivalent impedance of 8.5 Ω while transformer 2 is rated 75 kVA, 7, 620/240-V
single-phase and has an equivalent impedance of 5.1 Ω. The two transformers
operate with the same power factors. What is the kVA load of each transformer?
A. 48 & 72 B. 45 & 75 C. 42 & 78 D. 40 & 80
Solution:
𝑍𝑒 1 𝑆2
=
𝑍𝑒 2 𝑆1
8.5 𝑆2
=
5.1 𝑆1
 𝑆2 = 1.67𝑆1 → 𝑒𝑞′𝑛 1
 𝑆𝐿 = 𝑆1 + 𝑆2 → 𝑒𝑞′ 𝑛 2
𝑆𝑢𝑏𝑠𝑡𝑖𝑡𝑢𝑡𝑒 𝑒𝑞’𝑛 1 𝑖𝑛 2
120 = 𝑆1 + 1.67𝑆1
 𝑺𝟏 = 𝟒𝟒. 𝟗𝟒 𝒌𝑽𝑨
𝑆2 = 1.67(44.94)
 𝑺𝟐 = 𝟕𝟓. 𝟎𝟓 𝒌𝑽𝑨

39. Two single-phase transformers are connected in parallel at no-load. One has a turns
ratio of 5, 000/440 and rating of 200 kVA, the other has a ratio of 5, 000/480 and
rating of 350 kVA the leakage reactance of each is 3.5%. The no-load circulating
current is .
A. 207 A B. 702 A C. 720 A D. 270 A
Solution:
2
(𝑉𝑏𝑎𝑠𝑒1 ) (440)2
𝑋𝑏𝑎𝑠𝑒1 = =
𝑆𝑏𝑎𝑠𝑒1 200,000

 𝑋𝑏𝑎𝑠𝑒1 = 0.968 Ω
2
(𝑉𝑏𝑎𝑠𝑒2 ) (480)2
𝑋𝑏𝑎𝑠𝑒2 = =
𝑆𝑏𝑎𝑠𝑒2 350,000

 𝑋𝑏𝑎𝑠𝑒2 = 0.658 Ω
𝑋𝑒−𝑠1 = 𝑋𝑒 𝑋𝑏𝑎𝑠𝑒1 = (0.035)(0.968)
 𝑿𝒆−𝒔𝟏 = 𝟎. 𝟎𝟑𝟑𝟗 Ω
𝑋𝑒−𝑠2 = 𝑋𝑒 𝑋𝑏𝑎𝑠𝑒2 = (0.035)(0.658)
 𝑿𝒆−𝒔𝟐 = 𝟎. 𝟎𝟐𝟑 Ω
REE – October 1997
40. A power transformer rated 50, 000 kVA, 34.5/13.8 kV is connected Y-grounded
primary and delta on the secondary. Determine the full load phase current at the
secondary side.
A. 2, 092 A B. 1, 725 A C. 1, 449 A D. 1, 208 A
Solution:
𝑆 = √3 𝑉𝐿 𝐼𝐿
50,000,000
𝐼𝐿 𝑝 =
(34,500)(√3)

 𝐼𝐿 𝑝 = 836.74 𝐴𝑚𝑝𝑒𝑟𝑒𝑠
𝐼𝐿 𝑝 = 𝐼∅ 𝑝
𝑉𝑝 𝐼
= 𝐼𝑠
𝑉𝑠 𝑝
(34,500)(836.74)
𝐼∅ 𝑠 = 13,800 (√3)
 𝑰𝒔 ∅ = 𝟏, 𝟐𝟎𝟕. 𝟕𝟑 𝑨𝒎𝒑𝒆𝒓𝒆𝒔
𝑎𝑛𝑜𝑡ℎ𝑒𝑟 𝑠𝑜𝑙𝑢𝑡𝑖𝑜𝑛:
50,000,000
𝐼𝐿 𝑠 = (
√3) (13,800)
 𝐼𝐿 𝑠 = 2, 091.849 𝐴𝑚𝑝𝑒𝑟𝑒𝑠
𝐼𝐿 𝑠 2,091.849
𝐼∅ 𝑠 = =
√3 √3
 𝑰∅ 𝒔 = 𝟏, 𝟐𝟎𝟕. 𝟕𝟑 𝑨𝒎𝒑𝒆𝒓𝒆𝒔

REE – April 2006


41. A 2, 000 kW, 2, 400-V, 75% p.f load is to be supplied from a 34, 5000-V, 3-phase line
through a single bank of transformers. Give the primary and secondary line currents
in amperes for the wye-wye connections.
A. 50/700 B. 48/650 C. 60/800 D. 45/642
Solution:
𝑌 − ∆ 𝐶𝑜𝑛𝑛𝑒𝑐𝑡𝑖𝑜𝑛
2 𝑀𝑊 ∗
∠𝑐𝑜𝑠 −1 (0.75)
0.75
𝐼𝐿 𝑝 = [ ]
√3 (34,500)

𝐼𝐿 𝑝 = 44.626∠44.41°
 𝑰𝑳 𝒑 = 𝟒𝟒. 𝟔𝟐𝟔 𝑨𝒎𝒑𝒆𝒓𝒆𝒔
2 𝑀𝑊 ∗
∠𝑐𝑜𝑠 −1 (0.75)
0.75
𝐼𝐿 𝑠 = [ ]
√3 (2,400)

𝐼𝐿 𝑠 = 641.5∠44.41°
 𝑰𝑳 𝒔 = 𝟔𝟒𝟏. 𝟓 𝑨𝒎𝒑𝒆𝒓𝒆𝒔
REE – April 2005
42. A 3, 000 kVA, 2, 400 V, 75% power factor load is to be supplied from a 34, 500-V,
three-phase line through a single bank of transformers. What is the voltage rating of
each transformer if the connection is wye-wye?
A. 20, 000/1, 380 B. 18, 500/1, 350 C. 18, 000/1, 850 D. 19, 000/1, 350
Solution:
34,500
𝑉∅ 𝑝 =
√3
 𝑽∅ 𝒑 = 𝟏𝟗, 𝟗𝟏𝟖. 𝟓𝟖𝟒 𝑽𝒐𝒍𝒕𝒔
2,400
𝑉∅ 𝑠 =
√3
 𝑽∅ 𝒔 = 𝟏, 𝟑𝟖𝟓. 𝟔𝟒𝟏 𝑽𝒐𝒍𝒕𝒔

REE – March 1998


43. A 13.8 kV/480 V, 10 MVA three-phase transformer has 5% impedance. What is the
impedance in ohms referred to the primary?
A. 0.952 ohm B. 0.03 ohm C. 5.125 ohm D. 9.01 ohm
Solution:
𝑍𝑎𝑐𝑡𝑢𝑎𝑙
𝑍𝑝𝑢 =
𝑍𝑏
(𝑉𝑏 )2 (13,800)2
𝑍𝑏 = =
𝑆𝑏 10,000,000
 𝑍𝑏 = 19.044 𝑜ℎ𝑚𝑠
𝑍𝑒 𝑝 = 𝑍𝑝𝑢 𝑍𝑏 = (0.05)(19.044)
 𝒁𝒆 𝒑 = 𝟎. 𝟗𝟓𝟐 𝒐𝒉𝒎𝒔

REE – May 2009


44. A three-phase transformer is rated 15 MVA, 69/13.2 kV has a series impedance of
5%. What is the new per unit impedance if the system study requires a 100 MVA
base and 67 kV base?
A. 0.354 B. 0.347 C. 0.372 D. 0.333
Solution
𝑆 𝑉 2
𝑍𝑝𝑢 𝑛𝑒𝑤 = 𝑍𝑝𝑢 𝑜𝑙𝑑 ( 𝑛 ) ( 𝑜 )
𝑆𝑜 𝑉𝑁
100 69 2
𝑍𝑝𝑢 𝑛𝑒𝑤 = (0.05) ( )( )
15 67
 𝒁𝒑𝒖 = 𝟎. 𝟑𝟓𝟒
𝒏𝒆𝒘
REE – April 2004
45. A transformer rated 2, 000 kVA, 34, 500/240 volts has 5.75% impedance. What is
the per unit impedance?
A. 0.0635 B. 0.0656 C. 0.0575 D. 34.2
Solution:
𝑍𝑝𝑢 = 57.5%
 ∴ 𝑍𝑝𝑢 = 0.0575

46. Three 5:1 transformers are connected in delta-wye to step up the voltage at the
beginning of a 13, 200-Volt three-phase transmission line. Calculate the line voltage
on the high side of the transformers.
A. 114, 300 V B. 66, 000 V C. 132, 000 V D. 198, 000 V
Solution:
5 𝑉∅ 2 𝑉∅ 2
= =
1 𝑉∅ 1 13,200 𝑉

𝑉∅ 2 = 𝑎𝑉𝑝
𝑉∅ 2 = (5)(13, 200)
 𝑉∅ 2 = 66, 000 𝑉𝑜𝑙𝑡𝑠
𝑉𝐿 = √3 𝑉∅ 2 = √3 (66, 000)
 𝑽𝑳 = 𝟏𝟏𝟒, 𝟑𝟏𝟓. 𝟑𝟓𝟑 𝑽𝒐𝒍𝒕𝒔

47. A 150 kVA, 2, 400/480-V, three-phase transformer with an equivalent impedance of


4%is connected to an infinite bus and without load. If a three-phase fault occurs at
the secondary terminals, the fault current in amperes is .
A. 4, 512 A B. 3, 908 A C. 7, 815 A D. 1, 504 A
Solution:
3 ∅ 𝑓𝑎𝑢𝑙𝑡 𝑜𝑛 𝑡ℎ𝑒 𝑠𝑒𝑐𝑜𝑛𝑑𝑎𝑟𝑦 𝑡𝑒𝑟𝑚𝑖𝑛𝑎𝑙𝑠
𝑆𝐵
𝐼𝐹 3∅ =
√3 𝑉𝐵 𝑋𝑝𝑢
150,000
𝐼𝐹 3∅ =
√3 (480)(0.04)
 𝑰𝑭 𝟑∅ = 𝟒, 𝟓𝟏𝟎. 𝟓𝟒𝟗 𝑨𝒎𝒑𝒆𝒓𝒆𝒔
48. Transformer 1 is in parallel with Transformer 2
Transformer 1 Transformer 2
150 kVA, single-phase 300 kVA, single-phase
6, 600/240 V 6, 600/240 V
𝑍𝑒−𝑠1 = 0.02425∠62.9° Ω 𝑍𝑒−𝑠2 = 0.01067∠62.9° Ω
Determine the maximum kVA load the bank can carry without overloading any of
the two transformers, assuming that the two transformers operate at the same
power factors.
A. 450 kVA B. 432 kVA C. 420 kVA D. 412 kVA
Solution:
𝑆1 + 𝑆2 = 𝑆𝑙𝑜𝑎𝑑
𝐶𝑎𝑠𝑒 𝐼:
𝑇1 @ 150 𝑘𝑉𝐴
𝑍𝑒−𝑠1 𝑆2
=
𝑍𝑒−𝑠2 𝑆1
𝑍𝑒−𝑠1 0.02425∠62.9°
=
𝑍𝑒−𝑠2 0.01067∠62.9°
𝑍𝑒−𝑠1
 = 2.273
𝑍𝑒−𝑠2
𝑆2
2.273 =
𝑆1
𝑆2 = 2.273 (150, 000)
 𝑆2 = 340.95 𝑘𝑉𝐴
 ∴ 𝑜𝑣𝑒𝑟𝑙𝑜𝑎𝑑 𝑎𝑡 𝑇2
𝐶𝑎𝑠𝑒 𝐼𝐼:
𝑇2 @ 300 𝑘𝑉𝐴
300,000
𝑆1 =
2.273
 𝑆1 = 131.984 𝑘𝑉𝐴
𝑆𝑙𝑜𝑎𝑑 = 𝑆1 + 𝑆2 = (131.984) + (300)
 𝑺𝒍𝒐𝒂𝒅 = 𝟒𝟑𝟏. 𝟗𝟖𝟏 𝒌𝑽𝑨
Unit-1
Transformers
Objectives: After completing this unit, you will be able to:
• State the applications of a transformer in electrical and electronic circuits.
• State the principle of operation of a two-winding transformer.
• Draw the circuit symbol of a transformer.
• Derive the basic emf equation of a transformer.
• State the four conditions for transformer to be ideal.
• State the relations for transformation of voltage, current and impedance by a transformer in terms of its turns-ratio.
• State why a transformer should have no-load current.
• Define the two components of the no-load current.
• Explain why the hysteresis and eddy-current losses occur in the core, and how these can be reduced.
• Explain the construction of core-type and shell-type transformers.
• State what is meant by load component of primary current.
• State why in the equivalent circuit of a transformer we include a resistance and a leakage reactance in both the primary
and secondary side.
• State how we obtain a simplified equivalent circuit of a transformer as referred to the primary or to the secondary.
• State the meaning of ‘regulation down’ and ‘regulation up’ of a transformer.
• Derive the condition for zero regulation and condition for maximum regulation of a transformer.
• Define ‘commercial efficiency’ and ‘all-day efficiency’ of a transformer.
• Derive the condition of maximum efficiency of a transformer.
• Explain how to convert a two-winding transformer into an autotransformer, and state the advantages and disadvantages
of doing it.
• Explain how to get the ‘equivalent circuit parameters’ of a transformer by conducting ‘open-circuit test’ and ‘short-
circuit test’.

1.1 INTRODUCTION
A transformer is a highly efficient device for changing ac voltage from one value to another, without any change in its
frequency. There exists no simple device that can accomplish such changes in dc voltages. Thus, the transformer has provided a
feature to ac power system that lacks in dc power system.
The general practice is to generate ac voltage at about 11 kV, then step up by means of a transformer to higher voltages of 132
kV, 220 kV and 400 kV for the transmission lines. This conversion aids the transmission of huge electrical power at low cost.
High-voltage lines carry low currents, and hence the cost of lines and the power loss are tremendously reduced. At distribution
points, other transformers are used to step the voltage down to 400 V or 220 V for use in industries, offices and homes. Since
there are no moving parts in a transformer, it practically needs almost no maintenance and supervision.
Apart from the above, the transformers are also used in communication circuits, radio and TV circuits, telephone circuits,
instrumentation and control systems.
1.2 PRINCIPLE OF OPERATION
A transformer operates on the principle of mutual induction between two coils. Figure 1.1a shows the general construction of a
transformer. The vertical portions of the steel-core are termed limbs, and the top and bottom portions are called yokes. The two
coils P and S, having N1 and N2 turns, are wound on the limbs. These two windings are electrically unconnected but are linked
with one another through a magnetic flux in the core. The coil P is connected to the supply and is therefore called primary; coil S
is connected to the load and is termed the secondary.

(a) Construction. (b) Circuit symbol.


Fig. 1.1 A transformer.

1
Figure 1.1b shows the circuit symbol of a transformer. The thick line denotes the iron core. By having different ratios
N1/N2 of the two windings, power at lower or at higher voltage can be obtained. When N2 > N1, the transformer is called a step up
transformer; and when N2 < N1, the transformer is called a step down transformer.
EMF Equation
Consider a sinusoidally varying voltage V1 applied to the primary of the transformer shown in Fig. 1.1a. Due to this voltage, a
sinusoidally varying magnetic flux is set up in the core, which can be represented as
Φ = Φm sin ωt = Φm sin 2π ft
...(1.1)
where, Φm is the peak value of the flux and f is the frequency of sinusoidal variation of flux. As per the law of electromagnetic
induction, the induced emf in a winding of N turns is given as
dΦ d
e = −N = − N (Φm sin ωt ) = − N ωΦm cos ωt = ω NΦm sin (ωt − π / 2)
dt dt
...(1.2)
Thus, the peak value of the induced emf is Em = ωNΦm. Therefore, the rms value of the induced emf E is given as
Em ω NΦm 2π fNΦm
E= = = = 4.44 fNΦm
2 2 2
or E = 4.44 fNΦm
...(1.3)
This equation, known as emf equation of transformer, can be used to find the emf induced in any winding (primary or secondary)
linking with flux Φ.

Example 1.1 The primary of a 50-Hz step-down transformer has 480 turns and is fed from 6400 V supply. Find (a) the peak
value of the flux produced in the core, and (b) the voltage across the secondary winding if it has 20 turns.
Solution: (a) Using Eq. 1.3, we get
E 6400
Φm = = = 0.06 Wb = 60 mWb
4.44 fN1 4.44 × 50 × 480
(b) The voltage induced in the secondary winding is given as
E = 4.44 fN 2Φm = 4.44 × 50 × 20 × 0.06 = 266.4 V

1.3 IDEAL TRANSFORMER


We shall describe the physical construction and equivalent circuit of an actual transformer a little later. Here, we define the ideal
transformer as a circuit element. We shall then explore its properties in voltage, current, and impedance transformation. Primary
and secondary voltage and current variables are defined in Fig. 1.1b, which shows the circuit model of an ideal transformer.
The complete behaviour of a practical transformer can be better understood by initially assuming the transformer to be
ideal, and then allowing the imperfections of the actual transformer by suitably introducing some impedance.
Conditions for Ideal Transformer:
(i) The permeability (µ) of the magnetic circuit (the core) is infinite, i.e., the magnetic circuit has zero reluctance so
that no magneto-motive force (mmf) is needed to set up the flux in the core.
(ii) The core of the transformer has no losses.
(iii) The resistance of its windings is zero, hence no I2R losses in the windings.
(iv) Entire flux in the core links both the windings, i.e., there is no leakage flux.
Thus, an ideal transformer has no losses and stores no energy. However, an ideal transformer has no physical existence. But, the
concept of ideal transformer is very helpful in understanding the working of an actual transformer.
Consider an ideal transformer whose secondary is connected to a load ZL and primary is supplied from an ac source V1 (Fig.
1.2a). The voltage across the load is V2. The primary and secondary windings of the ideal transformer have zero impedance.
Hence, the induced emf E1 in the primary exactly counter balances the applied voltage V1, that is, V1 = -E1. Also, the induced emf
E2 is the same as voltage V2, that is, E2 = V2. Here, E1 is called counter emf or back emf induced in the primary, and E2 called
mutually induced emf in the secondary.

2
(a) The circuit. (b) The phasor diagram.
Fig. 1.2 Ideal transformer.
Figure 1.2b shows the phasor diagram of the ideal transformer. We have taken flux Φ as reference phasor, as it is common to
both the primary and secondary. As per Eq. 1.2, the induced emfs E1 and E2 lag flux Φ by 90°. The voltage V1 is equal and
opposite to emf E1. Thus, the applied voltage V1 leads the flux Φ by 90°. According to the first condition of ideality, the
reluctance of the magnetic circuit is zero and hence the required magnetizing current to produce flux Φ is also zero.
Transformation Ratio
The ratio of secondary voltage to the primary voltage is known as transformation ratio or turns-ratio. It is denoted by letter K.
Let N1 and N2 be the number of turns in primary and secondary windings, and E1 and E2 be the rms values of the primary and
secondary induced emfs. Using Eq. 1.3, we can write
E1 = 4.44 fN1Φ m
...(1.4)
and E2 = 4.44 fN 2Φ m
...(1.5)
Then, the transformation ratio or turns-ratio can be expressed as
V2 E2 N 2
K= = =
V1 E1 N1
...(1.6)
Thus, the side of the transformer with the larger number of turns has the larger voltage. Indeed, the voltage per turn is
constant for a given transformer. By selecting K properly, the transformation of voltage can be done from any value to any other
convenient value. There can be two1 cases:
(i) When K > 1 (i.e., N2 > N1); V2 > V1: the device is known as step-up transformer.
(ii) When K < 1 (i.e., N2 < N1); V2 < V1: the device is known as step-down transformer.
In general, a transformer can have more than 2 windings. The windings of a three-winding transformer are called primary,
secondary and tertiary. The primary is connected to an ac supply. Different loads may be connected across the secondary and
tertiary2. The induced emf in a winding is still proportional to its number of turns,
E1 : E2 : E3 :: N1 : N 2 : N3
Volt-Amperes
Consider again the two-winding transformer of Fig. 1.2a. For an ideal transformer, the current I1 in the primary is just sufficient to
provide mmf I1N1 to overcome the demagnetizing effect of the secondary mmf I2N2. Hence,
I 2 N1 1
∴ I1 N1 = I 2 N 2 or = =
I1 N 2 K
...(1.7)
Thus, we find that the current is transformed in the reverse ratio of the voltage. That is, the side of the transformer with the larger
number of turns has the smaller current. For example, a step-up transformer would have a primary with few turns of thick wire
(small voltage, large current) and the secondary would have many turns of thin wire (large voltage, small current).
Combining Eqs. 1.5 and 1.7, we have
E1I1 = E2 I 2
Hence, in an ideal transformer the input VA and output VA are identical.

1
The third case, when K = 1 (i.e., N1 = N2) is not important. We hardly ever use a transformer with unity turns ratio. Such a
transformer is used only when you need electrical isolation between two electrical circuits.
2
Sometimes, the tertiary winding has a centre-tap; the two halves having same number of turns, N3. The voltage of such a
winding is then specified as E3/0/ E3, or E3 - 0 - E3.

3
Impedance Transformation
Equations 1.6 and 1.7 reveal a very useful property of transformers, called impedance transformation. Figure 1.3 shows an ideal
transformer. It has N1 and N2 turns in its primary and secondary windings respectively. A load impedance ZL is connected across
its secondary, and an equivalent impedance Zeq is defined at its primary.

Fig. 1.3 The transformer changes the impedance ZL to equivalent impedance Zeq.
The equivalent impedance Zeq as faced by a source V1 is given as
V1 V1 × (V2 I 2 ) ⎛ V1 ⎞ ⎛ I 2 ⎞ ⎛ V2 ⎞ ⎛ 1 ⎞ ⎛ 1 ⎞
Z eq = = = ⎜ ⎟ × ⎜ ⎟ × ⎜ ⎟ = ⎜ ⎟ × ⎜ ⎟ × ZL
I1 I1 × (V2 I 2 ) ⎝ V2 ⎠ ⎝ I1 ⎠ ⎝ I 2 ⎠ ⎝ K ⎠ ⎝ K ⎠

or Z eq = Z L / K 2
...(1.8)
Therefore, the impedance is transformed in inverse proportion to the square of the turns-ratio. The concept of impedance
transformation is used for impedance matching. As per maximum power transfer theorem, the load impedance has to be properly
matched with the source impedance, as illustrated in Example 1.3 given below.

Example 1.2 A single-phase, 50-Hz transformer has 30 primary turns and 350 secondary turns. The net cross-sectional area of
the core is 250 cm2. If the primary winding is connected to a 230-V, 50-Hz supply, calculate (a) the peak value of flux density in
the core, (b) the voltage induced in the secondary winding, and (b) the primary current when the secondary current is 100 A.
(Neglect losses.)
Solution: (a) The peak value of the flux in the core is given as
E1 230
Φm = = = 0.034534 Wb
4.44 fN1 4.44 × 50 × 30
Therefore, the peak value of the flux density in the core is
Φm 0.034534
Bm = = = 1.3814 T
A 250 × 10 −4
(b) The voltage induced in the secondary winding is
N2 350
E2 = E1 × = 230 × = 2683.33 V = 2.683 kV
N1 30
(c) The primary current is
⎛N ⎞ ⎛ 350 ⎞
I1 = I 2 ⎜ 2 ⎟ = 100 × ⎜ ⎟ = 1166.67 A 1.167 kA
⎝ N1 ⎠ ⎝ 30 ⎠
Example 1.3 A source with an output resistance of 50 Ω is required to deliver power to a load of 800 Ω. Find the turns-ratio of
the transformer to be used for maximizing the load power.
Solution: For delivering maximum power to the load, the equivalent resistance must be equal to the source resistance. This
requires a resistance of 50 Ω looking into the primary of the transformer. That is,
Req = RL / K 2 or 50 = 800 / K 2 ⇒ K = 800 / 50 = 16 = 4
Thus,
N2
K= =4
N1
Example 1.4 Determine the load current IL in the ac circuit shown in Fig. 1.4a.

4
Ip 20 Ω j20 Ω 2:1 IL Ip 20 Ω j20 Ω

( 2)
2
×2 Ω
+ 2Ω +

30 V ⇒ −
30 V
( 2 ) × ( − j10 ) Ω
2
-j10 Ω

(a) (b)
Fig. 1.4
Solution: We first transform the load impedance into the primary to simplify the circuit, as shown in Fig. 1.4b. The primary
current is then calculated as
30∠0°
Ip = = 0.872∠35.53° A
20 + j 20 + 22 (2 − j10)
The load current, which is the same as the secondary current, is given by Eq. 1.7 as
I L = 2 × I p = 2 × 0.872∠35.53° = 1.74∠35.53° A
Example 1.5 A single-phase transformer has a core with cross-sectional area of 150 cm2. It operates at a maximum flux density
of 1.1 Wb/m2 from a 50-Hz supply. If the secondary winding has 66 turns, determine the output in kVA when connected to a
load of 4-Ω impedance. Neglect any voltage drop in the transformer.
Solution: Φm = Bm A = 1.1× 0.015 = 0.0165 Wb . Since the voltage drop in the transformer is negligible, we have
V2 = E2 = 4.44 fN 2Φm = 4.44 × 50 × 66 × 0.0165 = 241.76 V
V2 241.76
The output current, I 2 = = = 60.44 V
ZL 4
∴ Output volt-amperes = 241.76 × 60.44 = 14 612 VA = 14.612 kVA
Example 1.6 A single-phase, 50-Hz transformer has a square core having a net cross-sectional area of 9 cm2, and three
windings designed for the following voltages:
(i) Primary: 230 V; (ii) Secondary: 110 V; and (iii) Tertiary: 6/0/6 V.
Find the number of turns in each winding if the flux density is not to exceed 1 T.
Solution: Φm = Bm A = 1× 9 × 10 −4 = 9 × 10−4 Wb . The tertiary winding is divided into two halves; each half having a
voltage E3 = 6 V. Thus, the number of turns in each half of the tertiary is
E3 6
N3 = = = 30 turns
4.44 f Φ m 4.44 × 50 × 9 × 10−4
∴ Total number of turns on the tertiary winding = 2 × 30 = 60 turns .
We have seen that across 30 turns of the tertiary winding, the induced emf is 6 V. Therefore, the number of turns on the
primary and secondary can be calculated as follows :
N1 E1 N 3 E1 30 × 230
= or N1 = = = 1150 turns
N 3 E3 E3 6
N 2 E2 N 3 E2 30 ×110
and = or N2 = = = 550 turns
N 3 E3 E3 6
1.4 PRACTICAL TRANSFORMER AT NO LOAD
In actual practice, a transformer can never satisfy any of the conditions specified above for the ideal transformer. Nevertheless, the
concept of ideal transformer is helpful to understand the working of an actual transformer. We shall consider these conditions one
by one, and see in what way a practical transformer deviates from the ideal transformer. In this Section, we shall consider only the
first two ideality conditions. The remaining two conditions shall be considered in Section 1.7.
Consider a transformer with its primary connected to an alternating voltage source V1, and no load connected across its
secondary (Fig. 1.5a). With open circuit, the current in the secondary winding is zero. If the transformer were truly ideal, the
primary current too would be zero, as per 713.5. But, in practice there does flow a little no-load current I0 in the primary. This
current I0 is also called the exciting current of the transformer. Following are the two reasons why the no-load current I0 flows in
the primary.

5
(1) Effect of Magnetization
Consider the first ideality condition. No magnetic material can have infinite permeability so as to offer zero reluctance to the
magnetic circuit. Hence, in a practical transformer a finite mmf is needed to establish magnetic flux in the core. As a result, an
in-phase magnetizing current Im in the primary is needed to set up flux Φ in the core. This current Im is purely reactive and lags
the voltage V1 by 90°. The flux Φ induces emfs E1 and E2 in the primary and the secondary windings. As per Eq. 1.2, both these
emfs lag flux Φ by 90°, as shown in Fig. 1.5b.

(a) The circuit. (b) The phasor diagram. (c) The equivalent circuit.
Fig. 1.5 Transformer on no load.
As the current I2 in the secondary is zero (no load connected), the voltage drop in the secondary winding is zero. Hence, V2 =
E2. The induced emf E1 counter balances the applied voltage V1 and establishes an electrical equilibrium. If the third and fourth
ideality conditions (i.e., the effect of the resistance of the winding and the leakage of flux) are ignored, the magnitude of V1 will be
the same as that of emf E1. Thus, V1 = -E1.
(2) Effect of Core Losses
Let us now consider the second ideality condition. There exist two reasons (hysteresis and eddy current) for the energy loss in
the core of the transformer. The source must supply enough power to the primary to meet the core losses. Therefore, a core-loss
current Iw (in phase with V1) flows through the primary, as shown in the phasor diagram of Fig. 1.5b.
Thus, we find that the no-load current I0 has two components, Im and Iw. The magnetizing current Im lags voltage V1 by 90° and
the loss component Iw is in phase with voltage V1. The angle φ0 is the no-load phase angle. Thus, from the phasor diagram of Fig.
1.5b, we have`

I 0 = I w2 + I m2 ; φ0 = tan −1 ( I m / I w ); and Input power = V1 I w = V1 I 0 cos φ0


In the equivalent circuit shown in Fig. 1.5c, the no-load current I0 is divided into two parallel branches. The component Iw
accounts for the core loss, and hence is shown to flow through a resistance R0. The component Im represents magnetizing current.
Hence, it is shown to flow through a pure inductive reactance X0. The R0-X0 parallel circuit is called exciting circuit of the
transformer.

Example 1.7 A single-phase, 230-V/110-V, 50-Hz transformer takes an input of 350 volt amperes at no load while working at
rated voltage. The core loss is 110 W. Find the loss component of no-load current, the magnetizing component of no-load
current and the no-load power factor.
Solution: Given: V1 I 0 = 350 VA .
VA 350
∴ I0 = = = 1.52 A
V1 230
The core loss = Input power at no load, Pi = V1 I 0 cos φ0
Therefore, the power factor at no load is given as
Pi 110 W
pf = cos φ0 = = = 0.314
V1 I 0 350 VA
The loss component of no-load current is given as
I w = I 0 cos φ0 = 1.52 × 0.314 = 0.478 A
The magnetizing component of no-load current is given as

I m = I 02 − I w2 = (1.52) 2 − (0.478) 2 = 1.44 A

Since the core losses occur in the iron core, these are also called iron losses. These losses have two components: (i) Hysteresis
Loss, and (ii) Eddy-Current Loss

6
(i) Hysteresis Loss: When alternating current flows through the windings, the core material undergoes cyclic process of
magnetization and demagnetization. It is found that there is a tendency of the flux density B to lag behind the field strength H.
This tendency is called hysteresis3. The effect of this phenomenon on the core material can be best understood from the B-H plot
shown in Fig. 1.6.

Fig. 1.6 Hysteresis loop and energy relationship per half-cycle.


During positive half-cycle, when H increases from zero to its positive maximum value, the energy is stored in the core. This
energy is given by the area abda. However, when H decreases from its positive maximum value to zero, the energy is released
which is given by the area bdcb. The difference between these two energies is the net loss and is dissipated as heat in the core.
Thus, as H varies over one complete cycle, the total energy loss (per cubic metre) is represented by the area abcea of the
hysteresis loop. The hysteresis loss (usually expressed in watts) is given as
Ph = K h Bmn f V
...(1.9)
where Kh = hysteresis coefficient whose value depends upon the material
(Kh = 0.025 for cast steel, Kh = 0.001 for silicon steel)
Bm = maximum flux density (in tesla)
n = a constant, 1.5 ≤ n ≤ 2.5 depending upon the material
f = frequency (in hertz)
V = volume of the core material (in m3)
This loss can be minimized by selecting suitable ferromagnetic material for the core.
(ii) Eddy-Current Loss: The eddy currents are the circulating currents set up in the core due to alternating magnetic flux
(shown with dotted lines in Fig. 1.7a). These currents (shown in Fig. 1.7b) may be quite high since the resistance of the iron is
quite low. This results in unnecessary heating of the core and loss of power. The eddy-current loss (in watts) is given by
Pe = K e Bm2 f 2t 2V
...(1.10)
where Ke = a constant dependent upon the material
t = thickness of laminations (in metre)
The eddy-current loss can be minimized by dividing the solid iron core into thin sheets or laminations insulated from one
another (Fig. 1.7c). The path of the induced eddy currents in the core is broken by the insulating material between the sheets. The
eddy currents and hence the eddy-current loss is thus substantially reduced.

(a) (b) (c)


Fig. 1.7 Laminated core helps in reducing the eddy currents.

3
In Greek, hysterein means ‘to lag’.

7
Note that the eddy-current loss varies as the square of the frequency, whereas the hysteresis loss varies directly with the
frequency. The total iron loss is given as
Pi = Ph + Pe
...(1.11)
Example 1.8 A single-phase, 230-V/110-V, transformer has iron loss of 100 W at 60 Hz, and 60 W at 40 Hz. Determine the
hysteresis and eddy-current losses at 50 Hz.
Solution: We know that the hysteresis loss, Ph ∝ f ⇒ Ph = Af
and the eddy-current loss, Pe ∝ f 2
⇒ Pe = Bf 2
Then, the total iron loss, Pi = Ph + Pe = Af + Bf . Therefore, at the given two frequencies, we have
2

At 60 Hz : 100 = 60 A + 3600 B
At 40 Hz : 60 = 40 A + 1600 B
Solving the above two equations, we get A = 1.167 and B = 0.00834. We can now calculate the two losses at 50 Hz,
Hysteresis loss at 50 Hz, Ph = Af = 1.167 × 50 = 58.35 W
Eddy-current loss at 50 Hz, Pe = Bf = 0.00834 × (50) 2 = 20.85 W
2

1.5 CONSTRUCTION OF TRANSFORMER


The main parts of an actual transformer used in power circuits are as follows :
(i) An iron core that provides a magnetic circuit.
(ii) Two inductive coils wound on the core. These are suitably insulated from each other and also from the core. The
individual turns of a coil are also insulated from each other.
(iii) A suitable container for the assembled core and windings.
(iv) A suitable medium (called transformer oil) for insulating the core and windings from the container. This medium also
cools the windings and core of the transformer.
(v) Suitable porcelain bushings for insulating and bringing out winding terminals from the tank.
Core of Transformer
The core is made of steel laminations so as to minimize eddy-current loss. The laminations are about 0.35 mm thick and are
insulated from each other by a light coat of varnish on the surface. The laminations are pressed together so as to form a continuous
magnetic path, with minimum air gap. Depending upon the construction of the core, there are two types of transformers:
(1) Core Type Transformer: In this type, the windings surround a considerable part of the core. Both the windings are
divided into two parts and half of each winding is placed on each limb, side by side (shown schematically in Fig. 1.8a). This is
done to reduce the leakage of the magnetic flux. Furthermore, the low voltage (LV) winding is placed adjacent to the core and
high voltage (HV) winding is placed around the LV winding, as shown in Fig. 1.8b. This is done to minimize the cost of
insulation.

(a) (b)
Fig. 1.8 Core type transformer.
(2) Shell Type Transformer: It has three limbs. Both the windings are placed on the central limb (shown schematically in
Fig. 1.9a). The LV and HV windings are sandwiched as shown in Fig. 1.9b. Here, the core surrounds a considerable part of the
windings.

8
(a) (b)
Fig. 1.9 Shell type transformer.
In core type transformer, the flux has single path. But in shell type transformer, the flux divides equally in the central limb and
returns through the outer two legs. Since there is more space for insulation in the core type transformer, it is preferred for high
voltages. On the other hand, the shell type construction is more economical for low voltages.
1.6 TRANSFORMER ON LOAD
Let us examine what happens when a load is connected to the secondary of the transformer. Note that for simplicity we are still
considering a partially ideal transformer (i.e., a transformer satisfying only the ideality conditions (iii) and (iv) stated on page
000). Before connecting the load, there exists a flux Φ in the core due to the no-load current I0 flowing in the primary. On
connecting the load, a current I2 flows through the secondary, as shown in Fig. 1.10. The magnitude and phase of I2 with respect
to the secondary voltage V2 depends upon the nature of the load.

Fig. 1.10 Transformer on load.



The current I2 sets up a flux Φ in the core, which opposes the main flux Φ. This momentarily weakens the main flux, and the
primary back emf E1 gets reduced. As a result, the difference V1 - E1 increases and more current is drawn from the supply. This
again increases the back emf E1, so as to balance the applied voltage V1. In this process, the primary current increases by I1’. This
current is known as primary balancing current, or load component of primary current. Under such a condition, the secondary
ampere-turns must be counterbalanced by the primary ampere-turns. That is, N1 I1
'
= N 2 I 2 . Hence, we have
⎛N ⎞
I1' = ⎜ 2 ⎟ I 2 = KI 2
⎝ N1 ⎠
...(1.12)
The total primary current I1 is the phasor sum of the no-load current I0 and the primary balancing current I1’. That is,
I1 = I 0 + I1'
...(1.13)
Phasor Diagram
The phasor diagrams for the (partial) ideal transformer with resistive, inductive and capacitive loads are shown in Fig. 1.11. We
take flux Ф as reference phasor, as it is common to both the primary and secondary. The induced emfs E1 and E2 lag flux Ф by
90°. Assuming a step-up transformer, E2 > E1. The voltage V2 is same as E2. However, the voltage V1 has the same magnitude as
E1 but opposite in phase. Note that the primary balancing current I1’ is always in phase opposition to the secondary current I2.
For a step-up transformer, I1’ > I2. Since the no-load current I0 is negligibly small (compared to I1’), the primary current I1 is
almost opposite in phase to the secondary current I2. (Note that, in the phasor diagrams of Fig. 1.11, for the sake of clarity the no-
load current I0 is shown much larger than the practical values.)

9
(a) Resistive load. (b) Inductive load. (c) Capacitive load.
Fig. 1.11 Phasor diagrams for a transformer on load.

Example 1.9 A single-phase, 440-V/110-V, 50-Hz transformer takes a no-load current of 5A at 0.2 power factor lagging. If the
secondary supplies a current of 120 A at a power factor of 0.8 lagging to a load, determine the primary current and the primary
power factor.
Solution: φ0 = cos −1 0.2 = 78.46° and φ2 = cos −1 0.8 = 36.87°
V2 110 1
The transformation ratio, K = = =
V1 440 4
∴ I1' = K × I 2 = (1/ 4) ×120 = 30 A
The angle between I 0 and I1' ,θ = φ0 − φ2 = 78.46° − 36.87° = 41.59°
∴ I1 = I 02 + I1'2 + 2 I 0 I1' cos θ = 52 + 302 + 2 × 5 × 30 × cos 41.59° = 33.9 A
The phasor diagram is shown in Fig. 1.12. Note that for the sake of clarity, the phasors are not drawn to the scale. The angle
between I0 and I1 is given as
I1' sin θ 30sin 41.59°
α = tan −1 = tan −1 = 35.97°
I 0 + I1 cos θ
'
5 + 30 cos 41.59°
∴ angle φ1 = φ0 − α = 78.46° − 35.97° = 42.49°
Thus,
Primary power factor = cos φ1 = cos 42.49° = 0.737

Fig. 1.12 Phasor diagram.

10
1.7 PRACTICAL TRANSFORMER ON LOAD
We now consider the ideality conditions (iii) and (iv) stated on page 3. The effects of deviations from these conditions become
more prominent when a practical transformer is put on load.
Effect of Winding Resistance
The windings are made of copper wire, since copper has good conductivity. In actual practice, each winding has some resistance.
Current flow through the windings causes not only a voltage drop but also a power loss called I2R loss or copper loss. This effect
is accounted for by including a resistance R1 in the primary and resistance R2 in the secondary, as shown in Fig. 1.13.

Fig. 1.13 Effect of winding resistance.


Effect of Flux Leakage
In a practical transformer, the entire magnetic flux does not remain confined to the magnetic core. Not all the flux produced by
the primary winding links with the secondary. As shown in Fig. 1.14a, a part of it leaks through the air paths. The difference
between the total flux linking with the primary and the useful mutual flux Φu linking with both the windings is called the primary
leakage flux, ΦL1. Similarly, ΦL2 represents the secondary leakage flux.

(a) Its definition. (b) Its effect accounted for.


Fig. 1.14 Leakage flux in a transformer.
It is the useful mutual flux Φu that is responsible for the transformer action. The primary leakage flux ΦL1 induces an emf EL1
in the primary winding. Similarly, flux ΦL2 induces an emf EL2 in the secondary. The effect of flux leakage can then be accounted
for by including reactances X1 and X2 in the primary and secondary windings, respectively (as shown in Fig. 1.14b), such that
EL1 = I1 X 1 and EL 2 = I 2 X 2
The reactances X1 and X2 are called primary and secondary leakage reactances, respectively. Note that these reactances are
fictitious quantities. These are introduced just as a convenience to represent the effect of the flux leakage.
The reluctance of the paths of the leakage fluxes ΦL1 and ΦL2 is almost entirely due to the long air paths and is therefore
practically constant. Consequently, the value of the leakage flux is proportional to the current. On the other hand, the value of the
useful flux Φu remains almost independent of the load. Furthermore, the reluctance of the paths of the leakage flux is very high.
Hence, the value of this flux is relatively small even on full load.
1.8 EQUIVALENT CIRCUIT OF A TRANSFORMER
The function of an ideal transformer is to transform electric power from one voltage level to another without incurring any loss
and without needing any magnetizing current. For such a transformer, the volt-amperes in the primary are exactly balanced by the
volt-amperes in the secondary. An ideal transformer is supposed to operate at 100 percent efficiency.
We stated the four conditions that must be satisfied by a transformer to be ideal. We then examined the effects of each of these
conditions and explored how to account for the deviations in a practical transformer. Based on this, we can now draw the
equivalent circuit of a practical transformer (Fig. 1.15). This circuit is merely a representation of the following KVL equations for
the primary and secondary sides of the transformer.
V1 = I1 R1 + jI1 X 1 − E1 = I1 ( R1 + jX 1 ) − E1
...(1.14)
and E2 = I 2 R2 + jI 2 X 2 + V2 = I 2 ( R2 + jX 2 ) + V2
...(1.15)

11
Equation 1.14 states that the applied voltage V1 is the phasor sum of the negative of induced emf E1 and the voltage drops in
primary resistance, R1, and leakage reactance, X1, due to the flow of current I1. The induced emf E2 forces a current I2 in the
secondary circuit. Hence, Eq. 1.15 states that the induced emf E2 is phasor sum of the load voltage V2 and the voltage drops in
secondary resistance, R2, and leakage reactance, X2, due to the flow of current I2.

Fig. 1.15 Equivalent circuit of a transformer.


Furthermore, it can be seen from Fig. 1.15 that the primary current I1 is composed of two components, the no-load current I0
and the load component of primary current I1’. Moreover, the current I0 consists of two components Iw and Im. The current Iw
flows through resistance R0 and accounts for the iron loss of the transformer. The current Im, called magnetizing current, is
required to establish working magnetic flux in the core. The ac voltage source connected to the primary winding does not have to
supply any power in making this current flow. Hence, in the equivalent circuit, it is shown to flow through a pure reactance X0.
Phasor Diagram
We can draw the phasor diagram of a transformer circuit with a given load, provided all its parameters (as used in the equivalent
circuit of Fig. 1.15) are known. While drawing the phasor diagram, we should keep the following points in mind :
1. It is most convenient to commence the phasor diagram with the phasor representing the quantity that is common to the
two windings, namely, the flux Φ.
2. The induced emfs E1 and E2 lag behind flux Φ by 90°.
3. The values of emfs E1 and E2 are proportional to the number of turns on the primary and secondary windings.
4. The magnitude and phase of the current I2 is decided by the load.
5. The resistive voltage drops are always in phase with the respective current phasor.
6. The inductive voltage drops lead the respective current phasor by 90°.
7. The secondary induced emf E2 is obtained by vector sum of the terminal voltage V2 and the impedance drop I2Z2. Hence,
V2 must be drawn such that the phasor sum of V2 and I2Z2 is E2.
8. The primary balancing current I1’ and the secondary current I2 are in inverse proportion to the number of turns on the
primary and secondary windings.
9. The primary current I1 is the vector sum of the no-load current I0 and the primary balancing current I1’.
10. The primary voltage V1 is obtained by adding vectorially the impedance drop I1Z1 to the negative of E1.
11. The phase angle φ1 between V1 and I1 is the power factor-angle of the transformer.
The phasor diagrams for different types of loads (resistive, inductive and capacitive) are shown in Fig. 1.16.

(a) Resistive load. (b) Inductive load. (c) Capacitive load.


Fig. 1.16 Phasor diagrams for different types of loads.
Simplified Equivalent Circuit

12
Since the no-load current I0 of a transformer is only about 3-5 percent of the full-load primary current, not much error will be
introduced if the exciting circuit R0-X0 in Fig. 1.15 is shifted to the left of impedance R1-X1. This results in a circuit shown in Fig.
1.17a.

(a) The exciting circuit shifted to the left.

(b) The impedances transferred from secondary side to the primary side.

(c) Equivalent resistance and reactance referred to the primary side.


Fig. 1.17 Simplified equivalent circuit of a transformer as referred to the primary side.

Using the impedance transformation, we can now transform the impedances from the secondary side to the primary side and
remove the ideal transformer from the circuit, as shown in Fig. 1.17b. This can further be simplified by combining the two
resistances together and the two leakage reactances together, as shown in Fig. 1.17c. Here, the total resistance and total leakage
reactance as referred to primary are given as
Re1 = R1 + ( R2 / K 2 ) and X e1 = X 1 + ( X 2 / K 2 )
...(1.16)
Approximate Equivalent Circuit
Compared to the full-load primary current, the no-load current of a transformer is very small (only 3-5 percent). Therefore, while
considering the behaviour of a transformer on full-load, we can omit the exciting circuit R0-X0 without introducing much error.
The resulting approximate equivalent circuit is shown in Fig. 1.18a.

(a) As referred to primary side. (b) As referred to secondary side.


Fig. 1.18 Approximate equivalent circuit of a transformer.
Alternatively, we could transfer the impedances from primary side to the secondary side, so as to get the approximate
equivalent circuit as shown in Fig. 1.18b. Here, the total resistance and total leakage reactance as referred to secondary are given
as
Re 2 = K 2 R1 + R2 and X e2 = K 2 X1 + X 2
...(1.17)

13
Example 1.10 A single-phase, 50-kVA, 4400-V/220-V, 50-Hz transformer has primary and secondary resistances R1 = 3.45 Ω
and R2 = 0.009 Ω, respectively. The values of the leakage reactances are X1 = 5.2 Ω and X2 = 0.015 Ω. Calculate for this
transformer (a) the equivalent resistance as referred to the primary, (b) the equivalent resistance as referred to the secondary, (c)
the equivalent reactance as referred to the primary, (d) the equivalent reactance as referred to the secondary, (e) the equivalent
impedance as referred to the primary, (f) the equivalent impedance as referred to the secondary, (g) the total copper loss first by
using the individual resistances of the two windings and then by using the equivalent resistances as referred to each side.
kVA 50 000
Solution: Full-load primary current, I1 = = = 11.36 A
V1 4400
kVA 50 000
Full-load secondary current, I 2 = = = 227.27 A
V2 220
V2 220 1
Transformation ratio, K = = = = 0.05
V1 4400 20
(a) Re1 = R1 + ( R2 / K ) = 3.45 + [0.009 /(0.05) ] = 7.05 Ω
2 2

(b) Re 2 = K R1 + R2 = (0.05) × 3.45 + 0.009 = 0.0176 Ω


2 2

(c) X e1 = X 1 + ( X 2 / K ) = 5.2 + [0.015 /(0.05) ] = 11.2 Ω


2 2

(d) X e 2 = K X 1 + X 2 = (0.05) × 5.2 + 0.015 = 0.028 Ω


2 2

(e) Z e1 = Re21 + X e21 = (7.05) 2 + (11.2) 2 = 13.23 Ω

(f) Z e 2 = Re22 + X e22 = (0.0176)2 + (0.028) 2 = 0.0331 Ω


(g) Total copper loss = I1 R1 + I 2 R2 = (11.36) × 3.45 + (227) × 0.009 = 909 W
2 2 2 2

By considering equivalent resistances,


Total copper loss = I1 Re1 = (11.36) × 7.05 = 909.8 W
2 2

Total copper loss = I 2 Re 2 = (227.27) × 0.0176 = 909 W


2 2

1.9 VOLTAGE REGULATION OF A TRANSFORMER


With the increase in the load on a transformer, there is a change in its secondary terminal voltage. The voltage falls if the load
power-factor is lagging. It increases if the power factor is leading. The voltage regulation of a transformer is defined as the
change in its secondary terminal voltage from no load to full load, the primary voltage being assumed constant. Let
V2(0) = secondary terminal voltage at no load,
and V2 = secondary terminal voltage at full load.
Then, the voltage drop V2(0) - V2 is called the inherent regulation. We can compare this change in voltage with respect to either
the no-load voltage or the full-load voltage, and it can be expressed as either per unit basis or percentage basis. Thus, we have
V2(0) − V2
(i) Per unit regulation down =
V2(0)
V2(0) − V2
% regulation down = × 100
V2(0)
V2(0) − V2
(ii) Per unit regulation up =
V2
V2(0) − V2
% regulation up = ×100
V2
Normally, when nothing is specified, ‘regulation’ means ‘regulation down’.
Approximate Voltage Drop
At no load, V1 E1 and E2 = V2(0) . Also, if K is the transformation ratio, the secondary terminal voltage at no
load, V2(0) = E2 = KE1 = KV1 . Thus, using the equivalent circuit referred to the secondary (shown in Fig. 1.18b), we can draw
the phasor diagram of the transformer, as given in Fig. 1.19. In this diagram,
OA represents the full-load terminal voltage, V2
AB represents the resistive voltage drop, I2Re2

14
BC represents the reactive voltage drop, I2Xe2
AC represents the total impedance voltage drop, I2Ze2
OC represents the no-load terminal voltage, V2(0)
(same as the input voltage referred to the secondary, KV1)

Fig. 1.19 Phasor diagram of a transformer referred to the secondary.


With point O as centre and OC as radius, draw an arc which intersects the extended OA at point G, so that OC = OG. From
point C drop a perpendicular on OG, which intersects it at F. Thus, we have
Exact voltage drop = V2(0) − V2 = OC − OA = OG − OA = AG = AF + FG
Here, AF is the approximate voltage drop, which we intend to determine. The error in this approximation is FG. Since the
impedance voltage drop I2Ze2 (= AC) is very small compared to the full-load voltage V2 (= OA), the error committed in the
determination of the approximate voltage drop would be negligibly small.
From point B, drop a perpendicular BE on OG, and draw BD parallel to AF. We can now write
Approximate voltage drop, AF = AE + EF = AE + BD
= I 2 Re 2 cos φ + I 2 X e 2 sin φ
...(1.18)
In case of leading power factor of the load (instead of lagging), we can determine the approximate voltage drop either by putting -
φ in place of φ in Eq. 1.18 or by redrawing the phasor diagram for leading power factor, to get
Approximate voltage drop, AF = AE − EF = AE − BD
= I 2 Re 2 cos φ − I 2 X e 2 sin φ
...(1.19)
Thus, in general, we can write
Approximate voltage drop = I 2 Re 2 cos φ ± I 2 X e 2 sin φ
...(1.20)
in which + sign is to be used for lagging power factor and – sign for leading power factor. The voltage drop expressed in
percentage is the percent regulation, and is given as
I 2 Re 2 cos φ ± I 2 X e 2 sin φ
% Regulation = × 100
V2(0)
= Vr cos φ ± Vx sin φ
...(1.21)
I 2 Re 2
where Vr = % resistive drop = ×100
V2(0)
...(1.22)
I X
and Vx = % reactive drop = 2 e 2 × 100
V2(0)
...(1.23)
Exact Voltage Drop
Referring to Fig. 1.19, the exact voltage drop is AG. We have already determined the approximate value of voltage drop given by
AF. To determine FG, we proceed as follows. For the right angle triangle OFC, we can write

15
OC 2 = OF 2 + FC 2
or OC 2 − OF 2 = FC 2
or (OC − OF ) (OC + OF ) = FC 2
or (OG − OF ) (OC + OF ) = FC 2
or FG (2OC ) = FC 2 [Taking OF OC ]
FC 2
( DC − DF ) ( DC − BE )
2
( I X cos φ − I 2 Re 2 sin φ ) 2
2
∴ FG = = = = 2 e2
2OC 2OC 2OC 2V2(0)
Thus, for lagging power factor,
Exact voltage drop = AF + FG
( I 2 X e 2 cos φ − I 2 Re 2 sin φ ) 2
= ( I 2 Re 2 cos φ + I 2 X e 2 sin φ ) +
2V2(0)
For leading power factor,
( I 2 X e 2 cos φ + I 2 Re 2 sin φ ) 2
Exact voltage drop = ( I 2 Re 2 cos φ − I 2 X e 2 sin φ ) +
2V2(0)
Thus, in general, we have
( I 2 X e 2 cos φ m I 2 Re 2 sin φ ) 2
Exact voltage drop = ( I 2 Re 2 cos φ ± I 2 X e 2 sin φ ) +
2V2(0)
...(1.24)
( I R cos φ ± I 2 X e 2 sin φ ) ( I X cos φ m I 2 Re 2 sin φ ) 2
% Exact voltage drop = 2 e 2 × 100 + 2 e 2 × 100
V2(0) 2V2(0) × V2(0)
or
1
= (Vr cos φ ± Vx sin φ ) + (Vx cos φ m Vr sin φ ) 2
2V2(0)
...(1.25)
Keep in mind that upper signs are to be used for lagging power factor, and the lower signs for leading power factor.
Condition for Zero Regulation
It is possible to obtain zero regulation for a transformer. For this, the voltage drop (as given by Eq. 1.20) from no-load to full-
load should be zero. It is possible only if the sign in Eq. 1.20 is negative, i.e., only if the load has leading power factor. Thus, the
condition of zero regulation is given as
Re 2
I 2 Re 2 cos φ − I 2 X e 2 sin φ = 0 or tanφ =
X e2
...(1.26)
Also, note that for leading power factor, if the magnitude of the phase angle φ is high, the magnitude of I 2 X e 2 sin φ may
become greater than that of I 2 Re 2 cos φ . The regulation then becomes negative. It means that on increasing the load the
terminal voltage increases.
Condition for Maximum Regulation
We can derive the condition for maximum regulation (the worst case) using Eq. 1.20. The maximum value of regulation occurs
when the voltage drop is maximum (for which we use + sign). Therefore, the condition of maximum regulation can be obtained
by differentiating Eq. 1.20 with respect to the phase angle φ and equating it to zero,
d
( I 2 Re 2 cos φ + I 2 X e 2 sin φ ) = 0 ⇒ (− I 2 Re 2 sin φ + I 2 X e 2 cos φ ) = 0

X
or tan φ = e 2
Re 2
...(1.27)

16
Example 1.11 A single-phase, 40-kVA, 6600-V/250-V, transformer has primary and secondary resistances R1 = 10 Ω and R2 =
0.02 Ω, respectively. The equivalent leakage reactance as referred to the primary is 35 Ω. Find the full-load regulation for the
load power factor of (a) unity, (b) 0.8 lagging, and (c) 0.8 leading.
Solution: Given : R1 = 10 Ω; R2 = 0.02 Ω; Xe1 = 35 Ω
250
the turns-ratio, K = = 0.0379
6600
40 000
the full-load current, I 2 = = 160 A
250
∴ Re 2 = K 2 R1 + R2 = (0.0379) 2 × 10 + 0.02 = 0.0343 Ω
and X e 2 = K 2 X e1 = (0.0379) 2 × 35 = 0.0502 Ω
(a) For power factor, cos φ = 1; sin φ = 0. Hence,
I 2 Re 2 cos φ + I 2 X e 2 sin φ
∴ % Regulation = × 100
V2(0)
160 × 0.0343 ×1 + 0
= ×100 = 2.195 %
250
(b) For power factor, cos φ = 0.8 (lagging, φ positive); sin φ = 1 − cos φ = 0.6 .
2
Hence,
I 2 Re 2 cos φ + I 2 X e 2 sin φ
∴ % Regulation = × 100
V2(0)
160 × 0.0343 × 0.8 + 160 × 0.0502 × 0.6
= × 100 = 3.68 %
250
(c) For power factor, cos φ = 0.8 (leading, φ negative); sin φ = −0.6 . Hence,
I 2 Re 2 cos φ − I 2 X e 2 sin φ
∴ % Regulation = × 100
V2(0)
160 × 0.0343 × 0.8 − 160 × 0.0502 × 0.6
= ×100 = −0.172 %
250

1.10 EFFICIENCY OF A TRANSFORMER


Like any other machine, the efficiency of a transformer is defined as
Power output Power output Po
η= = =
Power input Power output +Power loss Po + Pl
1.10
There are two types of losses in a transformer:
(i) Copper losses or I2R losses in the primary and secondary windings, given as
Pc = I12 R1 + I 22 R2 = I12 Re1 = I 22 Re 2
1.11
The copper losses are variable with current. Let us see by what factor the copper losses get reduced when the load on the
transformer decreases. We know that the output power is given as
Po
Po = VI cos φ = VI × pf ⇒ VI =
pf
1.12
Thus, for a given load we can find the volt-ampere (VA) of the transformer. Assuming the voltage to remain constant, the
current is proportional to the VA of the transformer. Since the copper losses are proportional to the square of current, the
value of the copper losses for a given load (and hence for given VA) can be calculated from
2
⎛ VA ⎞
Pc = ⎜ ⎟ Pc (FL)
⎝ VA FL ⎠

17
1.13
(ii) Iron losses or core losses, due to hysteresis and eddy-currents, given by Eqs. 1.7 and 1.8, respectively. That is, Pi = Ph +
Pe. Since the maximum value of the flux Φm in a normal transformer does not vary more than about 2 % between no load
and full load, it is usual to assume the core losses constant at all loads.
The efficiency of a transformer can thus be written as
Po Po V2 I 2 cos φ2
η= = =
Po + Pl Po + Pc + Pi V2 I 2 cos φ2 + I 22 Re 2 + Pi
1.14
Condition for Maximum Efficiency
Assuming that the transformer is operating at a constant terminal voltage and a constant power factor, we are interested to know
for what load (i.e., what value of I2) the efficiency becomes maximum. To determine this, we first divide the numerator and
denominator of Eq. 1.30 by I2, to get
V2 cos φ2
η=
V2 cos φ2 + I 2 Re 2 + Pi / I 2
Obviously, the efficiency will be maximum when the denominator of the above equation is minimum, for which we must have
d Pi
(V2 cos φ2 + I 2 Re 2 + Pi / I 2 ) = 0 or Re 2 − =0
dI 2 I 22
or I 22 Re 2 = Pi or Pc = Pi
1.15
Thus, the efficiency at a given terminal voltage and load power factor is maximum for such a load current I2 which makes the
variable losses (copper losses) equal to the constant losses (iron losses).
All-day Efficiency
The efficiency defined in Eq. 1.26 is called commercial efficiency. This efficiency is not of much use in case of a distribution
transformer. The primary of a distribution transformer remains energized all the time. But the load on the secondary is intermittent
and variable during the day. It means that the core losses occur throughout the day, but the copper losses occur only when the
transformer is loaded. Such transformers, therefore, are designed to have minimum core losses. This gives them better all-day
efficiency, defined below.
Output energy (in kW h) in a cycle of 24 hours
ηall-day =
Total input energy (in kW h)
1.16
Example 1.12 For a single-phase, 150-kVA transformer, the required no-load voltage ratio is 5000-V/250-V. Find (a) the
number of turns in each winding for a maximum core flux of 0.06 Wb, (b) the efficiency at half rated kVA, and unity power
factor, (c) the efficiency at full load, and 0.8 power factor lagging, and (d) the kVA load for maximum efficiency, if the full-load
copper losses are 1800 W and core losses are 1500 W.
Solution: (a) Using the emf equation, we have
E2 250
E2 = 4.44 fN 2 Φ m ⇒ N2 = = = 18.8(say, 19 turns)
4.44 f Φ m 4.44 × 50 × 0.06
E1 5000
and N1 = N2 = × 19 = 380 turns
E2 250
(b) At half rated-kVA, the current is half the full-load current, and hence the output power too reduces by 0.5. Thus,
Output power, Po = 0.5 × (kVA) × (power factor) = 0.5 × 150 × 1 = 75 kW
Since copper losses is proportional to the square of current, we have
Copper losses, Pc = (0.5) 2 × (full-load copper loss) = (0.5) 2 × 1800 W = 0.45 kW
Iron losses being fixed, we have
Iron losses, Pi = 1500 W = 1.5 kW
Po 75
∴ η= × 100 = × 100 = 97.47 %
Po + Pc + Pi 75 + 0.45 + 1.5
(c) At full load and 0.8 power factor,
Output power, Po = (kVA) × (power factor) = 150 × 0.8 = 120 kW
Copper losses, Pc = 1800 W = 1.8 kW

18
Iron losses, Pi = 1500 W = 1.5 kW
Po 120
∴ η= × 100 = × 100 = 97.3 %
Po + Pc + Pi 120 + 1.8 + 1.5
(d) Let x be the fraction of full-load kVA at which the efficiency becomes maximum (that is, when the variable copper losses are
equal to the fixed iron losses). Then
Pc = Pi or x 2 × 1800 = 1500 x = 1500 /1800 = 0.913
Therefore, the load kVA under the condition of maximum efficiency is
Load kVA = (Full-load kVA) × x = 150 × 0.913 = 137 kVA
Example 1.13 For a single-phase, 200-kVA, distribution transformer has full-load copper losses of 3.02 kW and iron losses of
1.6 kW. It has following load distribution over a 24-hour day:
(i) 80 kW at unity power factor, for 6 hours.
(ii) 160 kW at 0.8 power factor (lagging), for 8 hours.
(iii) No load, for the remaining 10 hours.
Determine its all-day efficiency.
Solution: (a) For 80 kW load at unity power factor (for 6 hours):
Output energy = 80 × 6 = 480 kW h
Po 80
kVA = = = 80 kVA
pf 1
Using Eq. 1.29, we have
2 2
⎛ kVA ⎞ ⎛ 80 ⎞
Copper losses, Pc = ⎜ ⎟ Pc (FL) = ⎜ ⎟ × (3.02) = 0.4832 kW
⎝ kVA FL ⎠ ⎝ 200 ⎠
Iron losses, Pi = 1.6 kW
Total losses, Pl = Pc + Pi = 0.4832 kW + 1.6 kW = 2.0832 kW
∴ Total energy losses in 6 hours = 2.0832 × 6 = 12.50 kW h
(b) For 160-kW load at 0.8 power factor (for 8 hours):
Output energy = 160 × 8 = 1280 kW h
Po 160
kVA = = = 200 kVA = kVA FL
pf 0.8
∴ Copper losses, Pc = Pc (FL) = 3.02 kW
Iron losses, Pi = 1.6 kW
Total losses, Pl = Pc + Pi = 3.02 kW + 1.6 kW = 4.62 kW
∴ Total energy losses in 8 hours = 4.62 × 8 = 36.96 kW h
(c) For the no-load period of 10 hours:
Output energy = 0
Copper losses, Pc = 0
Iron losses, Pi = 1.6 kW
Total losses, Pl = Pc + Pi = 0 + 1.6 = 1.6 kW
∴ Total energy losses in 10 hours = 1.6 × 10 = 16 kW h
For 24-hour period:
Total output energy, Wo = 480 + 1280 = 1760 kW h
Total energy losses, Wl = 12.50 + 36.96 +16 = 65.46 kW h
Wo 1760
∴ All-day efficiency, ηall-day = × 100 = × 100 = 96.41%
Wo + Wl 1760 + 65.46

1.11 AUTOTRANSFORMERS
An autotransformer is a special transformer-connection that is useful in power systems, motor starters, variable ac sources, and
other applications. Figure 1.20a shows the special connection with the primary and secondary drawn in the usual position. Figure
1.20b shows the autotransformer (in the step-down mode) drawn in a manner that clarifies its function.

19
(a) Special connection of a transformer (b) Redrawn in the standard way
Fig. 1.20 A two-winding transformer converted into an autotransformer.
Note that the primary and secondary windings are connected in series for the new primary; the secondary is the new
secondary. Also, the primary and secondary are not electrically isolated from each other. Obviously, the voltage V2 = Vo. From
Fig. 1.20b, it is obvious that
N1 N + N2
Vi = V1 + V2 = V2 + V2 = 1 Vo
N2 N2
N2
or Vo = Vi
N1 + N 2
1.17
Hence, the new turns-ratio becomes N 2 : ( N1 + N 2 ) . Thus, we find that an autotransformer works like a potential divider
circuit, except that numbers of turns are to be used instead of resistances.
The apparent power rating (kVA rating) of the transformer is increased by the special connection, as is illustrated in
Example 1.14, given below.

Example 1.14 A single phase, 12-kVA, 120-V/120-V transformer is connected as an autotransformer to make a 240-V/120-V
transformer. What is the apparent power rating of the autotransformer?
Solution: Figure 1.21 shows the transformer connection with rated voltage and current. The current rating on both primary and
secondary windings is
12 kVA
I1 = I 2 = =100 A
120 V
In the autotransformer mode, the input apparent power is 240 × 100 = 24 kVA , and the output apparent power
is 120 × 200 = 24 kVA . Thus, the apparent power capacity of the 12-kVA transformer is doubled by the autotransformer
connection. In effect, half the apparent power is transformed and half is conducted directly to the secondary side.

Fig. 1.21

Practical Autotransformers
In practice, an autotransformer is made by winding a single coil XZ of N1 turns on a magnetic core, as shown in Fig. 1.22a. This
winding is excited by a voltage V1, so that a flux is set up in the core and an emf E1 is induced in the winding. The winding is
tapped at point Y, such that there are N2 turns between Y and Z. An emf E2 exists between the terminals Y and Z such that the
ratio E2/E1 = N2/N1 = K becomes the turns-ratio of the autotransformer. For ideal conditions, the turns-ratio is given as
N 2 V2
K= =
N1 V1

20
If a load is connected across terminals Y and Z, a load-current I2 flows due to the emf E2. The mmf due to current I2 is
counterbalanced by the mmf due to current I1.
The portion YZ of the winding is common to both the primary and secondary sides. Hence, it is called common
winding. The portion XY is called series winding. In variacs (variable autotransformers), point Y is made a sliding contact so as
to give a variable output voltage.

(a) A step-down autotransformer. (b) A step-up autotransformer.


Fig. 1.22 In practice, a single winding is used in making an autotransformer.

(a) Step-down Autotransformer: In Fig. 1.22a, N 2 < N1 , hence V2 < V1. Hence, this arrangement is a step-down
autotransformer. The output current I2 is greater than the input current I1. The distribution of currents in the winding is shown in
the figure.
The apparent power (volt-amperes) on the two sides must be the same V1 I1 = V2 I 2 . We can write the volt-amperes
delivered to the load as
V2 I 2 = V2 I1 + V2 ( I 2 − I1 )
The part V2I1 represents the volt-amperes conductively transferred from ac source to the load through the winding portion XY.
Only the remaining part V2 ( I 2 − I1 ) of the total volt-amperes is inductively transferred from ac source to the load through the
winding portion YZ.
(b) Step-up Autotransformer: In Fig. 1.22b, N 2 > N1 , hence V2 > V1. Hence, this arrangement is a step-up
autotransformer. The output current I2 is less than the input current I1. The volt-amperes drawn from the ac source at the input of
the autotransformer can be written as
V1 I1 = V1 I 2 + V1 ( I1 − I 2 )
The part V1I2 represents the volt-amperes conductively transferred to the load through the winding portion XY. The remaining
part V1 ( I1 − I 2 ) is inductively transferred to the load through the winding portion YZ.

Saving in Copper
For the same voltage ratio and capacity (volt-ampere rating), an autotransformer needs much less copper (or aluminium) material
compared to a two-winding transformer. The cross-sectional area of a conductor is proportional to the current carried by it, and its
length is proportional to the number of turns. Therefore,
Weight of copper in a winding ∝ NI = kNI
For a two-winding transformer:
Weight of copper in primary = kN1 I1
Weight of copper in secondary = kN 2 I 2
Total weight of copper = k ( N1 I1 + N 2 I 2 )
For an autotransformer (see Fig. 1.22a):
The portion XY of the winding has N1 − N 2 turns and carries current I1. The portion YZ of the winding has N 2 turns and
carries current I2 - I1. Therefore,
Weight of copper in portion XY = k ( N1 − N 2 ) I1
Weight of copper in portion YZ = kN 2 ( I 2 − I1 )
Total weight of copper = k ( N1 − N 2 ) I1 + kN 2 ( I 2 − I1 ) = k[( N1 − 2 N 2 ) I1 + N 2 I 2 ]
Therefore, the ratio of copper- weights for the two cases is

21
⎡ ⎛ N 2 ⎞ ⎤ ⎛ I1 ⎞ ⎛ N 2 ⎞
⎢1 − 2 ⎜ ⎟⎥ ⎜ ⎟ + ⎜ ⎟
k[( N1 − 2 N 2 ) I1 + N 2 I 2 ] ⎣ ⎝ N1 ⎠ ⎦ ⎝ I 2 ⎠ ⎝ N1 ⎠ [1 − 2 K ]K + K
= = = 1− K
k ( N1 I1 + N 2 I 2 ) ⎛ I1 ⎞ ⎛ N 2 ⎞ K+K
⎜ ⎟+⎜ ⎟
⎝ I 2 ⎠ ⎝ N1 ⎠
Evidently, the saving is large if K is close to unity. A unity transformation ratio means that no copper is needed at all for the
autotransformer. The winding can be removed all together. The volt-amperes are conductively transformed directly to the load !
Disadvantages: The use of autotransformer has following disadvantages:
1. No electrical isolation between the two sides.
2. Should an open-circuit occur between points Y and Z, full primary high voltage appears across the load.
3. The short-circuit current is larger than that in two-winding transformer.
Applications: The autotransformers find applications in following areas:
1. Boosting or buckling of supply voltage by a small amount.
2. Starting of ac machines, where the voltage is raised in two or more steps.
3. Continuously varying ac supply as in variacs.

1.12 THREE-PHASE TRANSFORMERS


Modern large transformers are usually of the three-phase core type, schematically shown in Fig. 1.23. Three similar limbs are
connected by top and bottom yokes. Each limb has primary and secondary windings arranged concentrically. In Fig. 1.23, the
primary is shown star-connected and the secondary delta-connected. In actual practice, the windings may be connected star/delta,
delta/star, star/star or delta/delta, depending upon the conditions under which the transformer is to be used.

Fig. 1.23 Three-phase core-type star/delta connected transformer.


Example 1.15 A three-phase, 50-Hz transformer has 840 turns on the primary and 72 turns on the secondary winding. The
supply voltage is 3300 V. Determine the secondary line voltage on no load when the windings are connected (a) star/delta, (b)
delta/star.
Solution: (a) For star/delta connection:
VL1 3300
Primary phase voltage, Vph1 = = = 1905.3 V
3 3
72
Secondary phase voltage, Vph2 = 1905.3 × = 163.3 V
840
∴ Secondary line voltage, VL 2 = V ph 2 = 163.3 V
(b) For delta/star connection:
Primary phase voltage, V ph1 = VL1 = 3300 V

72
Secondary phase voltage, Vph2 = 3300 × = 283 V
840
∴ Secondary line voltage,VL 2 = V ph 2 × 3 = 283 × 3 = 490 V
1.13 TRANSFORMER TESTING
There are two simple tests that may be conducted on a transformer to determine its efficiency and regulation. These are called
open-circuit test and short-circuit test. The power required to carry out these tests is very small compared with the full-load output
of the transformer.

22
(1) Open-Circuit Test
This test determines the no-load current and the parameters of the exciting circuit of the transformer. The transformer is connected
as shown in Fig. 1.24. Generally, the low voltage (LV) side is supplied rated voltage and frequency through an autotransformer
(also called a variac). The high voltage (HV) side is left open. The ratio of the voltmeter readings, V2/ V1, gives the
transformation ratio of the transformer. The reading of ammeter A, Io, gives the no-load current I0, and its reading is a check on
the magnetic quality of the ferromagnetic core and joints.
The primary current on no load is usually less than 5 per cent of the full-load current. Hence, the I2R loss on no load is less
than 1/400 of the primary I2R loss of full load and is therefore negligible compared with the core loss. Hence the wattmeter
reading, Wo, can be assumed to give the core loss of the transformer.

Fig. 1.24 Open-circuit test on a transformer.


Various parameters of the transformer can be calculated as under.
Wo V1 V1
Pi = Wo ; I0 = Io ; Iw = ; I m = I 02 − I w2 ; R0 = ; X0 =
V1 Iw Im
(2) Short-Circuit Test
This test determines the equivalent resistance and leakage reactance of the transformer. The connections are made as shown in
Fig. 1.25. Generally, the LV side of the transformer is short-circuited through a suitable ammeter A2. A low voltage is applied to
the primary (HV) side. This voltage is adjusted with the help of a variac so as to circulate full-load currents in the primary and
secondary circuits. The reading of ammeter A1, Isc, gives the full-load current in the primary winding. On the other hand, the core
loss is negligibly small, since the applied voltage (and hence the flux) is less than about one-twentieth of the rated voltage.
Hence, the wattmeter reading, Wsc, gives the copper loss (Pc).

Fig. 1.25 Short-circuit test on a transformer.


Equivalent resistance, reactance and impedance as referred to the primary side can be calculated as under.
Wsc Vsc
Re1 = ; Z e1 = ; X e1 = Z e21 − Re21
I sc2 I sc

Example 1.16 A single-phase, 50-Hz, 12-kVA, 200-V/400-V transformer gives the following test results:
(i) Open-circuit test (with HV winding open) : 200 V, 1.3 A, 120 W
(ii) Short-circuit test (with LV winding short-circuited) : 22 V, 30 A, 200 W
Calculate (a) the magnetizing current and the core-loss current, and (b) the parameters of equivalent circuit as referred to the low
voltage winding.
Solution: (a) The wattmeter reading, 120 W, in the open-circuit test gives the core losses. Therefore, the core-loss current is given
as
Wo 120 W
Iw = = = 0.6 A
V1 200 V
Hence the magnetizing current is given as

I m = I 02 − I w2 = (1.3) 2 − (0.6) 2 = 1.15 A


(b) The parameters of the exciting circuit are given by the open-circuit test, as
V1 200 V V1 200 V
R0 = = = 333 Ω and X0 = = = 174 Ω
I w 0.6 A I m 1.15 A

23
The short-circuit test gives the equivalent resistance and reactance as referred to the primary side (high voltage winding).
From the given specification of the transformer,
V2 200 V 1
The transformation ratio, K = = =
V1 400 V 2
12 kVA
The rated full-load current in the high voltage side, I FL = = 30 A
400 V
This confirms that the short-circuit test has been done at the rated full-load. Thus,
Wsc 200 W Vsc 22 V
Re1 = = = 0.222 Ω and Z e1 = = = 0.733 Ω
I sc2 (30 A) 2 I sc 30 A
∴ X e1 = Z e21 − Re21 = (0.733) 2 − (0.222) 2 = 0.699 Ω
We can now determine the equivalent resistance and reactance as referred to the secondary side (low voltage winding), as
2 2
⎛1⎞ ⎛1⎞
Re 2 = K 2 Re1 = ⎜ ⎟ × 0.222 = 0.055 Ω and X e 2 = K 2 X e1 = ⎜ ⎟ × 0.699 = 0.175 Ω
⎝2⎠ ⎝2⎠

ADDITIONAL SOLVED EXAMPLES

Example 1.17 A 25-kVA transformer has 500 turns on the primary and 40 turns on the secondary winding. The primary
winding is connected to a 3-kV, 50-Hz ac source. Calculate (a) the secondary emf, (b) the primary and secondary currents on full
load, and (c) the maximum flux in the core.
Solution: (a) The transformation ratio is given as
N2 40
K= = = 0.08
N1 500
∴ Secondary emf, E2 = KE1 ≈ KV1 = 0.08 × 3000 = 240 V
(b) The primary and secondary full-load currents are given as
kVA 25 kVA I1 8.33 A
I1 = = = 8.33 A and I2 = = = 104.125 A
V1 3 kV K 0.08
(c) The maximum flux in the core is given by emf equation,
E1 3000
Φm = = = 0.027 Wb
4.44 fN1 4.44 × 50 × 500
Example 1.18 A 230-V, 50-Hz, single-phase transformer has 50 turns on its primary. It is required to operate with a maximum
flux density of 1 T. Calculate the active cross-sectional area of the core. Find suitable dimensions for a square core.
Solution: From the emf equation, we have
E1 230
Φm = = = 0.02072 Wb
4.44 fN1 4.44 × 50 × 50
Φ m 0.02072
∴ Active core area, A = = = 0.02072 m 2 = 207.2 cm 2
Bm 1
Due to the insulation of laminations from each other, the gross area is about 10 % greater than the active area. Thus,
Gross area = 207.2 ×1.1 = 227.92 cm 2
If the core has square cross-section, the side of the square is
a = 227.92 = 15.09 ≈ 15 cm
Example 1.19 A single-phase transformer has a core whose cross-sectional area is 150 cm2, operates at a maximum flux density
of 1.1 Wb/m2 from a 50-Hz supply. If the secondary winding has 66 turns, determine the output in kVA when connected to a load
of 4-Ω impedance. Neglect any voltage drop in the transformer.
Solution: Φ m = Bm A = 1.1× 0.015 = 0.0165 Wb .

24
E2 = 4.44Φm fN 2 = 4.44 × 0.0165 × 50 × 66 = 242 V = V2
(Neglecting the voltage drops)
V2 242
Secondary current, I 2 = = = 60.5 A
ZL 4
V2 I 2 242 × 60.5
∴ output in kVA = = = 14.6 kVA
1000 1000
Example 1.20 A 11-kV/400-V distribution transformer takes a no-load primary current of 1 A at a power factor of 0.24 lagging.
Find (a) the core-loss current, (b) the magnetizing current, and (c) the iron loss.
Solution: (a) The core-loss current, I w = I 0 cos φ0 = 1.0 × 0.24 = 0.24 A

(b) The magnetizing current, I m = I 02 − I w2 = (1)2 − (0.24)2 = 0.971 A


(c) The iron loss, Pi = V1 I 0 cos φ0 = 11000 ×1.0 × 0.24 = 2640 W
Example 1.21 A two-winding, step-down transformer has a turns-ratio (N2/ N1) of 0.5. The primary winding resistance and
reactance are 2.5 Ω and 6 Ω, whereas the secondary winding resistance and reactance are 0.25 Ω and 1 Ω, respectively. Its
magnetizing current and core-loss current are 51.5 mA and 20.6 mA, respectively. While in operation, the output voltage for a
load of 25∠30° Ω is found to be 50 V. Determine the supply voltage, the current drawn from the supply and the power factor.
Solution: Given: Z1 = (2.5 + j 6) Ω = 6.5∠67.38° Ω ; Z 2 = (0.25 + j1) Ω = 1.03∠75.96° Ω
Let us take V2 as the reference phasor, i.e., V2 = 50∠0° V .
V2 50∠0° V
∴ I2 = = = 2∠ − 30° A
Z L 25∠30° Ω
E 2 = V2 + I 2 Z 2 = (50 + j 0) + (2∠ − 30°)(1.03∠75.96°) = 51.45∠1.65° V
⎛N ⎞
E1 = E2 ⎜ 1 ⎟ = (51.45∠1.65°) × 2 = 102.9∠1.65° V; − E1 = 102.9∠181.65° V
⎝ N2 ⎠
⎛N ⎞
I1' = −I 2 ⎜ 2 ⎟ = −(2∠ − 30°) × 0.5 = 1∠150° A
⎝ N1 ⎠
As seen from the phasor diagram of Fig. 1.5b, Im lags –E1 by 90° and Iw is in phase with –E1.
∴ I m = 0.0515∠(181.65° − 90°) = 0.0515∠91.65° A and I w = 0.0206∠181.65° A
∴ I1 = I1' + I m + I w = (1∠150° + 0.0515∠91.65° + 0.0206∠181.65°)
= 1.044556∠148.17° A
V1 = −E1 + I1Z1 = 102.9∠181.65° + (1.044556∠148.17°)(6.5∠67.38°)
= 108.601∠183.647° V
The angle between V1 and I1 = 183.647° − 148.17° = 35.477°
∴ Primary power factor, pf = cos (35.477°) = 0.814 lagging
Example 1.22 A single-phase, step-down transformer has turns-ratio of 4. The resistance and reactance of the primary winding
are 1.4 Ω and 5.5 Ω, respectively, and those of the secondary winding are 0.06 Ω and 0.04 Ω, respectively. If the LV winding is
short-circuited and the HV winding is connected to a 24-V, 50-Hz source, determine (a) the current in the LV winding, (b) the
copper loss in the transformer, and (c) the power factor. Ignore the no-load current I0.
Solution: Given: K = (1/4)=0.25. The equivalent resistance and reactance as referred to HV side are given as
Re1 = R1 + R2 / K 2 = 1.4 + (0.06) /(0.25) 2 = 2.36 Ω
and X e1 = X 1 + X 2 / K 2 = 5.5 + (0.04) /(0.25) 2 = 6.14 Ω

∴ Z e1 = Re21 + X e21 = (2.36) 2 + (6.14) 2 = 6.578 Ω


(a) The current in the HV winding,
VSC 24
I SC = = = 3.648 A
Z e1 6.578

25
Therefore, ignoring I0, we have I1 = I1' = 3.648 A
Thus, the current in the LV winding, I 2 = I1' / K = 3.648 /(0.25) = 14.592 A
(b) The copper loss in transformer = I1 Re1 = (3.648) × 2.36 = 31.4 W
2 2

P 31.4
(c) The power factor = cos φ1 = = = 0.3586
V1 I1 24 × 3.648
Example 1.23 The results of tests conducted on a single-phase, 20-kVA, 2200-V/220-V, 50-Hz transformer are given as under:
OC test (HV winding open) : 220 V, 4.2 A, 148 W.
SC test (LV winding short-circuited) : 86 V, 10.5 A, 360 W.
Determine (a) the regulation and efficiency at 0.8 pf lagging at full load, and (b) the power factor on short-circuit.
Solution: (a) From the short-circuit test, we have VSC = 86 V, ISC = 10.5 A, PSC = 360 W. Then,
VSC 86 V PSC 360 W
Z e1 = = = 8.19 Ω; Re1 = 2
= = 3.265 Ω
I SC 10.5 A I SC (10.5 A) 2
∴ X e1 = Z e21 − Re21 = (8.19) 2 − (3.265)2 = 7.51 Ω
VA 20 000
The full-load primary current, I1 = = = 9.09 A
V1 2200
Since power factor is 0.8, we have cos φ = 0.8, and sin φ = sin[cos −1 0.8] = 0.6
Using Eq. 1.18, we can determine regulation in terms of quantities referred to the primary side,
I1 ( Re1 cos φ + X e1 sin φ )
% Regulation = × 100
V1
9.09(3.265 × 0.8 + 7.51× 0.6)
= × 100 = 2.94 %
2200
The short-circuit test has been conducted for a short-circuit primary current of 10.5 A, whereas full-load primary current is
only 9.09 A. Therefore, the full-load copper loss is given as
2
⎛ 9.09 ⎞
Full-load copper loss = ⎜ ⎟ × 360 = 269.58 W
⎝ 10.5 ⎠
The open-circuit test gives the core loss. Hence, Pi = 148 W.
The full-load output power, Po = VA × pf = 20 000 × 0.8 = 16 000 W
Po 16 000
∴ % Efficiency, η = × 100 = × 100 = 97.45 %
Po + Pc + Pi 16 000 + 269.8 + 148
(b) The power factor on short-circuit is given as
Re1 3.265
pf = cos φSC = = = 0.399 (lagging)
Z e1 8.19
Example 1.24 A single-phase, 200-kVA transformer has an efficiency of 98 % at full-load. If the maximum efficiency occurs at
three-quarter of full-load, calculate the efficiency at half of full-load current, assuming the power factor to be 0.8 lagging.
Solution: At a power factor of 0.8, the full-load output power,
Po = (kVA) × (pf ) = (200 kVA)×0.8 = 160 kW
Po 160 kW
Since the efficiency is only 98 %, the input power, Pin = = = 163.26 kW
η 0.98
Therefore, total losses (copper loss and iron loss) on full load is
Pc + Pi = Pin − Po = 163.26 − 160 = 3.26 kW …(i)
We know that maximum efficiency occurs when the variable loss (copper loss) equals the fixed loss (iron loss). Hence, we must
have
2
⎛3⎞
⎜ ⎟ Pc = Pi …(ii)
⎝4⎠

26
Solving Eqs. (i) and (ii), we get Pc = 2.09 kW and Pi = 1.17 kW
Now, at half load, the total loss is given as
2 2
⎛1⎞ ⎛1⎞
Total losses, Pl = ⎜ ⎟ Pc + Pi = ⎜ ⎟ × 2.09 + 1.17 = 1.69 kW
⎝2⎠ ⎝2⎠
Po (160 / 2)
∴ % Efficiency,η half-load = × 100 = = 97.93 %
Po + Pl (160 / 2) + 1.69
Example 1.25 A single-phase, 150-kVA, 5000-V/250-V, 50-Hz transformer has the full-load copper losses of 1.8 kW and core
losses of 1.5 kW. Find (a) the number of turns in each winding for a maximum core flux of 60 mWb, (b) the efficiency at full
rated kVA, with power factor of 0.8 lagging, (c) the efficiency at half the rated kVA, with unity power factor, and (d) the kVA
load for maximum efficiency.
Solution: (a) Since E = 4.44 fN Φ m , the number of turns on the secondary winding,
E2 250
N2 = = = 19 turns
4.44 f Φ m 4.44 × 50 × 0.06
E1 5000
∴ N1 = N 2 = 19 × = 380 turns
E2 250
(b) At full rated kVA, the current is also full-load. Therefore,
Power output, Po = V2 I 2 × cos φ = (150 kVA) × 0.8 = 120 kW
Copper losses, Pc = 1.8 kW and the iron losses, Pi = 1.5 kW
Po 120
∴ % Efficiency,η = × 100 = × 100 = 97.32 %
Po + Pc + Pi 120 + 1.8 + 1.5
(c) At half the rated kVA, the current is half the full-load current. Thus,
Power output, Po = 0.5 × V2 I 2 × cos φ = 0.5 × (150 kVA) × 1 = 75 kW

Copper losses, Pc = (0.5 I 2 ) Re 2 = 0.25 I 2 Re 2 = 0.25 × (1.8 kW)=0.45 kW


2 2

Iron losses, Pi = 1.5 kW (Iron losses do not change with load)


Po 75
∴ % Efficiency,η = × 100 = × 100 = 97.47 %
Po + Pc + Pi 75 + 0.45 + 1.5
(d) Let x be the fraction of the full-load kVA at which maximum efficiency occurs. Then, according to the condition for
maximum efficiency, we should have
x 2 (copper losses at full load) = Pi
1.5
or x 2 ×1.8 = 1.5 ⇒ x=
= 0.913
1.8
Hence, the required kVA load for maximum efficiency = (150 kVA) × 0.913 = 137 kVA
Example 1.26 A single-phase, 50-kVA, 2400-V/240-V, 50-Hz transformer is used to step down the voltage of a distribution
system. The low tension voltage is required to be kept constant at 240 V.
(a) What load impedance connected to the LV side will be loading the transformer fully at 0.8 power factor lagging?
(b) What is the value of this impedance referred to the high voltage side?
(c) What is the value of the current referred to the high voltage side?
Solution: (a) Since the secondary voltage is required to be constant, the secondary current remains the same whatever be the value
of the power factor. The full-load secondary current I2 can be calculated from the kVA ratings,
50 kVA
I2 = = 208.33 A
240 V
V2 240 V
Therefore, the load impedance, Z L = = = 1.152 Ω
I 2 208.33 A
V2 240
(b) Transformation ratio, K = = = 0.1
V1 2400

27
The load impedance referred to the primary side,
Z eq = Z L / K 2 = 1.152 /(0.1)2 = 115.2 Ω
(c) The current referred to the high voltage side,
I1' = KI 2 = 0.1× 208.33 = 20.833 A
Example 1.27 A single-phase, 10-kVA, 2300-V/230-V, 50-Hz transformer is connected as an autotransformer with LT winding
in series with the HT winding as shown in Fig. 1.26. The autotransformer is excited from a 2530-V source, and it is fully loaded
such that the rated currents of the windings are not exceeded. Determine (a) the current distribution in the windings, (b) the kVA
output, (c) the volt-amperes transferred conductively and inductively from the input to the output, and (d) the saving in copper as
compared to the two-winding transformer for the same output.

Fig. 1.26 A two-winding transformer connected as autotransformer.


Solution:
10 000
Current rating of HT winding = = 4.35 A
2300
10 000
Current rating of LT winding = = 43.48 A
230
(a) The autotransformer can supply a load current I2 = 43.48 + 4.35 = 47.83 A at 2300 V, as shown in Fig. 1.26. The input
current I1 = 43.48 A. The current distribution is shown in the figure.
2300 × 47.83
(b) The kVA output = = 110kVA
1000
(c) The volt-amperes transferred conductively = V2 I1 = 2300 × 43.48 VA = 100 kVA
The volt-amperes transferred inductively = V2 ( I 2 − I1 ) = 2300 × 4.35 VA = 10 kVA
N 2 V2 2300
(d) Saving in copper = K = = = = 0.909 or 90.9 %
N1 V1 2530

28
SUMMARY
1. A transformer operates on the principle of mutual induction between two coils called primary and secondary.
2. The emf equation of a transformer: E = 4.44Φ m fN
3. A transformer is said to be ideal, if it has (i) infinite permeability, (ii) no core losses, (iii) no copper loss, and (iv) no leakage
flux.
V2 E2 N 2 I 2 N1 1
Transformation ratio, K = = = = = and Z eq1 = Z L 2 / K
2
4. ;
V1 E1 N1 I1 N 2 K
5. The exciting current or no-load current, I0 has two components: (i) magnetizing current, Im, and (ii) core-loss current, Iw.
6. The exciting circuit in the equivalent circuit of a transformer has a resistance R0 (which accounts for the core losses) and
reactance X0 (which accounts for the required magnetic flux in the core) in parallel.
7. The hysteresis loss varies directly with the frequency, and is given as
Ph = K h Bmn f V
8. The eddy-current loss varies directly with the square of the frequency, and is given as
Pe = K e Bm2 f 2t 2V
9. In core-type transformer, the windings surround a considerable part of the core, whereas in the shell-type transformer, the
core surrounds a considerable part of the windings.
10. On loading a transformer, the flux remains the same but the load current I2 gets reflected in the primary side as primary
balancing current or load component of primary current, I1’.
11. To account for the copper loss, resistances R1 and R2 are included in the primary and secondary of the equivalent circuit.
12. To account for the leakage flux, reactances X1 and X2 are included in the primary and secondary of the equivalent circuit.
13. Total resistance and leakage reactance referred to primary are
Re1 = R1 + ( R2 / K 2 ) and X e1 = X 1 + ( X 2 / K 2 )
14. Total resistance and leakage reactance referred to secondary are
Re 2 = K 2 R1 + R2 and X e 2 = K 2 X 1 + X 2
V2(0) − V2
15. Per unit regulation down =
V2(0)
V2(0) − V2
16. Per unit regulation up =
V2
17. Approximate voltage drop = I 2 Re 2 cos φ ± I 2 X e 2 sin φ (+ for lagging, −for leading pf).
Re 2
18. Condition for zero regulation: tanφ = (occurs only for leading pf).
X e2
X e2
19. Condition for maximum regulation: tan φ =
Re 2
Power output Po
20. Efficiency,η = =
Power output +Power loss Po + Pl
21. Condition for maximum efficiency: Variable losses = Fixed losses, or Pc = Pi
22. An autotransformer has only one winding, a part of which is common to primary and secondary.
23. The OC test determines the following:
W V V
Pi = Wo ; I 0 = I o ; I w = o ; I m = I 02 − I w2 ; R0 = 1 ; X 0 = 1
V1 Iw Im
24. The SC test determines the following:
W V
Re1 = 2sc ; Z e1 = sc ; X e1 = Z e21 − Re21
I sc I sc

29
CHECK YOUR UNDERSTANDING

Before you proceed to the next Chapter, take this Test. Give yourself two marks for each correct answer and minus one for each
wrong answer. If your score is 12 or more, go to the next Chapter; otherwise study this Chapter again.

No. Statement True False Marks


1. When the primary winding of a transformer is connected to an ac
source, an emf is induced only in the secondary winding.
2. In a step-up transformer, the current is stepped down.
3. In a core-type transformer, the two windings are put on two separate
limbs.
4. The magnetizing current Im leads the mutual flux Φm by 90°.
5. The no-load current in a transformer is about 2 – 5 % of the full-load
current.
6. If the power factor of the load on a transformer is lagging, it is
possible to make voltage regulation zero.
7. A transformer is designed such that it has maximum efficiency at
full load. If it has total losses P at full load, the total losses at half
load will be 0.625P.
8. The primary and secondary leakage fluxes are simulated by primary
and secondary leakage reactances in the equivalent circuit of a
transformer.
9. There is electrical isolation between primary and secondary
windings of an autotransformer.
10. In a short-circuit test of a transformer, usually the high voltage side
is shorted.
Your Score

Answers

1. False 2. True 3. True 4. False 5. True


6. False 7. True 8. True 9. False 10. False

REVIEW QUESTIONS
1. Explain the principle of working of a transformer. What is meant by step-up and step-down transformers? Which quantity
is being stepped up or stepped down?
2. Deduce the emf equation of a transformer.
3. State the conditions to be satisfied by a transformer to be ‘ideal’. How is the concept of an ‘ideal transformer’ useful in
understanding the behaviour of an actual transformer?
4. How do you justify that the voltage per turn is constant for a given transformer.
5. What do you understand by ‘transformation ratio’? A resistive load is connected across the secondary. What will be its
equivalent resistance as referred to the primary?
6. What is meant by ‘resistance referred to the primary’ and ‘resistance referred to the secondary’?
7. Draw the no-load phasor diagram of a transformer. Express the magnetizing current and the core-loss current in terms of the
no-load current and the power factor.
8. Explain why the hysteresis loss and the eddy-current loss occur in the core of a transformer.
9. Explain how the hysteresis loss and the eddy-current loss are related to the frequency of the ac supply. Also state how these
losses can be reduced.
10. Give the constructional differences between a core-type and a shell-type transformer.
11. Assuming that the windings of a practical transformer have no resistance and there is no leakage of flux, draw complete
phasor diagram for (a) a resistive load, (b) an inductive load, and (c) a capacitive load.
12. Explain what you understand by leakage flux and leakage reactance in reference to a transformer.
13. Explain how the effects of winding resistance and flux leakage represented in the equivalent circuit of a transformer.
14. Draw the complete equivalent circuit of a practical transformer. Show how this equivalent circuit can further be simplified
without introducing much error.

30
15. Draw the equivalent circuit of a transformer with (a) the primary quantities referred to the secondary side, and (b) the
secondary quantities referred to the primary side.
16. Define voltage regulation of a transformer. Explain what is meant by (a) inherent regulation, (b) percentage regulation up,
and (c) percentage regulation down.
17. Assuming that the impedance voltage drop is quite small compared to the full-load voltage, derive an expression for the
voltage drop from the no-load condition to the full-load condition in a transformer for both the lagging and leading power
factors.
18. Show that one can achieve zero voltage-regulation in a transformer only if the load has a leading power factor. Also, derive
the condition for zero regulation.
19. Derive the condition for maximum regulation of a transformer.
20. Explain what you understand by the efficiency of a transformer. Deduce the condition for the maximum efficiency.
21. Explain how the all-day efficiency differs from the commercial efficiency in case of a transformer. For what application of a
transformer, the all-day efficiency assumes more importance?
22. Explain how a two-winding transformer can be converted into an autotransformer.
23. Deduce an expression for the amount of copper saved in an autotransformer.
24. State the advantages, disadvantages and applications of an autotransformer.
25. Explain how you can determine the efficiency and regulation of a transformer by conducting ‘open-circuit test’ and ‘short-
circuit test’ on it.

MULTIPLE CHOICE QUESTIONS


Here are some incomplete statements. Four alternatives are provided below each. Tick the alternative that completes the
statement correct:
1. The core of a transformer is assembled with laminated sheets so as to
(a) reduce hysteresis loss
(b) reduce eddy-current loss
(c) reduce both the hysteresis and eddy-current losses
(d) ensure good magnetic coupling between primary and secondary windings
2. A close magnetic coupling between primary and secondary windings results in
(a) high commercial efficiency (b) high all-day efficiency
(c) good voltage regulation (d) none of the above
3. The number of turns in the primary winding of a transformer depends on
(a) the input voltage (b) the input current
(c) both the input voltage and current (d) the kVA
4. Cooling of transformers is required so as to
(a) increase the efficiency
(b) reduce the losses
(c) reduces the humming
(d) dissipate the heat generated in the windings
5. The emf induced in the windings of a transformer
(a) is in phase with the core flux
(b) is out of phase with the core flux
(c) lags the core flux by π/2
(d) leads the core flux by π/2
6. A single-phase, 150-kVA, 1100-V/400-V transformer has 100 turns on the secondary winding. The number of turns on its
primary winding will be
(a) 5500 (b) 2200 (c) 550 (d) 275
7. A single-phase, 5-kVA, 200-V/100-V transformer delivers 50 A at the rated voltage. The input current will be
(a) 25 A (b) more than 25 A (c) less than 25 A (d) 100 A
8. In a single-phase, 10-kVA, 230-V/1000-V transformer, the no-load current will be about
(a) 0.5 A (b) 2 A (c) 10 A (d) 15 A
9. A single-phase transformer has a turns-ratio of 4:1. If the secondary winding has a resistance of 1 ohm, this resistance as
referred to the primary will be
(a) 16 Ω (b) 4 Ω (c) 0.25 Ω (d) 0.0625 Ω
10. A transformer is supplying a unity power-factor load. The power factor at the primary terminals will be

31
(a) about 0.95 lagging (b) about 0.95 leading
(c) about 0.8 lagging (d) unity
11. When the secondary winding of a transformer is shorts-circuited, the power factor of the input is
(a) unity (b) about 0.8 leading (c) about 0.8 lagging (d) about 0.2 lagging
12. Under no-load condition, the power factor of a transformer is
(a) unity (b) zero (c) about 0.4 lagging (d) about 0.4 leading
13. If the full-load copper loss of a transformer is 100 W, its copper loss at half load will be
(a) 200 W (b) 100 W (c) 50 W (d) 25 W
14. If the full-load core loss of a transformer is 100 W, its core loss at half load will be
(a) 200 W (b) 100 W (c) 50 W (d) 25 W
15. A single-phase transformer is supplying power to a load at a terminal voltage of 11 kV. When the load is disconnected, the
terminal voltage becomes 11.5 kV. The voltage regulation of this transformer for this load is
(a) 55 % (b) 11.55 % (c) 5 % (d) 2.5 %
16. A transformer operates at maximum efficiency, when
(a) its hysteresis loss and eddy-current loss are minimum
(b) the sum of its hysteresis loss and eddy-current loss is equal to its copper loss
(c) the power factor of the load is leading
(d) its hysteresis loss is equal to its eddy-current loss
17. A distribution transformer should be selected on the basis of its
(a) all-day efficiency (b) regulation
(c) commercial efficiency (d) all the above
18. In a transformer, the iron losses do not vary with load current because
(a) the core area is constant
(b) the core flux remains constant
(c) the iron losses are equal to the copper losses
(d) the iron losses are very small
19. It is economical to use an autotransformer when the turns-ratio is
(a) low (b) high (c) more than 10 (d) none of the above
20. An auto-transformer steps down voltage V1 to V2. If an open-circuit develops in the common winding, the voltage across the
load may become
(a) V1 (b) V1 – V2 (c) V1 +V2 (d) V2
Answers
1. b 2. c 3. a 4. d 5. c 6. d 7. b 8. b 9. a 10. a
11. d 12. c 13. d 14. b 15. c 16. b 17. a 18. a 19. a 20. a

PROBLEMS
(A) Simple Problems
1. A single-phase, 250-kVA, 11-kV/415-V, 50-Hz transformer has 80 turns on the secondary. Calculate (a) the approximate
values of the primary and secondary currents, (b) the approximate number of primary turns, and (c) the maximum value of
the flux. [Ans.: (a) 22.7 A, 602 A; (b) 2121; (c) 23.4 mWb]
2. The primary winding of a 50-Hz transformer has 480 turns and is fed from a 6400-V supply. Determine (a) the peak value of
the flux in the core, and (b) the secondary voltage if the secondary winding has 20 turns. [Ans.: (a) 0.06 Wb; (b) 266.4 V]
3. A single phase, 50-Hz transformer has 80 turns on the primary winding and 400 turns on the secondary winding. The net
cross-sectional area of the core is 200 cm2. If the primary winding is connected to 240-V, 50-Hz supply, determine (a) the
emf induced in the secondary winding, and (b) the maximum flux density in the core. [Ans.: (a) 1200 V; (b) 0.675 T]
4. A 10-kVA, single-phase transformer has its primary connected to a 2000-V supply. It has 60 turns on the secondary winding
and the voltage across it is found to be 240 V. Assuming the transformer to be ideal, calculate (a) the number of turns on its
primary winding; (b) the full-load primary and secondary currents. [Ans.: (a) 500; (b) 5 A, 41.67 A]
5. A 200-kVA, 3300-V/240-V, 50-Hz, single-phase transformer has 80 turns on the secondary winding. Assuming an ideal
transformer, calculate (a) primary and secondary currents on full load, (b) the maximum value of flux, and (c) the number of
primary turns. [Ans.: (a) 60.6 A, 833.33 A; (b) 0.0135 Wb; (c) 1100]
6. A single-phase transformer with 10:1 turns-ratio and rated at 50 kVA, 2400-V/240-V, 50 Hz is used to step down the voltage
of a distribution system. The low tension (LT) voltage is to be kept constant at 240 V. Find the value of the load impedance
of the LT side so that the transformer is loaded fully. Find also the value of the maximum flux inside the core if the LT side
has 23 turns. [Ans.: 1.152 Ω, 0.047 Wb]

32
7. The primary of a single-phase transformer takes 1 A at power factor of 0.4 when connected to a 240-V, 50-Hz supply and the
secondary is on open circuit. The number of turns on the primary is twice that on the secondary. A load taking 50 A at a
lagging power factor of 0.8 is now connected across the secondary. What is now the value of the primary current?
(Neglect the voltage drops in the transformer.) [Ans.: 25.9 A]
8. A single-phase, 50-Hz transformer has 100 turns on the primary winding and 400 turns on the secondary winding. The net
cross-sectional area of the core is 250 cm2. If the primary winding is connected to a 50-Hz, 230-V supply, calculate (a) the
emf induced in the secondary winding, and (b) the maximum value of the flux density in the core.
[Ans.: (a) 920 V; (b) 0.414 T]
9. The no-load current of a single-phase transformer is 5.0 A at 0.3 power factor when supplied from a 240-V, 50-Hz source.
The number of turns on the primary is 200. Calculate (a) the maximum value of the flux in the core, (b) the core losses, and
(c) the magnetizing current. [Ans.: (a) 5.4 mWb; (b) 360 W; (c) 4.77 A]
10. A transformer on no-load takes 1.5 A at a power factor of 0.2 lagging when its primary is connected to a 50-Hz, 230-V
supply. Its transformation ratio (N2/N1) is 1/3. Determine the primary current when the secondary is supplying a current of
300 A at a power factor of 0.8 lagging. Neglect the voltage drops in the windings. [Ans.: 14.5 A]
11. The no-load current of a 50-Hz, 230-V transformer is 4.5 A at a power factor of 0.25 lagging. The number of turns on the
primary winding is 250. Calculate (a) the magnetizing current, (b) the core loss, and (c) the maximum value of the flux in
the core. [Ans.: (a) 4.35 A; (b) 258.75 W; (c) 4.14 mWb]
12. A 100-kVA transformer has 400 turns on the primary and 80 turns on the secondary winding. The primary and secondary
resistances are 0.3 Ω and 0.1 Ω, respectively. The primary and secondary leakage reactances are 1.1 Ω and 0.035 Ω,
respectively. Calculate the equivalent impedance referred to the primary side. [Ans.: 3.426 Ω]
13. A single-phase, 100-kVA, 1100-V/220-V transformer has following parameters: R1 = 0.1 Ω, X1 = 0.3 Ω, R2 = 0.004 Ω and
X2 = 0.012 Ω. Determine (a) the equivalent resistance and leakage reactance as referred to the high voltage winding, and (b)
the equivalent resistance and leakage reactance as referred to the low voltage winding.
[Ans.: (a) 0.2 Ω, 0.6 Ω; (b) 0.008 Ω, 0.024 Ω]
14. In a 50-kVA, 11-kV/400-V, single-phase transformer, the iron and copper losses are 500 W and 600 W, respectively under
rated conditions. Calculate (a) the efficiency at unity power factor at full load, (b) the load for maximum efficiency, and (c)
the iron and copper losses for this load. [Ans.: (a) 97.85 %; (b) 45.64 kVA; (c) 500 W, 500 W]
15. A 10-kVA, 200-V/400-V, 50-Hz, single-phase transformer gave the following test-results:
OC test (HV winding open) : 200 V, 1.3 A, 120 W.
SC test (LV winding shorted) : 22 V, 30 A, 200 W.
Calculate (a) the magnetizing current, and (b) the equivalent resistance and leakage reactance as referred to the low voltage
side. [Ans.: (a) 1.15 A; (b) 0.0555 Ω, 0.1745 Ω]
(B) Tricky Problems
16. A single-phase, 50-Hz transformer has 30 turns on primary and 350 turns on secondary. The net cross-sectional area of the
core is 250 cm2. If the primary winding is connected to a 230-V, 50-Hz supply, calculate (a) peak value of the flux density in
the core, (b) the voltage induced in the secondary winding, and (c) the primary current when the secondary current is 100 A.
Neglect the losses. [Ans.: (a) 1.3814 T; (b) 2683.33 V; (c) 1166.67 A]
17. A single-phase, 50-Hz, 100-kVA, 2400-V/240-V transformer has no-load current of 0.64 A and core loss of 700 W, when its
high voltage (HV) side is energized at rated voltage and frequency. Calculate the two components of the no-load current. If
this transformer supplies a load current of 40 A at 0.8 lagging pf on its low voltage side, determine the primary current and
its power factor. Ignore the resistance and leakage reactance drops. [Ans.: 0.5697 A, 0.2917 A; 4.584 A, 0.762 lagging]
18. An ideal 50-Hz, core-type transformer has 100 primary-turns and 200 secondary-turns. The primary rated voltage is 220 V.
If the maximum permissible flux density is 1.2 T, what should be the cross-sectional area? Also, find (a) the secondary
voltage, and (b) the primary current in complex form with reference to the secondary voltage vector when secondary delivers
−3
a current of 8 A at a lagging power factor of 0.8. [Ans.: 8.26 × 10 m ;(a) 440 V; (b) 16∠143.1° A ]
2

19. A 500-kVA, 11000-V/400-V, 50-Hz, single-phase transformer has 100 turns on the secondary winding. Calculate (a) the
approximate number of turns on the primary winding, (b) the approximate values of the primary and secondary current, and
(c) the maximum value of the flux in the core. [Ans.: (a) 2750; (b) 45.45 A, 1250 A; (c) 0.018 Wb]
20. A single-phase transformer has a turns-ratio of 144/432 and operates at a maximum flux of 7.5 mWb at 50 Hz. When
working on no load, it takes 0.24 kVA at a power factor of 0.26 lagging from the supply. If it supplies a load of 1.2 kVA at a
power factor of 0.8 lagging, determine (a) the magnetizing current, (b) the primary current, and (c) the primary power
factor. [Ans.: (a) 0.97 A; (b) 5.8 A; (c) 0.731 (lagging)]
21. The primary and secondary windings of a 30-kVA, 6000-V/230-V, single-phase transformer have resistances of 10 Ω and
0.0016 Ω, respectively. The total reactance of the transformer as referred to the primary side is 23 Ω. Calculate the
percentage regulation of the transformer when supplying full-load current at a power factor of 0.8 lagging. [Ans.: 2.54 %]
22. A transformer with turns-ratio 8: 1, has the resistances of the primary and secondary windings as 0.85 Ω and 0.012 Ω,
respectively, and the leakage reactances as 4.8 Ω and 0.07 Ω, respectively. Determine the voltage to be applied to the

33
primary to obtain a current of 150 A in the secondary when the secondary terminals are short-circuited. Ignore the
magnetizing current. [Ans.: 176.6 V]
23. A single-phase, 50-Hz transformer has a turns ratio 6:1. The primary and secondary winding resistances are 0.90 Ω and
0.03 Ω, respectively, and leakage reactances are 5 Ω and 0.13 Ω, respectively. Find (a) the voltage to be applied to the high
voltage side to get a current of 100 A in the low voltage winding on short-circuit, and (b) the primary power factor on short-
circuit. Neglect the no-load current. [Ans.: (a) 164.67 V; (b) 0.2 lagging]
(C) Challenging Prolems
24. A single-phase transformer has Z1 = (1.4 + j 5.2) Ω and Z 2 = (0.0117 + j 0.0465) Ω . The input voltage is 6600 V
and turns-ratio is 10.6:1. The secondary feeds a load which draws 300 A at 0.8 power factor lagging. Neglecting no-load
current I0, determine the secondary voltage and the output in kW. [Ans.: 600 V, 144 kW]
25. A single-phase, 10-kVA, 4000-V/400-V transformer has following parameters: R1 = 13 Ω, X1 = 20 Ω, R2 = 0.15 Ω, X2 = 0.25
Ω, R0 = 12000 Ω and X0 = 6000 Ω. Determine (a) the equivalent resistance and leakage reactance as referred to the primary
winding, (b) the input current with secondary terminals open-circuited, and (c) the input current when the secondary
supplies a load current of 25 A at a power of 0.8 lagging.
[Ans.: (a) 28 Ω, 45 Ω; (b) 0.745∠ − 63.5° A ; (c) 3.18∠ − 42.9° A ]
26. A single-phase, 200-V/2000-V transformer is fed from a 200-V supply. The equivalent winding resistance and leakage
reactance as referred to the low voltage side are 0.16 Ω and 0.7 Ω, respectively. The resistance representing core losses is
400 Ω and the magnetizing reactance is 231 Ω. A load impedance of ZL = (596 + j444) Ω is connected across the secondary
terminals. Calculate (a) the input current, (b) the secondary terminal voltage, and (c) the primary power factor.
[Ans.: (a) 25.96∠ − 40.78° A ; (b) 1859 V; (c) 0.757 lagging]
27. A single-phase, 100-kVA, 2000-V/200-V, 50-Hz transformer has impedance drop of 10 % and resistance drop of 5 %. (a)
What is the regulation at full-load 0.8 power factor lagging? (b) At what power factor is the regulation zero?
[Ans.: (a) 9.196 %; (b) 0.866 leading]
28. The primary and secondary windings of a 500-kVA, 11-kV/415-V, single-phase transformer have resistances of 0.42 Ω and
0.0019 Ω, respectively. Its core losses are 2.9 kW. Assuming the power factor to be 0.8, calculate its efficiency on (a) full
load, and (b) half load. [Ans.: (a) 98.39 %; (b) 98.13 %]
29. For the transformer in Prob. 28, assuming the power factor to be 0.8, find the output at which the efficiency is maximum and
calculate its value. [Ans.: 447 kVA, 98.4 %]
30. A single phase transformer has percentage regulations of 4 and 4.4 for lagging power factors of 0.8 and 0.6, respectively.
The full-load copper loss is equal to the iron loss. Calculate (a) the lagging power factor at which the full-load regulation is
maximum, and (b) the full-load efficiency at unity power factor. [Ans.: (a) 0.4472 lagging; (b) 96.15 %]
31. A 250-kVA, single-phase transformer has an efficiency of 96 % on full load at 0.8 power factor lagging and on half load 0.8
power factor lagging. Find iron loss and full-load copper loss. [Ans.: 2.78 kW, 5.56 kW]

EXPERIMENTAL EXERCISE 1.1


Load Test on a Single-Phase Transformer
OBJECTIVES:
1. To determine the polarity of the primary and secondary windings.
2. To find the turns-ratio.
3. To determine the efficiency at different loads.
4. To determine the voltage regulation.
APPARATUS: Single-phase ac power supply 230 V; One single-phase transformer 2 kVA, 220-V/110-V; One Variac 0-270 V, 15
A; One wattmeter 10 A, 230 V; Two voltmeters (MI type) 0-230 V; Two ammeters (MI type) 0-10 A and 0-20 A; One resistive
load.
CIRCUIT DIAGRAMS: The circuit diagrams are shown in Fig. 1.27.

(a) Circuit for polarity test and voltage ratio test.

34
(b) Circuit for determining the efficiency and the voltage regulation.
Fig. 1.27 Circuit diagrams for load test on a single-phase transformer.
BRIEF THEORY: (i) When a transformer is used, a terminal of primary (as well as of secondary) winding is alternately positive
and negative with respect to other terminal. It becomes important to know the relative polarities of the primary and secondary
terminals in situations such as follows:
(1) When two single-phase transformers are connected in parallel so as to share the total load on the system.
(2) When three single-phase transformers are connected to make a three-phase transformer.
The relative polarities can be determined by shorting one of the terminals of the primary and secondary windings (say, P2 and
S2 in Fig. 1.27a), and then measuring the voltage across the other terminals of the primary and secondary windings (say, P1 and S1
in Fig. 1.27a).
If P1 and S1 have same polarity (say, positive at an instant), the situation is as shown in Fig. 1.28a. Obviously, in such a case,
the voltage V3 = V1 – V2. On the other hand, if P1 and S1 have opposite polarity (say, positive and negative, respectively, at an
instant), the situation becomes as shown in Fig. 1.28b. Obviously, in such a case, the voltage V3 = V1 + V2.

(a) P1 and S1 of same polarity. (b) P1 and S1 of opposite polarity.


Fig. 1.28 Reading of V3 depends on the relative polarity of primary and secondary.
(ii) The induced emf in winding is proportional to the number of turns on it. Therefore, the turns-ratio is given as
N 2 E2 V2
K= =
N1 E1 V1
(iii) If Pin is the input power and Po is the output power of a transformer, its efficiency is given as
Po Po
η= =
Pin Po + Pl
where, Pl represents copper loss and iron loss in the transformer. The copper loss in the windings is proportional to the square of
the current. However, the iron loss (hysteresis and eddy-current loss in the core) remains constant. As the load is increased, the
efficiency also increases. It becomes maximum when the variable copper-loss becomes same as the fixed iron-loss. On further
increasing the load, the efficiency starts decreasing (Fig. 1.29a).

(a) Efficiency versus load-current. (b) Output voltage versus load-current.


Fig. 1.29 Both efficiency and output voltage of the transformer vary with load current.
(iv) If V2(0) is the output no-load voltage and V2(FL) is the output full-load voltage, the percentage voltage-regulation is defined
as

35
V2(0) − V2(FL)
% Regulation = ×100 %
V2(FL)
PROCEDURE:
(i) For Polarity and Turns-Ratio Test:
1. Make connections as given in Fig. 1.27a.
2. Switch on the ac supply and adjust the variac so that voltmeter V1 reads about 100 V.
3. Record the reading of voltmeter V3.
4. If V3 < V1, the terminals P1 and S1 have same polarity.
5. If V3 > V1, the terminals P1 and S1 have opposite polarity.
6. Using variac, vary the input voltage.
7. For various values of the input voltage V1, note down the readings of the output voltage V2.
8. Calculate the ratio V2/V1.
9. Switch off the supply.
(ii) For Efficiency and Regulation Test:
1. Make connections as given in Fig. 1.27b.
2. Disconnect the load.
3. Switch on the ac supply and adjust the variac so that voltmeter V1 reads 220 V, the rated value for the given transformer.
4. Note down the reading of the ammeter A1. This gives no-load primary current.
5. Note down the reading of the voltmeter V2. This gives no-load output voltage V(0).
6. Note down the reading of the wattmeter W. This gives the iron loss Pi in the transformer.
7. Switch on the load in parts till the full-load current [ I 2(FL) = (2 kVA/110 V)=18.18 A ] is reached. For each load,
note down the readings of wattmeter W, voltmeter V2 and ammeter A2.
8. Calculate the efficiency for each value of load and then plot the curve of efficiency versus the load current.
9. Calculate the percentage regulation.
10. Switch off the supply.
OBSERVATIONS AND CALCULATIONS:
(i) Polarity Test:
Reading of the voltmeter V1 =
Reading of the voltmeter V2 =
Reading of the voltmeter V3 =
If V3 < V1, the terminals P1 and S1 have same polarity, and if V3 > V1, the terminals P1 and S1 have opposite polarity.
(ii) Table for Turns-Ratio Test:
Sr. No. V1 V2 Turns-Ratio = V 2/V 1
1
2
3
(iii) Table for Efficiency Test:
Sr. No. W V1 V2 Output = V 2 I 2 Efficiency = V 2I 1/W

Plot the graph of efficiency versus load current (as in Fig. 1.29a).
(iv) Regulation Test:
Secondary voltage on no-load, V2(0) =
Secondary voltage on full-load, V2(FL) =
V2(0) − V2( FL )
∴ % Regulation = ×100 % =
V2( FL )
RESULTS:

36
1. The polarities of the primary and secondary windings have been marked.
2. The turns-ratio has been found almost same as the specified value.
3. The efficiency versus load-current curve has been plotted. The maximum efficiency occurs at a load current of _____
amperes.
PRECAUTIONS:
1. Before switching on the supply, the zero reading of the wattmeter, voltmeters and ammeters should be checked.
2. The meters of proper range should be selected.
3. While determining efficiency, use an ammeter in series with the wattmeter so as to keep a check that the current through
the wattmeter does not exceed its rating.
VIVA-VOCE:
1. Can you name some applications of transformers?
Ans.: To step-up generated voltage before transmission and to step-down at consumer end. Current and voltage
transformers are used in instrumentation. Small size transformers are used in electronics and communication circuits,
e.g., radio, TV, etc.
2. Can you name the material normally used for making the core of transformers?
Ans.: Cold rolled grain oriented (CRGO) silicon steel.
3. What is the purpose of adding silicon to steel?
Ans.: By adding silicon (about 4 %) to the steel, its resistance increase. Hence the eddy-current loss is reduced.
4. Any harm if you add more than 4 % silicon?
Ans.: By adding more silicon, the resistance increases further. But, the steel become very brittle.
5. Why do we use laminations for making the core of a transformer?
Ans.: To decrease the eddy-current loss.
6. Usually, how much is the thickness of laminations used.
Ans.: It is about 0.35 mm.
7. A transformer is rated as 0.5 kVA, 220 V/110 V, 50 Hz. Which winding, primary or secondary, is made of thinner wire?
Ans.: Since the primary carries less current, it is made of thinner wire.
8. If I apply 220 V at 40 Hz, how much voltage will you get across the secondary?
Ans.: Since voltage transformation ratio does not depend on the frequency of the supply, we will still get 110 V across
the secondary.
9. If I apply 220 V at 0 Hz, will I still get 110 V across the secondary?
Ans.: No, we will get 0 V, as the flux is not varying at all for zero frequency (it is a dc source). Moreover, the
transformer will get burnt due to flow of large current, as there will be no back emf induced in the primary.
10. There are 780 turns on the primary and 240 turns on the secondary winding of a transformer. The primary winding is
connected to an ac supply and secondary is connected to a load such that the primary current is 10 A. Now, suppose 100
turns of the secondary winding suddenly get short-circuited. Will the primary current increase or decrease?
Ans.: The short-circuited 100 turns of the secondary will draw a large current. Hence the primary current will increase
by large amount.

EXPERIMENTAL EXERCISE 1.2


Open-Circuit and Short-Circuit Tests on a Transformer

OBJECTIVES:
1. To determine the parameters Iw, Im, R0 and X0 of the transformer by conducting open-circuit test on it.
2. To determine the equivalent resistance Req, the equivalent reactance Xeq and the equivalent impedance Zeq of the
transformer by conducting short-circuit test on it.
APPARATUS: Single-phase ac power supply 220 V; One single-phase transformer 2 kVA, 220-V/110-V; One variac 0-270 V, 20
A; One wattmeter 20 A, 250 V; One voltmeter (MI type) 0-50 V; One voltmeter (MI type) 0-250 V; One ammeter (MI type) 0-2
A; One ammeter (MI type) 0-20 A.
CIRCUIT DIAGRAMS: The circuit diagrams are shown in Fig. 1.30.

(a) Circuit for open-circuit test.

37
(b) Circuit for short-circuit test.
Fig. 1.30 Circuit diagrams for testing a transformer.
BRIEF THEORY: (i) In open-circuit test, one winding is left open and the other winding is applied rated voltage (see Fig. 1.30a).
The readings of the wattmeter (Wo), the voltmeter (Vo) and the ammeter (Io) are noted. Since the no-load current is very small, the
copper loss can be ignored. Therefore, the wattmeter indicates the iron losses, i.e.,
Iron losses, Pi = Wo
The parameters Iw, Im, R0 and X0 can be calculated as follows:
Wo Vo Vo
Iw = ; I m = I 02 − I w2 ; R0 = ; and X0 =
Vo Iw Im
(ii) In short-circuit test, one winding (preferably, low voltage) is short-circuited and the other winding is applied a low voltage
such that full-load current flows in the windings. The readings of the wattmeter (Wsc), the voltmeter (Vsc) and the ammeter (Isc) are
noted. Since the applied voltage is low, the iron losses are negligibly small compared to the copper loss. Therefore, the wattmeter
indicates the full-load copper loss, i.e.,
Copper loss, Pc = Wsc
The equivalent resistance Req, the equivalent reactance Xeq and the equivalent impedance Zeq, as referred to the winding on which
the measurements are made, can be calculated as follows:
Wsc Vsc
Re1 = ; Z e1 = ; and X e1 = Z e21 − Re21
I sc2 I sc
PROCEDURE:
(i) For Open-Circuit Test:
1. Connect the circuit as shown in Fig. 1.30a.
2. Put the variac at low voltage output.
3. Switch on the ac supply.
4. Adjust the variac to the rated voltage of the transformer.
5. Record the ammeter, wattmeter and voltmeter readings.
6. Switch off the supply.
(ii) For Short-Circuit Test:
1. Connect the circuit as shown in Fig. 1.30b.
2. Put the variac at lowest voltage output.
3. Slowly increase the applied voltage till the ammeter reading equals the rated value.
4. Record the ammeter, wattmeter and voltmeter readings.
5. Switch off the supply.
OBSERVATIONS:
(i) Open-Circuit Test:
Reading of the ammeter, Io =
Reading of the wattmeter, Wo =
Reading of the voltmeter, Vo =
(ii) Short-Circuit Test:
Reading of the ammeter, Isc =
Reading of the wattmeter, Wsc =
Reading of the voltmeter, Vsc =
CALCULATIONS:
Wo
( ) −( )
2 2
(i) Iw = = = A; I m = I 02 − I w2 = = A;
Vo

38
Vo Vo
R0 = = = Ω; X0 = = = Ω.
Iw Im
Wsc Vsc
(ii) Re1 = = = Ω; Z e1 = = = Ω;
I sc2 I sc

( ) −( )
2 2
X e1 = Z e21 − Re21 = = Ω.
RESULTS:
1. The values of the equivalent circuit parameters are as follows:
R0 = Ω; X0 = Ω; Re1 = Ω; X e1 = Ω; Z e1 = Ω.
2. The magnetizing current, Im = A; The iron-loss current, Iw = A.
3. The no-load current, I0 = A.
PRECAUTIONS:
1. Before switching on the supply, the zero reading of the wattmeter, voltmeters and ammeter should be checked.
2. The meters of proper range should be selected.
3. While conducting the short-circuit test, the voltage applied should be initially set at zero. It should then be increased
slowly. If, by mistake, higher voltage than needed is applied there is every possibility that the transformer may be
damaged.
VIVA-VOCE:
1. What is the importance of open-circuit and short-circuit tests on a transformer?
Ans.: The purpose of these tests is to find the iron loss, copper loss and hence the efficiency of the transformer at
different loads. This is an indirect method of testing, since the transformer is not actually loaded during the testing.
Especially, for large size transformers this type of indirect testing is important as the arrangement for actual loading just
cannot be done.
2. How do you justify that the reading of the wattmeter in open-circuit test indicates the iron losses?
Ans.: The iron losses (hysteresis and eddy-current) depend upon the magnetization level of the core, which in turn
depends upon the voltage applied. In open-circuit test, full rated voltage is applied to the transformer. Hence, the iron
losses occur at full rated value. Furthermore, as the secondary winding is open, the secondary current is zero. Hence
there is no copper loss in the secondary winding. However, in the primary winding, there is small no-load current
flowing. The copper loss in the primary due to this small current is negligible. Hence, the wattmeter reading gives the
iron losses.
3. What does the reading of wattmeter indicate in the short-circuit test? Justify your answer.
Ans.: It indicates the full-load copper losses of the transformer. In short-circuit test, full rated current is made to flow
through both the secondary and primary windings, by applying very low voltage to the primary. Because of low voltage
the magnetization level of the core is very low. Hence the iron losses are negligibly small. Thus, the wattmeter indicates
only the full-rated copper losses of the transformer.
4. Suppose that the full rated voltage is applied in the short-circuit test. What do you expect to happen?
Ans.: Since the secondary winding is shorted, a heavy current will be drawn by the transformer. As a result, the circuit
breaker should trip off. If it does not, the transformer will get excessively overheated resulting in burning of the
insulation.
5. How do the iron losses vary with the load on the transformer?
Ans.: Iron losses do not depend on the load. These losses depend only on the magnetization level of the core, which in
turn depend upon the voltage applied.
6. How do the copper losses vary with the load on the transformer?
Ans.: Copper losses or I2R losses depend on the square of the currents flowing in the windings.
7. What is the phasor relationship between Iw, Im and I0?

Ans.: The current I0 is the phasor sum of Iw and Im. I 0 = I w2 + I m2 . The iron-loss component Iw is in phase with the
applied voltage. The magnetizing component Im lags behind the applied voltage by 90°.
8. What is the magnitude of no-load current as compared to the full-load current?
Ans.: The no-load current is about 3 – 5 % of the full-load current.
9. Of what order is the power factor of a transformer under no-load condition?
Ans.: It is about 0.2 lagging.

**********

39
TRANSFOMER
TRANSFOMER is a device that:
a. Transfer electrical energy from one electric circuit to another
b. Does so without a change in frequency
c. Does so by the principle of electromagnetic induction
d. Has electric circuit that are linked by a common magnetic circuit
From the formula:
P=VI
Transformer transfer electrical energy from one circuit (Primary side) to another (Secondary side) without a change in power and extremely efficient because the only losses are
those that occur in the copper windings (𝐼 2 𝑅 𝑙𝑜𝑠𝑠𝑒𝑠) 𝑎𝑛𝑑 𝑖𝑛 𝑡ℎ𝑒 𝑖𝑟𝑜𝑛 (ℎ𝑦𝑠𝑡𝑒𝑟𝑒𝑠𝑖𝑠 𝑎𝑛𝑑 𝑒𝑑𝑑𝑦 𝑐𝑢𝑟𝑟𝑒𝑛𝑡; there are also no losses resulting from rotation, such as are present in rotating
machines. If the input (Primary Side) to a transformer is assumed to equal the output (Output Side) of a transformer and voltage drops are negligible, then
Thus, 𝑃𝑝 = 𝑃𝑠
𝐸𝑝 𝑥𝐼𝑝 = 𝐸𝑠 𝑥𝐼𝑠
𝐸𝑝 𝐼 𝑁𝑝
𝑎= 𝐸𝑠
= 𝐼𝑠 = 𝑁𝑠
𝑝

Where:
Ep-primary voltage, volts
Es-secondary voltage, volts
Is-secondary current, ampere
Ip-primary current, ampere Ratio of Transformation indicates how much the primary voltage is lowered or raised
Np-primary winding
Step down transformer- When the primary impressed voltage Ep is reduced to a lower secondary voltage Es thus a>1
Ns-secondary winding
Step up transformer- When the primary impressed voltage Ep is increased to a higher secondary voltage Es thus a<1
a-ratio of transformation
Recall:
∅ ∅𝑥𝑍 ∅𝑥𝑁 ∅𝑥𝑁
𝐸𝑎𝑣 = = = = 1 V
𝑡 𝑥 108 𝑡 𝑥 108 𝑡 𝑥 108 𝑥 108
4𝑓

4f∅ 𝑁
Eav= 108 V

E=1.11 Eav=1.11 x (4f∅𝑁x10−8 ) = 4.44f∅𝑁𝑥10−8 𝑉


Thus,
Ep= 4.44f∅𝑁𝑝 𝑥10−8 𝑉
Es= 4.44f∅𝑁𝑠 𝑥10−8 𝑉
EXAMPLE PROBLEMS
1. The 2,300 volt primary winding of a 60 cycle transformer has 4,800 turns. Calculate (a) the mutual flux (b) number of turns in the 230 volt secondary winding.
Given:
f= 60 cps
Vp= 2,300 volt
Np=4,800 turns

Solution:
𝐸𝑝 𝑥 108 2300 𝑥108
a. 𝟇m= 4.44 𝑥 𝑓 𝑥 𝑁 = 4.44 𝑥 60 𝑥 4800 = 1.8 𝑥 105 𝑚𝑎𝑥𝑤𝑒𝑙𝑙𝑠
𝑝

𝐸𝑠 𝑥 108 230 𝑥 108


b. 𝑁𝑠 = 4.44 𝑥 𝑓 𝑥 𝑁 = 4.44 𝑥 60 𝑥 1.8 𝑥 105 = 480 𝑡𝑢𝑟𝑛𝑠
𝑝

2. The maximum flux in the core of a 60 cycle transformer that has 1,320 primary turns and 46 secondary turns is 3.76 x 10^6 maxwells. Calculate the primary and secondary induced
voltages.
Given:
f=60 cps
Np=1,320 turns
Ns=46 turns
𝟇m=3.76 x 10^6 maxwells

Solution:
Ep= 4.44f∅𝑁𝑝 𝑥10−8 𝑉 = 4.44 𝑥 60 𝑥 3.76 𝑥 106 𝑥 1320 𝑥 10−8 = 13,200 𝑉
Es= 4.44f∅𝑁𝑠 𝑥10−8 𝑉 = 4.44 𝑥 60 𝑥 3.76 𝑥 106 𝑥 46 𝑥 10−8 = 460 𝑉
EXAMPLE PROBLEMS
3. The secondary winding of a 4600/230 V transformer has 36 turns. How many turns are there in the primary winding?
Given:
Ep= 4,600 V
Es=230 V
Ns=36 turns
Solution:
𝐸𝑝 𝑁𝑝
=
𝐸𝑠 𝑁𝑠
𝐸𝑝 4600
𝑁𝑝 = 𝑥𝑁𝑠 = 𝑥36 = 720 𝑡𝑢𝑟𝑛𝑠
𝐸𝑠 230

4. The volts per turn in a 25 cycle 2400/230 volt transformer is 8. Calculate (a) the primary and secondary turns (b) maximum flux in the core.
Given:
E/N=8 volt/turn
Ep=2,400 volts
Es=230 volts

Solution:
𝑡𝑢𝑟𝑛
a. 𝑁𝑝 = 2400 𝑉 𝑥 = 300 𝑡𝑢𝑟𝑛𝑠
8𝑉
𝑡𝑢𝑟𝑛
𝑁𝑠 = 230 𝑉 𝑥 = 29 𝑡𝑢𝑟𝑛𝑠
8𝑉
𝐸𝑝 𝑥 108 2400 𝑥108
b. 𝟇m= 4.44 𝑥 𝑓 𝑥 𝑁 = 4.44 𝑥 60 𝑥 300 = 7.21 𝑥 106 𝑚𝑎𝑥𝑤𝑒𝑙𝑙𝑠
𝑝
EXAMPLE PROBLEMS
5. The secondary load current of a 2300/115 V transformer is 46 A. Calculate the primary current.
Given:
Ep= 2300 volts
Es= 115 volts
Is= 46 A
Solution:
𝐸 115
𝐼𝑝 = 𝐸𝑠 𝑥𝐼𝑠 = 2300 𝑥46 = 2.3 𝐴
𝑝

6. The primary and secondary of a transformer were measured and found to be 3.8 A and 152 A, respectively. If
the secondary load voltage is 116 volts, what is the primary emf?
Given:
Ip= 3.8 A
Is= 152 A
Es= 116 V
Solution:
𝐼 152
𝐸𝑝 = 𝐼𝑠 𝑥𝐸𝑠 = 𝑥116 = 4640 𝑣𝑜𝑙𝑡𝑠
𝑝 3.8
Since the voltage drops are all directly proportional to the load current, Is in the secondary and Ip in the
primary, it should be clear that at no load there will be no voltage drop in either winding. Therefore, if a
transformer delivers rated load at secondary terminal voltage, that voltage will change if the load is removed.
Thus,

𝐸𝑁𝐿 −𝐸𝐹𝐿
Percent Regulation= 𝑥 100
𝐸𝐹𝐿

Where:
Enl=No Load Voltage, volts
Efl= Full Load Voltage, volts
EXAMPLE PROBLEMS
7. Calculate the percent regulation of a 2300/115 volt transformer whose no load voltage was measured and found to be 118 V
Given:
Ep=2300 volt
𝐸𝑠𝐹𝐿 = 115 volts
𝐸𝑠𝑁𝐿 = 118 volts

Solution:
118−115
Percent regulation= 𝑥100 = 2.61 %
115
8. The percent regulation of a 4800/240 volt distribution transformer is 3.33 percent. Calculate the voltage to which the secondary voltage will
rise when full load is removed.
Given:
Percent regulation=3.33%
Ep=4800 volts
𝐸𝑠𝐹𝐿 = 240 volts

Solution:
𝐸𝑠𝑁𝐿 −240
Percent regulation= 3.33 = 𝑥100
240
Thus,
𝐸𝑠𝑁𝐿 = 248 𝑣𝑜𝑙𝑡𝑠
Equivalent Resistance, Reactance and Impedance

When regulation calculations are made for transformers, it is convenient to combine the resistance and reactance
drops that actually occur on the primary and secondary sides into a single value of IR and a single value of IX. One
method of simplifying the calculations is to make use of idea that a transformer having a ratio of transformation a can be
converted into an equivalent transformer having a ratio of 1:1
𝐼
Converting primary value to secondary value through the transformation ratio (𝐼𝑝 = 𝑎𝑠 ),
𝐼𝑠 𝑅𝑠 𝐼 𝑅𝑝 𝑅𝑝
𝐼𝑠 𝑅𝑠 + 𝑎
= 𝐼𝑠 𝑅𝑠 + ( 𝑎𝑠 𝑥 𝑎
)=𝐼𝑠 𝑅𝑠 + 𝑎2
𝐼𝑠 𝑋𝑠 𝐼 𝑋𝑝 𝑋𝑝
𝐼𝑠 𝑋𝑠 + 𝑎
= 𝐼𝑠 𝑋𝑠 + ( 𝑎𝑠 𝑥 𝑎
)=𝐼𝑠 𝑋𝑠 + 𝑎2
𝑅𝑝 𝑋𝑝
Thus,𝑅𝑒 = 𝑅𝑠 + 𝑎𝑛𝑑 𝑋𝑒 = 𝑋𝑠 + 𝑖𝑛 𝑠𝑒𝑐𝑜𝑛𝑑𝑎𝑟𝑦 𝑡𝑒𝑟𝑚𝑠
𝑎2 𝑎2
𝑅𝑒 = 𝑎2 𝑅𝑠 + 𝑅𝑝 𝑎𝑛𝑑 𝑋𝑒 = 𝑋𝑠 𝑎2 + 𝑋𝑝 𝑖𝑛 𝑝𝑟𝑖𝑚𝑎𝑟𝑦 𝑡𝑒𝑟𝑚𝑠
𝑍𝑒 = 𝑅𝑒2 + 𝑋𝑒2
Where,
𝑅𝑝 = 𝑝𝑟𝑖𝑚𝑎𝑟𝑦 𝑠𝑖𝑑𝑒 𝑟𝑒𝑠𝑖𝑠𝑡𝑎𝑛𝑐𝑒, 𝑜ℎ𝑚
𝑅𝑠 = 𝑠𝑒𝑐𝑜𝑛𝑑𝑎𝑟𝑦 𝑠𝑖𝑑𝑒 𝑟𝑒𝑠𝑖𝑠𝑡𝑎𝑛𝑐𝑒, 𝑜ℎ𝑚 𝑅𝑒 = 𝑒𝑞𝑢𝑖𝑣𝑎𝑙𝑒𝑛𝑡 𝑟𝑒𝑠𝑖𝑠𝑡𝑎𝑛𝑐𝑒, 𝑜ℎ𝑚
𝑋𝑝 = 𝑝𝑟𝑖𝑚𝑎𝑟𝑦 𝑠𝑖𝑑𝑒 𝑟𝑒𝑎𝑐𝑡𝑎𝑛𝑐𝑒, 𝑜ℎ𝑚
𝑋𝑠 = 𝑠𝑒𝑐𝑜𝑛𝑑𝑎𝑟𝑦 𝑠𝑖𝑑𝑒 𝑟𝑒𝑎𝑐𝑡𝑎𝑛𝑐𝑒, 𝑜ℎ𝑚 𝑅𝑒 = 𝑒𝑞𝑢𝑖𝑣𝑎𝑙𝑒𝑛𝑡 𝑟𝑒𝑎𝑐𝑡𝑎𝑛𝑐𝑒, 𝑜ℎ𝑚
𝑍𝑒 = 𝑒𝑞𝑢𝑖𝑣𝑎𝑙𝑒𝑛𝑡 𝑖𝑚𝑝𝑒𝑑𝑎𝑛𝑐𝑒, 𝑜ℎ𝑚
EXAMPLE PROBLEMS
9. A 25 kva 2300/230 volt distribution transformer has the following resistance and leakage reactance values; Rp=0.8 ohm; Xp=3.2 ohm ohm;Rs= 0.009 ohm; Xs=0.03 ohm. Calculate the
equivalent values of resistance ,reactance and impedance. (a) in secondary terms (b) in primary terms.
Given:
Ep=2300 volt
Es= 230 volts
Rp=0.8 ohm
Xp=3.2 ohm
2300
𝑎= 230
= 10

Solution:
0.8 3.2
a. 𝑅𝑒 = 0.009 + 102 = 0.017 𝑜ℎ𝑚 𝑋𝑒 = 0.03 + 102 = 0.062 𝑜ℎ𝑚 𝑍𝑒 = 0.0172 + 0.0622 = 0.0642 𝑜ℎ𝑚

or 𝑍𝑒 = 𝑅𝑒 + 𝑗𝑋𝑒 = 𝑍𝑒 < 𝜃 = 0.017 + 𝑗0.062 = 0.0642 < 74.667 𝑜ℎ𝑚


a. b. 𝑅𝑒 = (0.009𝑥102 ) + 0.8 = 1.7 𝑜ℎ𝑚 𝑋𝑒 = (0.03𝑥102 ) + 3.2 = 6.2 𝑜ℎ𝑚 𝑍𝑒 = 1.72 + 6.22 = 6.42 𝑜ℎ𝑚
or 𝑍𝑒 = 𝑅𝑒 + 𝑗𝑋𝑒 = 𝑍𝑒 < 𝜃 = 1.7 + 𝑗6.2 = 6.42 < 74.667 𝑜ℎ𝑚

10. For the transformer of last example, calculate the equivalent resistance and reactance voltage drops for a secondary load current of 109 A in (a) secondary terms (b) primary
terms.
Given:
Is=109 A
109
Ip= 10 = 10.9 𝐴

Solution:
a. Vr=IsRe=109 x 0.017=1.85 V Vx=IsXe=109 x 0.062=6.75 V
b. Vr=IsRe=10.9 x 1.7=18.5 V Vx=IsXe=10.9 x 6.2=67.5 V
Equivalent Circuit of a Transformer

Note that the transformer, as an electric circuit, merely acts like an impedance voltage drop. Thus, it is also
possible to represent a transformer as an ordinary series electric circuit that has three elements, equivalent
resistance (𝑅𝑒 ), equivalent leakage reactance (𝑋𝑒 ) and the load.
The Short Circuit Test
In order to determine experimentally the value of the equivalent resistance, impedance and reactance and it
is an attempt to make the windings carry rated currents without requiring that the transformer deliver a load thus
the power input to the transformer will be extremely low. It is also use to determine the copper loss of the
transformer.
𝑃𝑠𝑐
𝑅𝑒 = 2
𝐼𝑠𝑐
𝐸𝑠𝑐
𝑍𝑒 =
𝐼𝑠𝑐

𝑋𝑒 = 𝑍𝑒2 − 𝑅𝑒2
Where,
𝑅𝑒 = 𝑒𝑞𝑢𝑖𝑣𝑎𝑙𝑒𝑛𝑡 𝑟𝑒𝑠𝑖𝑠𝑡𝑎𝑛𝑐𝑒, 𝑜ℎ𝑚 𝑍𝑒 = 𝑒𝑞𝑢𝑖𝑣𝑎𝑙𝑒𝑛𝑡 𝑖𝑚𝑝𝑒𝑑𝑎𝑛𝑐𝑒, 𝑜ℎ𝑚
𝑋𝑒 = 𝑒𝑞𝑢𝑖𝑣𝑎𝑙𝑒𝑛𝑡 𝑟𝑒𝑎𝑐𝑡𝑎𝑛𝑐𝑒, 𝑜ℎ𝑚
𝑃𝑠𝑐 = 𝑠ℎ𝑜𝑟𝑡 𝑐𝑖𝑟𝑐𝑢𝑖𝑡 𝑝𝑜𝑤𝑒𝑟 𝑐𝑜𝑝𝑝𝑒𝑟 𝑙𝑜𝑠𝑠 , 𝑤𝑎𝑡𝑡𝑠
𝐼𝑠𝑐 = 𝑠ℎ𝑜𝑟𝑡 𝑐𝑖𝑟𝑐𝑢𝑖𝑡 𝑐𝑢𝑟𝑟𝑒𝑛𝑡, 𝑎𝑚𝑝𝑒𝑟𝑒
𝐸𝑠𝑐 = 𝑠ℎ𝑜𝑟𝑡 𝑐𝑖𝑟𝑐𝑢𝑖𝑡 𝑣𝑜𝑙𝑡𝑎𝑔𝑒, 𝑣𝑜𝑙𝑡𝑠
The Open Circuit Test
When one side of a transformer is left open circuited and the other side is connected to a source of
alternating current whose voltage is rated value, the current will be extremely low- about 2 to 10 percent of the
rated load current. Two components of power loss are developed in the iron and depends on the magnetic
properties of the materials used to construct the core of transformer and its design.
a. Hysteresis Loss- which is purely magnetic, and results because the tiny magnetic particles produce a kind of
molecular friction as they tend to change alignment with the rapid reversals of alternating current
1.6
𝑃ℎ = 𝑘ℎ 𝑓𝐵𝑚
b. Eddy current loss- which is electromagnetic in character and is caused by the flow of currents in the iron in
exactly the same way as in transformer windings.
Recall:
𝑃𝑒 = 𝑘𝑒 𝑓 2 𝐵𝑚
2
V= 4.44f∅𝑁𝑥10−8 ; ∅= 𝐵𝑚 x A
𝐸 𝑥 108 108 𝐸 𝐸
∅ = 𝐵𝑚 𝑥 𝐴 = ; 𝐵𝑚 = =𝑘
4.44𝑓𝑁 4.44𝑁𝐴 𝑓 𝑓

thus,

1.6 𝐸 1.6 𝐸 1.6 𝐸 1.6


𝑃ℎ = 𝑘ℎ 𝑓𝐵𝑚 = 𝑘ℎ 𝑓 𝑘 = 𝑘ℎ 𝑥𝑘 1.6 = 𝑘1
𝑓 𝑓0.6 𝑓0.6

𝐸 2
𝑃𝑒 = 𝑘𝑒 𝑓 2 𝐵𝑚
2
= 𝑘𝑒 𝑓 2 𝑘 = 𝑘𝑒 𝑥𝑘 2 𝐸 2 = 𝑘2 (𝐸 2 )
𝑓

𝐸 1.6
𝑃𝑐 = 𝑃ℎ + 𝑃𝑒 = 𝑘1 + 𝑘2 (𝐸 2 )
𝑓0.6

Where,

𝑃ℎ = ℎ𝑦𝑠𝑡𝑒𝑟𝑒𝑠𝑖𝑠 𝑙𝑜𝑠𝑠, 𝑤𝑎𝑡𝑡𝑠 𝑃𝑒 = 𝑒𝑑𝑑𝑦 𝑐𝑢𝑟𝑟𝑒𝑛𝑡 𝑙𝑜𝑠𝑠, 𝑤𝑎𝑡𝑡𝑠 𝑃𝑐 = 𝑐𝑜𝑟𝑒 𝑙𝑜𝑠𝑠, 𝑤𝑎𝑡𝑡𝑠

𝐸 = 𝑎𝑝𝑝𝑙𝑖𝑒𝑑 𝑒𝑚𝑓, 𝑣𝑜𝑙𝑡𝑠

𝑓 = 𝑎𝑝𝑝𝑙𝑖𝑒𝑑 𝑓𝑟𝑒𝑞𝑢𝑒𝑛𝑐𝑦, 𝑐𝑝𝑠 𝑜𝑟 ℎ𝑒𝑟𝑡𝑧

𝑘1 = ℎ𝑦𝑠𝑡𝑒𝑟𝑒𝑠𝑖𝑠 𝑝𝑟𝑜𝑝𝑜𝑟𝑡𝑖𝑜𝑛𝑎𝑙𝑖𝑡𝑦 𝑐𝑜𝑛𝑠𝑡𝑎𝑛𝑡


EXAMPLE PROBLEMS
11. A 4400 volt 60 cycle transformer has core loss of 840 watts of which one third is eddy current loss.
Determine the core loss when the transformer is connected (a) to a 4600 volt 60 cycle source (b) 4400 volt 50
cycle source (c) to a 4600 volt 50 cycle source.
Given:
840
𝑃𝑐 = 840 𝑤𝑎𝑡𝑡𝑠; 𝑃𝑒 = 𝑤𝑎𝑡𝑡𝑠 = 280 𝑤𝑎𝑡𝑡𝑠; 𝑃ℎ = 840 − 280 = 560 𝑤𝑎𝑡𝑡𝑠
3
𝑃ℎ 𝑓0.6 560 600.6 𝑃 280
𝑘1 = = = 9.673𝑥10−3 𝑘2 = 𝐸𝑒2 = 44002 = 1.446 𝑥10−5
𝐸 1.6 44001.6

Solution:
𝐸 1.6 46001.6
a. 𝑃𝑐 = 𝑃ℎ + 𝑃𝑒 = 𝑘1 𝑓0.6 + 𝑘2 𝐸2 = 9.673𝑥10−3 + 1.446 𝑥10−5 46002 = 907.262 watts
600.6

44001.6
b. 𝑃𝑐 = 𝑃ℎ + 𝑃𝑒 = 9.673𝑥10−3 + 1.446 𝑥10−5 44002 = 904.692 watts
500.6
46001.6
c. 𝑃𝑐 = 𝑃ℎ + 𝑃𝑒 = 9.673𝑥10−3 500.6
+ 1.446 𝑥10−5 46002 = 976.772 watts
EXAMPLE PROBLEMS
12. The following data were obtained when a short circuit test was performed upon a 100 kva 2400/240 volt
distribution transformer: 𝐸𝑠𝑐 = 72 𝑣𝑜𝑙𝑡𝑠; 𝐼𝑠𝑐 = 41.6 𝑎𝑚𝑝; 𝑃𝑠𝑐 = 1,180 𝑤𝑎𝑡𝑡𝑠. All instrument were placed on the high side,
and the low side was short circuited. Calculate (a) the equivalent resistance, impedance and reactance (b) voltage
drop IR and IX at primary and secondary side.
Given:
𝑃 = 100 𝑘𝑉𝐴 = 100,000 𝑉𝐴
𝐸𝑝 = 2400 𝑣𝑜𝑙𝑡 ; 𝐸𝑠 = 240 𝑣𝑜𝑙𝑡
𝐸𝑠𝑐 = 72 𝑣𝑜𝑙𝑡𝑠; 𝐼𝑠𝑐 = 41.6 𝑎𝑚𝑝; 𝑃𝑠𝑐 = 1,180 𝑤𝑎𝑡𝑡𝑠

Solution:
𝑃𝑠𝑐 1180 𝐸𝑠𝑐 72
a. 𝑅𝑒 = 2 = = 0.682 𝑜ℎ𝑚 ; 𝑍𝑒 = = = 1.707 𝑜ℎ𝑚; 𝑋𝑒 = 𝑍𝑒2 − 𝑅𝑒2 = 1.7072 − 0.6822 = 1.565 𝑜ℎ𝑚
𝐼𝑠𝑐 41.62 𝐼𝑠𝑐 41.6
𝑃 100,000 𝑃 100,000
b. 𝐼𝑝 = = = 41.667 𝐴; 𝐼𝑠 = = = 416.667 𝐴
𝐸𝑝 2400 𝐸𝑠 240

IpRe=(41.667)(0.682)=28.417 V
IpXe=(41.667)(1.565)=65.209 V
IsRe=(416.667)(0.682)=284.167 V
IsXe=(416.667)(1.565)=652.084 V
EXAMPLE PROBLEMS
13. For the transformer of the previous example , calculate the copper losses when the load is (a) 125 kva (b) 75
kva (c) 85 kw at a power factor of 0.772.
Given:
𝑃𝑐𝑢 𝑖𝑠 𝑑𝑖𝑟𝑒𝑐𝑡𝑙𝑦 𝑝𝑟𝑜𝑝𝑜𝑟𝑡𝑖𝑜𝑛𝑎𝑙 𝑡𝑜 𝑡ℎ𝑒 𝑠𝑞𝑢𝑎𝑟𝑒 𝑜𝑓 𝑖𝑡𝑠 𝑘𝑉𝐴 𝑟𝑎𝑡𝑖𝑛𝑔
Thus, 𝑃𝑐𝑢 = 𝑘 𝑆 2
𝑃𝑠𝑐 = 𝑃𝑐𝑢 = 1180 𝑤𝑎𝑡𝑡𝑠
Solution:
𝑃𝑐𝑢2 𝑆2 2 𝑆2 2 125 2
a. = ; 𝑃𝑐𝑢2 = 𝑃𝑐𝑢1 𝑆 = 1180 = 1843.75 𝑤𝑎𝑡𝑡𝑠
𝑃𝑐𝑢1 𝑆1 1 100

𝑆2 2 75 2
b. 𝑃𝑐𝑢2 = 𝑃𝑐𝑢1 = 1180 = 663.75 𝑤𝑎𝑡𝑡𝑠
𝑆1 100

𝑆2 2 85/.772 2
c. 𝑃𝑐𝑢2 = 𝑃𝑐𝑢1 𝑆 = 1180 = 1430.491 𝑤𝑎𝑡𝑡𝑠
1 100
Equivalent Calculations Using Short Circuit and Open Circuit Data

When the secondary of a transformer delivers power to a load, an equivalent amount of power is supplied to
the primary by the AC source; the power output is generally delivered at a voltage that is different from that of a
source. As previous discussions have shown, there are only two kinds of losses in a static transformer:
a. Copper losses in the primary and secondary windings which measured by wattmeter when the short circuit
test is performed.
b. Hysteresis and eddy current losses in the laminated core which where Core loss is made up and is measured
when open circuit test is performed.
the efficiency of a transformer is given by,
𝑃𝑜𝑢𝑡
𝑒𝑓𝑓𝑖𝑐𝑖𝑒𝑛𝑐𝑦 % = 𝑥100
𝑃𝑜𝑢𝑡 +𝑃𝑙𝑜𝑠𝑠

𝑤ℎ𝑒𝑟𝑒, 𝑃𝑙𝑜𝑠𝑠 = 𝑃𝑐𝑢 + 𝑃𝑐


It can ,in fact, be shown that, because of these relationships, the efficiency of a transformer is a maximum when the copper losses are equal to the iron losses.
𝑃𝑐
𝐼𝑠2 𝑅𝑒 = 𝑃𝑐 𝑡ℎ𝑢𝑠, 𝐼𝑠 =
𝑅𝑒

𝐸𝑠 𝐼𝑠 𝐸𝐼
𝑠 𝐹𝐿 𝑃𝑐 𝐸𝑠 𝐼𝐹𝐿 𝑃𝑐
1000
= 1000𝑥𝐼 𝑅𝑒
= 1000 2 𝑥𝑅
𝐼𝐹𝐿
𝐹𝐿 𝑒

𝐸𝑠 𝐼𝑠 𝐸𝑠 𝐼𝑓𝑙
where, = 𝑘𝑣𝑎 𝑙𝑜𝑎𝑑 𝑓𝑜𝑟 𝑚𝑎𝑥𝑖𝑚𝑢𝑚 𝑒𝑓𝑓𝑖𝑐𝑖𝑒𝑛𝑐𝑦; = 𝑘𝑣𝑎 𝑟𝑎𝑡𝑖𝑛𝑔 𝑜𝑓 𝑡𝑟𝑎𝑛𝑠𝑓𝑜𝑟𝑚𝑒𝑟
1000 1000

𝑃𝑐
𝑘𝑣𝑎max 𝑒𝑓𝑓 = 𝑘𝑣𝑎𝑟𝑎𝑡𝑒𝑑
𝑃𝑠
EXAMPLE PROBLEMS
14. A 5 kva 2300/230 volt 60 cycle standard distribution transformer was tested, with the following results; short
circuit input= 112 watts, open circuit input= 40 watts. Calculate the efficiencies of the transformer for a power factor
of 0.8 for the following fractions of rated kilovolt ampere. (a) ¼ (b) ½ (c) 1 ¼
Given:
𝑆𝑜𝑢𝑡 = 5 𝑘𝑣𝑎 𝐸𝑝 = 2300 𝑉 𝐸𝑠 = 230 𝑉 𝑓 = 60 𝑐𝑝𝑠
𝑃𝑠𝑐 = 𝑃𝑐𝑢 = 112 𝑤𝑎𝑡𝑡𝑠
𝑃𝑜𝑐 = 𝑃𝑐𝑜 = 40 𝑤𝑎𝑡𝑡𝑠
Solution:
𝑃𝑐𝑢2 𝑆2 2 2; 𝑆2 2 112 2
a. = = 𝑟𝑎𝑡𝑖𝑜 𝑃𝑐𝑢2 = 𝑃𝑐𝑢1 = 0.25 = 0.007 𝑘𝑤 𝑃𝑐𝑜1 = 𝑃𝑐𝑜2 = 0.04 𝑘𝑤
𝑃𝑐𝑢1 𝑆1 𝑆1 1000
𝑃𝑜𝑢𝑡 1
𝑃𝑜𝑢𝑡 = 5 𝑘𝑣𝑎 𝑥0.8𝑥0.25 = 1 𝑘𝑤 𝑒𝑓𝑓𝑖𝑐𝑖𝑒𝑛𝑐𝑦 % = 𝑃 x 100 = 1+(0.007+0.04) 𝑥100 = 95.51%
𝑜𝑢𝑡 +𝑃𝑙𝑜𝑠𝑠

𝑆2 2 112
b. 𝑃𝑐𝑢2 = 𝑃𝑐𝑢1 = 0.5 2 = 0.028 𝑘𝑤 𝑃𝑐𝑜1 = 𝑃𝑐𝑜2 = 0.04 𝑘𝑤
𝑆1 1000
𝑃𝑜𝑢𝑡 2
𝑃𝑜𝑢𝑡 = 5 𝑘𝑣𝑎 𝑥0.8𝑥0.5 = 2 𝑘𝑤 𝑒𝑓𝑓𝑖𝑐𝑖𝑒𝑛𝑐𝑦 % = 𝑃 x 100 = 2+(0.028+0.04) 𝑥100 = 96.712 %
𝑜𝑢𝑡 +𝑃𝑙𝑜𝑠𝑠

𝑆2 2 112
c. 𝑃𝑐𝑢2 = 𝑃𝑐𝑢1 𝑆 = 1000
1.25 2
= 0.175 𝑘𝑤 𝑃𝑐𝑜1 = 𝑃𝑐𝑜2 = 0.04 𝑘𝑤
1
𝑃𝑜𝑢𝑡 5
𝑃𝑜𝑢𝑡 = 5 𝑘𝑣𝑎 𝑥0.8𝑥1.25 = 5 𝑘𝑤 𝑒𝑓𝑓𝑖𝑐𝑖𝑒𝑛𝑐𝑦 % = 𝑃 x 100 = 5+(0.175+0.04) 𝑥100 = 98.877 %
𝑜𝑢𝑡 +𝑃𝑙𝑜𝑠𝑠
EXAMPLE PROBLEMS
15. Using the data of the previous example , calculate the kva load (at a power factor of 0.8) when the
efficiency is a maximum and the maximum.
Given:
𝑆𝑜𝑢𝑡 = 5 𝑘𝑣𝑎
𝑃𝑠𝑐 = 𝑃𝑐𝑢 = 112 𝑤𝑎𝑡𝑡𝑠
𝑃𝑜𝑐 = 𝑃𝑐𝑜 = 40 𝑤𝑎𝑡𝑡𝑠
Solution:
𝑃𝑐 40
a. 𝑘𝑣𝑎max 𝑒𝑓𝑓 = 𝑘𝑣𝑎𝑟𝑎𝑡𝑒𝑑 =5 = 2.988 𝑘𝑣𝑎
𝑃𝑠 112

𝑎𝑡 𝑚𝑎𝑥𝑖𝑚𝑢𝑚 𝑒𝑓𝑓𝑖𝑐𝑖𝑒𝑛𝑐𝑦, 𝑃𝑐𝑜 = 𝑃𝑐𝑢 ,


𝑃𝑜𝑢𝑡 2.988 𝑥 0.8
𝑒𝑓𝑓𝑖𝑐𝑖𝑒𝑛𝑐𝑦 % = 𝑃 x 100 = (2.988𝑥0.8)+(0.04+0.04) 𝑥100 = 96.762 %
𝑜𝑢𝑡 +𝑃𝑙𝑜𝑠𝑠
Parallel Operation of Transformer

Several important conditions must be fulfilled if two or more transformers are to operate successfully in
parallel to deliver a common load. Theses important conditions are:
a. The voltage rating of both primaries and secondaries must be identical. This obviously implies that the
transformation ratios are the same.
b. The transformers must be properly connected with regard to polarity.
c. The equivalent impedances should be inversely proportional to the respective kilovolt ampere ratings.
d. The ratio of the equivalent resistance to the equivalent reactance of all transformers should be the same.

No load operation- When the secondary load is removed, the primaries will still be energized and the secondaries will
remain connected in parallel.
(𝑎1 −𝑎2 )𝐸𝑠
𝐼𝑐 = 𝑎
1 𝑍𝑒1 +𝑎2 𝑍𝑒2

𝑤ℎ𝑒𝑟𝑒,
𝐼𝑐 = 𝑐𝑖𝑟𝑐𝑢𝑙𝑎𝑡𝑖𝑛𝑔 𝑐𝑢𝑟𝑟𝑒𝑛𝑡, 𝑎𝑚𝑝
𝑎1 = 𝑡𝑟𝑎𝑛𝑠𝑓𝑜𝑟𝑚𝑎𝑡𝑖𝑜𝑛 𝑟𝑎𝑡𝑖𝑜 𝑜𝑓 1𝑠𝑡 𝑡𝑟𝑎𝑛𝑠𝑓𝑜𝑟𝑚𝑒𝑟 𝐸𝑠 = 𝑠𝑒𝑐𝑜𝑛𝑑𝑎𝑟𝑦 𝑣𝑜𝑙𝑡𝑎𝑔𝑒, 𝑣𝑜𝑙𝑡
𝑎2 = 𝑡𝑟𝑎𝑛𝑠𝑓𝑜𝑟𝑚𝑎𝑡𝑖𝑜𝑛 𝑟𝑎𝑡𝑖𝑜 𝑜𝑓 2𝑛𝑑 𝑡𝑟𝑎𝑛𝑠𝑓𝑜𝑟𝑚𝑒𝑟 𝑍𝑒1 = 𝑒𝑞𝑢𝑖𝑣𝑎𝑙𝑒𝑛𝑡 𝑖𝑚𝑝𝑒𝑑𝑎𝑛𝑐𝑒 𝑜𝑓 1𝑠𝑡 𝑡𝑟𝑎𝑛𝑠𝑓𝑜𝑚𝑒𝑟, 𝑣𝑜𝑙𝑡
𝑍𝑒2 = 𝑒𝑞𝑢𝑖𝑣𝑎𝑙𝑒𝑛𝑡 𝑖𝑚𝑝𝑒𝑑𝑎𝑛𝑐𝑒 𝑜𝑓 2𝑛𝑑 𝑡𝑟𝑎𝑛𝑠𝑓𝑜𝑚𝑒𝑟, 𝑣𝑜𝑙𝑡
Load operation –Equal Ratios of Transformation- When two transformers having equal ratios of transformation ratios are connected in parallel, the total
load current will divide between them inversely as their equivalent impedances.
𝐸𝑝
𝑆𝑖𝑛𝑐𝑒 𝐼1 𝑍𝑒1 𝑎𝑛𝑑 𝐼2 𝑍𝑒2 𝑏𝑜𝑡ℎ 𝑒𝑞𝑢𝑎𝑙 − 𝐸𝑠 , 𝑖𝑡 𝑓𝑜𝑙𝑙𝑜𝑤𝑠 𝑡ℎ𝑎𝑡,
𝑎
𝐼1 𝑍𝑒1 = 𝐼2 𝑍𝑒2
𝐼1 𝑍𝑒1
Thus, 𝐼2
=
𝑍𝑒2

It implies that when two transformer of different kVA ratings are connected in parallel, they divide the total load in proportion to their respective kVA
ratings only when their equivalent impedances are inversely proportional to their respective ratings.
Where,
𝐼1 = 𝑐𝑢𝑟𝑟𝑒𝑛𝑡 𝑠ℎ𝑎𝑟𝑒 𝑜𝑓 1𝑠𝑡 𝑡𝑟𝑎𝑛𝑠𝑓𝑜𝑟𝑚𝑒𝑟, 𝑎𝑚𝑝𝑒𝑟𝑒
𝐼2 = 𝑐𝑢𝑟𝑟𝑒𝑛𝑡 𝑠ℎ𝑎𝑟𝑒 𝑜𝑓 2𝑛𝑑 𝑡𝑟𝑎𝑛𝑠𝑓𝑜𝑚𝑒𝑟, 𝑎𝑚𝑝𝑒𝑟𝑒 𝑍𝑒1 = 𝑒𝑞𝑢𝑖𝑣𝑎𝑙𝑒𝑛𝑡 𝑖𝑚𝑝𝑒𝑑𝑎𝑛𝑐𝑒 𝑜𝑓 1𝑠𝑡 𝑡𝑟𝑎𝑛𝑠𝑓𝑜𝑚𝑒𝑟, 𝑣𝑜𝑙𝑡
𝑍𝑒2 = 𝑒𝑞𝑢𝑖𝑣𝑎𝑙𝑒𝑛𝑡 𝑖𝑚𝑝𝑒𝑑𝑎𝑛𝑐𝑒 𝑜𝑓 2𝑛𝑑 𝑡𝑟𝑎𝑛𝑠𝑓𝑜𝑚𝑒𝑟, 𝑣𝑜𝑙𝑡
Load operation –Unequal Ratios of Transformation- When two transformers having unequal ratios of transformation ratios are connected in parallel, the
total load current will divide in accordance with the following equations.

(𝑎2 −𝑎1 )𝐸𝑠 +(𝑎2 𝑍𝑒2 𝐼𝑡 )


𝐼1 =
𝑎1 𝑍𝑒1 +𝑎2 𝑍𝑒2
(𝑎1 −𝑎2 )𝐸𝑠 +(𝑎1 𝑍𝑒1 𝐼𝑡 )
𝐼2 =
𝑎1 𝑍𝑒1 +𝑎2 𝑍𝑒2

Where,
𝐼𝑡 = 𝑡𝑜𝑡𝑎𝑙 𝑐𝑢𝑟𝑟𝑒𝑛𝑡, 𝑎𝑚𝑝𝑒𝑟𝑒
EXAMPLE PROBLEMS
16. The following information is given in connection with two transformer that are connected in parallel. Determine the circulating current.

Transformer 1 Transformer 2
25 kva 35 kva
2360/230 volts 2300/230 volts
Ze=0.08 ohm, in secondary terms Ze=0.06 ohm, in secondary terms
Solution:
2360 2300
𝑎1 = = 10.26 𝑎2 = = 10
230 230
(𝑎1 −𝑎2 )𝐸𝑠 10.26−10 230
𝐼𝑐 = = = 𝟒𝟐. 𝟏 𝒂𝒎𝒑
𝑎1 𝑍𝑒1 +𝑎2 𝑍𝑒2 10.26 0.08 +(10)(0.06)

17. The following information is given for two transformers connected in parallel and delivering a total load of 300 kva. Calculate the load current and kilovolt amperes delivered by each transformer.

Transformer 1 Transformer 2

150 kva 250 kva


6900/230 volts 6900/230 volts
Ze=9.4 ohm, in primary terms Ze=5.8 ohm, in primary terms

300,000 𝑉𝐴 𝐼1 𝑍𝑒1 5.8 𝑍𝑒1 5.8 5.8


𝐼𝑡 = = 43.478 𝐴 = = ; 𝐼1 = 𝐼2 = 𝐼2 ; 43.478 = 𝐼𝑡 = 𝐼1 + 𝐼2 = 𝐼2 + 𝐼2 ; 𝐼2 = 26.888 𝐴
6900 𝑉 𝐼2 𝑍𝑒2 9.4 𝑍𝑒2 9.4 9.4

𝐼1 = 𝐼𝑡 − 𝐼2 = 43.478 − 26.888 = 16.59 𝐴

𝑆1 = 6.9 𝑘𝑣 𝑥 16.59 𝐴 = 114.471 𝑘𝑣𝑎 𝑆2 = 6.9 𝑘𝑣 𝑥 26.888 𝐴 = 185.527 𝑘𝑣𝑎

𝑆𝑡 = 𝑆1 + 𝑆2 = 114.471 𝑘𝑣𝑎 + 185.527 𝑘𝑣𝑎 = 300 𝑘𝑣𝑎


EXAMPLE PROBLEMS
18. The transformer of example 16 deliver a total load of 46 kva. Calculate the secondary currents and the kilovolt ampere load of each one.

Transformer 1 Transformer 2
25 kva 35 kva
2360/230 volts 2300/230 volts

Solution: Ze=0.08 ohm, in secondary terms Ze=0.06 ohm, in secondary terms

2360 2300 46000


𝑎1 = = 10.26 𝑎2 = = 10 𝐼𝑡 = = 200 𝑎𝑚𝑝
230 230 230
(𝑎2 −𝑎1 )𝐸𝑠 +(𝑎2 𝑍𝑒2 𝐼𝑡 ) 10.26−10 230+(10)(0.06)(200)
𝐼1 = = = 𝟏𝟐𝟔. 𝟓𝟒𝟖 𝒂𝒎𝒑
𝑎1 𝑍𝑒1 +𝑎2 𝑍𝑒2 10.26 0.08 +(10)(0.06)
(𝑎1 −𝑎2 )𝐸𝑠 +(𝑎1 𝑍𝑒1 𝐼𝑡 ) 10−10.26 230+(10.26)(0.08)(200)
𝐼2 = = = 𝟕𝟑. 𝟒𝟓𝟐 𝒂𝒎𝒑
𝑎1 𝑍𝑒1 +𝑎2 𝑍𝑒2 10.26 0.08 +(10)(0.06)
𝑆1 = 0.23 𝑘𝑣 𝑥 126.548 𝐴 = 𝟐𝟗. 𝟏𝟎𝟔 𝒌𝒗𝒂 𝑆2 = 0.23 𝑘𝑣 𝑥 73.452 𝐴 = 𝟏𝟔. 𝟖𝟗𝟒 𝐤𝐯𝐚

You might also like